Friday, November 5, 2010

IS MASTURBATION A SIN?

"'When a man has an emission of semen, he must bathe his whole body with water, and he will be unclean till evening. Any clothing or leather that has semen on it must be washed with water, and it will be unclean till evening. When a man lies with a woman and there is an emission of semen, both must bathe with water, and they will be unclean till evening. (Lev 15:16-18)
Notice verse 18 deals with seminal emission during sexual intercourse, but verse 16 is speaking of a seminal emission that does not occur as a result of intercourse. He simply said, "Go take a bath." The phrase has been misunderstood by many especially those that uses their head knowledge and intellect to interprete the Word of God as a permission from Hm to embark on the subject matter. Even research has found that masturbation serves to release sexual tension. If a man is prevented from having sexual release, then the seminal fluids can back up, causing increased sexual tension and physical discomfort. Can we because of scientific point of view accept this as a proper act? Can we start or continue in this act and we think God is pleasant with it? We should draw a curtain between nightmare and masturbation as verse 16 is only talking about nightmare.
Oxford dictionary defines masturbation as a process of giving yourself sexual pleasure by rubbing your sexual organs. In order words it is a deliberate act by self in order to satisfy the desire of your flesh. While the Bible nowhere explicitly states that masturbation is a sin, there is no question as to whether the actions that lead to masturbation are sinful. Masturbation is a sin because a person consents in their own heart to take sexual gratification from impure thoughts. It is sinful to indulge in the sexual pleasure that is derived from the satisfaction of using one's imagination to fixate upon obscene images, or obscene actions, or forbidden sex. Lustfully fixating upon genitals, or sexual acts, is idolatry. This is called: the worship of obscenities. This is the sin of those addicted to pornography. The act of masturbation is an outward act that confirms that one's heart and mind has consented to an inward act of impurity. To attain sexual release, a person's will consent to love something that is sexually impure.
A sin consists in loving something that God hates, or in hating something that God loves. Go Masturbation is nearly always the result of lustful thoughts, sexual stimulation, and/or pornographic images. It is these problems that need to be dealt with. If the sins of lust, immoral thoughts, and pornography are forsaken and overcome, masturbation will become a non-issue. Many people struggle with guilty feelings concerning masturbation, when in reality, the things that led to the act are far more worthy of repentance.

There are some biblical principles that can be applied to the issue of masturbation. Ephesians 5:3 declares, “Among you there must not be even a hint of sexual immorality, or of any kind of impurity.” It is hard to see how masturbating can pass that particular test. The Bible teaches us, “So whether you eat or drink or whatever you do, do it all for the glory of God” 1 Corinthians 10:31. If you cannot give God glory for something, you should not do it. If a person is not fully convinced that an activity is pleasing to God, then it is a sin: “Everything that does not come from faith is sin” Romans 14:23. Further, we need to remember that our bodies have been redeemed and belong to God. “Do you not know that your body is a temple of the Holy Spirit, who is in you, whom you have received from God? You are not your own; you were bought at a price. Therefore honor God with your body” 1 Corinthians 6:19-20.
CONCLUSION: This great truth should have a real bearing on what we do with our bodies. In light of these principles, the conclusion that masturbation is a sin is biblical. Clearly, masturbation is not glorifying to God; it does not avoid the appearance of immorality, nor does it pass the test of God’s having ownership over our bodies.
Olatunji Olusegun
Redeemed Christian Fellowship
victorolat@gmail.com

Sunday, October 31, 2010

LoSOLUTIONS MANUAL FUNDAMENTALS OF MODERN MANUFACTURING: MATERIALS, PROCESSES, AND SYSTEMS Second Edition MIKELL P. GROOVER

1

Professor of Industrial and Manufacturing Systems Engineering
Lehigh University
John Wiley & Sons, Inc., New York
2
PREFACE
This is the Solutions Manual for the textbook Fundamentals of Modern Manufacturing:
Materials, Processes, and Systems (Second Edition). It contains the answers to the Review
Questions and Multiple Choice Quizzes at the end of the Chapters 2 through 44, as well as the
Problems at the end of Chapters 3, 4, 6, 10, 11, 13, 16, 18, 19, 20, 21, 22, 23, 24, 25, 26, 29, 30,
31, 33, 34, 35, 38, 40, 42, and 43. There are approximately 740 review questions, 500 quiz questions,
and 500 problems (nearly all of them quantitative) in the text.
I have personally answered all of the questions and solved all of the quizzes and problems and have
personally recorded the solutions in this booklet. Many of the problems have been tested in class, thus
giving me an opportunity to compare my own answers with those developed by the students. Despite
my best efforts to avoid errors in this solutions manual, I am sure that errors are present. I would
appreciate hearing from those of you who discover these errors, so that I can make the necessary
corrections in subsequent editions of the Solutions Manual. Similarly, I would appreciate any
suggestions from users of the text itself that might help to make any subsequent editions more accurate,
more relevant, and easier to use. My address is:
Dr. Mikell P. Groover
Department of Industrial and Manufacturing Systems Engineering
Lehigh University
200 West Packer Avenue
Bethlehem, PA 18015
Office telephone number 610-758-4030.
Fax machine number 610-758-4886.
E-mail addresses: either
Mikell.Groover@Lehigh.edu
or
mpg0@Lehigh.edu
I hope you find the text and this Solutions Manual to be helpful teaching aids in your particular
manufacturing course.
Mikell P. Groover
3
TABLE OF CONTENTS:
Chapter Chapter Title* Page
1. Introduction (No questions or problems)
2. The Nature of Materials 4
3. Mechanical Properties of Materials (P) 7
4. Physical Properties of Materials (P) 18
5. Dimensions, Tolerances, and Surfaces 21
6. Metals (P) 24
7. Ceramics 29
8. Polymers 32
9. Composite Materials 36
10. Fundamentals of Casting (P) 39
11. Metal Casting Processes (P) 49
12. Glassworking 57
13. Shaping Processes for Plastics (P) 60
14. Rubber Processing Technology 70
15. Shaping Processes for Polymer Matrix Composites 73
16. Powder Metallurgy (P) 76
17. Processing of Ceramics and Cermets 84
18. Fundamentals of Metal Forming (P) 87
19. Bulk Deformation Processes (P) 92
20. Sheet Metalworking (P) 112
21. Theory of Metal Machining (P) 122
22. Machining Operations and Machine Tools (P) 134
23. Cutting Tool Technology (P) 142
24. Economic and Product Design Considerations in Machining (P) 153
25. Grinding and Other Abrasive Processes (P) 166
26. Nontraditional Machining and Thermal Cutting Processes (P) 173
27. Heat Treatment of Metals 180
28. Cleaning and Surface Treatments 182
29. Coating and Deposition Processes (P) 184
30. Fundamentals of Welding (P) 190
31. Welding Processes (P) 197
32. Brazing, Soldering, and Adhesive Bonding 207
33. Mechanical Assembly (P) 211
34. Rapid Prototyping (P) 218
35. Processing of Integrated Circuits (P) 222
36. Electronics Assembly and Packaging 230
37. Microfabrication Technologies 233
38. Numerical Control and Industrial Robotics (P) 235
39. Group Technology and Flexible Manufacturing Systems 244
40. Production Lines (P) 246
41. Manufacturing Engineering 253
42. Production Planning and Control (P) 256
43. Quality Control (P) 263
44. Measurement and Inspection 271
*(P) indicates chapters with problem sets.
4
2 THE NATURE OF MATERIALS
Review Questions
2.1 The elements listed in the Periodic Table can be divided into three categories. What are these
categories and give an example of each?
Answer. The three types of elements are metals (e.g., aluminum), nonmetals (e.g., oxygen), and
semimetals (e.g., silicon).
2.2 Which elements are the noble metals?
Answer. The noble metals are copper, silver, and gold.
2.3 What is the difference between primary and secondary bonding in the structure of materials?
Answer. Primary bonding is strong bonding between atoms in a material, for example to form a
molecule; while secondary bonding is not as strong and is associated with attraction between
molecules in the material.
2.4 Describe how ionic bonding works?
Answer. In ionic bonding, atoms of one element give up their outer electron(s) to the atoms of
another element to form complete outer shells.
2.5 What is the difference between crystalline and noncrystalline structures in materials?
Answer. The atoms in a crystalline structure are located at regular and repeating lattice positions in
three dimensions; thus, the crystal structure possesses a long-range order which allows a high
packing density. The atoms in a noncrystalline structure are randomly positioned in the material, not
possessing any repeating, regular pattern.
2.6 What are some common point defects in a crystal lattice structure?
Answer. Some of the common point defects are: (1) vacancy - a missing atom in the lattice
structure; (2) ion-pair vacancy (Schottky defect) - a missing pair of ions of opposite charge in a
compound; (3) interstitialcy - a distortion in the lattice caused by an extra atom present; and (4)
Frenkel defect - an ion is removed from a regular position in the lattice and inserted into an
interstitial position not normally occupied by such an ion.
2.7 Define the difference between elastic and plastic deformation in terms of the effect on the crystal
lattice structure.
Answer. Elastic deformation involves a temporary distortion of the lattice structure that is
proportional to the applied stress. Plastic deformation involves a stress of sufficient magnitude to
cause a permanent shift in the relative positions of adjacent atoms in the lattice. Plastic deformation
generally involves the mechanism of slip - relative movement of atoms on opposite sides of a plane
in the lattice.
2.8 How do grain boundaries contribute to the strain hardening phenomenon in metals?
Answer. Grain boundaries block the continued movement of dislocations in the metal during
straining. As more dislocations become blocked, the metal becomes more difficult to deform; in
effect it becomes stronger.
2.9 Identify some materials that have a crystalline structure.
5
Answer. Materials typically possessing a crystalline structure are metals and ceramics other than
glass. Some plastics have a partially crystalline structure.
2.10 Identify some materials that possess a noncrystalline structure.
Answer. Materials typically having a noncrystalline structure include glass (fused silica), rubber,
and certain plastics (specifically, thermosetting plastics).
2.11 What is the basic difference in the solidification (or melting) process between crystalline and
noncrystalline structures?
Answer. Crystalline structures undergo an abrupt volumetric change as they transform from liquid
to solid state and vice versa. This is accompanied by an amount of energy called the heat of fusion
that must be added to the material during melting or released during solidification. Noncrystalline
materials melt and solidify without the abrupt volumetric change and heat of fusion.
Multiple Choice Quiz
There are a total of 20 correct answers in the following multiple choice questions (some questions have
multiple answers that are correct). To attain a perfect score on the quiz, all correct answers must be
given, since each correct answer is worth 1 point. For each question, each omitted answer or wrong
answer reduces the score by 1 point, and each additional answer beyond the number of answers required
reduces the score by 1 point. Percentage score on the quiz is based on the total number of correct
answers.
2.1 The basic structural unit of matter is which one of the following? (a) atom, (b) electron, (c) element,
(d) molecule, or (e) nucleus.
Answer. (a)
2.2 Approximately how many different elements have been identified (one answer)? (a) 10, (b) 50, (c)
100, (d) 200, or (e) 500.
Answer. (c)
2.3 In the Periodic Table, the elements can be divided into which of the following categories (more than
one)? (a) ceramics, (b) gases, (c) liquids, (d) metals, (e) nonmetals, (f) polymers, (g) semi-metals,
and (h) solids.
Answer. (d), (e), and (g).
2.4 The element with the lowest density and smallest atomic weight is which one of the following? (a)
aluminum, (b) argon, (c) helium, (d) hydrogen, or (e) magnesium.
Answer. (d)
2.5 Which of the following bond types are classified as primary bonds (more than one)? (a) covalent
bonding, (b) hydrogen bonding, (c) ionic bonding, (d) metallic bonding, and (e) van der Waals forces.
Answer. (a), (c), and (d).
2.6 How many atoms are there in the unit cell of the face- centered cubic (FCC) unit cell (one
answer)? (a) 8, (b) 9, (c) 10, (d) 12, or (e) 14.
Answer. (e)
2.7 Which of the following are not point defects in a crystal lattice structure (more than one)? (a) edge
dislocation, (b) interstitialcy, (c) Schottky defect, or (d) vacancy.
6
Answer. (b), (c), (d)
2.8 Which one of the following crystal structures has the fewest slip directions and therefore the metals
with this structure are generally more difficult to deform at room temperature? (a) BCC, (b) FCC,
or (c) HCP.
Answer. (c)
2.9 Grain boundaries are an example of which one of the following types of crystal structure defects?
(a) dislocation, (b) Frenkel defect, (c) line defects, (d) point defects, or (e) surface defects.
Answer. (e)
2.10 Twinning is which of the following (more than one)? (a) elastic deformation, (b) mechanism of
plastic deformation, (c) more likely at high deformation rates, (d) more likely in metals with HCP
structure, (e) slip mechanism, and (f) type of dislocation.
Answer. (b), (c), and (d).
2.11 Polymers are characterized by which of the following bonding types (more than one)? (a) adhesive,
(b) covalent, (c) hydrogen, (d) ionic, (e) metallic, and (f) van der Waals.
Answer. (b) and (f).
7
3 MECHANICAL PROPERTIES OF MATERIALS
Review Questions
3.1 What is the dilemma between design and manufacturing in terms of mechanical properties?
Answer. To achieve design function and quality, the material must be strong; for ease of
manufacturing, the material should not be strong, in general.
3.2 What are the three types of static stresses to which materials are subjected?
Answer. tensile, compressive, and shear.
3.3 State Hooke's Law.
Answer. Hooke's Law defines the stress-strain relationship for an elastic material: = E, where
E = a constant of proportionality called the modulus of elasticity.
3.4 What is the difference between engineering stress and true stress in a tensile test?
Answer. Engineering stress divides the load (force) on the test specimen by the original area; while
true stress divides the load by the instantaneous area which decreases as the specimen stretches.
3.5 Define tensile strength of a material.
Answer. The tensile strength is the maximum load experienced during the tensile test divided by the
original area.
3.6 Define yield strength of a material.
Answer. The yield strength is the stress at which the material begins to plastically deform. It is
usually measured as the .2% offset value - the point at which the stress-strain for the material
intersects a line which is offset from the elastic region of the stress-strain curve by 0.2%.
3.7 Why cannot a direct conversion be made between the ductility measures of elongation and
reduction in area using the assumption of constant volume?
Answer. Because of necking that occurs in the test specimen.
3.8 What is work hardening?
Answer. Strain hardening is the increase in strength that occurs in metals when they are strained.
3.9 In what case does the strength coefficient have the same value as the yield strength?
Answer. When the material does not strain harden.
3.10 How does the change in cross-sectional area of a test specimen in a compression test differ from its
counterpart in a tensile test specimen?
Answer. In a compression test, the specimen cross-sectional are increases as the test progresses;
while in a tensile test, the cross-sectional area decreases.
3.11 What is the complicating factor that occurs in a compression test?
Answer. Barreling of the test specimen due to friction at the interfaces with the testing machine
platens.
8
3.12 Tensile testing is not appropriate for hard brittle materials such as ceramics. What is the test
commonly used to determine the strength properties of such materials?
Answer. A three-point bending test is commonly used to test the strength of brittle materials. The
test provides a measure called the transverse rupture strength for these materials.
3.13 How is the shear modulus of elasticity G related to the tensile modulus of elasticity E, on average?
Answer. G = 0.4 E, on average.
3.14 How is shear strength S related to tensile strength TS, on average?
Answer. S = 0.7 TS, on average.
3.15 What is hardness and how is it generally tested?
Answer. Hardness is defined as the resistance to indentation of a material. It is tested by pressing
a hard object (sphere, diamond point) into the test material and measuring the size (depth, area) of
the indentation.
3.16 Why are different hardness tests and scales required?
Answer. Different hardness tests and scales are required because different materials possess
widely differing hardnesses. A test whose measuring range is suited to very hard materials is not
sensitive for testing very soft materials.
3.17 Define the recrystallization temperature for a metal.
Answer. The recrystallization temperature is the temperature at which a metal recrystallizes (forms
new grains) rather than work hardens when deformed.
3.18 Define viscosity of a fluid.
Answer. Viscosity is the resistance to flow of a fluid material; the thicker the fluid, the greater the
viscosity.
3.19 What is the defining characteristic of a Newtonian fluid?
Answer. A Newtonian fluid is one for which viscosity is a constant property at a given
temperature. Most liquids (water, oils) are Newtonian fluids.
3.20 What is viscoelasticity, as a material property?
Answer. Viscoelasticity refers to the property most commonly exhibited by polymers that defines
the strain of the material as a function of stress and temperature over time. It is a combination of
viscosity and elasticity.
Multiple Choice Quiz
There are a total of 18 correct answers in the following multiple choice questions (some questions have
multiple answers that are correct). To attain a perfect score on the quiz, all correct answers must be
given, since each correct answer is worth 1 point. For each question, each omitted answer or wrong
answer reduces the score by 1 point, and each additional answer beyond the number of answers required
reduces the score by 1 point. Percentage score on the quiz is based on the total number of correct
answers.
3.1 Which one of the following are the three basic types of static stresses to which a material can be
subjected (three answers)? (a) compression, (b) hardness, (c) reduction in area, (d) shear, (e)
tensile, (f) true stress, and (f) yield.
9
Answer. (a), (d), and (e).
3.2 Which of the following is the correct definition of ultimate tensile strength, as derived from the
results of a tensile test on a metal specimen? (a) the stress encountered when the stress-strain
curve transforms from elastic to plastic behavior, (b) the maximum load divided by the final area of
the specimen, (c) the maximum load divided by the original area of the specimen, or (d) the stress
observed when the specimen finally fails.
Answer. (c)
3.3 If stress values were measured during a tensile test, which of the following would have the higher
value? (a) engineering stress, or (b) true stress.
Answer. (b)
3.4 If strain measurements were made during a tensile test, which of the following would have the
higher value? (a) engineering stain, or (b) true strain.
Answer. (a)
3.5 The plastic region of the stress-strain curve for a metal is characterized by a proportional
relationship between stress and strain: (a) true or (b) false.
Answer. (b) It is the elastic region that is characterized by a proportional relationship between
stress and strain. The plastic region is characterized by a power function - the flow curve.
3.6 Which one of the following types of stress strain relationship best describes the behavior of brittle
materials such as ceramics and thermosetting plastics: (a) elastic and perfectly plastic, (b) elastic
and strain hardening, (c) perfectly elastic, or (d) none of the above.
Answer. (c)
3.7 Which one of the following types of stress strain relationship best describes the behavior of most
metals at room temperature: (a) elastic and perfectly plastic, (b) elastic and strain hardening, (c)
perfectly elastic, or (d) none of the above.
Answer. (b)
3.8 Which of the following types of stress strain relationship best describes the behavior of metals at
temperatures above their respective recrystallization points: (a) elastic and perfectly plastic, (b)
elastic and strain hardening, (c) perfectly elastic, or (d) none of the above.
Answer. (a)
3.9 Which one of the following materials has the highest modulus of elasticity? (a) aluminum, (b)
diamond, (c) steel, (d) titanium, or (e) tungsten.
Answer. (b)
3.10 The shear strength of a metal is usually (a) greater than, or (b) less than its tensile strength.
Answer. (b)
3.11 Most hardness tests involve pressing a hard object into the surface of a test specimen and
measuring the indentation (or its effect) that results: (a) true or (b) false.
Answer. (a)
3.12 Which one of the following materials has the highest hardness? (a) alumina ceramic, (b) gray cast
iron, (c) hardened tool steel, (d) high carbon steel, or (e) polystyrene.
10
Answer. (a)
3.13 Viscosity can be defined as the ease with which a fluid flows: (a) true or (b) false.
Answer. (b) Viscosity is the resistance to flow.
3.14 Viscoelasticity has features of which of the following more traditional material properties (more
than one)? (a) elasticity, (b) plasticity, (c) viscosity.
Answer. (a), (b), (c). This answer may require some justification. Viscoelasticity is usually
considered to be a property that combines elasticity and viscosity. However, in deforming over time
it involves plastic flow (plasticity). Strictly speaking, the shape return feature in viscoelastic
behavior violates the definition of plastic flow; however, many materials considered to be
viscoelastic do not completely return to their original shape.
Problems
Strength and Ductility in Tension
3.1 A tensile test uses a test specimen that has a gage length of 50 mm and an area = 200 mm2. During
the test the specimen yields under a load of 98,000 N. The corresponding gage length = 50.23 mm.
This is the 0.2 percent yield point. The maximum load = 168,000 N is reached at a gage length =
64.2 mm. Determine: (a) yield strength Y, (b) modulus of elasticity E, and (c) tensile strength TS.
Solution: (a) Y = 98,000/200 = 490 MPa.
(b) = E e
Subtracting the 0.2% offset, e = (50.23 - 50.0)/50.0 - 0.002 = 0.0026
E = /e = 490/0.0026 = 188.5 x 103 MPa.
(c) TS = 168,000/200 = 840 MPa.
3.2 A test specimen in a tensile test has a gage length of 2.0 in and an area = 0.5 in2. During the test
the specimen yields under a load of 32,000 lb. The corresponding gage length = 2.0083 in. This is
the 0.2 percent yield point. The maximum load = 60,000 lb is reached at a gage length = 2.60 in.
Determine: (a) yield strength Y, (b) modulus of elasticity E, and (c) tensile strength TS.
Solution: (a) Y = 32,000/0.5 = 64,000 lb/in2
(b) = E e
Subtracting the 0.2% offset, e = (2.0083 - 2.0)/2.0 - 0.002 = 0.00215
E = /e = 64,000/0.00215 = 29.77 x 106 lb/in2
(c) TS = 60,000/0.5 = 120,000 lb/in2
3.3 In Problem 3.1, (a) determine the percent elongation. (b) If the specimen necked to an area = 92
mm2, determine the percent reduction in area.
Solution: (a) % elongation = (64.2 - 50)/50 = 14.2/50 = 0.284 = 28.4%
(b) % area reduction = (200 - 92)/200 = 0.54 = 54%
3.4 In Problem 3.2, (a) determine the percent elongation. (b) If the specimen necked to an area = 0.25
in2, determine the percent reduction in area.
Solution: (a) % elongation = (2.60 - 2.0)/2.0 = 0.6/2.0 = 0.3 = 30%
(b) % area reduction = (0.5 - 0.25)/0.5 = 0.50 = 50%
11
3.5 The following data are collected during a tensile test in which the starting gage length = 125.0 mm
and the cross- sectional area = 62.5 mm2:
Load (N) 0 17,793 23,042 27,579 28,913 27,578 20,462
Length (mm) 0 125.23 131.25 140.05 147.01 153.00 160.10
The maximum load is 28,913 N and the final data point occurred immediately prior to failure. (a)
Plot the engineering stress strain curve. Determine: (b) yield strength Y, (c) modulus of elasticity E,
(d) tensile strength TS.
Solution: (a) Student exercise.
(b) From the plot, Y = 310.27 MPa.
(c) First data point is prior to yielding.
Strain e = (125.23 - 125)/125 = 0.00184, E = 310.27/0.00184 = 168,625 MPa.
(d) From the plot, TS = 426.6 MPa.
Flow Curve
3.6 In Problem 3.5, determine the strength coefficient and the strain hardening exponent. Be sure not to
use data after the point at which necking occurred.
Solution: Starting volume of test specimen V = 125(62.5) = 7812.5 mm3.
Select two data points: (1) F = 23042 N and L = 131.25 mm; (2) F = 28913 N and L = 147.01 mm.
(1) A = V/L = 7812.5/131.25 = 59.524 mm2.
Stress = 23042/59.524 = 387.1 MPa. Strain = ln(131.25/125) = 0.0488
(2) A = 7812.5/147.01 = 53.143 mm2.
Stress = 28913/53.143 = 544.1 MPa. Strain = ln(147.01/125) = 0.1622
Substituting these values into the flow curve equation, we have
(1) 387.1 = K(0.0488)n and (2) 544.1 = K(0.1622)n
544.1/387.1 = (0.1622/0.0488)n
1.4056 = (3.3238)n
ln(1.4056) = n ln(3.3238) 0.3405 = 1.2011 n n = 0.283
Substituting this value with the data back into the flow curve equation, we obtain the value of the
strength coefficient K:
K = 387.1/(0.0488).283 = 909.9 MPa
K = 544.1/(0.1622).283 = 910.4 MPa Use average K = 910.2 MPa
The flow curve equation is: s = 910.2 e 0.283
3.7 In a tensile test on a metal specimen, true strain = 0.08 at a stress = 265 MPa. When the true stress
= 325 MPa, the true strain = 0.27. Determine the flow curve parameters n and K.
Solution: (1) 265 = K(0.08)n and (2) 325 = K(0.27)n
325/265 = (0.27/0.08)n 1.2264 = (3.375)n
n ln(3.375) = ln(1.2264) 1.2164 n = 0.2041 n = 0.1678
Substituting this value with the data back into the flow curve equation, we obtain the value of the
strength coefficient K:
(1) K = 265/(0.08).1678 = 404.85 MPa
(2) K = 325/(0.27).1678 = 404.85 MPa
12
The flow curve equation is: s = 404.85 e 0.1678
3.8 During a tensile test, a metal has a true strain = 0.10 at a true stress = 37,000 lb/in2. Later, at a true
stress = 55,000 lb/in2, the true strain = 0.25. Determine the flow curve parameters n and K.
Solution: (1) 37,000 = K(0.10)n and (2) 55,000 = K(0.25)n
55,000/37,000 = (0.25/0.10)n 1.4865 = (2.5)n
n ln(2.5) = ln(1.4865) 0.9163 n = 0.3964 n = 0.4326
Substituting this value with the data back into the flow curve equation, we obtain the value of the
strength coefficient K:
(1) K = 37,000/(0.10).4326 = 100,191 lb/in2
(2) K = 55,000/(0.25).4326 = 100,191 lb/in2
The flow curve equation is: s = 100,191 e 0.4326
3.9 In a tensile test a metal begins to neck at a true strain = 0.28 with a corresponding true stress =
345.0 MPa. Without knowing any more about the test, can you estimate the flow curve parameters
n and K?
Solution: If we assume that n = when necking starts, then n = 0.28.
Using this value in the flow curve equation, we have K = 345/(0.28).28 = 492.7 MPa
The flow curve equation is: s = 492.7 e 0.28
3.10 A tensile test for a certain metal provides flow curve parameters: n = 0.3 and K = 600 MPa.
Determine: (a) the flow stress at a true strain = 1.0, and (b) true strain at a flow stress = 600 MPa.
Solution: (a) Yf = 600(1.0).3 = 600 MPa
(b) = (600/600)1/.3 = (1.0)3.33 = 1.00
3.11 The flow curve for a certain metal has parameters: n = 0.22 and K = 54,000 lb/in2. Determine: (a)
the flow stress at a true strain = 0.45, and (b) the true strain at a flow stress = 40,000 lb/in2.
Solution: (a) Yf = 54,000(0.45).22 = 45,300 lb/in2
(b) = (40,000/54,000)1/.22 = (0.7407)4.545 = 0.256
3.12 A metal is deformed in a tension test into its plastic region. The starting specimen had a gage length
= 2.0 in and an area = 0.50 in2. At one point in the tensile test, the gage length = 2.5 in and the
corresponding engineering stress = 24,000 lb/in2; and at another point in the test prior to necking, the
gage length = 3.2 in and the corresponding engineering stress = 28,000 lb/in2. Determine the
strength coefficient and the strain hardening exponent for this metal.
Solution: Starting volume V = LoAo = 2.0(0.5) = 1.0 in3
(1) A = V/L = 1.0/2.5 = 0.4 in2
So, true stress = 24,000(.5)/.4 = 31,250 lb/in2 and = ln(2.5/2.0) = 0.223
(2) A = 1.0/3.2 = 0.3125 in2
So, true stress = 28,000(.5)/.3125 = 44,800 lb/in2 and = ln(3.2/2.0) = 0.470
These are two data points with which to determine the parameters of the flow curve equation.
(1) 31,250 = K(0.223)n and (2) 44,800 = K(0.470)n
44,800/31,250 = (0.470/0.223)n
1.4336 = (2.1076)n
ln(1.4336) = n ln(2.1076)
13
.3602 = .7455 n n = 0.483
(1) K = 31,250/(0.223).483 = 64,513 lb/in2
(2) K = 44,800/(0.470).483 = 64,516 lb/in2 Use average K = 64,515 lb/in2
The flow curve equation is: s = 64,515 e 0.483
3.13 A tensile test specimen has a starting gage length = 75.0 mm. It is elongated during the test to a
length = 110.0 mm before necking occurs. (a) Determine the engineering strain. (b) Determine the
true strain. (c) Compute and sum the engineering strains as the specimen elongates from: (1) 75.0 to
80.0 mm, (2) 80.0 to 85.0 mm, (3) 85.0 to 90.0 mm, (4) 90.0 to 95.0 mm, (5) 95.0 to 100.0 mm, (6)
100.0 to 105.0 mm, and (7) 105.0 to 110.0 mm. (d) Is the result closer to the answer to part (a) or
part (b)? Does this help to show what is meant by the term true strain?
Solution: (a) Engineering strain e = (110 - 75)/75 = 35/75 = 0.4667
(b) True strain = ln(110/75) = ln(1.4667) = 0.383
(c)L = 75 to 80 mm: e = (80 - 75)/75 = 5/75 = 0.0667
L = 80 to 85 mm: e = (85 - 80)/80 = 5/80 = 0.0625
L = 85 to 90 mm: e = (90 - 85)/85 = 5/85 = 0.0588
L = 90 to 95 mm: e = (95 - 90)/90 = 5/90 = 0.0556
L = 95 to 100 mm: e = (100 - 95)/95 = 5/95 = 0.0526
L = 100 to 105 mm: e = (105 - 100)/100 = 5/100 = 0.0500
L = 105 to 110 mm: e = (110 - 105)/105 = 5/105 = 0.0476
_____________________________________________
Sum of incremental engineering strain values = 0.3938
(d) The resulting sum in (c) is closer to the true strain value in (b). The summation process is an
approximation of the integration over the range from 75 to 110 mm in (b). As the interval size is
reduced, the summation becomes closer to the integration value.
3.14 A tensile specimen is elongated to twice its original length. Determine the engineering strain and
true strain for this test. If the metal had been strained in compression, determine the final
compressed length of the specimen such that: (a) the engineering strain is equal to the same value
as in tension (it will be negative value because of compression), and (b) the true strain would be
equal to the same value as in tension (again, it will be negative value because of compression). Note
that the answer to part (a) is an impossible result. True strain is therefore a better measure of strain
during plastic deformation.
Solution: Engineering strain e = (2.0 - 1.0)/1.0 = 1.0
True strain = ln(2.0/1.0) = ln(2.0) = 0.693
(a) To be compressed to the same engineering strain (e = -1.0) the final height of the compression
specimen would have to be zero, which is impossible.
(b) To be compressed to the same true strain value (e = -0.693) the final height of the compression
specimen can be determined as follows:
= -.693 = ln(Lf/Lo)
Lf/Lo = exp.(-0.693) = 0.500 Therefore, Lf = 0.5 Lo
3.15 Derive an expression for true strain as a function of D and Do for a tensile test specimen of round
cross-section.
Solution: Starting with the definition of true strain as = ln(L/Lo) and assuming constant volume,
we have V = AoLo = AL
14
Therefore, L/Lo = Ao/A
A = D2 and Ao = Do
2
Ao/A = Do
2 /D2 = (Do/D)2
e = ln(Do/D)2 = 2 ln(Do/D)
3.16 Show that true strain = ln(1 + e).
Solution: Starting definitions: (1) = ln(L/Lo) and (2) e = (L - Lo)/Lo
Consider definition (2): e = L/Lo - Lo/Lo = L/Lo - 1
Rearranging, 1 + e = L/Lo
Substituting this into definition (1), e = ln(1 + e)
3.17 Based on results of a tensile test, the flow curve has parameters calculated as n = 0.40 and K =
551.6 MPa. Based on this information, calculate the (engineering) tensile strength for the metal.
Solution: Tensile strength occurs at maximum value of load. Necking begins immediately
thereafter. At necking, n = . Therefore, = 551.6(.4).4 = 382.3 MPa. This is a true stress.
TS is defined as an engineering stress. From Problem 3.15, we know that = 2 ln(Do/D).
Therefore,
0.4 = 2 ln(Do/D)
ln(Do/D) = .4/2 = 0.2
Do/D = exp.(.2) = 1.221
Area ratio = (Do/D)2 = (1.221)2 = 1.4918
The ratio between true stress and engineering stress would be the same ratio.
Therefore, TS = 1.4918(382.3) = 570.3 MPa
3.18 A copper wire of diameter 0.80 mm fails at an engineering stress = 248.2 MPa. Its ductility is
measured as 75% reduction of area. Determine the true stress and true strain at failure.
Solution: Area reduction AR = (Ao - Af)/Ao = 0.75
Ao - Af = 0.75 Ao
Ao - 0.75Ao = 0.25 Ao = Af
If engineering stress = 248.2 MPa, then true stress = 248.2/0.25 = 992.8 MPa
True strain = ln(Lf/Lo) = ln(Ao/Af) = ln(4) = 1.386. However, it should be noted that these values
are associated with the necked portion of the test specimen.
3.19 A steel tensile specimen with starting gage length = 2.0 in and cross-sectional area = 0.5 in2 reaches
a maximum load of 37,000 lb. Its elongation at this point is 24%. Determine the true stress and true
strain at this maximum load.
Solution: Elongation = (L - Lo)/Lo = 0.24
L - Lo = 0.24 Lo
L = 1.24 Lo
A = Ao/1.24 = 0.8065 Ao
True stress = 37,000/0.8065(0.5) = 91,754 lb/in2
True strain = ln(1.24) = 0.215
Compression
3.20 A metal alloy has been tested in a tensile test to determine the following flow curve parameters: K =
620.5 MPa and n = 0.26. The same metal is now tested in a compression test in which the starting
height of the specimen = 62.5 mm and its diameter = 25 mm. Assuming that the cross- section
increases uniformly, determine the load required to compress the specimen to a height of (a) 50 mm
and (b) 37.5 mm.
15
Solution: Starting volume of test specimen V = hDo
2/4 = 62.5(25)2/4 = 30679.6 mm3.
(a) At h = 50 mm, = ln(62.5/50) = ln(1.25) = 0.223
Yf = 620.5(.223).26 = 420.1 MPa
A = V/L = 30679.6/50 = 613.6 mm2
F = 420.1(613.6) = 257,770 N
(b) At h = 37.5 mm, = ln(62.5/37.5) = ln(1.667) = 0.511
Yf = 620.5(0.511).26 = 521.1 MPa
A = V/L = 30679.6 /37.5 = 818.1 mm2
F = 521.1(818.1) = 426,312 N
3.21 The flow curve parameters for a certain stainless steel are K = 1100 MPa and n = 0.35. A
cylindrical specimen of starting cross-section area = 1000 mm2 and height = 75 mm is compressed
to a height of 58 mm. Determine the force required to achieve this compression, assuming that the
cross-section increases uniformly.
Solution: For h = 58 mm, = ln(75/58) = ln(1.293) = 0.257
Yf = 1100(.257).35 = 683.7 MPa
Starting volume V = 75(1000) = 75,000 mm3
At h = 58 mm, A = V/L = 75,000/58 = 1293.1 mm2
F = 683.7(1293.1) = 884,095 N.
3.22 A steel test specimen (E = 30 x 106 lb/in2) in a compression test has a starting height = 2.0 in and
diameter = 1.5 in. The metal yields (0.2% offset) at a load = 140,000 lb. At a load of 260,000 lb, the
height has been reduced to 1.6 in. Determine: (a) yield strength Y, (b) flow curve parameters K and
n. Assume that the cross-sectional area increases uniformly during the test.
Solution: (a) Starting volume of test specimen V = hD2/4 = 2(1.5)2/4 = 3.534 in3.
Ao = Do/4 = (1.5)2/4 = 1.767 in2
Y = 140,000/1.767 = 79,224 lb/in2
(b) Elastic strain at Y = 79,224 lb/in2 is e = Y/E = 79,224/30,000,000 = 0.00264
Strain including offset = 0.00264 + 0.002 = 0.00464
Height h at strain = 0.00464 is h = 2.0(1 - 0.00464) = 1.9907 in.
Area A = 3.534/1.9907 = 1.775 in2.
True strain = 140,000/1.775 = 78,862 lb/in2.
At F = 260,000 lb, A = 3.534/1.6 = 2.209 in2.
True stress = 260,000/2.209 = 117,714 lb/in2.
True strain = ln(2.0/1.6) = 0.223
Given the two points: (1) = 78,862 lb/in2 at = 0.00464, and (2) = 117,714 lb/in2 at = 0.223.
117,714/78,862 = (0.223/0.00464)n
1.493 = (48.06)n
ln(1.493) = n ln(48.06)
0.4006 = 3.872 n n = 0.103
K = 117,714/(0.223).103 = 137,389 lb/in2.
The flow curve equation is: s = 137,389 e .103
Bending and Shear
16
3.23 A bend test is used for a certain hard material. If the transverse rupture strength of the material is
known to be 1000 MPa, what is the anticipated load at which the specimen is likely to fail, given that
its dimensions are: b = 15 mm, h = 10 mm, and L = 60 mm?
Solution: F = (TRS)(bh2)/1.5L = 1000(15 x 102)/(1.5 x 60) = 16,667 N.
3.24 A special ceramic specimen is tested in a bend test. Its cross-sectional dimensions are b = 0.50 in
and h = 0.25 in. The length of the specimen between supports = 2.0 in. Determine the transverse
rupture strength if failure occurs at a load = 1700 lb.
Solution: TRS = 1.5FL/bh2 = 1.5(1700)(2.0)/(0.5 x 0.252) = 163,200 lb/in2.
3.25 A piece of metal is deformed in shear to an angle of 42as shown in Figure P3.25. Determine the
shear strain for this situation.
Solution: = a/b = tan 42= 0.9004.
3.26 A torsion test specimen has a radius = 25 mm, wall thickness = 3 mm, and gage length = 50 mm. In
testing, a torque of 900 N-m results in an angular deflection = 0.3. Determine: (a) the shear stress,
(b) shear strain, and (c) shear modulus, assuming the specimen had not yet yielded.
Solution: (a) = T/(2R2t) = (900 x 1000)/(2(25)2(3)) = 76.39 MPa.
(b) = R/L, = .3(2/360) = 0.005236 rad.
= 25(0.005236)/50 = 0.002618
(c) = G, G = /= 76.39/0.002618 = 29,179 MPa.
3.27 In a torsion test, a torque of 5000 ft-lb is applied which causes an angular deflection = 1on a
thin-walled tubular specimen whose radius = 1.5 in, wall thickness = 0.10 in, and gage length = 2.0
in. Determine: (a) the shear stress, (b) shear strain, and (c) shear modulus, assuming the specimen
had not yet yielded.
Solution: (a) = T/(2R2t) = (5000 x 12)/(2(1.5)2(0.1)) = 42,441 lb/in2.
(b) = R/L, = 1(2/360) = 0.01745 rad., = 1.5(0.01745)/2.0 = 0.01309
(c) = G, G = /= 42,441/0.01309 = 3.24 x 106 lb/in2.
3.28 In Problem 3.26, failure of the specimen occurs at a torque = 1200 N-m and a corresponding
angular deflection = 10. What is the shear strength of the metal?
Solution: S = (1200 x 1000)/(2(25)2(3)) = 101.86 MPa.
3.29 In Problem 3.27, the specimen fails at a torque = 8000 ft-lb and an angular deflection = 23.
Calculate the shear strength of the metal.
Solution: S = (8000 x 12)/(2(1.5)2(0.1)) = 67,906 lb/in2.
Hardness
3.30 In a Brinell hardness test, a 1500 kg load is pressed into a specimen using a 10 mm diameter
hardened steel ball. The resulting indentation has a diameter = 3.2 mm. Determine the BHN for the
metal.
Solution: BHN = 2(1500)/(10(10 - (102 - 3.22).5) = 3000/(10x 0.5258) = 182 BHN
3.31 One of the inspectors in the quality control department has frequently used the Brinell and Rockwell
hardness tests, for which equipment is available in the company. He claims that all hardness tests
are based on the same principle as the Brinell test, which is that hardness is always measured as the
17
applied load divided by the area of the impressions made by an indentor. (a) Is he correct? (b) If
not, what are some of the other principles involved in hardness testing, and what are the associated
tests?
Solution: (a) No, the claim is not correct. Not all hardness tests are based on the applied load
divided by area, but many of them are.
(b) Some of the other hardness tests and operating principles include: (1) Rockwell hardness test,
which measures the depth of indentation of a cone resulting from an applied load; (2) Scleroscope,
which measures the rebound height of a hammer dropped from a certain distance against a surface
specimen; and (3) Durometer, which measures elastic deformation by pressing an indentor into the
surface of rubber and similar soft materials.
3.32 Suppose in Problem 3.30 that the specimen is steel. Based on the BHN determined in that problem,
estimate the tensile strength of the steel.
Solution: The estimating formula is: TS = 500(BHN). For a tested hardness of BHN = 182, TS =
500(182) = 91,000 lb/in2.
3.33 A batch of annealed steel has just been received from the vendor. It is supposed to have a tensile
strength in the range 60,000 to 70,000 lb/in2. A Brinell hardness test in the receiving department
yields a value of BHN = 118. (a) Does the steel meet the specification on tensile strength? (b)
Estimate the yield strength of the material.
Solution: (a) TS = 500(BHN) = 500(118) = 59,000 lb/in2. This lies outside the specified range of
60,000 to 70,000 lb/in2. However, from a legal standpoint, it is unlikely that the batch can be rejected
on the basis of its measured BHN without using an actual tensile test to measure TS. The above
formula for converting from BHN to TS is only an approximating equation.
(b) Based on Table 3.2 in the text (page 47), the ratio of Y to TS for low carbon steel =
25,000/45,000 = 0.555. Using this ratio, we can estimate the yield strength to be Y = 0.555(59,000)
= 32,700 lb/in2.
Viscosity of Fluids
3.34 Two flat plates, separated by a space of 4 mm, are moving relative to each other at a velocity of 5
m/sec. The space between them is occupied by a fluid of unknown viscosity. The motion of the
plates is resisted by a shear stress of 10 Pa due to the viscosity of the fluid. Assuming that the
velocity gradient of the fluid is constant, determine the coefficient of viscosity of the fluid.
Solution: Shear rate = (5 m/s x 1000 mm/m)/(4 mm) = 1250 s-1
= (10N/m2)/(1250 s-1) = 0.008 N-s/m2.
3.35 Two parallel surfaces, separated by a space of 0.5 in that is occupied by a fluid, are moving relative
to each other at a velocity of 25 in/sec. The motion is resisted by a shear stress of 0.3 lb/in2 due to
the viscosity of the fluid. If the velocity gradient in the space between the surfaces is constant,
determine the viscosity of the fluid.
Solution: Shear rate = (25 in/sec)/(0.5 in) = 50 sec-1
= (0.3 lb/in2)/(50 sec-1) = 0.0006 lb-sec/in2.
3.36 A 125.0 mm diameter shaft rotates inside a stationary bushing whose inside diameter = 125.6 mm
and length = 50.0 mm. In the clearance between the shaft and the bushing is contained a lubricating
oil whose viscosity = 0.14 Pas. The shaft rotates at a velocity of 400 rev/min; this speed and the
action of the oil are sufficient to keep the shaft centered inside the bushing. Determine the
magnitude of the torque due to viscosity that acts to resist the rotation of the shaft.
18
Solution: Bushing internal bearing area A = (125.6)2 x 50/4 = 19729.6 mm2 = 19729.2(10-6) m2
d = (125.6 - 125)/2 = 0.3 mm
v = (125mm/rev)(400 rev/min)(1 min/60 sec) = 2618.0 mm/s
Shear rate = 2618/0.3 = 8726.6 s-1
= (0.14)(8726.6) = 1221.7 Pa = 1221.7 N/mm2
Force on surface between shaft and bushing = (1221.7 N/mm2)(19729.2(10-6)) = 24.1 N
Torque T = 24.1 N x 125/2 mm = 1506.4 N-mm = 1.506 N-m
19
4 PHYSICAL PROPERTIES OF MATERIALS
Review Questions
4.1 Define the property density of a material.
Answer. Density is the weight per unit volume.
4.2 What is the difference in melting characteristics between a pure metal element and an alloy metal?
Answer. A pure metal element melts at one temperature (the melting point), while an alloy begins
melting at a certain temperature called the solidus and finally completes the transformation to the
molten state at a higher temperature called the liquidus. Between the solidus and liquidus, the metal
is a mixture of solid and liquid.
4.3 Describe the melting characteristics of a noncrystalline material such as glass.
Answer. In the heating of a noncrystalline material such as glass, the material begins to soften as
temperature increases, finally converting to a liquid at a temperature defined for these materials as
the melting point.
4.4 Define the specific heat property of a material.
Answer. Specific heat is defined as the quantity of heat required to raise the temperature of a unit
mass of the material by one degree.
4.5 What is the thermal conductivity of a material?
Answer. Thermal conductivity is the capacity of a material to transfer heat energy through itself by
thermal movement only (no mass transfer).
4.6 Define thermal diffusivity.
Answer. Thermal diffusivity is the thermal conductivity divided by the volumetric specific heat.
4.7 What are the important variables that affect mass diffusion?
Answer. According to Fick's first law, mass diffusion depends on: diffusion coefficient which rises
rapidly with temperature (so temperature could be listed as an important variable), concentration
gradient, contact area, and time.
4.8 Define the resistivity of a material.
Answer. Resistivity is the material's capacity to resist the flow of an electric current.
4.9 Why are metals better conductors of electricity than ceramics and polymers?
Answer. Metals are better conductors because of metallic bonding, which permits electrons to
move easily within the metal. Ceramics and polymers have covalent and ionic bonding, in which the
electrons are tightly bound to particular molecules.
4.10 What is the dielectric strength of a material?
Answer. The dielectric strength is defined as the electrical potential required to break down the
insulator per unit thickness.
4.11 What is an electrolyte?
20
Answer. An electrolyte is an ionized solution capable of conducting electric current by movement
of the ions.
Multiple Choice Quiz
There are a total of 12 correct answers in the following multiple choice questions (some questions have
multiple answers that are correct). To attain a perfect score on the quiz, all correct answers must be
given, since each correct answer is worth 1 point. For each question, each omitted answer or wrong
answer reduces the score by 1 point, and each additional answer beyond the number of answers required
reduces the score by 1 point. Percentage score on the quiz is based on the total number of correct
answers.
4.1 Which one of the following metals has the lowest density? (a) aluminum, (b) copper, (c) magnesium,
or (d) tin.
Answer. (c)
4.2 Polymers typically exhibit greater thermal expansion properties than metals: (a) true, or (b) false.
Answer. (a)
4.3 In the heating of most metal alloys, melting begins at a certain temperature and concludes at a
higher temperature. In these cases, which of the following temperatures marks the beginning of
melting? (a) liquidus, of (b) solidus.
Answer. (b)
4.4 Which of the following materials has the highest specific heat? (a) aluminum, (b) concrete, (c)
polyethylene, or (d) water.
Answer. (d)
4.5 Copper is generally considered easy to weld, because of its high thermal conductivity: (a) true, or
(b) false.
Answer. (b) The high thermal conductivity of copper makes it difficult to weld because the heat
flows away from the joint rather than being concentrated to permit melting of the metal.
4.6 The mass diffusion rate dm/dt across a boundary between two different metals is a function of
which of the following variables (more than one): (a) concentration gradient dc/dx, (b) contact area,
(c) density, (d) melting point, (e) temperature, and (f) time.
Answer. (a), (b), (e), and (f). This is perhaps a trick question. Choices (a) and (b) are included in
Eq. (4.5). Temperature (e) has a strong influence on the diffusion coefficient. Time (f) figures into
the process because it affects the concentration gradient; as time elapses, the concentration
gradient is reduced so that the rate of diffusion is reduced.
4.7 Which of the following pure metals is the best conductor of electricity? (a) aluminum, (b) copper,
(c) gold, or (d) silver.
Answer. (d)
4.8 A superconductor is characterized by which of the following (choose one best answer): (a) very
low resistivity, (b) zero conductivity, or (c) resistivity properties between those of conductors and
semiconductors?
Answer. (b)
21
4.9 In an electrolytic cell, the anode is the electrode which is (a) positive, or (b) negative.
Answer. (a)
Problems
4.1 The starting diameter of a shaft is 25.00 mm. This shaft is to be inserted into a hole in an expansion
fit assembly operation. To be readily inserted, the shaft must be reduced in diameter by cooling.
Determine the temperature to which the shaft must be reduced from room temperature (20C) in
order to reduce its diameter to 24.98 mm. Refer to Table 4.1.
Solution: For steel, = 12(10-6) mm/mm/C according to Table 4.1.
Revise Eq. (4.1) to D2 - D1 = D1 (T2 - T2).
24.98 - 25.00 = 12(10-6)(25.00)(T2 - 20)
-0.02 = 300(10-6)(T2 - 20)
-0.02 = 0.0003(T2 - 20) = 0.0003T2 - 0.006
-.02 + 0.006 = 0.0003T2
-0.014 = 0.0003T2 T2 = -46.67°C
4.2 Aluminum has a density of 2.70 g/cm3 at room temperature (20C). Determine its density at 650C,
using data in Table 4.1 as a reference.
Solution: Assume a 1 cm3 cube, 1 cm on each side.
From Table 4.1, = 24(10-6) mm/mm/C
L2 - L1 = L1 (T2 - T2).
L2 = 1.0 + 24(10-6)(1.0)(650 - 20) = 1.01512 cm
(L2 )3 = (1.01512)3 = 1.04605 cm3
Assume weight remains the same; thus at 650C = 2.70/1.04605 = 2.581 g/cm3
4.3 With reference to Table 4.1, determine the increase in length of a steel bar whose length = 10.0 in,
if the bar is heated from room temperature (70F) to 500F.
Solution: Increase = (6.7 x 10-6 in/in/F)(10.0 in)(500F - 70F) = 0.0288 in.
4.4 With reference to Table 4.2, determine the quantity of heat required to increase the temperature of
an aluminum block that is 10 cm x 10 cm x 10 cm from room temperature (21C) to 300C.
Solution. Heat = (0.21 cal/g-C)(103 cm3)(2.70 g/cm3)(300C - 21C) = 158,193 cal.
Conversion: 1.0 cal = 4.184J, so heat = 662,196 J.
4.5 What is the resistance R of a length of copper wire whose length = 10 m and whose diameter =
0.10 mm? Use Table 4.3 as a reference.
Solution: R = rL/A, A = (0.1)2/4 = 0.007854 mm2 = 0.007854(10-6) m2
From Table 4.3, r = 1.7 x 10-8 -m2/m
R = (1.7 x 10-8 -m2/m)(10 m)/( 0.007854(10-6) m2) = 2164.5(10-2) = 21.65 W
22
5 DIMENSIONS, TOLERANCES, AND SURFACES
Review Questions
5.1 What is a tolerance?
Answer. A tolerance is defined as the total amount by which a specified dimension is permitted to
vary.
5.2 What are some of the reasons why surfaces are important?
Answer. The reasons why surfaces are important include: aesthetics, safety, friction and wear,
effect of surface on mechanical and physical properties, mating of components in assembly, and
thermal electrical contacts.
5.3 Define nominal surface.
Answer. The nominal surface is the ideal part surface represented on an engineering drawing. It is
assumed perfectly smooth; perfectly flat if referring to a planar surface; perfectly round if referring
to a round surface, etc.
5.4 Define surface texture.
Answer. Surface texture is the random and repetitive deviations from the nominal surface, including
roughness, waviness, lay, and flaws.
5.5 How is surface texture distinguished from surface integrity?
Answer. Surface texture refers only to the surface geometry; surface integrity includes not only
surface but the altered layers beneath the surface.
5.6 Within the scope of surface texture, how is roughness distinguished from waviness?
Answer. Roughness consists of the finely-spaced deviations from the nominal surface, while
waviness refers to the deviations of larger spacing. Roughness deviations lie within waviness
deviations.
5.7 Surface roughness is a measurable aspect of surface texture; what does surface roughness mean?
Answer. Surface roughness is defined as the average value of the vertical deviations from the
nominal surface over a specified surface length.
5.8 What is the difference between AA and RMS in surface roughness measurement?
Answer. AA and RMS are alternative methods by which the average roughness value is
computed; see Eqs. (5.1) and (5.3) in the text.
5.9 Indicate some of the limitations of using surface roughness as a measure of surface texture.
Answer. Surface roughness measurement provides only a single value of surface texture. Among
its limitations are: (1) it varies depending on direction; (2) it does not indicate lay; (3) its value
depends on the roughness width cutoff L used to measure the average.
5.10 Identify some of the changes and injuries that can occur at or immediately below the surface of a
metal.
23
Answer. The changes and injuries include: cracks, craters, variations in hardness near the surface,
metallurgical changes resulting from heat, residual stresses, intergranular attack, etc. (see Table
5.1).
5.11 What causes the various types of changes that occur in the altered layer just beneath the surface?
Answer. Energy input resulting from the manufacturing process used to generate the surface. The
energy forms can be any of several types, including mechanical, thermal, chemical, and electrical.
5.12 Name some manufacturing processes that produce very poor surface finishes.
Answer. Processes that produce poor surfaces include: sand casting, hot rolling, sawing, and
thermal cutting (e.g., flame cutting).
5.13 Name some manufacturing processes that produce very good or excellent surface finishes.
Answer. Processes that produced very good and excellent surfaces include: honing, lapping,
polishing, and superfinishing.
Multiple Choice Quiz
There are a total of 19 correct answers in the following multiple choice questions (some questions have
multiple answers that are correct). To attain a perfect score on the quiz, all correct answers must be
given, since each correct answer is worth 1 point. For each question, each omitted answer or wrong
answer reduces the score by 1 point, and each additional answer beyond the number of answers required
reduces the score by 1 point. Percentage score on the quiz is based on the total number of correct
answers.
5.1 A tolerance is which one of the following? (a) clearance between a shaft and a mating hole, (b)
measurement error, (c) total permissible variation from a specified dimension, or (d) variation in
manufacturing.
Answer. (c)
5.2 Which of the following two geometric terms have the same meaning? (a) circularity, (b)
concentricity, (c) cylindricity, and (d) roundness.
Answer. (a) and (d).
5.3 Surface texture includes which of the following characteristics of a surface (may be more than
one)? (a) deviations from the nominal surface, (b) feed marks of the tool that produced the surface,
(c) hardness variations, (d) oil films, and (e) surface cracks.
Answer. (a), (b), and (e).
5.4 Which averaging method generally yields the higher value of surface roughness, (a) AA or (b)
RMS?
Answer. (b)
5.5 Surface texture is included within the scope of surface integrity: (a) true or (b) false.
Answer. (a)
5.6 Thermal energy is normally associated with which of the following changes in the altered layer? (a)
cracks, (b) hardness variations, (c) heat affected zone, (d) plastic deformation, (e) recrystallization,
or (f) voids.
Answer. (b), (c), and (e).
24
5.7 A better finish (lower roughness value) will tend to have which of the following effects on fatigue
strength of a metal surface? (a) increase, (b) decrease, or (c) no effect.
Answer. (b)
5.8 Which of the following are included within the scope of surface integrity? (a) chemical absorption,
(b) microstructure near the surface, (c) microcracks beneath the surface, (d) substrate
microstructure, (e) surface roughness, or (f) variation in tensile strength near the surface.
Answer. (a), (b), (c), (e), and (f)
5.9 Which one of the following manufacturing processes will likely result in the best surface finish? (a)
arc welding, (b) grinding, (c) machining, (d) sand casting, or (e) sawing.
Answer. (b)
5.10 Which one of the following manufacturing processes will likely result in the worst surface finish? (a)
cold rolling, (b) grinding, (c) machining, (d) sand casting, or (e) sawing.
Answer. (d). Also, sawing (e) will yield a poor finish. Accept either answer.
25
6 METALS
Review Questions
6.1 What are some of the general properties that distinguish metals from ceramics and polymers?
Answer. Metallic properties include: high strength and stiffness, good electrical and thermal
conductivity, and higher density than ceramics or polymers.
6.2 What are the two major groups of metals? Define them.
Answer. Ferrous metals, which are based on iron; and nonferrous, which includes all others.
6.3 What is the definition of an alloy?
Answer. An alloy is a metal comprised of two or more elements, at least one of which is metallic.
6.4 What is a solid solution in the context of alloys?
Answer. A solid solution is an alloy in which one of the metallic elements is dissolved in another to
form a single phase.
6.5 Distinguish between a substitutional solid solution and an interstitial solid solution.
Answer. A substitutional solid solution is where the atoms of dissolved element replace atoms of
the solution element in the lattice structure of the metal. An interstitial solid solution is where the
dissolved atoms are small and fit into the vacant spaces (the interstices) in the lattice structure of
the solvent metal.
6.6 What is an intermediate phase in the context of alloys?
Answer. An intermediate phase is an alloy formed when the solubility limit of the base metal in the
mixture is exceeded and a new phase, such as a metallic compound (e.g., Fe3C) or intermetallic
compound (e.g., Mg2Pb) is formed.
6.7 The copper-nickel system is a simple alloy system, as indicated by its phase diagram. Why is it so
simple?
Answer. The Cu-Ni alloy system is simple because it is a solid solution alloy throughout its entire
composition range.
6.8 What is the range of carbon percentages which defines an iron-carbon alloy as a steel?
Answer. The carbon content ranges from 0.02% to 2.11%.
6.9 What is the range of carbon percentages which defines an iron-carbon alloy as cast iron?
Answer. The carbon content ranges from 2.11% to about 5%.
6.10 Identify some of the common alloying elements other than carbon in low alloy steels.
Answer. The common alloying elements in low alloy steel are Cr, Mn, Mo, Ni, and V; we should
also mention the most important, which is C.
6.11 What are some of the mechanisms by which the alloying elements other than carbon strengthen
steel.
Answer. All of the alloying elements other than C strengthen the steel by solid solution alloying. Cr,
Mn, Mo, and Ni increase hardenability during heat treatment. Cr and Mo improve hot hardness.
26
Several of the alloying elements (Cr, Mo, V) form hard carbides with C, which increases wear
resistance. Vanadium inhibits grain growth during heat treatment which improves strength and
toughness.
6.12 What is the mechanism by which carbon strengthens steel in the absence of heat treatment?
Answer. If no heat treatment carbon strengthens by creating a two-phase structure in the steel.
6.13 What is the predominant alloying element in all of the stainless steels?
Answer. Chromium.
6.14 Why is austenitic stainless steel called by that name?
Answer. It is called austenitic because this alloy exists in its austenitic phase at room temperature.
The reason is that nickel has the effect of enlarging the austenitic temperature range to include
room temperature.
6.15 Besides high carbon content, what other alloying element is characteristic of the cast irons?
Answer. Silicon.
6.16 Identify some of the properties for which aluminum is noted?
Answer. Aluminum is noted for its low density, high electrical and thermal conductivity, formability,
good corrosion resistance due to the formation of a tough oxide film on its surface, and ability to be
alloyed and strengthened to achieve good strength-to-weight ratios.
6.17 What are some of the noteworthy properties of magnesium?
Answer. Magnesium is noted for its very low density (lightest of the structural metals), propensity
to oxidize (which can cause problems in processing), and low strength; however, it can be alloyed
and strengthened by methods similar to those used for aluminum alloys to achieve respectable
strength-to-weight ratios.
6.18 What is the most important engineering property of copper which determines most of its
applications?
Answer. Its high electrical conductivity.
6.19 What elements are traditionally alloyed with copper to form (a) bronze and (b) brass?
Answer. (a) tin, (b) zinc.
6.20 What are some of the important applications of nickel?
Answer. The important applications are: (1) as an alloying ingredient in steel, e.g., stainless steel;
(2) for plating of steel to resist corrosion; and (3) to form nickel-based alloys noted for
high-temperature performance and corrosion resistance.
6.21 What are the noteworthy properties of titanium?
Answer. Titanium is noted for its high strength-to-weight ratio, corrosion resistance (due to the
formation of a thin but tough oxide film), and high temperature strength.
6.22 Identify some of the important applications of zinc.
Answer. The important applications are: (1) die castings - zinc is an easy metal to cast; (2) as a
coating in galvanized steel; (3) as an alloying element with copper to form brass.
6.23 What important alloy is formed from lead and tin?
27
Answer. Solder.
6.24 (a) Name the important refractory metals. (b) What does the term refractory mean?
Answer. (a) The refractory metals include columbium (Cb), molybdenum (Mo), tantalum (Ta), and
tungsten (W). Mo and W are the most important. (b) Refractory means the capability to withstand
high temperature service.
6.25 (a) Name the four principal noble metals. (b) Why are they called noble metals?
Answer. (a) The principal noble metals are copper, gold, platinum, and silver. (b) Nobel metals are
so-named because they are chemically inactive.
6.26 The superalloys divide into three basic groups, according to the base metal used in the alloy. Name
the three groups.
Answer. The three groups are: (1) iron-based alloys, (2) nickel-based alloys, and (3) cobalt-based
alloys.
6.27 What is so special about the superalloys? What distinguishes them from other alloys?
Answer. The superalloys are generally distinguished by their strength and resistance to corrosion
and oxidation at elevated temperatures.
6.28 What are the three basic methods by which metals can be strengthened?
Answer. The three basic methods are: (1) alloying to form solid solutions and two-phase structures
which are stronger than the elemental metals; (2) cold working, in which the strain-hardened metal
is stronger and harder than the unstrained metal; and (3) heat treatment - most of the commercial
heat treatments are designed to increase the strength of the metal.
Multiple Choice Quiz
There are a total of 23 correct answers in the following multiple choice questions (some questions have
multiple answers that are correct). To attain a perfect score on the quiz, all correct answers must be
given, since each correct answer is worth 1 point. For each question, each omitted answer or wrong
answer reduces the score by 1 point, and each additional answer beyond the number of answers required
reduces the score by 1 point. Percentage score on the quiz is based on the total number of correct
answers.
6.1 Which of the following properties or characteristics are inconsistent with the metals (more than
one)? (a) good thermal conductivity, (b) high strength, (c) high electrical resistivity, (d) high
stiffness, or (e) ionic bonding.
Answer. (c) and (e).
6.2 Which of the metallic elements is the most abundant on the earth? (a) aluminum, (b) copper, (c)
iron, (d) magnesium, or (e) silicon.
Answer. (a)
6.3 The predominant phase in the iron-carbon alloy system for a composition with 99% Fe at room
temperature is which of the following? (a) austenite, (b) cementite, (c) delta, (d) ferrite, or (e)
gamma.
Answer. (d)
28
6.4 A steel with 1.0% carbon is known as which of the following: (a) eutectoid, (b) hypoeutectoid, (c)
hypereutectoid, or (d) wrought iron.
Answer. (c)
6.5 The strength and hardness of steel increases as carbon content increases: (a) true of (b) false.
Answer. (a)
6.6 Plain carbon steels are designated in the AISI code system by which of the following? (a) 01XX,
(b) 10XX, (c) 11XX, (d) 12XX, or (e) 30XX.
Answer. (b)
6.7 Which of the following elements is the most important alloying ingredient in steel? (a) carbon, (b)
chromium, (c) nickel, (d) molybdenum, or (e) vanadium.
Answer. (a)
6.8 Which of the following is not a common alloying ingredient in steel? (a) chromium, (b) manganese,
(c) nickel, (d) vanadium, (e) zinc.
Answer. (e)
6.9 Solid solution alloying is the principal strengthening mechanism in high-strength low-alloy (HSLA)
steels: (a) true or (b) false.
Answer. (a)
6.10 Which of the following alloying elements are most commonly associated with stainless steel (name
two)? (a) chromium, (b) manganese, (c) molybdenum, (d) nickel, and (e) tungsten.
Answer. (a) and (d).
6.11 Which of the following is the most important cast iron commercially? (a) ductile cast iron, (b) gray
cast iron, (c) malleable iron, or (d) white cast iron.
Answer. (b)
6.12 Which of the following metals has the lowest density? (a) aluminum, (b) magnesium, (c) tin, or (d)
titanium.
Answer. (b)
6.13 Which of the following metals has the highest density? (a) gold, (b) lead, (c) platinum, (d) silver, or
(e) tungsten.
Answer. (c)
6.14 From which of the following ores is aluminum derived? (a) alumina, (b) bauxite, (c) cementite, (d)
hematite, or (e) scheelite.
Answer. (b)
6.15 Which of the following metals possess good electrical conductivity (more than one)? (a) aluminum,
(b) copper, (c) gold, (d) silver, or (e) tungsten.
Answer. (a), (b), (c), and (d).
6.16 Traditional brass is an alloy of which of the following metallic elements? (a) aluminum, (b) copper,
(c) gold, (d) tin, (e) zinc.
29
Answer. (b) and (e).
6.17 Which of the following has the lowest melting point? (a) aluminum, (b) lead, (c) magnesium, (d) tin,
or (e) zinc.
Answer. (d)
Problems
6.1 For the copper-nickel phase diagram in Figure 6.2, find the compositions of the liquid and solid
phases for a nominal composition of 70% Ni and 30% Cu at 1371C (2500F).
Solution: From Fig 6.2, the compositions are observed as follows:
Liquid phase composition = 65% Ni - 35% Cu.
Solid phase composition = 83% Ni - 17% Cu.
6.2 For the preceding problem, use the inverse lever rule to determine the proportions of liquid and solid
phases present in the alloy.
Solution: From Fig 6.2, measured values of CL and CS are: CL = 5 mm, CS = 12 mm.
Liquid phase proportion = 12/(12 + 5) = 12/17 = 0.71
Solid phase proportion = 5/17 = 0.29
6.3 For the lead-tin phase diagram of Figure 6.3, is it possible to design a solder (lead-tin alloy) with a
melting point of 260C (500F). If so, what would be its nominal composition?
Solution: It is possible to obtain such a solder, if the lead-tin proportion is 67%-33%.
6.4 Using the lead-tin phase diagram in Figure 6.3, determine the liquid and solid phase compositions for
a nominal composition of 40% Sn and 60% Pb at 204C (400F).
Solution: From Fig 6.3, the compositions are observed as follows:
Liquid phase composition = 56% Sn - 44% Pb.
phase composition = 18% Sn - 82% Pb.
6.5 For the preceding problem, use the inverse lever rule to determine the proportions of liquid and solid
phases present in the alloy.
Solution: From Fig 6.3, measured values of CL and CS are: CL = 10.5 mm, CS = 15 mm.
Liquid phase proportion = 15/(15 + 10.5) = 15/25.5 = 0.59
phase proportion = 10.5/25.5 = 0.41
6.6 Using the lead-tin phase diagram in Figure 6.3, determine the liquid and solid phase compositions for
a nominal composition of 90% Sn and 10% Pb at 204C (400F).
Solution: From Fig 6.3, the compositions are observed as follows:
Liquid phase composition = 78% Sn - 22% Pb.
phase composition = 98% Sn - 2% Pb.
6.7 For the preceding problem, use the inverse lever rule to determine the proportions of liquid and solid
phases present in the alloy.
Solution: From Fig 6.3, measured values of CL and CS are: CL = 7.8 mm, CS = 4.2 mm.
Liquid phase proportion = 4.2/(13) = 0.32
phase proportion = 7.8/13 = 0.68
30
6.8 In the iron-iron carbide phase diagram of Figure 6.4, identify the phase or phases present at the
following temperatures and nominal compositions: (a) 650C (1200F) and 2% Fe3C, (b) 760C
(1400F) and 2% Fe3C, and (c) 1095C (2000F) and 1% Fe3C.
Solution: (a) Alpha + iron carbide, (b) gamma + iron carbide, and (c) gamma.
31
7 CERAMICS
Review Questions
7.1 What is a ceramic is.
Answer. A ceramic is an inorganic, nonmetallic compound, usually formed into useful products by a
heating process.
7.2 What are the four most common elements in the earth's crust?
Answer. Oxygen, silicon, aluminum, and iron.
7.3 What is the difference between the traditional ceramics and the new ceramics?
Answer. Traditional ceramics are based primarily on clay products (e.g., pottery, bricks) while new
ceramics are more recently developed ceramics which are generally simpler in chemical
composition (e.g., oxides, carbides).
7.4 What is the feature that distinguishes glass from the traditional and new ceramics?
Answer. Glass is noncrystalline (amorphous), while most other ceramics assume a crystalline
structure.
7.5 Why are graphite and diamond not classified as ceramics?
Answer. Because they are not compounds; they are alternative forms of the element carbon.
7.6 What are the general mechanical properties of ceramic materials?
Answer. Usually high hardness, brittle, no ductility.
7.7 What are the general physical properties of ceramic materials?
Answer. Usually electrical and thermal insulators, medium density (typically below the density of
metals), high melting temperatures, thermal expansion usually less than metals.
7.8 What type of atomic bonding characterizes the ceramics?
Answer. Covalent and ionic bonding.
7.9 What do bauxite and corundum have in common?
Answer. They are both minerals of alumina.
7.10 What is clay, used in making ceramic products?
Answer. Clay most commonly consists of hydrous aluminum silicate, the usually kaolinite
(Al2(Si2O5)(OH)4).
7.11 What is glazing, as applied to ceramics?
Answer. Glazing involves the application of a surface coating of oxides such as alumina and silica,
usually to a porous ceramic product such as earthenware, to make the product more impervious to
moisture and more attractive.
7.12 What does the term refractory mean?
Answer. Refractories are heat resistant ceramic materials. The term is sometimes also applied to
metals that are heat resistant.
32
7.13 What are some of the principal applications of the cemented carbides, such as WC-Co?
Answer. Important applications of WC-Co include: cutting tool inserts, drawing dies, rock drilling
bits, dies for powder metallurgy, and other applications where hardness is a critical factor.
7.14 What is one of the important applications of titanium nitride, as mentioned in the text?
Answer. As a thin coating on cutting tools to prolong tool life.
7.15 What elements comprise the ceramic material Sialon?
Answer. Silicon, aluminum, oxygen, and nitrogen.
7.16 Define glass.
Answer. Glass is an inorganic, nonmetallic material which cools to a rigid solid without
crystallization.
7.17 What is the primary mineral in glass products?
Answer. Silica, or silicon dioxide (SiO2).
7.18 What are some of the functions of the ingredients that are added to glass in addition to silica.
Answer. The functions of the additional ingredients include: (1) acting as flux (promoting fusion)
during heating; (2) increasing fluidity in the molten glass during processing; (3) retarding
devitrification - the tendency to crystallize from the glassy state; (4) reducing thermal expansion in
the final product; (5) increasing the chemical resistance against attack by acids, basic substances, or
water; (6) adding color to the glass; and (7) altering the index of refraction for optics applications
(e.g., lenses).
7.19 What does the term devitrification mean?
Answer. Devitrification is the transformation from the glassy state into a polycrystalline state.
7.20 What is graphite?
Answer. Graphite is carbon in the form of hexagonal crystalline layers, in which covalent bonding
exists between atoms in the layers, and the (parallel) layers are bonded by van der Waals forces,
thus leading to highly anisotropic properties.
Multiple Choice Quiz
There are a total of 18 correct answers in the following multiple choice questions (some questions have
multiple answers that are correct). To attain a perfect score on the quiz, all correct answers must be
given, since each correct answer is worth 1 point. For each question, each omitted answer or wrong
answer reduces the score by 1 point, and each additional answer beyond the number of answers required
reduces the score by 1 point. Percentage score on the quiz is based on the total number of correct
answers.
7.1 Which one of the following is the most common element in the earth's crust? (a) aluminum, (b)
calcium, (c) iron, (d) oxygen, or (e) silicon.
Answer. (d)
7.2 Glass products are based primarily on which one of the following minerals? (a) alumina, (b)
corundum, (c) feldspar, (d) kaolinite, or (e) silica.
Answer. (e)
33
7.3 Which of the following contains significant amounts of aluminum oxide (more than one)? (a)
alumina, (b) bauxite, (c) corundum, (d) quartz, or (e) sandstone.
Answer. (a), (b), and (c).
7.4 Which of the following ceramics are commonly used as abrasives in grinding wheels (two best
answers)? (a) aluminum oxide, (b) calcium oxide, (c) carbon monoxide, (d) silicon carbide, or (e)
silicon dioxide.
Answer. (a) and (d)
7.5 Which one of the following is generally the most porous of the clay-based pottery ware? (a) china,
(b) earthenware, (c) porcelain, or (d) stoneware.
Answer. (b)
7.6 Which of the following is fired at the highest temperatures? (a) china, (b) earthenware, (c)
porcelain, or (d) stoneware.
Answer. (c)
7.7 Which one of the following comes closest to expressing the chemical composition of clay? (a)
Al2O3, (b) Al2(Si2O5)(OH)4, (c) 3AL2O3-2SiO2, (d) MgO, or (e) SiO2.
Answer. (b)
7.8 Glass ceramics are polycrystalline ceramic structures that have been transformed into the glassy
state: (a) true, or (b) false.
Answer. (b) It's reversed in the statement. Glass ceramics are glasses that have been transformed
into a mostly crystalline form through heat treatment.
7.9 Which one of the following materials is closest to diamond in hardness? (a) aluminum oxide, (b)
carbon dioxide, (c) cubic boron nitride, (d) silicon dioxide, or (e) tungsten carbide.
Answer. (c)
7.10 Which of the following best characterizes the structure of glass-ceramics? (a) 95% polycrystalline,
(b) 95% vitreous, or (b) 50% polycrystalline.
Answer. (a)
7.11 Properties and characteristics of the glass-ceramics include which of the following (may be more
than one)? (a) efficiency in processing, (b) electrical conductor, (c) high thermal expansion, or (d)
strong, relative to other ceramics.
Answer. (a) and (d).
7.12 Diamond is the hardest material known: (a) true, or (b) false.
Answer. (a)
7.13 The specific gravity of graphite is closest to which one of the following: (a) 1.0 (b) 2.0, (c) 4.0, (d)
8.0, or (e) 16.0.
Answer. (b)
7.14 Synthetic diamonds date to: (a) ancient times, (b) 1800s, (c) 1950s, or (d) 1980.
Answer. (c)
34
8 POLYMERS
Review Questions
8.1 What is a polymer?
Answer. A polymer is a compound that consists of long-chain molecules. The molecules consist of
repeating units, called mers, connected end to end.
8.2 What are the three basic categories of polymers?
Answer. The categories are: (1) thermoplastics, (2) thermosetting polymers, and (3) elastomers.
8.3 How do the properties of polymers compare with those of metals?
Answer. In general, polymers have lower strength, hardness, stiffness, density, and temperature
resistance compared to metals. In addition, polymers are low in electrical and thermal conductivity.
8.4 What are the two methods by which polymerization occurs? Briefly describe the two methods.
Answer. The two types of polymerization are: (1) addition or chain polymerization and (2) step
polymerization, also known as condensation polymerization. See Article 10.1.1 for descriptions.
8.5 What does the degree of polymerization indicate?
Answer. The degree of polymerization indicates the average number of mers or repeating units in
the polymer molecule.
8.6 Define the term tacticity as it applies to polymers.
Answer. Tacticity refers to the way the atoms or atom groups replacing H atoms in the molecule
are arranged.
8.7 What is cross-linking in a polymer and what is its significance?
Answer. Cross-linking is the formation of connections between the long-chain molecules in a
polymer. It causes the polymer structure to be permanently altered. If the amount of cross-linking
is low, the polymer is transformed into an elastomer; if cross-linking is significant, the polymer is
transformed into a thermosetting polymer.
8.8 What is a copolymer?
Answer. A copolymer is a polymer made up of two different types of mers, such as ethylene and
propylene.
8.9 The arrangement of repeating units in a copolymer can vary. What are some of the possible
arrangements?
Answer. There are four possible arrangements of the mers along the chain: (1) alternating, (2)
random, (3) block, and (4) graft. See Article 10.1.4 for descriptions.
8.10 What is a terpolymer?
Answer. A terpolymer is a polymer with three different mer types. An example is ABS
(acrylonitrile-butadiene-styrene) plastic.
8.11 How are a polymer's properties affected when it takes on a crystalline structure?
35
Answer. Density, stiffness, and melting temperature increase.
8.12 Does any polymer ever become 100% crystalline?
Answer. No.
8.13 What are some of the factors that influence a polymer's tendency to crystallize?
Answer. Factors are: (1) only linear polymers can form crystals; (2) copolymers do not form
crystals; (3) stereoregularity - isotactic polymers always form crystals, atactic polymers never form
crystals, and syndiotactic polymers sometimes form crystals; (4) slow cooling from the molten states
promotes crystal formation; (5) plasticizers inhibit crystal formation; and (6) stretching the polymer
tends to promote crystallization.
8.14 Why are fillers added to a polymer?
Answer. Fillers are added to increase strength or simply to reduce the cost of the polymer.
8.15 What is a plasticizer?
Answer. A plasticizer is a chemical added to the polymer to make it softer and more flexible. It is
often added to improve the polymer's flow characteristics for shaping.
8.16 In addition to fillers and plasticizers, what are some other additives used with polymers?
Answer. Other additives include: lubricants - to reduce friction and improve flow; flame retardents;
colorants; cross-linking agents, antioxidants, and ultraviolet light absorbers.
8.17 Describe the difference in mechanical properties as a function of temperature between a highly
crystalline thermoplastic and an amorphous thermoplastic.
Answer. A highly crystalline TP retains rigidity during heating until just before its Tm is reached.
An amorphous TP shows a significant drop in deformation resistance at its Tg as temperature is
raised; it becomes increasingly like a liquid as temperature continues to increase.
8.18 What is unique about the polymer cellulose?
Answer. Cellulose is a polymer that grows in nature. Wood fiber contains about 50% cellulose and
cotton fiber is about 95% cellulose.
8.19 The nylons are members of which polymer group?
Answer. Polyamides.
8.20 What is the chemical formula of ethylene, the monomer for polyethylene?
Answer. C2H4
8.21 What is the basic difference between low density and high density polyethylene?
Answer. LDPE has a branched structure and is amorphous. HDPE is linear and highly crystalline.
These differences account for HDPE higher density, stiffness, and melting point.
8.22 How do the properties of thermosetting polymers differ from those of thermoplastics?
Answer. Thermosets are more rigid, brittle, capable of higher service temperatures, and cannot be
remelted.
8.23 Cross-linking (curing) of thermosetting plastics is accomplished by one of three ways. Name the
three ways.
36
Answer. The three ways are: (1) temperature-activated systems, in which elevated temperatures
accomplish curing; (2) catalyst-activated systems, in which small amounts of a catalyst cause
cross-linking; and (3) mixing-activated systems, in which two reactive components are mixed and
curing occurs by their chemical reaction.
8.24 Elastomers and thermosetting polymers are both cross- linked. Why are their properties so
different?
Answer. Elastomers are lightly cross-linked, whereas thermosets are highly cross-linked. Light
cross-linking allows extensibility; a highly cross-linked structure makes the polymer rigid.
8.25 What happens to an elastomer when it is below its glass transition temperature?
Answer. An elastomer is hard and brittle below its Tg.
8.26 What is the primary polymer ingredient in natural rubber?
Answer. Polyisoprene.
8.27 How are thermoplastic elastomers different from conventional rubbers?
Answer. TPEs are different in two basic ways: (1) they exhibit thermoplastic properties, and (2)
their extensibility derives from physical connections between different phases in the polymer.
Multiple Choice Quiz
There are a total of 25 correct answers in the following multiple choice questions (some questions have
multiple answers that are correct). To attain a perfect score on the quiz, all correct answers must be
given, since each correct answer is worth 1 point. For each question, each omitted answer or wrong
answer reduces the score by 1 point, and each additional answer beyond the number of answers required
reduces the score by 1 point. Percentage score on the quiz is based on the total number of correct
answers.
8.1 Of the three polymer types, which one is the most important commercially? (a) thermoplastics, (b)
thermosets, or (c) elastomers.
Answer. (a)
8.2 Which one of the three polymer types is not normally considered to be a plastic? (a) thermoplastics,
(b) thermosets, or (c) elastomers.
Answer. (c)
8.3 Which one of the three polymer types does not involve cross- linking? (a) thermoplastics, (b)
thermosets, or (c) elastomers.
Answer. (a)
8.4 As the degree of crystallinity in a given polymer increases, the polymer becomes denser and stiffer,
and its melting temperature decreases: (a) true or (b) false.
Answer. (b) Melting temperature increases with higher degree of crystallinity.
8.5 Which of the following is the chemical formula for the repeating unit in polyethylene? (a) CH2, (b)
C2H4, (c) C3H6, (d) C5H8, or (e) C8H8.
Answer. (b)
37
8.6 Degree of polymerization is which one of the following? (a) average number of mers in the
molecule chain; (b) proportion of the monomer that has been polymerized; (c) sum of the molecule
weights of the mers in the molecule; or (d) none of the above.
Answer. (a)
8.7 A branched molecular structure is stronger in the solid state and more viscous in the molten state
than a linear structure for the same polymer: (a) true or (b) false.
Answer. (a)
8.8 A copolymer is a mixture consisting of macromolecules of two different homopolymers: (a) true or
(b) false.
Answer. (b)
8.9 As temperature of a polymer increases, its density (a) increases, (b) decreases, or (c) remains fairly
constant.
Answer. (b)
8.10 Which answers complete the following sentence correctly (more than one): As the temperature of
an amorphous thermoplastic polymer is gradually reduced, the glass transition temperature Tg is
indicated when (a) the polymer transforms to a crystalline structure, (b) the coefficient of thermal
expansion increases markedly, (c) the slope of specific volume versus temperature changes
markedly, (d) the polymer becomes stiff, strong, and elastic, or (e) the polymer solidifies from the
molten state.
Answer. (c) and (d).
8.11 Which of the following plastics has the highest market share? (a) phenolics, (b) polyethylene, (c)
polypropylene, (d) polystyrene, or (e) polyvinylchloride.
Answer. (b)
8.12 Which of the following polymers are normally thermoplastic (more than one): (a) acrylics, (b)
cellulose acetate, (c) nylon, (d) polychloroprene, (e) polyethylene, or (f) polyurethane.
Answer. (a), (b), (c), and (e).
8.13 Polystyrene (without plasticizers) is amorphous, transparent, and brittle: (a) true or (b) false.
Answer. (a)
8.14 The fiber rayon used in textiles is based on which of the following polymers: (a) cellulose, (b) nylon,
(c) polyester, (d) polyethylene, or (e) polypropylene.
Answer. (a)
8.15 The basic difference between low density polyethylene and high density polyethylene is that the
latter has a much higher degree of crystallinity: (a) true or (b) false.
Answer. (a)
8.16 Among the thermosetting polymers, the most widely used commercially is which of the following:
(a) epoxies, (b) phenolics, (c) silicones, or (d) urethanes.
Answer. (b)
8.17 Polyurethanes can be which of the following (more than one): (a) thermoplastic, (b) thermosetting,
or (c) elastomeric.
38
Answer. (a), (b), and (c).
8.18 The chemical formula for polyisoprene in natural rubber is which of the following: (a) CH2, (b)
C2H4, (c) C3H6, (d) C5H8, or (e) C8H8.
Answer. (d)
8.19 The leading commercial synthetic rubber is which of the following: (a) butyl rubber, (b) isoprene
rubber, (c) polybutadiene, (d) polyurethane, (e) styrene-butadiene rubber, or (f) thermoplastic
elastomers.
Answer. (e)
39
9 COMPOSITE MATERIALS
Review Questions
9.1 What is a composite material?
Answer. A composite material is a materials system consisting of two or more distinct phases
whose combination results in properties that differ from those of its constituents.
9.2 Identify some of the characteristic properties of composite materials.
Answer. Typical properties include: (1) high strength-to- weight and stiffness-to-weight ratios; (2)
good fatigue properties and toughness; (3) anisotropic properties in many cases; and (4) other
properties and features that are difficult or impossible to obtain with metals, ceramics, or polymers
alone.
9.3 What does the term anisotropic mean?
Answer. Anisotropic means that the properties of a material vary depending on the direction in
which they are measured.
9.4 How are traditional composites distinguished from synthetic composites?
Answer. Traditional composites have been used for decades or centuries; some of them are
obtained from sources in nature, such as wood. Synthetic composites are manufactured.
9.5 Name the three basic categories of composite materials.
Answer. Metal matrix composites (MMCs), ceramic matrix composites (CMCs), and polymer
matrix composites (PMCs).
9.6 What are the common forms of the reinforcing phase in composite materials?
Answer. The forms are: (1) fibers, (2) particles and flakes, and (3) an infiltrated phase in skeletal
structures.
9.7 What is a whisker?
Answer. A whisker is a thin, hairlike crystal of very high strength.
9.8 What are the two forms of sandwich structure among laminar composite structures? Briefly
describe each.
Answer. The two forms are: (1) foamed-core sandwich, in which the core is polymer foam
between two solid skins; and (2) honeycomb, in which the core is a honeycomb structure
sandwiched between two solid skins.
9.9 Give some examples of commercial products which are laminar composite structures.
Answer. Examples given in Table 9.2 are: automotive tires, honeycomb sandwich structures, fiber
reinforced polymer structures such as boat hulls, plywood, printed circuit boards, snow skis made
from fiber reinforced polymers, and windshield glass.
9.10 What are the three general factors that determine the properties of a composite material?
40
Answer. Three factors are given in the text: (1) the component materials; (2) the geometric shapes
of the constituents - the reinforcing phase in particular - and the resulting structure of the material;
and (3) the interaction of the phases.
9.11 What is the rule of mixtures?
Answer. The rule of mixtures applies to certain properties of composite materials; it states that the
property value is a weighted average of the property values of the components, the weighting being
by proportions of the components in the composite.
9.12 What is a cermet?
Answer. A cermet is a composite material consisting of a ceramic and a metal. In the text, it is
defined as a composite consisting of ceramic grains imbedded in a metallic matrix.
9.13 Cemented carbides are what class of composites?
Answer. Yes; although the cemented carbide industry does not generally think of cemented
carbides as cermets, they fit within the definition.
9.14 What are some of the weaknesses of ceramics that might be corrected in fiber-reinforced ceramic
matrix composites?
Answer. Weaknesses of ceramics include: low tensile strength, poor toughness, and susceptibility to
thermal cracking.
9.15 What is the most common fiber material in fiber-reinforced plastics?
Answer. E-glass.
9.16 What does the term advanced composites mean?
Answer. An advanced composite is a PMC in which carbon, Kevlar, or boron fibers are used as
the reinforcing material.
9.17 What is a hybrid composite?
Answer. A hybrid composite is a fiber-reinforced PMC in which two or more fibers materials are
combined in the FRP.
9.18 Identify some of the important properties of fiber- reinforced plastic composite materials.
Answer. Properties include: high strength-to-weight ratio, high modulus-to-weight ratio, low density,
good fatigue strength, good corrosion resistance, and low thermal expansion for many FRPs.
9.19 Name some of the important applications of FRPs.
Answer. FRPs are used in modern aircraft as skin parts, automobile body panels, printed circuit
boards, tennis rackets, boat hulls, and a variety of other items.
9.20 What is meant by the term interface in the context of composite materials?
Answer. The interface is the boundary between the component phases in a composite material.
Multiple Choice Quiz
There are a total of 22 correct answers in the following multiple choice questions (some questions have
multiple answers that are correct). To attain a perfect score on the quiz, all correct answers must be
given, since each correct answer is worth 1 point. For each question, each omitted answer or wrong
answer reduces the score by 1 point, and each additional answer beyond the number of answers required
41
reduces the score by 1 point. Percentage score on the quiz is based on the total number of correct
answers.
9.1 Anisotropic means which one of the following: (a) composite materials with composition consisting
of more than two materials, (b) properties are the same in every direction, (c) properties vary
depending on the direction in which they are measured, or (d) strength and other properties as a
function of curing temperature.
Answer. (c)
9.2 The reinforcing phase is the matrix within which the secondary phase is imbedded: (a) true or (b)
false.
Answer. (b)
9.3 Which one of the following reinforcing geometries offers the greatest potential for strength and
stiffness improvement in the resulting composite material? (a) fibers, (b) flakes, (c) particles, or (d)
infiltrated phase.
Answer. (a)
9.4 Wood is which one of the following composite types? (a) CMC, (b) MMC, or (c) PMC.
Answer. (c)
9.5 Which of the following materials are used as fibers in fiber-reinforced plastics (more than one): (a)
aluminum oxide, (b) boron, (c) carbon/graphite, (d) epoxy, (e) Kevlar 49, (f) S-glass, and (g)
unsaturated polyester.
Answer. (a), (b), (c), (e), and (f).
9.6 Which of the following metals are most commonly used as the matrix material in fiber-reinforced
MMCs (name three)? (a) aluminum, (b) copper, (c) iron, (d) magnesium, (e) titanium, or (f) zinc.
Answer. (a), (d), and (e).
9.7 Which of the following metals is used as the matrix metal in nearly all WC cemented carbides? (a)
aluminum, (b) chromium, (c) cobalt, (d) lead, (e) nickel, (f) tungsten, or (g) tungsten carbide.
Answer. (c)
9.8 Ceramic matrix composites are designed to overcome which of the following weaknesses of
ceramics (more than one)? (a) compressive strength, (b) hardness, (c) hot hardness, (d) modulus of
elasticity, (e) tensile strength, or (f) toughness.
Answer. (e) and (f).
9.9 Which of the following polymer types are most commonly used in polymer matrix composites? (a)
elastomers, (b) thermoplastics, or (c) thermosets.
Answer. (c)
9.10 Which one of the following is the most common reinforcing material in FRPs? (a) Al2O3, (b) boron,
(c) carbon, (d) cobalt, (e) graphite, (f) Kevlar 49, or (g) SiO2.
Answer. (g)
9.11 Identify which of the following materials are composites (more than one)? (a) cemented carbide, (b)
phenolic molding compound, (c) plywood, (d) Portland cement, (e) rubber in automobile tires, (f)
wood, or (g) 1020 steel.
42
Answer. (a), (b), (c), (e), and (f).
43
10 FUNDAMENTALS OF METAL CASTING
Review Questions
10.1 Identify some of the important advantages of shape casting processes.
Answer. Advantages include: (1) complex part geometries are possible; (2) some casting
operations are net shape processes, meaning that no further manufacturing operations are needed
to accomplish the final part shape; (3) very large parts are possible; (4) applicable to any metal
that can be melted; and (5) some casting processes are suited to mass production.
10.2 What are some of the limitations and disadvantages of casting?
Answer. Disadvantages include: (1) limitations on mechanical strength properties; (2) porosity; (3)
poor dimensional accuracy; (4) safety hazards due to handling of hot metals; and (5)
environmental problems.
10.3 What is a factory that performs casting operations usually called?
Answer. A foundry.
10.4 What is the difference between an open mold and a closed mold?
Answer. An open mold is open to the atmosphere at the top; it is an open container in the desired
shape which must be flat at the top. A closed mold has a cavity that is entirely enclosed by the
mold, with a passageway (called the gating system) leading from the outside to the cavity. Molten
metal is poured into this gating system to fill the mold.
10.5 Name the two basic mold types that distinguish casting processes.
Answer. The two types are: (1) expendable molds and (2) permanent molds.
10.6 Which casting process is the most important commercially?
Answer. The most important casting process is sand casting.
10.7 What is the difference between a pattern and a core in sand molding?
Answer. The pattern determines the external shape of the casted part, while a core determines its
internal geometry.
10.8 What is meant by the term superheat?
Answer. Superheat is the temperature difference above the melting point at which the molten
metal is poured. The term also refers to the amount of heat that is removed from the molten
metal between pouring and solidification.
10.9 Why should turbulent flow of molten metal into the mold be avoided?
Answer. Turbulence causes several problems: (1) accelerates formation of oxides in the solidified
metal, and (2) mold erosion or gradual wearing away of the mold due to impact of molten metal.
10.10 What is the continuity law as it applies to the flow of molten metal in casting?
Answer. The continuity law, or continuity equation, indicates that the volumetric flow rate is
constant throughout the liquid flow.
44
10.11 What are some of the factors affecting the fluidity of a molten metal during pouring into a mold
cavity?
Answer. Factors include: (1) pouring temperature, (2) metal alloy composition, (3) viscosity of
liquid metal, and (4) heat transfer to the surroundings.
10.12 What does heat of fusion mean in casting?
Answer. Heat of fusion is the amount of heat energy required to transform the metal from solid
state to liquid state.
10.13 How does solidification of alloys differ from solidification of pure metals?
Answer. Pure metals solidify at a single temperature equal to the melting point. Most alloys
(exceptions are eutectic alloys) start to solidify at the liquidus and complete solidification at the
solidus, where the liquidus is a higher temperature than the solidus.
10.14 What is a eutectic alloy?
Answer. A eutectic alloy is a particular composition in an alloy system for which the solidus and
liquidus temperatures are equal. The temperature is called the eutectic temperature. Hence,
solidification occurs at a single temperature, rather than over a temperature range.
10.15 What is the relationship known as Chvorinov's Rule in casting?
Answer. Chvorinov's Rule is summarized: TST = Cm(V/A)2, where TST = total solidification time,
Cm = constant, V = volume of casting, and A = surface area of casting.
10.16 Identify the three sources of contraction in a metal casting after pouring.
Answer. The three contractions occur due to: (1) contraction of the molten metal after pouring,
(2) solidification shrinkage during transformation of state from liquid to solid, and (3) thermal
contraction in the solid state.
10.17 What is a chill in casting?
Answer. A chill is a heat sink placed to encourage rapid freezing in certain regions of the casting.
Multiple Choice Quiz
There are a total of 13 correct answers in the following multiple choice questions (some questions have
multiple answers that are correct). To attain a perfect score on the quiz, all correct answers must be
given, since each correct answer is worth 1 point. For each question, each omitted answer or wrong
answer reduces the score by 1 point, and each additional answer beyond the number of answers required
reduces the score by 1 point. Percentage score on the quiz is based on the total number of correct
answers.
10.1 Sand casting is which of the following types? (a) expendable mold, or (b) permanent mold.
Answer. (a)
10.2 The upper half of a sand casting mold is called which of the following? (a) cope, or (b) drag.
Answer. (a)
10.3 In casting, a flask is which one of the following? (a) beverage bottle for foundrymen, (b) box
which holds the cope and drag, (c) container for holding liquid metal, or (d) metal which extrudes
between the mold halves.
Answer. (b)
45
10.4 In foundry work, a runner is which one of the following? (a) channel in the mold leading from the
downsprue to the main mold cavity, (b) foundryman who moves the molten metal to the mold, or
(c) vertical channel into which molten metal is poured into the mold.
Answer. (a)
10.5 Total solidification time is defined as which one of the following? (a) time between pouring and
complete solidification, (b) time between pouring and cooling to room temperature, (c) time
between solidification and cooling to room temperature, or (d) time to give up the heat of fusion.
Answer. (a)
10.6 During solidification of an alloy when a mixture of solid and liquid metals are present, the
solid-liquid mixture is referred to as whic h one of the following? (a) eutectic composition, (b) ingot
segregation, (c) liquidus, (d) mushy zone, or (e) solidus.
Answer. (d)
10.7 Chvorinov's Rule states that total solidification time is proportional to which one of the following
quantities? (a) (A/V)n, (b) Hf, (c) Tm, (d) V, (e) V/A, or (f) (V/A)2; where A = surface area of
casting, Hf = heat of fusion, Tm = melting temperature, and V = volume of casting.
Answer. (f)
10.8 A riser in casting is described by which of the following (may be more than one answer)? (a) an
insert in the casting that inhibits buoyancy of the core, (b) gating system in which the sprue feeds
directly into the cavity, (c) metal that is not part of the casting, (d) source of molten metal to feed
the casting and compensate for shrinkage during solidification, and (e) waste metal that is usually
recycled.
Answer. (c), (d), and (e).
10.9 In a sand casting mold, the V/A ratio of the riser should be which one of the following relative to
the V/A ratio of the casting itself? (a) equal, (b) greater, or (c) smaller.
Answer. (b)
10.10 A riser that is completely enclosed within the sand mold and connected to the main cavity by a
channel to feed the molten metal is called which of the following (may be more than one)? (a)
blind riser, (b) open riser, (c) side riser, and (d) top riser.
Answer. (a) and (c).
Problems
Heating and Pouring
10.1 A disk 40 cm in diameter and 5 cm thick is to be casted of pure aluminum in an open mold
operation. The melting temperature of aluminum = 660C and the pouring temperature will be
800C. Assume that the amount of aluminum heated will be 5% more than needed to fill the mold
cavity. Compute the amount of heat that must be added to the metal to heat it to the pouring
temperature, starting from a room temperature of 25C. The heat of fusion of aluminum = 389.3
J/g. Other properties can be obtained from Tables 4.1 and 4.2 in this text. Assume the specific heat
has the same value for solid and molten aluminum.
Solution: Volume V = D2h/4= (40)2(5)/4 = 6283.2 cm3
Volume of aluminum to be heated = 6283.2(1.05) = 6597.3 cm3
46
From Table 4.1 and 4.2, density = 2.70 g/cm3 and specific heat C = 0.21 Cal/g-C = 0.88 J/g-C
Heat required = 2.70(6597.3){0.88(660-25) + 389.3 + 0.88(800-660)}
= 17,812.71{558.8 + 389.3 + 123.2} = 19,082,756 J
10.2 A sufficient amount of pure copper is to be heated for casting a large plate in an open mold. The
plate has dimensions: L = 20 in, W = 10 in, and D = 3 in. Compute the amount of heat that must be
added to the metal to heat it to a temperature of 2150 F for pouring. Assume that the amount of
metal heated will be 10% more than needed to fill the mold cavity. Properties of the metal are:
density = 0.324 lbm/in3, melting point = 1981 F, specific heat of the metal = 0.093 Btu/lbm-F in the
solid state and 0.090 Btu/lbm-F in the liquid state; and heat of fusion = 80 Btu/lbm.
Solution: Volume V = (20 x 10 x 3)(1 + 10%) = 600(1.1) = 660.0 in3
Assuming To = 75 F and using Eq. (12.1),
H = 0.324 x 660{0.093(1981 - 75) + 80 + 0.090(2150 - 1981)} = 213.84{177.26 + 80 + 15.21}
H = 58,265 Btu
10.3 The downsprue leading into the runner of a certain mold has a length = 175 mm. The
cross-sectional area at the base of the sprue is 400 mm2. The mold cavity has a volume = 0.001 m3.
Determine: (a) the velocity of the molten metal flowing through the base of the downsprue, (b) the
volume rate of flow, and (c) the time required to fill the mold cavity.
Solution: (a) Velocity v = (2 x 9815 x 175)0.5 = (3,435,096)0.5 = 1853 mm/s
(b) Volume flow rate Q = vA = 1853 x 400 = 741,200 mm3/s
(c) Time to fill cavity MFT = V/Q = 1,000,000/741,200 = 1.35 s
10.4 A mold has a downsprue of length = 6.0 in. The cross-sectional area at the bottom of the sprue is
0.5 in2. The sprue leads into a horizontal runner which feeds the mold cavity, whose volume = 75
in3. Determine: (a) the velocity of the molten metal flowing through the base of the downsprue, (b)
the volume rate of flow, and (c) the time required to fill the mold cavity.
Solution: (a) Velocity v = (2 x 32.2 x 12 x 6.0)0.5 = (4636.8)0.5 = 68.1 in/sec
(b) Volume flow rate Q = vA = 68.1 x 0.5 = 34.05 in3/sec
(c) Time to fill cavity MFT = V/Q = 75.0/34.05 = 2.2 sec.
10.5 The flow rate of liquid metal into the downsprue of a mold = 1 liter/sec. The cross-sectional area at
the top of the sprue = 800 mm2 and its length = 175 mm. What area should be used at the base of
the sprue to avoid aspiration of the molten metal?
Solution: Flow rate Q = 1.0 l/s = 1,000,000 mm3/s
Velocity v = (2 x 9815 x 175)0.5 = 1854 mm/s
Area at base A = 1,000,000/1854 = 540 mm2
10.6 The volume rate of flow of molten metal into the downsprue from the pouring cup is 50 in3/sec. At
the top where the pouring cup leads into the downsprue, the cross-sectional area = 1.0 in2.
Determine what the area should be at the bottom of the sprue if its length = 8.0 in. It is desired to
maintain a constant flow rate, top and bottom, in order to avoid aspiration of the liquid metal.
Solution: Velocity at base v = (2gh)0.5 = (2 x 32.2 x 12 x 8)0.5 = 78.6 in/sec
Assuming volumetric continuity, area at base A = (50 in/sec)/(78.6 in/sec) = 0.636 in2
10.7 Molten metal can be poured into the pouring cup of a sand mold at a steady rate of 1000 cm3/s. The
molten metal overflows the pouring cup and flows into the downsprue. The cross-section of the
sprue is round, with a diameter at the top = 3.4 cm. If the sprue is 25 cm long, determine the proper
diameter at its base so as to maintain the same volume flow rate.
47
Solution: Velocity at base v = (2gh)0.5 = (2 x 981 x 25)0.5 = 221.5 cm/s
Assuming volumetric continuity, area at base A = (1000 cm/s)/(221.5 cm/s) = 4.51 cm2
Area of sprue A = D2/4; rearranging, D2 = 4A/= 4(4.51)/= 5.74 cm2
D = 2.39 cm
10.8 During pouring into a sand mold, the molten metal can be poured into the downsprue at a constant
flow rate during the time it takes to fill the mold. At the end of pouring the sprue is filled and there is
negligible metal in the pouring cup. The downsprue is 6.0 in long. Its cross-sectional area at the top
= 0.8 in2 and at the base = 0.6 in2. The cross-sectional area of the runner leading from the sprue
also = 0.6 in2, and it is 8.0 in long before leading into the mold cavity, whose volume = 65 in3. The
volume of the riser located along the runner near the mold cavity = 25 in3. It takes a total of 3.0 sec
to fill the entire mold (including cavity, riser, runner, and sprue. This is more than the theoretical time
required, indicating a loss of velocity due to friction in the sprue and runner. Find: (a) the theoretical
velocity and flow rate at the base of the downsprue; (b) the total volume of the mold; (c) the actual
velocity and flow rate at the base of the sprue; and (d) the loss of head in the gating system due to
friction.
Solution: (a) Velocity v = (2 x 32.2 x 12 x 6.0)0.5 = 68.1 in/sec
Flow rate Q = 68.1 x 0.60 = 40.8 in3/sec
(b) Total V = 65.0 + 25.0 + 0.5(0.8 + 0.6)(6.0) + 0.6(8.0) = 99.0 in3
(c) Actual flow rate Q = 99.0/3 = 33.0 in3/sec
Actual velocity v = 33.0/0.6 = 55.0 in/sec
(d) v = (2 x 32.2 x 12 x h)0.5 = 27.8 h0.5 = 55.0 in/sec.
h0.5 = 55.0/27.8 = 1.978
h = 1.9782 = 3.914 in
Head loss = 6.0 - 3.914 = 2.086 in
Shrinkage
10.9 A mold cavity has the shape of a cube, 100 mm on a side. Determine the dimensions and volume
of the final cube after cooling to room temperature if the cast metal is copper. Assume that the
mold is full at the start of solidification and that shrinkage occurs uniformly in all directions.
Solution: For copper, solidification shrinkage is 4.9%, solid contraction during cooling is 7.5%.
Volume of cavity V = (100)3 = 1,000,000 mm3
Volume of casting V = 1,000,000(1-0.049)(1-0.075) = 1,000,000(.951)(.025) = 879,675 mm3
Dimension on each side of cube = (879,675)0.333 = 95.82 mm
10.10 The cavity of a casting mold has dimensions: L = 250 mm, W = 125 mm and H = 20 mm.
Determine the dimensions of the final casting after cooling to room temperature if the cast metal is
aluminum. Assume that the mold is full at the start of solidification and that shrinkage occurs
uniformly in all directions.
Solution: For aluminum, solidification shrinkage = 6.6%, solid contraction during cooling = 5.6%.
Total volumetric contraction = (1-0.066)(1-0.056) = 0.8817
Linear contraction = (0.8817)0.333 = 0.9589
Final casting dimensions: L = 250(0.9589) = 239.725 cm
W = 125(0.9589) = 119.863 cm
H = 20(0.9589) = 19.178 cm
48
10.11 Determine the scale of a "shrink rule" that is to be used by pattern-makers for low carbon steel.
Express your answer in terms of decimal fraction inches of elongation per foot of length compared
to a standard rule.
Solution: Low carbon steel: solidification shrinkage = 2.75%, solid contraction = 7.2%.
Total volumetric contraction = (1-0.0275)(1-0.072) = 0.9025
Linear contraction = (0.9025)0.333 = 0.9664
Shrink rule elongation = (0.9664)-1 = 1.0348
Elongation of a 12 inch rule = 12(1.0348 - 1.0) = 0.418 in/ft
10.12 Determine the scale of a "shrink rule" that is to be used by pattern-makers for brass which is 70%
copper and 30% zinc. Express your answer in terms of millimeters of elongation per meter of
length compared to a standard rule.
Solution: For brass, solidification shrinkage is 4.5%, solid contraction during cooling is 8.0%.
Total volumetric contraction = (1-0.045)(1-0.080) = 0.8786
Linear contraction = (0.8786)0.333 = 0.9578
Shrink rule elongation = (0.9578)-1 = 1.0441
Elongation of a 1 meter rule = 1000(1.0441 - 1.0) = 44.1 mm/m
10.13 Determine the scale of a "shrink rule" that is to be used by pattern-makers for gray cast iron. The
gray cast iron has a volumetric contraction of -2.5%, which means it expands during solidification.
Express your answer in terms of millimeters of elongation per meter of length compared to a
standard rule.
Solution: For gray CI, solidification shrinkage = -2.5%, solid contraction during cooling = 3.0%.
Total volumetric contraction = (1+0.025)(1-0.030) = 0.99425
Linear contraction = (0.99425)0.333 = 0.9981
Shrink rule elongation = (0.9981)-1 = 1.00192
Elongation of a 1 meter rule = 1000(1.00192 - 1.0) = 1.92 mm/m
10.14 The final dimensions of a disk-shaped casting of 1.0% carbon steel are: diameter = 12.0 in and
thickness = 0.75 in. Determine the dimensions of the mold cavity to take shrinkage into account.
Assume that shrinkage occurs uniformly in all directions.
Solution: For 1% carbon steel, solidification shrinkage is 4.0%, solid contraction during cooling is
7.2%.
Total volumetric contraction = (1-0.040)(1-0.072) = 0.8909
Linear contraction = (0.8909)0.333 = 0.9622
Oversize factor for mold = (0.9622)-1 = 1.03927
Mold cavity dimensions: D = 12.00(1.03927) = 12.471 in and t = 0.750(1.03927) = 0.779 in
Solidification Time and Riser Design
10.15 In the casting of steel under certain mold conditions, the mold constant in Chvorinov's Rule is
known to be Cm = 4.0 min/cm2, based on previous experience. The casting is a flat plate whose
length = 30 cm, width = 10 cm, and thickness = 20 mm. Determine how long it will take for the
casting to solidify.
Solution: Volume V = 30 x 10 x 2 = 600 cm3
Area A = 2(30 x 10 + 30 x 2 + 10 x 2) = 760 cm2
Chvorinov’s Rule: TST = Cm (V/A)2 = 4(600/760)2 = 2.49 min
10.16 Solve for total solidification time in the previous problem only using a value of n = 1.9 in
Chvorinov's Rule. What adjustment must be made in the units of Cm?
49
Solution: Chvorinov’s Rule: TST = Cm (V/A)1.9 = 4(600/760)1.9 = 2.55 min
The units for Cm become min/in1.9 - strange but consistent with Chvorinov’s empirical rule.
10.17 A disk-shaped part is to be cast out of aluminum. The diameter of the disk = 500 mm and its
thickness = 20 mm. If Cm = 2.0 sec/mm2 in Chvorinov's Rule, how long will it take the casting to
solidify?
Solution: Volume V = D2t/4 = (500)2(20)/4 = 3,926,991 mm3
Area A = 2D2/4 + Dt = (500)2/2 + (500)(20) = 424,115 mm2
Chvorinov’s Rule: TST = Cm (V/A)2 = 2.0(3,926,991/424,115)2 = 171.5 s = 2.86 min
10.18 In casting experiments performed using a certain alloy and type of sand mold, it took 155 sec for a
cube-shaped casting to solidify. The cube was 50 mm on a side. (a) Determine the value of the
mold constant Cm in Chvorinov's Rule. (b) If the same alloy and mold type were used, find the total
solidification time for a cylindrical casting in which the diameter = 30 mm and length = 50 mm.
Solution: (a) Volume V = (50)3 = 125,000 mm3
Area A = 6 x (50)2 = 15,000 mm2
(V/A) = 125,000/15,000 = 8.333 mm
Cm = TST/(V/A)2 = 155/(8.333)2 = 2.232 s/mm2
(b) Cylindrical casting with D = 30 mm and L = 50 mm.
Volume V = D2L/4 = (30)2(50)/4 = 35,343 mm3
Area A = 2D2/4 + DL = (30)2/2 + (30)(50) = 6126 mm2
V/A = 35,343/6126 = 5.77
TST = 2.232 (5.77)2 = 74.3 s = 1.24 min.
10.19 A steel casting has a cylindrical geometry with 4.0 in diameter and weighs 20 lb. This casting takes
6.0 min to completely solidify. Another cylindrical-shaped casting with the same diameter-to-length
ratio weighs 12 lb. This casting is made of the same steel and the same conditions of mold and
pouring were used. Determine: (a) the mold constant in Chvorinov's Rule; and (b) the dimensions,
and (c) the total solidification time of the lighter casting. Note: The density of steel = 490 lb/ft3.
Solution: (a) For steel, = 490 lb/ft3 = 0.2836 lb/in3
Weight W = V, V = W/= 20/0.2836 = 70.53 in3
Volume V = D2L/4 = (4)2L/4 = 4L = 70.53 in3
Length L = 70.53/4= 5.61 in
Area A = 2D2/4 + DL = 2(4)2/4 + (4)(5.61) = 95.63 in2
(V/A) = 70.53/95.63 = 0.7375
Cm = 6.0/(0.7353)2 = 11.03 min/in2
(b) Find dimensions of smaller cylindrical casting with same D/L ratio and w = 12 lb.
Weight is proportional to volume: V = (12/20)(70.53) = 42.32 in3
D/L ratio = 4.0/5.61 = 0.713; thus L = 1.4025D
Volume V = D2L/4 = (4)2(1.4025D)/4 = 1.1015D3
D3 = (42.32 in3)/1.1015 = 38.42 in3
D = (38.42)0.333 = 3.374 in
L = 1.4025(3.374) = 4.732 in
(c) V = D2L/4 = (3.374)2(4.732)/4 = 42.32 in3
A = 2D2/4 + DL = 0.5(3.374)2 + (3.374)(4.732) = 68.04 in2
V/A = 42.32/68.04 = 0.622 in.
TST = 11.03(.622)2 = 4.27 min.
50
10.20 The total solidification times of three casting shapes are to be compared: (1) a sphere with
diameter = 10 cm, (2) a cylinder with diameter and length both = 10 cm, and (3) a cube with each
side = 10 cm. The same casting alloy is used in the three cases. (a) Determine the relative
solidification times for each geometry. (b) Based on the results of part (a), which geometric
element would make the best riser? (c) If Cm = 3.5 min/cm2 in Chvorinov's Rule, compute the total
solidification time for each casting.
Solution: For ease of computation, make the substitution 10 cm = 1 decimeter (1 dm)
(a) Chvorinov’s Rule: TST = Cm(V/A)2
(1) Sphere volume V = D3/6 = (1)3/6 = /6 dm3
Sphere area A = D2 = (1)2 = dm2
V/A = (/6)/ = 1/6 = 0.1667 dm
Chvorinov’s Rule TST = (0.1667)2Cm = 0.02778Cm
(2) Cylinder volume V = D2H/4 = (1)2(1)/4 = /4 = 0.25dm3
Cylinder area A = 2D2/4 + DL = 2(1)2/4 + (1)(1) = /2 + = 1.5dm2
V/A = .25/1.5= 0.1667 dm
Chvorinov’s Rule TST = (0.1667)2Cm = 0.02778Cm
(3) Cube: V = L3 = (1)3 = 1.0 dm3
Cube area = 6L2 = 6(1)2 = 6.0 dm2
V/A = 1.0/6.0 = 0.1667 dm
Chvorinov’s Rule TST = (0.1667)2Cm = 0.02778Cm
(b) All three shapes are equivalent as risers.
(c) If Cm = 3.5 min/cm2 = 350 min/dm2, then TST = 0.02778(350) = 9.723 min. Note, however,
that the volumes of the three geometries are different: (1) sphere V = 0.524 dm3 = 524 cm3,
cylinder V = 0.25= 0.7854 dm3 = 785.4 cm3, and (3) cube V = 1.0 dm3 = 1000cm3. Accordingly,
we might revise our answer to part (b) and choose the sphere on the basis that it wastes less metal
than the other shapes.
10.21 The total solidification times of three casting shapes are to be compared: (1) a sphere, (2) a
cylinder, in which the L/D ratio = 1.0, and (3) a cube. For all three geometries, the volume V =
1000 cm3. The same casting alloy is used in the three cases. (a) Determine the relative
solidification times for each geometry. (b) Based on the results of part (a), which geometric
element would make the best riser? (c) If Cm = 3.5 min/cm2 in Chvorinov's Rule, compute the total
solidification time for each casting.
Solution: For ease of computation, make the substitution 10 cm = 1 decimeter (1 dm). Thus 1000
cm3 = 1.0 dm3.
(1) Sphere volume V = D3/6 = 1.0 dm3. D3 = 6/= 1.910 dm3. D = (1.910)0.333 = 1.241 dm
Sphere area A = D2 = (1.241)2 = 4.836 dm2
V/A = 1.0/4.836 = 0.2067 dm
Chvorinov’s Rule TST = (0.2067)2Cm = 0.0428Cm
(2) Cylinder volume V = D2H/4 = D3/4 = 1.0 dm3. D3 = 4/= 1.273 dm3
Therefore, D = H = (1.273)0.333 = 1.084 dm
Cylinder area A = 2D2/4 + DL = 2(1.084)2/4 + (1.084)(1.084) = 5.536 dm2
V/A = 1.0/5.536 = 0.1806 dm
Chvorinov’s Rule TST = (0.1806)2Cm = 0.0326Cm
51
(3) Cube: V = L3 =1.0 dm3. L = 1.0 dm
Cube area = 6L2 = 6(1)2 = 6.0 dm2
V/A = 1.0/6.0 = 0.1667 dm
Chvorinov’s Rule TST = (0.1667)2Cm = 0.02778Cm
(b) Sphere would be the best riser, since V/A ratio is greatest.
(c) Given that Cm = 3.5 min/cm2 = 350 min/dm3
Sphere: TST = 0.0428(350) = 14.98 min
Cylinder: TST = 0.0326(350) = 11.41 min
Cube: TST = 0.02778(350) = 9.72 min
10.22 A cylindrical riser is to be used for a sand casting mold. For a given cylinder volume, determine the
diameter-to- length ratio that will maximize the time to solidify.
Solution: To maximize TST, the V/A ratio must be maximized.
Cylinder volume V = D2L/4. L = 4V/D2
Cylinder area A = 2D2/4 + DL
Substitute the expression for L from the volume equation in the area equation:
A = D2/2 + DL = D2/2 + D(4V/D2) = D2/2 + 4V/D
Differentiate the area equation with respect to D:
dA/dD = D - 4V/D2 = 0 Rearranging, D = 4V/D2
D3 = 4V/
D = (4V/)0.333
From the previous expression for L, substituting in the equation for D that we have developed,
L = 4V/D2 = 4V/(4V/)0.667 = (4V/)0.333
Thus, optimal values are D = L = (4V/)0.333, and therefore the optimal D/L ratio = 1.0
10.23 A riser in the shape of a sphere is to be designed for a sand casting mold. The casting is a
rectangular plate, with length = 200 mm, width = 100 mm, and thickness = 18 mm. If the total
solidification time of the casting itself is known to be 3.5 min, determine the diameter of the riser so
that it will take 25% longer for the riser to solidify.
Solution: Casting volume V = LWt = 200(100)(18) = 360,000 mm3
Casting area A = 2(200 x 100 + 200 x 18 + 100 x 18) = 50,800 mm2
V/A = 360,000/50,800 = 7.0866
Casting TST = Cm(7.0866)2 = 3.50 min
Cm = 3.5/(7.0866)2 = 0.0697 min/mm2
Riser volume V = D3/6 = 0.5236D3
Riser area A = D2 = 3.1416D2
V/A = 0.5236D3/3.1416D2 = 0.1667D
TST = 1.25(3.5) = 4.375 min = 0.0697(0.1667D)2 = 0.001936D2
D2 = 4.375/0.001936 = 2259.7 mm2
D = 47.5 mm
10.24 A cylindrical riser is to be designed for a sand casting mold. The length of the cylinder is to be 1.25
times its diameter. The casting is a square plate, each side = 10 in and thickness = 0.75 inch. If the
metal is cast iron, and Cm = 16.0 min/in2 in Chvorinov's Rule, determine the dimensions of the riser
so that it will take 30% longer for the riser to solidify.
Solution: Casting volume V = tL2 = 0.75(10.0)2 = 75 in3
Casting area A = 2L2 + 4Lt = 2(10.0)2 + 4(10.0)(0.75) = 230.0 in2
52
V/A = 75/230 = 0.3261 Casting TST = 16(0.3261)2 = 1.70 min
Riser TST = 1.30(1.70) = 2.21 min
Riser volume V = D2H/4 = 0.25D2(1.25D) = 0.3125D3
Riser area A = 2D2/4 + DH = 0.5D2 + 1.25D2 = 1.75D2
V/A = 0.3125D3/1.75D2 = 0.1786D
Riser TST = 16.0(0.1786D)2 = 16.0(0.03189)D2 = 0.5102D2 = 2.21 min
D2 = 2.21/0.5102 = 4.3316
D = (4.3316)0.5 = 2.081 in
H = 1.25(2.081) = 2.602 in.
10.25 A cylindrical riser with diameter-to-length ratio = 1.0 is to be designed for a sand casting mold. The
casting geometry is illustrated in Figure P10.25, in which the units are inches. If Cm = 19.5 min/in2
in Chvorinov's Rule, determine the dimensions of the riser so that the riser will take 0.5 minute
longer to freeze than the casting itself.
Solution: Casting volume V = V(5 in x 10 in rectangular plate) + V(5 in. half disk) + V(upright
tube) - V(3 in x 6 in rectangular cutout).
V(5 in x 10 in rectangular plate) = 5 x 12.5 x 1.0 = 62.5 in3
V(5 in. half disk) = 0.5(5)2(1)/4 = 9.817 in3
V(upright tube) = 3.0(2.5)2/4 - 4(1.5)2/4) = 7.657 in3
V(3 in x 6 in rectangular cutout) = 3 x 6 x 1 = 18.0 in3
Total V = 62.5 + 9.817 + 7.657 - 18.0 = 61.974 in3
Total A = 1 x 5 + 1(12.5 + 2.5+ 12.5) + 2(6+3) + 2(5 x 12.5 - 3 x 6) + 2(.5(5)2/4) - 2(1.5)2/4 +
2.5(3) + 1.5(3+1) = 203.36 in2
V/A = 61.974/203.36 = 0.305 in
Casting TST = 19.5(0.305)2 = 1.81 min
Riser design: specified TST = 1.81 + 0.5 = 2.31 min
Riser volume V = D2L/4 = D3/4 = 0.25D3
Riser area A = DL + 2D2/4 = D2 + 0.5D2 = 1.5D2
V/A = .25D3/1.5D2 = D/6
TST = Cm(V/A)2
2.31 = 19.5(D/6)2 = 0.5417D2
D2 = 2.31/0.5417 = 4.266 in2 D = 2.065 in. and L = 2.065 in.
53
11 METAL CASTING PROCESSES
Review Questions
11.1 Name the two basic categories of casting processes?
Answer. The two categories are: (1) expendable mold processes, and (2) permanent mold
processes.
11.2 There are various types of patterns used in sand casting. What is the difference between a split
pattern and a match- plate pattern?
Answer. A split pattern is a pattern that consists of two pieces; a match-plate pattern consists of
the two split patterns attached to opposite sides of a plate.
11.3 What is a chaplet?
Answer. Chaplets are metal supports of various designs used to hold the core in place in the sand
mold.
11.4 What properties determine the quality of a sand mold for sand casting?
Answer. The usual properties are: (1) strength - ability to maintain shape in the face of the flowing
metal, (2) permeability - ability of the mold to allow hot air and gases to escape from the cavity, (3)
thermal stability - ability to resist cracking and buckling when in contact with the molten metal, (4)
collapsibility - ability of the mold to give way during shrinkage of the casting, (5) reusability - can
the sand be reused to make other molds?
11.5 What is the Antioch process?
Answer. The Antioch process refers to the making of the mold. The mold is 50% sand and 50%
plaster heated in an autoclave and then dried. This mold has greater permeability than a plaster
mold.
11.6 What is the difference between vacuum permanent-mold casting and vacuum molding?
Answer. Vacuum permanent-mold casting is a form of low- pressure casting in which a vacuum is
used to draw molten metal into the cavity. Vacuum molding is sand casting in which the sand mold
is held together by vacuum pressure rather than a chemical binder.
11.7 What are the most common metals processed using die casting?
Answer. Common die cast metals include: zinc, tin, lead, aluminum, brass, and magnesium.
11.8 Which die casting machines usually have a higher production rate, cold-chamber or hot-chamber,
and why?
Answer. Hot-chamber machines are faster because cold- chamber die casting machines require
molten metal to be ladled into the chamber from an external source.
11.9 What is flash in die casting?
Answer. Flash is a thin portion at the exterior of a casting that results from molten metal being
squeezed into the spaces between the die halves of the mold at the parting line, or into the
clearances around the cores and ejector pins.
11.10 What is the difference between true centrifugal casting and semicentrifugal casting?
54
Answer. In true centrifugal casting, a tubular mold is used and a tubular part is produced. In
semicentrifugal casting, the shape is solid; an example is a railway wheel. The mold is rotated so
that centrifugal force is used to distribute the molten metal to the exterior of the mold so that the
density of the final metal is greater at the outer sections.
11.11 What is a cupola?
Answer. A cupola is a vertical cylindrical furnace equipped with a tapping spout near its base.
Cupolas are used for melting cast irons.
11.12 What are some of the operations required of sand castings after removal from the mold?
Answer. These operations include: (1) trimming, in which the sprues, runners, risers, and flash are
removed, (2) core removal, (3) surface cleaning, (4) inspection, (5) repair if needed, (6) heat
treatment, and (7) machining.
11.13 What are some of the general defects encountered in casting processes?
Answer. General defects include: (1) misruns, (2) cold shuts, (3) cold shots, (4) shrinkage cavity,
(5) microporosity, and (6) hot tearing. See Article 11.6.1.
Multiple Choice Quiz
There are a total of 28 correct answers in the following multiple choice questions (some questions have
multiple answers that are correct). To attain a perfect score on the quiz, all correct answers must be
given, since each correct answer is worth 1 point. For each question, each omitted answer or wrong
answer reduces the score by 1 point, and each additional answer beyond the number of answers required
reduces the score by 1 point. Percentage score on the quiz is based on the total number of correct
answers.
11.1 Which one of the following casting processes is most widely used (one answer)? (a) centrifugal
casting, (b) die casting, (c) investment casting, (d) sand casting, or (e) shell casting.
Answer. (d)
11.2 In sand casting, the volumetric size of the pattern is which of the following relative to the cast part?
(a) bigger, (b) same size, or (c) smaller.
Answer. (a)
11.3 Silica sand has which one of the following compositions? (a) Al2O3, (b) SiO, (c) SiO2, or (d) SiSO4.
Answer. (c)
11.4 For which of the following reasons is a green mold so- named? (a) green is the color of the mold,
(b) moisture is contained in the mold, (c) mold is cured, or (d) mold is dry.
Answer. (b)
11.5 Given that Wm = weight of the molten metal displaced by a core and Wc = weight of the core, the
buoyancy force is which one of the following? (a) downward force = Wm + Wc, (b) downward
force = Wm - Wc, (c) upward force = Wm + Wc, or (d) upward force = Wm - Wc.
Answer. (d)
11.6 Which of the following casting processes are expendable mold operations (more than one)? (a)
investment casting, (b) low pressure casting, (c) sand casting, (d) shell molding, (e) slush casting,
and (f) vacuum molding.
55
Answer. (a), (c), (d), and (f).
11.7 Shell molding is which one of the following? (a) casting operation in which the molten metal has
been poured out after a thin shell has been solidified in the mold, (b) casting operation used to make
artificial sea shells, (c) casting process in which the mold is a thin shell of sand binded by a
thermosetting resin, or (d) sand casting operation in which the pattern is a shell rather than a solid
form.
Answer. (c)
11.8 Investment casting is also known by which one of the following names? (a) fast-payback molding,
(b) full-mold process, (c) lost-foam process, (d) lost pattern process, or (e) lost-wax process.
Answer. (e)
11.9 In plaster mold casting, the mold is made of which one of the following materials? (a) Al2O3, (b)
CaSO4-H2O, (c) SiC, or (d) SiO2.
Answer. (b)
11.10 Which of the following qualifies as a precision casting process (more than one)? (a) ingot casting,
(b) investment casting, (c) plaster mold casting, (d) sand casting, and (c) shell molding.
Answer. (b) and (c).
11.11 Which of the following casting processes are permanent mold operations (more than one)? (a)
centrifugal casting, (b) die casting, (c) low pressure casting, (d) shell molding, (e) slush casting, and
(f) vacuum permanent-mold casting.
Answer. (a), (b), (c), (e), and (f).
11.12 Which of the following metals would typically be die casted (more than one)? (a) aluminum, (b)
cast iron, (c) steel, (d) tin, (e) tungsten, and (f) zinc.
Answer. (a), (d), and (f).
11.13 Which of the following are advantages of die casting over sand casting (more than one)? (a) better
surface finish, (b) higher melting temperature metals, (c) higher production rates, (d) larger parts
can be casted, and (e) mold can be reused.
Answer. (a), (c), and (e).
11.14 Cupolas are furnaces used to melt which of the following metals (choose one best answer)? (a)
aluminum, (b) cast iron, (c) steel, or (d) zinc.
Answer. (b)
11.15 A misrun is which one of the following defects in casting? (a) globules of metal becoming
entrapped in the casting, (b) metal is not properly poured into the downsprue, (c) metal solidifies
before filling the cavity, (d) microporosity, and (e) "pipe" formation.
Answer. (c)
11.16 Which one of the following casting metals is most important commercially? (a) aluminum and its
alloys, (b) bronze, (c) cast iron, (d) cast steel, or (e) zinc alloys.
Answer. (c)
56
Problems
Buoyancy Force
11.1 An aluminum-copper alloy casting is made in a sand mold using a sand core that weighs 20 kg.
Determine the buoyancy force in Newtons tending to lift the core during pouring.
Solution: Core volume V = 20/1605.4 = 0.01246 m3.
Weight of displaced Al-Cu = 35.17 kg.
Difference = (35.17 - 20) x 9.815 = 149 N.
11.2 A sand core located inside a mold cavity has a volume of 157.0 in3. It is used in the casting of a
cast iron pump housing. Determine the buoyancy force that will tend to lift the core during pouring.
Solution: From Table 13.1, density of cast iron = 0.26 lb/in3.
Fb = Wm - Wc
Wc = 157(0.058) = 9.106 lb.
Wm = 157(0.26) = 40.82 lb.
Fb = 40.82 - 9.11 = 31.71 lb.
11.3 Caplets are used to support a sand core inside a sand mold cavity. The design of the caplets and
the manner in which they are placed in the mold cavity surface allows each caplet to sustain a
force of 10 lbs. Several caplets are located beneath the core to support it before pouring; and
several other caplets are placed above the core to resist the buoyancy force during pouring. If the
volume of the core = 325 in.3, and the metal poured is brass, determine the minimum number of
caplets that should be placed: (a) beneath the core, and (b) above the core.
Solution: From Table 13.1, density of brass = 0.313 lb/in3.
(a) Wc = 325(0.058) = 18.85 lb. At least 2 caplets are required beneath to resist the weight of
the core. Probably 3 or 4 caplets would be better to achieve stability.
(b) Wm = 325(.313) = 101.73 lb.
Fb = 101.73 - 18.85 = 82.88 lb. A total of 9 caplets are required above the core to resist the
buoyancy force.
11.4 A sand core used to form the internal surfaces of a steel casting experiences a buoyancy force of
23 kg. The volume of the mold cavity forming the outside surface of the casting = 5000 cm3. What
is the weight of the final casting? Ignore considerations of shrinkage.
Solution: Sand density = 1.6 g/cm3, steel casting density = 7.82 g/cm3
Fb = Wm - Wc = 7.82V - 1.6V = 6.22V = 23 kg = 23,000 g V = 3698 cm3.
Cavity volume V = 5000 cm3
Volume of casting V = 5000 - 3698 = 1302 cm3.
Weight of the final casting W = 1302(7.82) = 10,184 g = 10.184 kg
Centrifugal Casting
11.5 A horizontal true centrifugal casting operation will be used to make copper tubing. The lengths will
be 1.5 m with outside diameter = 15.0 cm, and inside diameter = 12.5 cm. If the rotational speed of
the pipe = 1000 rev/min, determine the G-factor.
Solution: From Eq. (11.4), GF = R(N/30)2/g = 7.5((1000)/30)2/981 = 83.8
11.6 A true centrifugal casting operation is to be performed in a horizontal configuration to make cast
iron pipe sections. The sections will have a length = 42.0 in., outside diameter = 8.0 in, and wall
57
thickness = 0.50 in. If the rotational speed of the pipe = 500 rev/min, determine the G-factor. Is the
operation likely to be successful?
Solution: Using outside wall of casting, R = 0.5(8)/12 = 0.333 ft.
v = RN/30 = (.333)(500)/30 = 17.45 ft/sec.
GF = v2/Rg = (17.45)2/(.333 x 32.2) = 28.38
Since the G-factor is less than 60, the rotational speed is not sufficient, and the operation is likely to
be unsuccessful.
11.7 A horizontal true centrifugal casting process is used to make brass bushings with dimensions: L =
10 cm, OD = 15 cm, and ID = 12 cm. (a) Determine the required rotational speed in order to obtain
a G-factor of 70. (b) When operating at this speed, what is the centrifugal force per square meter
(Pa) imposed by the molten metal on the inside wall of the mold?
Solution: (a) Using the outside wall diameter of the casting, which is equal to the inside wall
diameter of the mold, D = 15 cm
N = (30/)(2g x 70/15).5 = 913.7 rev/min.
(b) Use 1.0 cm of mold wall length as basis of area calculations.
Area of this length of mold wall A = DoL = (15 cm)(1 cm) = 15cm2 = 15(10-4) m2
Volume of cast metal V = (Ro
2 - Ri
2)(1.0) = ((7.5)2 - (6)2)(1.0) = 63.62 cm3
Mass m = (8.62g/cm3)(63.62 cm3) = 548.4 g = 0.5484 kg
v = RN/30 Use mean radius R = (7.5 + 6.0)/2 = 6.75 cm
v = (6.75)(913.7)/30 = 645.86 cm/s = 6.4585 m/s
Centrifugal force per square meter on mold wall = Fc/A where Fc = mv2/R
Fc = (0.5484 kg)(6.4586 m/s)2/(6.75 x 10-2 m) = 338.9 kg-m/s2
Given that 1 N = 9.81 kg-m/s2, Fc = 338.9/9.81 = 34.55 N
Fc/A = (34.55 N)/(15x 10-4 m2) = 0.7331(104) N/m2 = 7331 Pa
11.8 True centrifugal casting operation is performed horizontally to make large diameter copper tube
sections. The tubes have a length = 1.0 m, diameter = 0.25 m, and wall thickness = 15 mm. If the
rotational speed of the pipe = 700 rev/min, (a) determine the G-factor on the molten metal. (b) Is
the rotational speed sufficient to avoid "rain?" (c) What volume of molten metal must be poured
into the mold to make the casting if solidification shrinkage and contraction after solidification are
considered?
Solution: (a) GF = v2/Rg g = 9.8 m/s2
v = RN/30 = (.125)(700)/30 = 9.163 m/s
GF = (9.163)2/(.125 x 9.8) = 68.54
(b) G-factor is sufficient for a successful casting operation.
(c) Volume of final product after solidification and cooling is
V = (.252 - (.25-.03)2)x 1.0/4 = .25(.252 - .222) = 0.011074 m3
From Table 12.1, solidification shrinkage = 4.9% and solid thermal contraction = 7.5% for copper.
Taking these factors into account,
Volume of molten metal V = 0.011074/(1-.049)(1-.075) = 0.01259 m3
11.9 If a true centrifugal casting operation were to be performed in a space station circling the Earth,
how would weightlessness affect the process?
Solution: The mass of molten metal would be unaffected by the absence of gravity, but its weight
would be zero. Thus, in the G-factor equation (GF = v2/Rg), GF would theoretically go to infinity if
g = 0. Thus, it should be possible to force the metal against the walls of the mold in centrifugal
58
casting without the nuisance of “raining” inside the cavity. However, this all assumes that the
metal is inside the mold and rotating with it. In the absence of gravity, there would be a problem in
pouring the molten metal into the mold cavity and getting it to adhere to the mold wall as the mold
begins to rotate. With no gravity the liquid metal would not be forced against the lower surface of
the mold to initiate the centrifugal action.
11.10 A horizontal true centrifugal casting process is used to make aluminum rings with dimensions: L = 5
cm, OD = 65 cm, and ID = 60 cm. (a) Determine the rotational speed that will provide a G-factor
= 60. (b) Suppose that the ring were made out of steel instead of aluminum. If the rotational speed
computed in that problem were used in the steel casting operation, determine the G-factor and (c)
centrifugal force per square meter (Pa) on the mold wall. (d) Would this rotational speed result in a
successful operation?
Solution: (a) Use inside diameter of mold in Eq. (11.5), D = Do = 65 cm. Use g = 981 cm/s2,
N = 30(2g x GF/D).5/= 30(2 x 981 x 60/65).5/= 406.4 rev/min.
(b) Rotational speed would be the same as in part (a) because mass does not enter the computation
of rotational speed. N = 406.4 rev/min
(c) Use 5 cm ring length as basis of area calculations.
Area of this length of mold wall A = DoL = (65 cm)(5 cm) = 1021 cm2 = 0.1021 m2
Volume of cast metal V = (Ro
2 - Ri
2)(L) = ((65/2)2 - (60/2)2)(5.0) = 2454.4 cm3
Density of steel = 7.87 g/cm3
Mass m = (7.87g/cm3)(2454.4 cm3) = 19,315.9 g = 19.316 kg
v = RN/30 Use mean radius R = (65 + 60)/4 = 31.25 cm = 0.3125 m
v = (31.25)(406.4)/30 = 1329.9 cm/s = 13.299 m/s
Centrifugal force per square meter on mold wall = Fc/A where Fc = mv2/R
Fc = (19.316 kg)(13.299 m/s)2/(0.3125 m) = 10,932.1 kg-m/s2
Given that 1 N = 9.81 kg-m/s2, Fc = 10,932.1/9.81 = 1114.4 N
Fc/A = (1114.4 N)/(0.1021 m2) = 10,914.7 N/m2 = 10,914.7 Pa
(d) The G-factor of 60 would probably result in a successful casting operation.
11.11 For the steel ring of preceding Problem 11.10(b), determine the volume of molten metal that must
be poured into the mold, given that the liquid shrinkage is 0.5 percent, and the solidification
shrinkage and solid contraction after freezing can be determined from Table 10.1.
Solution: Volume of final casting V = (Ro
2 - Ri
2)L = (32.52 - 302)(5) = 2454.4 cm3
Given that the molten metal shrinkage = 0.5%, and from Table 10.1, the solidification shrinkage for
steel = 3% and the solid contraction during cooling = 7.2%, the total volumetric contraction is
1 - (1-.005)(1-.03)(1-.072) = 1 - .8957 = 0.1043
The required starting volume of molten metal V = 2454.4/(0.8957) = 2740.2 cm3
11.12 A horizontal true centrifugal casting process is used to make lead pipe for chemical plants. The
pipe has length = 0.5 m, outside diameter = 70 mm, and thickness = 6.0 mm. Determine the
rotational speed that will provide a G-factor = 60.
Solution: D = 70 mm = 0.07 m. g = 9.8 m/s2
N = 30(2g x GF/D).5/= 30(2 x 9.8 x 60/.07).5/= 1237.7 rev/min.
11.13 A vertical true centrifugal casting process is used to make tube sections with length = 10.0 in and
outside diameter = 6.0 in. The inside diameter of the tube = 5.5 in at the top and 5.0 in at the
bottom. At what speed must the tube be rotated during the operation in order to achieve these
specifications?
59
Solution: Use Eq. (11.6) to make the computation of N: N = (30/)(2gL/(Rt
2-Rb
2).5
L = 10 in. = 0.8333 ft
Rt = 5.5/2 = 2.75 in. = 0.22917 ft
Rb = 5.0/2 = 2.50 in. = 0.20833 ft
N = (30/)(2 x 32.2 x .8333/(.229172-.208332).5 = 9.5493(5888).5 = 732.7 rev/min
11.14 A vertical true centrifugal casting process is used to produce bushings that are 200 mm long and
200 mm in outside diameter. If the rotational speed during solidification is 500 rpm, determine the
inside diameter at the top of the bushing if the diameter at the bottom is 150 mm.
Solution: L = 200 mm = 0.2 m. Rb = 150/2 = 75 mm = 0.075 m.
N = (30/)(2gL/(Rt
2-Rb
2).5 = (30/)(2 x 9.8 x 0.2/(Rt
2-.0752).5
N = (30/)(3.92/(Rt
2-.005625)).5 = 500 rev/min
(3.92/(Rt
2-.005625)).5 = 500/30 = 52.36
(3.92/(Rt
2-.005625) = (52.36)2 = 2741.56
Rt
2-.005625 = 3.92/2741.56 = 0.00143
Rt
2 = .005625 + 0.001430 = 0.007055
Rt = 0.007055).5 = .08399 m = 83.99 mm.
11.15 A vertical true centrifugal casting process is used to cast brass tubing that is 15.0 in long and
whose outside diameter = 8.0 in. If the speed of rotation during solidification is 1000 rpm, determine
the inside diameters at the top and bottom of the tubing if the total weight of the final casting = 75.0
lbs.
Solution: For brass, density = 0.313 lb/in3 (Table 11.1).
Volume of casting V = 75.0/.313 = 239.6 in3
Assume the inside wall of the casting is straight from top to bottom (an approximation of the
parabolic shape). The average inside radius Ri = (Rt + Rb)/2
Volume V = (Ro
2 - Ri
2)L = (4.02 - Ri
2)(15.0) = 239.6 in3
(4.02 - Ri
2) = 239.6/15= 5.085
Ri
2 = 16.0 - 5.085 = 10.915 in2 Ri = 3.304 in
Let Rt = Ri + y = 3.304 + y and Rb = Ri - y = 3.304 - y, where y = one-half the difference between
Rt and Rb.
N = (30/)(2gL/(Rt
2-Rb
2).5 = (30/)(2 x 32.2 x 12 x 15/((3.304+y)2-(3.304-y)2)).5
Given N = 1000, thus
1000/30 = (11592/((3.304+y)2-(3.304-y)2)).5
((3.304+y)2-(3.304-y)2).5 = 30(11592).5/1000= 1.02814
(3.3042 + 6.608y + y2 - (3.3042 - 6.608y + y2)).5 = 1.02814
(3.3042 + 6.608y + y2 - 3.3042 + 6.608y - y2).5 = 1.02814
(2 x 6.608y).5 = (13.216y).5 = 1.02814
3.635 (y).5 = 1.02814 y = .080 in.
Rt = 3.304 + 0.080 = 3.384 in. Dt = 6.768 in.
Rb = 3.304 - 0.080 = 3.224 in. Db = 6.448 in.
Defects and Design Considerations
11.16 The housing for a certain machinery product is made of two components, both aluminum castings.
The larger component has the shape of a dish sink and the second component is a flat cover that is
attached to the first component to create an enclosed space for the machine parts. Sand casting is
used to produce the two castings, both of which are plagued by defects in the form of misruns and
cold shuts. The foreman complains that the thickness of the parts are too thin, and that is the
60
reason for the defects. However, it is known that the same components are cast successfully in
other foundries. What other explanation can be given for the defects?
Solution: Misruns and cold shuts result from low fluidity. One possible reason for the defects in
this case is that the thickness of the casting cross-sections is too small. However, given that the
casting of these parts is successfully accomplished at other foundries, two other possible
explanations are: (1) the pouring temperature is too low, and (2) the pouring operation is performed
too slowly.
11.17 A large steel sand casting shows the characteristic signs of penetration defect - a surface
consisting of a mixture of sand and metal. (a) What steps can be taken to correct the defect? (b)
What other possible defects might result from taking each of these steps?
Solution: (a) What are the possible corrective steps? (1) Reduce pouring temperature. (2)
Increase the packing of the mold sand to resist penetration. (3) Treat the mold cavity surface to
make it harder.
(b) What possible defects might result from each of these steps? In the case of step (1), the risk is
for cold shuts and misruns. Steps (2) and (3) would reduce permeability of the sand, thus increasing
the risk of sand blows and pin holes.
61
12 GLASSWORKING
Review Questions
12.1 We have classified glass as a ceramic material; yet glass is different from the traditional and new
ceramics. What is the difference?
Answer. Glass is vitreous - it is in the glassy state, whereas traditional and new ceramics are, by
and large, polycrystalline materials.
12.2 What is the predominant chemical compound in almost all glass products?
Answer. Silica - SiO2.
12.3 What are the three basic steps in the glassworking sequence?
Answer. (1) raw materials preparation and melting, (2) shaping, and (3) heat treatment. Finishing
operations (e.g., grinding, polishing, etching) are performed on some glass products, if needed.
12.4 Melting furnace for glassworking can be divided into four types. Name three of the four types.
Answer. The four types are: (1) pot furnaces, (2) day tanks, (3) continuous tank furnaces, and (4)
electric furnaces.
12.5 Describe the spinning process in glassworking.
Answer. Spinning in glassworking is similar to centrifugal casting in metalworking. A gob of
molten glass is dropped in to a conical mold which spins, causing centrifugal force to spread the
glass upward onto the mold surface.
12.6 What is the main difference between the press-and-blow and the blow-and-blow shaping processes
in glassworking?
Answer. In the press-and-blow process, the initial forming step is pressing of the part, while the
first step in the blow-and-blow process is blowing.
12.7 There are several ways of shaping plate or sheet glass. Name and briefly describe one of them.
Answer. The methods described in this test are: (1) rolling, in which the hot glass is squeezed
between opposing cylindrical rolls; and (2) the float process, in which the melted glass flows onto a
molten tin surface to achieve uniform thickness and smoothness.
12.8 Describe the Danner process?
Answer. In the Danner process, molten glass flows around a rotating hollow mandrel through
which air is blown while the glass is being drawn. The temperature of the air and its volumetric
flow rate as well as the drawing velocity determine the diameter and wall thickness of the tubular
cross-section. During hardening, the glass tube is supported by a series of rollers extending
beyond the mandrel.
12.9 Two processes for forming glass fibers are discussed in the text. Name and briefly describe one
of them.
Answer. The two processes in the text are: (1) drawing, in which fine glass fibers are pulled
through small orifices in a heated plate; and (2) centrifugal spraying, in which molten glass is forced
to flow through small orifices in a rapidly rotating bowl to form glass fibers.
62
12.10 What is the purpose of annealing in glassworking?
Answer. Annealing is performed on glass to remove internal stresses that result from shaping and
solidification.
12.11 Describe how a piece of glass is heat treated to produce tempered glass.
Answer. The glass is heated to a temperature above the annealing temperature and the surfaces
are then quenched by air jets to cool and harden them while the interior of the piece remains
plastic; as the interior cools and contracts, it puts the previously hardened surfaces in compression,
which strengthens it.
12.12 Describe the type of material that is commonly used to make windshields for automobiles.
Answer. Laminated glass, in which two sheets of glass are laminated on either side of a polymer
sheet. This has good impact resistance and does not splinter when broken.
12.13 What are some of the design recommendations for glass parts?
Answer. The guidelines are: (1) subject ceramic parts to compressive, not tensile loads; (2)
ceramics are brittle, so avoid impact loading; (3) use large radii on inside and outside corners; (4)
screw threads should be course. (See Section 14.4.)
Multiple Choice Quiz
There are a total of 10 correct answers in the following multiple choice questions (some questions have
multiple answers that are correct). To attain a perfect score on the quiz, all correct answers must be
given, since each correct answer is worth 1 point. For each question, each omitted answer or wrong
answer reduces the score by 1 point, and each additional answer beyond the number of answers required
reduces the score by 1 point. Percentage score on the quiz is based on the total number of correct
answers.
12.1 Which one of the following terms refers to the glassy state of a material? (a) crystalline, (b)
devitrified, (c) polycrystalline, (d) vitiated, or (e) vitreous.
Answer. (e)
12.2 Besides helping to preserve the environment, the use of recycled glass as an ingredient of the
starting material in glassmaking serves what other useful purpose (one only)? (a) adds coloring
variations to the glass for aesthetic value, (b) makes the glass easier to melt, (c) makes the glass
stronger, or (d) reduces odors in the plant.
Answer. (b)
12.3 The charge in glassworking is which one of the following? (a) the duration of the melting cycle, (b)
the electric energy required to melt the glass, (c) the name given to the melting furnace, or (d) the
starting materials in melting.
Answer. (d)
12.4 Typical glass melting temperatures are in which of the following ranges? (a) 400C to 500C, (b)
900C to 1000C, (c) 1500C to 1600C, or (d) 2000C to 2200C.
Answer. (c)
12.5 Casting is a glassworking process used for high production. (a) True, or (b) false.
Answer. (b) Casting is used in glassworking for large components like giant telescope lenses in
small lot sizes. It is a slow process for these large products.
63
12.6 Which one of the following processes or processing steps is not applicable in glassworking? (a)
annealing, (b) pressing, (c) quenching, (d) sintering, and (e) spinning.
Answer. (d) Sintering is used to cause bonding of particulate materials such as metal and ceramic
powders.
12.7 The press-and-blow process is best suited to the production of (narrow-necked) beverage bottles,
while the blow-and-blow process is more appropria te for producing (wide-mouthed) jars. (a) True,
or (b) false.
Answer. (b) It's the reverse.
12.8 Which one of the following processes is used to produce glass tubing? (a) Danner process, (b)
pressing, (c) rolling, or (d) spinning.
Answer. (a)
12.9 If a glass part with a wall thickness of 5 mm (0.20 in) takes 10 minutes to anneal, how much time
would a glass part of similar geometry but with a wall thickness of 7.5 mm (0.30 in) take to anneal
(choose the closest answer)? (a) 10 minutes, (b) 15 minutes, (c) 20 minutes, or (c) 30 minutes.
Answer. (c) The rule is that annealing time varies as the square of the wall thickness. That would
indicate an annealing time of 0.30/0.202 = 2.25 times 10 minutes or 22.5 minutes. 20 minutes is
closest.
12.10 A lehr is which of the following? (a) a lion's den, (b) a melting furnace, (c) a sintering furnace, (d)
an annealing furnace, or (e) none of the above.
Answer. (d)
64
13 SHAPING PROCESSES FOR PLASTICS
Review Questions
13.1 What are some of the reasons why the plastic shaping processes are important?
Answer. Some of the reasons are: (1) many of the processes are net shape processes; (2) in
general, less energy employed than in metalworking processes; (3) lower temperatures are
required to process plastics than metals or ceramics; (4) great flexibility in geometry; and (5)
painting and other finishing processes generally not required.
13.2 Identify the main categories of plastics shaping processes, as classified by the resulting product
geometry.
Answer. The categories are: (1) extrusion, (2) molding, (3) forming of continuous sheets and films,
(4) fibers, (5) foamed products, and (6) discrete formed sheets and films.
13.3 Viscosity is an important property of a polymer melt in plastics shaping processes. Upon what
parameters does viscosity depend?
Answer. Viscosity of a polymer melt depends on: (1) temperature, and (2) shear rate. Of course,
(3) the molecular weight of the polymer also affects viscosity.
13.4 How does the viscosity of a polymer melt differ from most fluids that are Newtonian.
Answer. A polymer melt exhibits pseudoplasticity, which means that its value decreases with
increasing shear rate.
13.5 Besides viscosity, what other properties of a polymer melt are important in plastics processing?
Briefly define each of the properties identified.
Answer. Other properties include: viscoelasticity, a combination of viscous and elastic properties
which cause the melt to exhibit memory - the tendency to return to its previous shape, as exhibited
by die swell in extrusion.
13.6 Define die swell in extrusion.
Answer. Die swell is the tendency of the extrudate to expand in cross-section dimensions
immediately on exiting the die orifice. It results from the viscoelastic properties of the polymer
melt.
13.7 Briefly describe the plastic extrusion process.
Answer. In plastic extrusion, a polymer melt is compressed to flow through a die orifice and thus
the continuous length of the plastic assumes a cross-sectional shape that is approximately the same
as the profile of the orifice.
13.8 The barrel and screw of an extruder are generally divided into three sections; identify the sections.
Answer. (1) feed section, in which the feed stock is fed from the hopper and heated; (2)
compression section, in which the polymer changes to a viscous fluid; and (3) metering section, in
which pressure is developed to pump the plastic through the die orifice.
13.9 What are the functions of the screen pack and breaker plate at the die end of the extruder barrel?
65
Answer. The functions are: (1) filter dirt and lumps, (2) build pressure, (3) straighten the flow and
remove memory.
13.10 What are the various forms of extruded shapes and corresponding dies?
Answer. (1) solid profiles, such as rounds and L-shapes; (2) hollow profiles, such as tubes; (3)
wire and cable coating; (4) sheet and film; and (5) filaments (continuous fibers).
13.11 What is the distinction between plastic sheet and film?
Answer. Thickness. Sheet stock has a thickness greater than 0.020 in (0.5 mm), while film stock
is less than 0.020 in (0.5 mm) thick.
13.12 What is the blown-film process for producing film stock?
Answer. The blown-film process is a widely used process for making thin polyethylene film for
packaging. It combines extrusion and blowing to produce a tube of thin film. The process begins
with the extrusion of a tube which is immediately drawn upward while still molten and
simultaneously expanded in size by air inflated into it through the die mandrel.
13.13 Describe the calendering process.
Answer. Calendering is a process for producing sheet and film stock out of rubber or rubbery
thermoplastics such as plasticized PVC. In the process, the initial feedstock is passed through a
series of rolls to work the material and reduce its thickness to the desired gage. See Figure 13.17.
13.14 Polymer fibers and filaments are used in several applications; what is the most important
application?
Answer. Textiles.
13.15 Technically, what is the difference between a fiber and a filament?
Answer. A fiber is a long, thin strand of material whose length is at least 100 times its diameter; a
filament is a fiber of continuous length.
13.16 Among the synthetic fiber materials, which are the most important?
Answer. Polyester is the most important commercially, followed by nylon, acrylics, and rayon.
13.17 Briefly describe the injection molding process.
Answer. See beginning of Section 15.6.
13.18 An injection molding machine is divided into two principal components; identify them.
Answer. The components of an injection molding machine are: (1) the injection unit and (2) the
clamping unit.
13.19 What are the two basic types of clamping units?
Answer. The clamping units are: (1) mechanical toggle clamp and (2) hydraulic. In addition, there
are hydromechanical units which combine hydraulic and mechanical actuations.
13.20 Gates in injection molds have several functions; name them.
Answer. The functions of gates in an injection mold are: (1) to increase shear rate to increase
viscosity and temperature of the polymer melt, (2) the thinner cross- section of the gate freezes
more rapidly to seal off the cavity; and (3) parts can be more easily broken off the runner at the
gate.
66
13.21 What are the advantages of a three-plate mold over a two- plate mold in injection molding?
Answer. As the mold opens, the three-plate mold automatically separates the molded part(s) from
the runner system.
13.22 Discuss some of the defects that can occur in plastic injection molding.
Answer. The defects include: short shots, in which the polymer melt solidifies before filling the
cavity; flashing, in which the polymer melt is squeezed into the parting surfaces between the mold
halves and around ejection pins; sink marks, in which the surface is drawn into the molding by
contraction of internal material; and weld lines where the melt has flowed around a core or
13.23 Describe structural foam molding.
Answer. Structural foam molding is an injection molding process in which a gas or gas-producing
ingredient is mixed with the polymer melt prior to injection into the mold cavity; this results in the
part having a tough outer skin surrounded by a foam core.
13.24 What are the significant differences in the equipment and operating procedures between injection
molding of thermoplastics and injection molding of thermosets?
Answer. The differences in injection molding of thermosets are: (1) shorter barrel length, (2) lower
temperatures in the barrel, these first two reasons to prevent premature curing; and (3) use of a
heated mold to cause cross-linking of the TS polymer.
13.25 What is reaction injection molding?
Answer. Reaction injection molding involves the mixing of two highly reactive liquid ingredients
and immediately injecting the mixture into a mold cavity where chemical reactions leading to
solidification occur. The two ingredients form the components used in catalyst-activated or
mixing-activated thermoset systems (Section 8.3.1).
13.26 What kinds of products are produced by blow molding?
Answer. Blow molding is used to produce hollow, seamless containers, such as bottles.
13.27 What is the starting material form in thermoforming?
Answer. Thermoforming starts with a thermoplastic sheet or film.
13.28 What is the difference between a positive mold and a negative mold in thermoforming?
Answer. A positive mold has a convex shape; a negative mold has a concave cavity.
13.29 Why are the molds generally more costly in mechanical thermoforming than in pressure or vacuum
thermoforming?
Answer. In mechanical thermoforming, matching mold halves are required; while in other
thermoforming processes, only one mold form is required.
13.30 What are the processes by which polymer foams are produced?
Answer. There are several foaming processes: (1) mechanical agitation - mixing a liquid resin with
air by, then hardening the polymer by means of heat or chemical reaction; (2) mixing a physical
blowing agent with the polymer - a gas such as nitrogen (N2) or pentane (C5H12) which can be
dissolved in the polymer melt under pressure, so that the gas comes out of solution and expands
when the pressure is subsequently reduced; and (3) mixing the polymer with chemical compounds,
called chemical blowing agents, that decompose at elevated temperatures to liberate gases such as
CO2 or N2 within the melt.
67
13.31 What are some of the general considerations that product designers must keep in mind when
designing components out of plastics?
Answer. Some of the general considerations are : (1) Plastics are not as strong or stiff as metals
and should not be used in applications where high stresses will be encountered. (2) Impact
resistance of plastics is general good, better than many ceramics. (3) Service temperatures of
plastics are limited relative to engineering metals and ceramics. (4) Thermal expansion is greater
for plastics than metals; so dimensional changes due to temperature variations are much more
significant than for metals. (5) Many types of plastics degrade from sunlight and certain other
forms of radiation. Also, some plastics degrade in oxygen and ozone atmospheres. Finally, plastics
are soluble in many common solvents.
Multiple Choice Quiz
There are a total of 36 correct answers in the following multiple choice questions (some questions have
multiple answers that are correct). To attain a perfect score on the quiz, all correct answers must be
given, since each correct answer is worth 1 point. For each question, each omitted answer or wrong
answer reduces the score by 1 point, and each additional answer beyond the number of answers required
reduces the score by 1 point. Percentage score on the quiz is based on the total number of correct
answers.
13.1 The shear viscosity of a polymer melt is affected by which of the following (more than one)? (a)
degree of polymerization, (b) polymer type, (c) rate of flow, (d) temperature.
Answer. (a), (b), (c), and (d). Degree of polymerization (a) is correlated with molecular weight,
and rate of flow (c) is related to shear rate. The other parameters, (b) and (d), are probably more
recognizable to the reader as correct answers to this question.
13.2 The forward movement of polymer melt in an extruder barrel is resisted by drag flow, which is
caused by the resistance to flow through the die orifice: (a) true or (b) false.
Answer. (b) Drag flow is the forward motion of the melt caused by the Archimedian screw
principle in the barrel. The resistance to forward flow is called back pressure flow.
13.3 Which three of the following are sections of a conventional extruder barrel for thermoplastics? (a)
compression section, (b) die section, (c) feed section, (d) heating section, (e) metering section, and
(f) shaping section.
Answer. (a), (c), and (e)
13.4 Which of the following processes is not associated with the production of plastic sheet and film
(more than one)? (a) blown-film extrusion process, (b) calendering, (c) chill-roll extrusion, (d)
doctor blade method, or (e) slit-die extrusion.
Answer. (d) Calendering is more closely associated with rubber coating of materials such as
textiles, but should be included in our list of plastic sheet and film-making processes.
13.5 Spinning in the production of synthetic fibers refers to which of the following: (a) extrusion of
polymer melt through small die openings, (b) drawing the strands to elongate and thin them, (c) both
of the above, or (d) none of the above.
Answer. (c)
13.6 The principal components of an injection molding machine are which two of the following? (a)
clamping unit, (b) hopper, (c) injection unit, (d) mold, and (e) part ejection unit.
Answer. (a) and (c)
68
13.7 The parting line in injection molding is which one of the following: (a) the lines formed where
polymer melt meets after flowing around a core in the mold, (b) the narrow gate sections where
the parts are separated from the runner, (c) where the clamping unit is joined to the injection unit in
the molding machine, (d) where the two mold halves come together, (e) none of the above.
Answer. (d)
13.8 The function of the ejection system is to (one best answer): (a) move polymer melt into the mold
cavity, (b) open the mold halves after the cavity is filled, (c) remove the molded parts from the
runner system after molding, (d) separate the part from the cavity after molding, (e) none of the
above.
Answer. (d)
13.9 A three-plate mold offers which of the following advantages when compared to a two-plate mold
(more than one)? (a) automatic separation of parts from runners, (b) gating is usually at the base of
the part to reduce weld lines, (c) sprue does not solidify, (d) stronger molded parts, (e) none of the
above.
Answer. (a) and (b)
13.10 Which of the following defects or problems is associated with injection molding (more than one)?
(a) bambooing, (b) die swell, (c) drag flow, (d) flash, (e) melt fracture, (f) short shots, or (g) sink
marks.
Answer. (d), (f), and (g)
13.11 In rotational molding, centrifugal force is used to force the polymer melt against the surfaces of the
mold cavity where solidification occurs: (a) true or (b) false.
Answer. (b) It is the force of gravity in the doubly rotating mold that forces the polymer against
the mold surfaces.
13.12 Use of a parison is associated with which one of the following plastic shaping processes? (a)
bi-injection molding, (b) blow molding, (c) compression molding, (d) pressure thermoforming, or (e)
sandwich molding.
Answer. (b)
13.13 A thermoforming mold with a convex form is called which one of the following (may be more than
one)? (a) a die, (b) a negative mold, (c) a positive mold, or (d) a three-plate mold.
Answer. (c)
13.14 The term encapsulation refers to which one of the following plastics shaping processes? (a)
casting, (b) compression molding, (c) extrusion of hollow forms, (d) injection molding in which a
metal insert is encased in the molded part, or (e) vacuum thermoforming using a positive mold.
Answer. (a)
13.15 Which of the following terms applies to the processing of foam plastics (more than one)? (a)
chemical blowing agents, (b) open cell structure, (c) powder injection molding, (d) sandwich
molding, (e) structural foam molding, (f) all of the above.
Answer. (a), (b), (d), and (e).
13.16 The two most common polymer foams are which of the following? (a) polyacetal, (b) polyethylene,
(c) polystyrene, (d) polyurethane, and (e) polyvinylchloride.
69
Answer. (c) and (d)
13.17 In which of the following property categories do plastic parts compare favorably with metals (more
than one)? (a) impact resistance, (b) resistance to ultraviolet radiation, (c) stiffness, (d) strength,
(e) strength-to- weight ratio, or (f) temperature resistance.
Answer. (a) and (e).
13.18 Which of the following processes are generally limited to thermoplastic polymers (more than one)?
(a) blow molding, (b) compression molding, (c) reaction injection molding, (d) thermoforming, (e)
transfer molding, (f) wire coating.
Answer. (a) and (d).
13.19 Which of the following processes would be applicable to produce hulls for small boats (more than
one)? (a) blow molding, (b) compression molding, (c) injection molding, (d) rotational molding, or (e)
vacuum thermoforming.
Answer. (a), (d), and (e).
Problems
Extrusion
13.1 The diameter of an extruder barrel is 65 mm and its length = 1.75 m. The screw rotates at 55
rev/min. The screw channel depth = 5.0 mm, and the flight angle = 18. The head pressure at the
die end of the barrel is 5.0 x 106 Pa. The viscosity of the polymer melt is given as 100 Pas. Find the
volume flow rate of the plastic in the barrel.
Solution: Qd = 0.52(65x10-3)2(55/60)(5x10-3)sin 18 cos 18 = 95,560x10-9(0.3090)(0.9510)
= 28.081 x 10-6 m3/s
p = 5 MPa = 5x106 n/m2
Qb = (5x106)(65x10-3)(5x10-3)3(sin 18)2/12(100)(1.75) = 5.804 x 10-6 m3/s
Qx = 28.081 - 5.804 = 22.277 x 10-6 m3/s.
13.2 An extruder barrel has a diameter of 120 mm and a length = 3.0 m. The screw channel depth = 8.0
mm, and its pitch = 95 mm. The viscosity of the polymer melt is 75 Pas, and the head pressure in
the barrel is 4.0 MPa. What rotational speed of the screw is required to achieve a volumetric flow
rate of 90 cm3/s?
Solution: A = tan-1(95/120) = 14.14
Qd = 0.52(.12)2(N)(8x10-3)sin 14.14 cos 14.14 = 0.5685x10-3(0.2444)(0.9697)
= 134.73 N x 10-6 m3/s
Qb = (4x106)(.12)(8x10-3)3(sin 14.14)2/12(75)(3.0) = 26.66 x 10-6 m3/s
Qx = Qd - Qb = 157.5 N x 10-6 - 26.66 x 10-6 = 90 x 10-6 m3/s
134.73 N = 90 + 26.66 = 116.66
N = 116.66/134.73 = 0.8659 rev/s = 51.95 rev/min.
13.3 An extruder has diameter = 80 mm and length = 2.0 m. Its screw has a channel depth = 5 mm,
flight angle = 18 degrees, and it rotates at 1 rev/sec. The plastic melt has a shear viscosity = 150
Pas. Determine the extruder characteristic by computing Qmax and pmax and then finding the
equation of the straight line between them.
Solution: Qmax = Qd = 0.52(.08)2(1)(5x10-3)sin 18 cos 18 = 0.158 x 10-3(0.3090)(0.9510)
= 46.4 x 10-6 m3/s
pmax = 6(.08)(1)(2)(150)(cot 18)/(5x10-3)2 = 452.4(3.077)/25x10-6 = 55 x 106 Pa = 55 MPa
70
Qx = 46.4 x 10-6 - (46.4x10-6/55)p
Qx = 46.4 x 10-6 - 0.8436 x 10-6 p, where p has units of MPa
13.4 Determine the helix angle A such that the screw pitch p is equal to the screw diameter D. This is
called the "square" angle in plastics extrusion - the angle that provides a flight advance equal to one
diameter for each rotation of the screw.
Solution: Assume flight land = zero.
From Eq. (15.4), tan A = pitch/D
If pitch = D, then A = tan-1(1/) = 17.66°
13.5 An extruder barrel has a diameter of 2.5 in. The screw rotates at 60 rev/min; its channel depth =
0.20 in, and its flight angle = 17.5. The head pressure at the die end of the barrel is 800 lb/in2 and
the length of the barrel is 50 in. The viscosity of the polymer melt is 122 x 10-4 lb-sec/in2.
Determine the volume flow rate of the plastic in the barrel.
Solution: Qd = 0.52(2.5)2(1)(.2)sin 17.5 cos 17.5 = 0.5(12.337)(0.3007)(0.9537) = 1.769 in3/sec
Qb = (800)(2.5)(.2)3(sin 17.5)2/12(122x10-4)(50) = 0.621 in3/sec
Qx = 1.769 - 0.621 = 1.148 in3/sec.
13.6 An extruder barrel has a diameter of 4.0 in and an L/D ratio of 28. The screw channel depth =
0.25 in, and its pitch = 4.8 in. It rotates at 60 rev/min. The viscosity of the polymer melt is 100 x
10-4 lb-sec/in2. What head pressure is required to obtain a volume flow rate = 150 in3/min?
Solution: A = tan-1(pitch/D) = tan-1(4.8/4) = 20.9
Qd = 0.52(4)2(1)(.25)sin 20.9 cos 20.9 = 19.74(0.3567)(0.9342) = 6.578 in3/sec = 394.66 in3/min
Qx = Qd - Qb = 394.66 - Qd = 150
Qb = 394.66 - 150 = 244.66 in3/min = 4.078 in3/sec
L = 4(28) = 112 in.
Qb = p(4)(.25)3(sin 20.9)2/12(100x10-4)(112) = 4.078
0.0018592 p = 4.078
p = 2193.4 lb/in2
13.7 An extrusion operation produces continuous tubing with outside diameter = 2.0 in and inside
diameter = 1.7 in. The extruder barrel has a diameter = 4.0 in and length = 10 ft. The screw rotates
at 50 rev/min; it has a channel depth = 0.25 in and flight angle = 16. The head pressure has a
value of 350 lb/in2 and the viscosity of the polymer melt is 80 x 10-4 lb-sec/in2. Under these
conditions, what is the production rate in length of tube/min, assuming the extrudate is pulled at a
rate that eliminates the effect of die swell (i.e., the tubing has the same OD and ID as the die
profile).
Solution: Qd = 0.52(4)2(50/60)(.25)sin 16 cos 16 = 16.45(0.2756)(0.9613) = 4.358 in3/sec
Qb = (350)(4)(.25)3(sin 16)2/12(80x10-4)(120) = 0.453 in3/sec
Qx = 4.358 - 0.453 = 3.905 in3/sec.
Ax = 0.25(22 - 1.72) = 0.872 in2
vx = 3.905/0.872 = 4.478 in/sec = 22.39 ft/min.
13.8 An extruder has barrel diameter and length of 100 mm and 2.8 m, respectively. The screw
rotational speed = 50 rev/min, channel depth = 7.5 mm, and flight angle = 17. The plastic melt has
a shear viscosity = 175 Pas. Determine: (a) the extruder characteristic, (b) the shape factor Ks for
a circular die opening with diameter = 3.0 mm and length = 12.0 mm, and (c) the operating point (Q
and p).
Solution: Qmax = Qd = 0.52(.1)2(50/60)(7.5x10-3)sin 17 cos 17 = 308.4 x 10-6(0.2924)(0.9563)
71
= 86.2 x 10-6 m3/s
pmax = 6(.1)(50/60)(2.8)(175)(cot 17)/(7.5x10-3)2 = 44.75 x 106 Pa =44.75 MPa
Qx = 86.2 x 10-6 - 1.926 x 10-12 p, where p has units of Pa
(b) Given: Dd = 3 mm, Ld = 12 mm.
Ks = (3 x 10-3)4/128(175)(12 x 10-3) = 0.9467 x 10-12
(c) 0.9467 x 10-12 p = 86.2 x 10-6 - 1.926 x 10-12 p
2.8727 x 10-12 p = 86.2 x 10-6
p = 30.0 x 106 Pa = 30 MPa
Qx = 0.9467 x 10-12 (30 x 106) = 28.4 x 10-6 m3/s
Check with extruder characteristic: Qx = 86.2 x 10-6 - 1.926 x 10-12 (30 x 106) = 28.4 x 10-6 m3/s.
13.9 Consider an extruder in which the barrel diameter = 4.5 in and length = 11 ft. The extruder screw
rotates at 60 rev/min; it has channel depth = 0.35 in and flight angle = 20. The plastic melt has a
shear viscosity = 125 x 10-4 lb-sec/in2. Determine: (a) Qmax and pmax, (b) the shape factor Ks for a
circular die opening in which Dd = 0.312 in and Ld = 0.75 in, and (c) the values of Q and p at the
operating point.
Solution: (a) Qmax = 0.52(4.5)2(1)(.35)sin 20 cos 20 = 34.975(0.342)(0.9397) = 11.24 in3/sec
pmax = 6(4.5)(1)(132)(.0125)(cot 20)/(0.35)2 = 3139 lb/in2
(b) Given: Dd = 0.312 in., Ld = 0.75 in.
Ks = (.312)4/128(.0125)(.75) = 0.024808
(c) From (a), Qx = Qmax - (Qmax/pmax)p = 11.24 - 0.003581p
From (b), Qx = 0.024808p
Combining, .024808p = 11.24 - .003581p
.02839p = 11.24 p = 395.9 lb/in2
Qx = 11.24 - 0.003581(395.9) = 9.82 in3/sec
13.10 An extruder has a barrel diameter = 5.0 in and length = 12 ft. The extruder screw rotates at 50
rev/min; it has channel depth = 0.30 in and flight angle = 17.7. The plastic melt has a shear
viscosity = 100 x 10-4 lb- sec/in2. Find: (a) the extruder characteristic, (b) the values of Q and p at
the operating point, given that the die characteristic is Qx = 0.00150 p.
Solution: (a) Qmax = 0.52(5)2(50/60)(.3)sin 17.7 cos 17.7 = 30.84(0.3040)(0.9527) = 8.93 in3/sec
pmax = 6(5)(50/60)(144)(.01)(cot 17.7)/(0.3)2 = 3937.6 lb/in2
Qx = Qmax - (Qmax/pmax)p = 8.93 - 0.002268p
(b) Given: die characteristic Qx = 0.0015p
Qx = 8.93 - 0.002268p = 0.0015p
0.00377p = 8.93 p = 2370 lb/in2
Qx = 8.93 - 0.002268(2370) = 3.55 in3/sec
13.11 An extruder has a barrel diameter = 4.0 in and length = 5.0 ft. The extruder screw rotates at 80
rev/min. It has a channel with depth = 0.15 in and flight angle = 20. The polymer melt has a shear
viscosity = 60 x 10-4 lb- sec/in2 at the operating temperature of the process. The specific gravity of
the polymer is 1.2. (a) Find the equation for the extruder characteristic. If a T-shaped cross-section
is extruded at a rate of 0.13 lb/sec, determine: (b) the operating point (Q and p), and (c) the die
characteristic that is indicated by the operating point.
Solution: (a) Qmax = 0.52(4)2(80/60)(.15)sin 20 cos 20 = 15.79(0.3420)(0.9397) = 5.075 in3/sec
pmax = 6(4)(80/60)(60)(.006)(cot 20)/(0.15)2 = 1104.8 lb/in2
72
Qx = Qmax - (Qmax/pmax)p = 5.075 - 0.004593p
(b) Given: T-shaped cross section extruded at 0.13 lb/sec.
Density of polymer = 1.2(62.5 lb/ft3)/(12 in3/ft3) = 0.0433 lb/in3
Qx = 0.13/0.0433 = 3.0 in3/sec.
3.0 = 5.075 - 0.004593p
0.004593p = 5.075 - 3.0 = 2.075
p = 451.8 lb/in2
(c) Qx = Ks p
Ks = Qx/p = 3.0/451.8 = 0.00664
Qx = 0.00664 p
Injection Molding
13.12 Compute the percentage volumetric contraction of a polyethylene molded part, based on the value
of shrinkage given in Table 13.1.
Solution: 15.12 S = 0.025 for polyethylene from Table 13.1.
Volumetric contraction = 1.0 - (1 - .025)3 = 1.0 - 0.92686 = 0.07314 = 7.314%
Note that we are not using the parameter S from Table 13.1 in the way it was intended to be used.
Its intended use is to compute the oversized dimension of a mold cavity in injection molding.
Instead, we are using the shrinkage term to calculate the amount of (volumetric) reduction in size
of the part after the polymer is injected into the cavity. In fact, a slightly different shrinkage
parameter value may apply in this case.
13.13 The specified dimension = 100.00 mm for a certain injection molded part made of nylon-6,6.
Compute the corresponding dimension to which the mold cavity should be machined, using the
value of shrinkage given in Table 13.1.
Solution: S = 0.020 for nylon-6,6 from Table 13.1.
Dc = 100.0 + 100(0.020) + 100(0.020)2 = 100 + 2.02 + 0.04 = 102.04 mm.
13.14 The part dimension for a certain injection molded part made of polycarbonate is specified as 3.75
in. Compute the corresponding dimension to which the mold cavity should be machined, using the
value of shrinkage given in Table 13.1.
Solution: S = 0.007 for polycarbonate from Table 13.1.
Dc = 3.75 + 3.75(0.007) + 3.75 (0.007)2 = 3.75 + 0.0263 + 0.0002 = 3.7765 in.
13.15 The foreman in the injection molding department says that a polyethylene part produced in one of
the operations has greater shrinkage than the calculations indicate it should have. The important
dimension of the part is specified as 112.5 0.25 mm. However, the actual molded part measures
112.02 mm. (a) As a first step, the corresponding mold cavity dimension should be checked.
Compute the correct value of the mold dimension, given that the shrinkage value for polyethylene is
0.025 (from Table 13.1). (b) What adjustments in process parameters could be made to reduce the
amount of shrinkage.
Solution: (a) Given: S = 0.025, Dc = 112.5 + 112.5(.025) + 112.5(.025)2 = 115.383 mm
(b) Adjustments to reduce shrinkage include: (1) increase injection pressure, (2) increase
compaction time, and (3) increase molding temperatures.
73
Other Molding Operations and Thermoforming
13.16 The extrusion die for a polyethylene parison used in blow molding has a mean diameter = 16.0 mm.
The size of the ring opening in the die = 1.5 mm. The mean diameter of the parison is observed to
swell to a size of 20.5 mm after exiting the die orifice. If the diameter of the blow molded container
is to be 100 mm, determine (a) the corresponding wall thickness of the container and (b) the wall
thickness of the parison.
Solution: (a) rsd = 20.5/160 = 1.281
tm = (1.281)2 (1.5)(16.0)/100.0 = 0.505 mm
(b) tp = (1.281)2(1.5) = 2. 462 mm
13.17 A blow molding operation produces a 6.25 in diameter bottle from a parison that is extruded in a die
whose outside diameter = 1.25 in and inside diameter = 1.00 in. The observed diameter swell ratio
= 1.24. What is the maximum air pressure that can be used if the maximum allowable tensile stress
for the polymer is 1000 lb/in2.
Solution: Mean extrusion die diameter Dd = (1.25 - 1.00)/2 = 1.125 in.
and wall thickness td = (1.25 - 1.0)/2 = 0.125 in.
Blow molding wall thickness tm = (1.24)3(.125)(1.125)/6.25 = 0.0429 in.
Maximum air pressure p = 2(1000)(0.0429)/6.25 = 13.73 lb/in2 .
13.18 A parison is extruded from a die with outside diameter = 11.5 mm and inside diameter = 7.5 mm.
The observed die swell is 1.25. The parison is used to blow mold a beverage container whose
outside diameter = 112 mm (a standard size 2-liter soda bottle). (a) What is the corresponding wall
thickness of the container? (b) Obtain an empty 2-liter plastic soda bottle and (carefully) cut it
across the diameter. Using a micrometer, measure the wall thickness to compare with your answer
in (a).
Solution: (a) Dd = (11.5 + 7.5)/2 = 9.5 mm, and td = (11.5 - 7.5)/2 = 2.0 mm
tm = (1.25)3(2.0)(9.5)/112 = 0.331 mm (= 0.013 in.)
(b) Measured value should be close to calculated value. Some wall thickness are less.
13.19 An extrusion operation is used to produce a parison whose mean diameter = 27 mm. The inside
and outside diameters of the die that produced the parison are 18 mm and 22 mm, respectively. If
the minimum wall thickness of the blow molded container is to be 0.40 mm, what is the maximum
possible diameter of the blow mold?
Solution: Dd = (22 + 18)/2 = 20 mm, and td = (22 - 18)/2 = 2 mm.
rsd = 27/20 = 1.35
Rearranging Eq. (15.25) in text, Dm = rsd
3tdDd/tm = (1.35)3(2)(20)/.40 = 246 mm.
13.20 A rotational molding operation is to be used to mold a hollow playing ball out of polyethylene. The
ball will be 1.5 ft in diameter and its wall thickness should be 1/16 in. What weight of PE powder
should be loaded into the mold in order to meet these specifications? The specific gravity of the PE
grade is 0.95.
Solution: Density = 0.95(62.4/1728) = 0.0343 lb/in3
Volume = (Do
3 - Di
3)/6 = 0.16667[(1.5x12)3 - (1.5x12 - 2/16)3] = 63.18 in3
Weight = (63.18)(0.0343) = 2.17 lb.
13.21 The problem in a certain thermoforming operation is that there is too much thinning in the walls of
the large cup- shaped part. The operation is conventional pressure thermoforming using a positive
mold, and the plastic is an ABS sheet with an initial thickness of 3.2 mm. (a) Why is thinning
74
occurring in the walls of the cup? (b) What changes could be made in the operation to correct the
problem?
Solution: (a) As the starting flat sheet is draped over the convex cup-shaped mold, the portion
contacting the base of the cup experiences little stretching. However, the remaining portions of the
sheet must be stretched significantly to conform to the sides of the cup. Hence, thinning in these
sides results.
(b) The problem could be solved by either: (1) fabricating a negative mold to replace the current
positive mold, since a negative mold will distribute the material more uniformly and result in
approximately equal thinning throughout the sheet; or (2) prestretch the sheet as in Figure 13.38 in
the text.
75
14 RUBBER PROCESSING TECHNOLOGY
Review Questions
14.1 How is the rubber industry organized?
Answer. The rubber industry is organized into three parts: (1) rubber growing plantations produce
natural rubber, (2) the petrochemical industry produces synthetic rubber, and (3) fabricators take
the NR and SR and produce finished rubber goods.
14.2 How is raw rubber recovered from the latex that is tapped from a rubber tree?
Answer. The rubber is usually recovered as follows: (1) the latex is collected into tanks and diluted
to half natural concentration; (2) formic or acetic or other acid is added to the solution which
causes the rubber to coagulate; (3) the coagulum is then squeezed through rolls to drive off water;
and (4) the resulting sheets are dried in smokehouses for several days. The resulting raw rubber is
called ribbed smoked sheet.
14.3 What is the sequence of processing steps required to produce finished rubber goods?
Answer. The typical sequence is: (1) production of the raw rubber, (2) compounding, (3) mixing,
(4) shaping, and (5) vulcanization.
14.4 What are some of the functions of the additives that are combined with rubber during
compounding?
Answer. The additives and functions are: vulcanizing chemicals, reinforcing fillers, extenders to
reduce cost, antioxidants, coloring pigments, plasticizers to soften the rubber, and blowing agents to
make foam rubber.
14.5 Name the four basic categories of processes used to shape rubber.
Answer. The categories are: (1) extrusion, (2) calendering, (3) coating, and (4) molding.
14.6 What are some of the operations used to coat rubber onto a fabric to produce reinforced rubber?
Answer. Calendering, skimming, dipping, and spraying; see Article 14.1.4.
14.7 What does vulcanization do to the rubber?
Answer. Vulcanization causes cross-linking of the rubber molecules; this strengthens and stiffens
the rubber while extensibility is retained.
14.8 Name the three basic tire constructions and briefly identify the differences in their construction.
Answer. (a) diagonal ply, (b) belted bias, and (c) radial ply. Diagonal ply and belted bias both have
their carcass plys running in a diagonal direction relative to the tire circumference; radial ply has its
carcass plies running in a radial direction; belted bias and radial ply tires use belts, which are
additional plies around the outside circumference of the tire; whereas diagonal ply tires do not have
these belts.
14.9 What are the three basic steps in the manufacture of a pneumatic tire?
Answer. The three steps are: (1) preform the components, (2) building the carcass and adding the
rubber for the sidewall and treads, and (3) molding and curing.
14.10 What is the purpose of the bead coil in a pneumatic tire?
76
Answer. The bead coil provides a rigid support for the tire when it is mounted onto the wheel rim.
14.11 What is a TPE?
Answer. TPE stands for thermoplastic elastomer; it is a thermoplastic polymer that behaves like a
rubber.
14.12 Many of the design guidelines that are applicable to plastics are also applicable to rubber.
However, the extreme flexibility of rubber results in certain differences. What are some examples
of these differences?
Answer. Three examples: (1) no draft is needed on the part for mold removal; (2) holes should be
molded into rubber parts rather than machined, whereas holes can be machined or molded in a
plastic part; and (3) screw threads are not normally used on rubber parts.
Multiple Choice Quiz
There are a total of 11 correct answers in the following multiple choice questions (some questions have
multiple answers that are correct). To attain a perfect score on the quiz, all correct answers must be
given, since each correct answer is worth 1 point. For each question, each omitted answer or wrong
answer reduces the score by 1 point, and each additional answer beyond the number of answers required
reduces the score by 1 point. Percentage score on the quiz is based on the total number of correct
answers.
14.1 The most important rubber product is: (a) footwear, (b) conveyor belts, (c) pneumatic tires, or (d)
tennis balls.
Answer. (c)
14.2 The chemical name of the ingredient recovered from the latex of the rubber tree is which one of
the following? (a) polybutadiene, (b) polyisobutylene, (c) polyisoprene, or (d) polystyrene.
Answer. (c)
14.3 Of the following rubber additives, which one would rank as the single most important? (a)
antioxidants, (b) carbon black, (c) clays and other hydrous aluminum silicates, (d) plasticizers and
softening oils, or (e) reclaimed rubber.
Answer. (b)
14.4 Which one of the following molding processes is the most important in the production of products
made of conventional rubber? (a) compression molding, (b) injection molding, (c) thermoforming, or
(d) transfer molding.
Answer. (a)
14.5 Which of the following ingredients do not contribute to the vulcanizing process (more than one)?
(a) calcium carbonate, (b) carbon black, (c) stearic acid, (d) sulfur, and (e) zinc oxide.
Answer. (a) and (b)
14.6 How many minutes are required to cure (vulcanize) a modern passenger car tire? (a) 5, (b) 15, (c)
25, or (d) 45.
Answer. (b)
14.7 When is the tread pattern imprinted onto the circumference of the tire? (a) during preforming, (b)
while building the carcass, (c) during molding, or (d) during curing.
77
Answer. (c)
14.8 Which of the following are not normally used in the processing of thermoplastic elastomers (more
than one)? (a) blow molding, (b) compression molding, (c) extrusion, (d) injection molding, or (e)
vulcanization.
Answer. (b) and (e)
14.9 Screw threads are not normally molded into rubber parts: (a) true or (b) false.
Answer. (a) Screw threads are not normally designed into rubber parts because of the extreme
flexibility of rubber.
78
15 SHAPING PROCESSES FOR POLYMER MATRIX
COMPOSITES
Review Questions
15.1 What are the principal polymers used in fiber-reinforced polymers?
Answer. Principal polymer matrices in FRPs are unsaturated polyesters and epoxies.
15.2 What is the difference between a roving and a yarn?
Answer. A roving consists of untwisted filaments, while a yarn consists of twisted fibers.
15.3 In the context of fiber reinforcement, what is a mat?
Answer. A mat is a felt consisting of randomly oriented fibers held loosely together in a binder.
15.4 Why do we say that particles and flakes are members of the same basic class of reinforcing
material?
Answer. Flakes are simply particles that possess very low width-to-thickness ratios.
15.5 What is sheet molding compound (SMC)?
Answer. SMC consists of TS polymer resin, fillers, and chopped glass fibers, all rolled into a sheet
of typical thickness = 0.250 inch.
15.6 How is a prepreg different from a molding compound?
Answer. Prepregs have continuous fibers rather than chopped fibers as in molding compounds.
15.7 Why are laminated FRP products made by the spray-up method not as strong as similar products
made by hand lay-up?
Answer. Because in hand lay-up, orientation of the fibers is controlled; whereas in spray-up, the
fibers in each layer are randomly oriented.
15.8 What is the difference between the wet lay-up approach and the prepreg approach in hand lay-up?
Answer. In wet lay-up, the layer of fiber reinforcement is placed into the mold dry, and the
uncured resin is then applied to it to form the composite laminate. In the prepreg approach, layers
of fiber preimpregnated with resin are laid into the mold.
15.9 What is an autoclave?
Answer. An autoclave is an enclosed chamber which can supply heat and/or pressure at
controlled levels.
15.10 What are some of the distinguishing characteristics of the closed mold processes for PMCs?
Answer. Characteristics include: (1) they use molds consisting of two halves that open and close
during the molding cycle; (2) high pressure is generally associated with the process; and (3) the
geometric shapes of the moldings are more complex in three dimensions.
15.11 Identify some of the different forms of PMC molding compounds.
Answer. PMC molding compounds include SMCs, TMCs, and BMCs.
79
15.12 What is preform molding?
Answer. Preform molding is a compression molding process in which a precut mat is placed into
the lower half of a mold together with a charge of thermosetting resin; the materials are then
pressed between heated molds to cure the resin and produce a fiber-reinforced molding.
15.13 Describe reinforced reaction injection molding (RRIM).
Answer. RRIM involves the injection of resins that cure by chemical reaction together with
reinforcing fibers into a closed mold. The resulting part is a fiber-reinforced (usually glass fiber)
plastic molding.
15.14 What is filament winding?
Answer. Filament winding is a process in which resin- impregnated continuous fibers are wrapped
around a rotating mandrel with the internal shape of the FRP product; the resin is cured and the
mandrel is removed.
15.15 What is the advantage of computer numerical control over mechanical control in filament winding?
Answer. CNC allows independent control over mandrel rotation and carriage speed for greater
flexibility in relative motions.
15.16 Describe the pultrusion process.
Answer. Pultrusion is a process in which continuous fibers are dipped into a resin and pulled
through a shaping die (somewhat like an extrusion die) where the resin cures. The resulting
sections are similar to extruded parts except that they are reinforced with continuous fibers.
15.17 How does pulforming differ from pultrusion?
Answer. Pulforming is pultrusion with the added operation of a shape change in the length (straight
length becomes curved) and cross-section (different cross-sections throughout the length).
15.18 With what kinds of products is tube rolling associated?
Answer. Typical products include: bicycle frames and space trusses.
15.19 How are FRPs cut?
Answer. Uncured FRPs are cut by methods which include: knives, scissors, power shears,
steel-rule blanking dies, laser beam cutting, and water jet cutting. Cured FRPs are cut by WC and
HSS cutting tools, diamond cutting tools, and water jet cutting.
Multiple Choice Quiz
There are a total of 12 correct answers in the following multiple choice questions (some questions have
multiple answers that are correct). To attain a perfect score on the quiz, all correct answers must be
given, since each correct answer is worth 1 point. For each question, each omitted answer or wrong
answer reduces the score by 1 point, and each additional answer beyond the number of answers required
reduces the score by 1 point. Percentage score on the quiz is based on the total number of correct
answers.
15.1 Which one of the following is the most common polymer type in fiber-reinforced polymer
composites? (a) elastomers, (b) thermoplastics, or (c) thermosets.
Answer. (c)
80
15.2 Most rubber products are properly classified into which of the following categories (more than
one)? (a) elastomer reinforced with carbon black, (b) fiber- reinforced composite, (c)
particle -reinforced composite, (d) polymer matrix composite, (e) pure elastomer, and (f) pure
polymer.
Answer. (a), (c), and (d).
15.3 Hand lay-up is classified in which of the following general categories of PMC shaping processes
(more than one)? (a) closed mold process, (b) compression molding, (c) contact molding, (d)
filament winding, or (e) open mold process.
Answer. (c) and (e).
15.4 A positive mold with a smooth surface will produce a good finish on which surface of the laminated
product in the hand lay-up method? (a) inside surface or (b) outside surface.
Answer. (a)
15.5 SMC molding is a form of which one of the following? (a) compression molding, (b) contact
molding, (c) injection molding, (d) open mold processing, (e) pultrusion, or (f) transfer molding.
Answer. (a)
15.6 Filament winding involves the use of which one of the following fiber reinforcements? (a)
continuous filaments, (b) fabrics, (c) mats, (d) prepregs, (e) short fibers, or (f) woven rovings.
Answer. (a)
15.7 In filament winding, when the continuous filament is wound around the cylindrical mandrel at a
helix angle close to 90, it is called which of the following (one best answer)? (a) bi-axial winding,
(b) helical winding, (c) hoop winding, (d) perpendicular winding, (e) polar winding, or (f) radial
winding.
Answer. (c)
15.8 Pultrusion is most similar to which one of the following plastic shaping processes? (a)
blow-molding, (b) extrusion, (c) injection molding, or (d) thermoforming.
Answer. (b)
15.9 Water jet cutting is one of several ways of cutting or trimming uncured or cured FRPs; in the case
of cured FRPs, the process is noted for its reduction of dust and noise: (a) true or (b) false.
Answer. (a)
81
16 POWDER METALLURGY
Review Questions
16.1 Name some of the reasons for the commercial importance of powder metallurgy technology.
Answer. PM is important because: (1) parts can be made to net or near net shape, (2) parts can
be made with a controlled level of porosity, (3) certain metals difficult to process by other methods
can be processed by PM, (4) PM allows the formulation of unusual alloys not easily obtained by
traditional alloying methods.
16.2 What are some of the disadvantages of PM methods?
Answer. Disadvantages include: (1) high tooling costs, (2) metal powders are expensive, (3)
difficulties in storing and handling, (4) certain limitations on part geometry imposed by the uniaxial
press methods, and (5) variations in density in a PM component can be troublesome.
16.3 In the screening of powders for sizing, what is meant by the term mesh count?
Answer. The mesh count of the screen is the number of openings per linear inch.
16.4 What is the difference between open pores and closed pores in a metallic powders?
Answer. Open pores are air spaces between particles, while closed pores are voids internal to a
particle.
16.5 What is meant by the term aspect ratio for a metallic particle?
Answer. The aspect ratio of a particle is ratio of the maximum dimension to the minimum
dimension of the given particle.
16.6 How would one measure the angle of repose for a given amount of metallic powder?
Answer. One measure would be to let the powders flow through a small funnel and measure the
angle taken by the resulting pile of powders relative to the horizontal.
16.7 Define bulk density and true density for metallic powders.
Answer. Bulk density refers to the weight per volume of the powders in the loose state, while true
density is the weight per volume of the true volume of metal in the powders (the volume that would
result if the powders were melted).
16.8 What are the principal methods used to produce metallic powders?
Answer. The methods are: (1) atomization - the conversion of molten metal into droplets which
solidify into powders; (2) chemical reduction - reducing metallic oxides by use of reducing agents
which combine with the oxygen to free the metals in the form of powders; and (3) electrolysis -
use of an electrolytic cell to deposit particles of the metal onto the cathode in the cell.
16.9 What are the three basic steps in the conventional powder metallurgy shaping process?
Answer. The steps are: (1) blending and/or mixing, (2) pressing, and (3) sintering.
16.10 What is the technical difference between mixing and blending in powder metallurgy?
Answer. Mixing refers to the combining of metal powders of different chemistries, while blending
means combining particles of the same chemistry but different sizes.
82
16.11 What are some of the ingredients usually added to the metallic powders during blending and/or
mixing?
Answer. The additives are: (1) lubricants, (2) binders, and (3) deflocculants.
16.12 What is meant by the term green compact?
Answer. The green compact is the pressed but not yet sintered PM part.
16.13 Describe what happens to the individual particles during compaction.
Answer. Starting with the initial powder arrangement, the particles are first repacked into a more
efficient arrangement, followed by deformation of the particles as pressure is increased.
16.14 Which of the following most closely typifies the sintering temperatures in PM? (a) 0.5 Tm, (b) 0.8
Tm, (c) Tm.
Answer. (b)
16.15 What are the three steps in the sintering cycle in PM?
Answer. The three steps in the cycle are: (1) preheat, in which lubricants and binders are burned
off, (2) sintering, and (3) cool down.
16.16 What are some of the reasons why a controlled furnace is desirable in sintering?
Answer. Some of the purposes of a controlled atmosphere furnace are: (1) oxidation protection,
(2) provide a reducing atmosphere to remove existing oxides, (3) provide a carburizing atmosphere,
and (4) remove lubricants and binders from pressing.
16.17 What are the advantages of infiltration in PM?
Answer. Advantages of infiltration are: resulting structure is nonporous structure, improved
toughness and strength.
16.18 What is the difference between powder injection molding and metal injection molding?
Answer. Metal injection molding is a subset of powder injection molding, in which the powders are
metallic. The more general term includes powders of ceramic.
16.19 How is isostatic pressing distinguished from conventional pressing and sintering in PM?
Answer. Isostatic pressing applies hydrostatic pressure to all sides of the mold, whereas
conventional pressing is uniaxial.
16.20 Describe liquid phase sintering.
Answer. Liquid phase sintering occurs when two metals of different melting temperatures are
sintered at a temperature between their melting points. Accordingly, one metal melts, thoroughly
wetting the solid particles and creating a strong bonding between the metals upon solidification.
16.21 What are the two basic classes of metal powders as far as chemistry is concerned?
Answer. The two classes are: (1) elemental powders - powders of pure metal such as iron or
copper, and (2) prealloyed powders - powders of alloys such as stainless steel or brass.
16.22 Why is PM technology so well suited to the production of gears and bearings?
Answer. The reasons are: (1) the geometries of these parts lend themselves to PM pressing, and
(2) the porosity allows impregnation of the PM parts with lubricants.
83
Multiple Choice Quiz
There are a total of 18 correct answers in the following multiple choice questions (some questions have
multiple answers that are correct). To attain a perfect score on the quiz, all correct answers must be
given, since each correct answer is worth 1 point. For each question, each omitted answer or wrong
answer reduces the score by 1 point, and each additional answer beyond the number of answers required
reduces the score by 1 point. Percentage score on the quiz is based on the total number of correct
answers.
16.1 The particle size that can pass through a screen is obtained by taking the reciprocal of the mesh
count of the screen. (a) true, or (b) false.
Answer. (b) The given description neglects consideration of the screen wire thickness.
16.2 Identify which of the phrases make the following statement correct: For a given weight of metallic
powders, the total surface area of the powders is increased by (more than one): (a) larger particle
size, (b) smaller particle size, (c) higher shape factor, (d) smaller shape factor.
Answer. (b) and (c)
16.3 As particle size increases, interparticle friction (a) increases, or (b) decreases.
Answer. (b)
16.4 Which of the following powder shapes would tend to have the lowest interparticle friction? (a)
acicular, (b) cubic, (c) flakey, (d) spherical, and (e) rounded.
Answer. (d)
16.5 Which of the following statements is correct in the context of metallic powders (more than one)?
(a) porosity + packing factor = 1.0, (b) packing factor = 1/porosity, (c) packing factor = 1.0 -
porosity, (d) packing factor = - porosity, (e) packing factor = bulk density/true density.
Answer. (a), (c), (e)
16.6 Repressing refers to a pressworking operation used to compress an unsintered part in a closed die
to achieve sizing and better surface finish. (a) true or (b) false.
Answer. (b) The repressed part has been sintered, not unsintered.
16.7 Impregnation refers to which of the following (more than one)? (a) soaking oil by capillary action
into the pores of a PM part, (b) putting polymers into the pores of a PM part, or (c) filling the pores
of the PM part with a molten metal.
Answer. (a), (b)
16.8 In cold isostatic pressing, the mold is most typically made of which one of the following? (a) rubber,
(b) sheetmetal, (c) tool steel, (d) textile, or (e) thermosetting polymer.
Answer. (a)
16.9 Which of the following processes combines pressing and sintering of the metal powders (more than
one)? (a) metal injection molding, (b) hot pressing, (c) spark sintering, and (d) hot isostatic pressing.
Answer. (b), (c), and (d)
16.10 Which of the following design features would be difficult or impossible to achieve by conventional
pressing and sintering (more than one)? (a) side holes, (b) threaded holes, (c) outside rounded
corners, (d) vertical stepped holes, or (e) vertical wall thickness of 1/8 inch (3 mm).
84
Answer. (a), (b), (c)
Problems
Characterization of Engineering Powders
16.1 A screen with 325 mesh count has wires with a diameter of 0.001377 in. Using Eq. (16.1),
determine: (a) the maximum particle size that will pass through the wire mesh, and (b) the
proportion of open space in the screen.
Solution: (a) By Eq. (16.1), particle size PS = 1/MC - tw = 1/325 - 0.001377
= 0.003077 - 0.001377 = 0.00170 in.
(b) There are 325 x 325 = 105,625 openings in one square inch of the mesh. By inference from
part (a), each opening is 0.00170 inch on a side, thus each opening is (0.0017)2 = 0.000002889 in2.
The total open area in one square inch of mesh = 105,625(0.000002889 in2) = 0.30523 in2. This is
total open space. Therefore, the percent open space in one square inch of mesh = 30.523%.
16.2 A screen with 10 mesh count has wires with a diameter of 0.0213 in. Using Eq. (16.1), determine:
(a) the maximum particle size that will pass through the wire mesh, and (b) the proportion of open
space in the screen.
Solution: (a) By Eq. (16.1), particle size PS = 1/MC - tw = 1/10 - 0.0213 = 0.0787 in.
(b) There are 10 x 10 = 100 openings in one square inch of the mesh. By inference from part (a),
each opening is 0.0787 inch on a side, thus each opening is (0.0787)2 = 0.00619 in2. The total open
area in one square inch of mesh = 100(0.00619 in2) = 0.619 in2. This is total open space. Therefore,
the percent open space in one square inch of mesh = 61.9%.
16.3 What is the aspect ratio of a cubic particle shape.
Solution: The aspect ratio is the ratio of the maximum dimension to the minimum dimension of the
particle shape. The minimum dimension is the edge of any face of the cube; call it L. The
maximum dimension is the diagonal of the cube, which is given by (L2 + L2 + L2)0.5 = (3 L2)0.5 =
(3)0.5 L = 1.732 L. Thus, the aspect ratio = 1.732:1.
16.4 Determine the shape factor for metallic particles of the following ideal shapes: (a) sphere, (b)
cubic, (c) cylindrical with length-to-diameter ratio of 1:1, (d) cylindrical with length-to-diameter
ratio of 2:1, and (e) a disk-shaped flake whose thickness-to-diameter ratio is 1:10.
Solution: (a) Sphere: Ks = 6.0 as shown in the text, Eq. (16.6).
(b) Cube: Let L = edge of one face. For a cube, A = 6L2 and V = L3.
Find diameter D of a sphere of equivalent volume.
V = D3/6 = L3
D3 = 6L3/= 1.90986 L3
D = (1.90986 L3)0.333 = 1.2407 L
Ks = A D/V = (6L2)(1.2407 L)/L3 = 7.444
(c) Cylinder with L/D = 1.0. For this cylinder shape, L = D. Thus, A = 2D2/4 + DL = .5L2 +
L2 = 1.5L2, and V = (D2/4)L = 0.25L3.
Find diameter D of a sphere of equivalent volume.
V = D3/6 = 0.25L3
D3 = 6(0.25L3)/ = 1.5L3
D = (1.5 L3)0.333 = 1.1447 L
Ks = A D/V = (1.5L2)(1.1447 L)/0.25L3 = 6.868
85
(d) Cylinder with L/D = 2.0. For this cylinder shape, 0.5L = D. Thus, A = 2D2/4 + DL =
0.5(0.5L)2 + (0.5L)L = 0.125L2 + 0.5L2 = 0.625L2, and V = (D2/4)L = 0.25(0.5L)2 L =
0.0625L3
Find diameter D of a sphere of equivalent volume.
V = D3/6 = 0.0625L3
D3 = 6(0.0625L3)/ = 0.375L3
D = (0.375 L3)0.333 = 0.721 L
Ks = A D/V = (0.625L2)(0.721 L)/0.0625L3 = 7.211
(e) Disk with L/D = 0.10. For this shape, 10L = D. Thus, A = 2D2/4 + DL = .5(10L)2 +
(10L)L = 50L2 + 10L2 = 60L2, and V = (D2/4)L = 0.25(10L)2 L = 25L3
Find diameter D of a sphere of equivalent volume.
V = D3/6 = 25L3
D3 = 6(25L3)/ = 150L3
D = (150 L3)0.333 = 5.313 L
Ks = A D/V = (60L2)(5.313 L)/25L3 = 12.75
16.5 A pile of iron powder weighs 2 lb. The particles are spherical in shape and all have the same
diameter of 0.002 in. (a) Determine the total surface area of all the particles in the pile. (b) If the
packing factor = 0.6, determine the volume taken by the pile. Note: the density of iron = 0.284
lb/in3.
Solution: (a) For a spherical particle of D = 0.002 in., V = D3/6 = (0.002)3/6
= 0.00000000418 = 4.18 x 10-9 in3/particle
Weight per particle W = V = 0.284(4.18 x 10-9 in3) = 1.19 x 10-9 lb/particle
Number of particles in 2 lb = 2.0/(1.19 x 10-9) = 1.681 x 109
A = D2 = (0.002)2 = 0.00001256 in2 = 12.56 x 10-6 in2
Total surface area = (1.681 x 109)(12.56 x 10-6) = 21.116 x 103 in2
(b) With a packing factor of 0.6, the total volume taken up by the pile = (2.0/0.284)/0.6 = 11.74 in3
16.6 Solve Problem 16.5, except that the diameter of the particles is 0.004 in. Assume the same packing
factor.
Solution: (a) For a spherical particle of D = 0.004 in., V = D3/6 = (0.004)3/6
= 0.00000003351 = 33.51 x 10-9 in3/particle
Weight per particle W = V = 0.284(33.51 x 10-9 in3) = 9.516 x 10-9 lb/particle
Number of particles in 2 lb = 2.0/(9.516 x 10-9) = 0.2102 x 109
A = D2 = (0.004)2 = 0.00005027 in2 = 50.27 x 10-6 in2
Total surface area = (0.2102 x 109)(50.27 x 10-6) = 10.565 x 103 in2
(b) With a packing factor of 0.6, the total volume taken up by the pile = (2.0/0.284)/0.6 = 11.74 in3
16.7 Suppose in Problem 16.5 that the average particle diameter = 0.002 in; however, the sizes vary,
forming a statistical distribution as follows: 25% of the particles by weight are 0.001 in, 50% are
0.002 in, and 25% are 0.003 in. Given this distribution, what is the total surface area of all the
particles in the pile.
Solution: For a spherical particle of D = 0.001 in., V = D3/6 = (0.001)3/6
= 0.5236 x 10-9 in3/particle
Weight per particle W = V = 0.284(0.5236 x 10-9 in3) = 0.1487 x 10-9 lb/particle
Particles of size D = 0.001 in. constitute 25% of total 2 lb. = 0.5 lb
Number of particles in 0.5 lb = 0.5/(0.1487 x 10-9) = 3.362 x 109
86
A = D2 = (0.001)2 = 3.142 x 10-6 in2/particle
Total surface area of particles of D = 0.001 in. = (3.362 x 109)(3.142 x 10-6) = 10.563 x 103 in2
For a spherical particle of D = 0.002 in., V = (0.002)3/6 = 4.18 x 10-9 in3/particle
Weight per particle W = V = 0.284(4.18 x 10-9 in3) = 1.19 x 10-9 lb/particle
Particles of size D = 0.002 in. constitute 50% of total 2 lb. = 1.0 lb
Number of particles in 1 lb = 1.0/(1.19 x 10-9) = 0.8406 x 109
A = D2 = (0.002)2 = 12.56 x 10-6 in2
Total surface area of particles of D = 0.002 in. = (0.8406 x 109)(12.566 x 10-6) = 10.563 x 103 in2
For a spherical particle of D = 0.003 in., V = (0.003)3/6 = 14.137 x 10-9 in3/particle
Weight per particle W = V = 0.284(14.137 x 10-9 in3) = 4.015 x 10-9 lb/particle
Particles of size D = 0.003 in. constitute 25% of total 2 lb. = 0.5 lb
Number of particles in 0.5 lb = 0.5/(4.015 x 10-9) = 0.124 x 109
A = D2 = (0.003)2 = 28.274 x 10-6 in2
Total surface area of particles of D = 0.003 in. = (0.124 x 109)(28.274 x 10-6) = 3.506 x 103 in2
Total surface area of all particles = 10.563 x 103 + 10.563 x 103 + 3.506 x 103 = 24.632 x 103 in2.
16.8 A solid cube of copper with each side = 1.0 ft is converted into metallic powders of spherical shape
by gas atomization. What is the percentage increase in total surface area if the diameter of each
particle is 0.004 in (assume that all particles are the same size)?
Solution: Area of initial cube A = 6(1 ft)2 = 6 ft2 = 864 in2
Volume of cube V = (1 ft)3 = 1728 in3
Surface area of a spherical particle of D = 0.004 in. is A = D2 = (0.004)2
= 50.265 x 10-6 in3/particle
Volume of a spherical particle of D = 0.004 in. is V = D3/6 = (0.004)3/6
= 33.51 x 10-9 in3/particle
Number of particles in 1 ft3 = 1728/33.51 x 10-9 = 51.567 x 109
Total surface area = (51.567 x 109)( 50.265 x 10-6 in3) = 2,592 x 103 = 2,592,000 in2
Percent increase = 100(2,592,000 - 864)/864 = 299,900%
16.9 A solid cube of aluminum with each side = 1.0 m is converted into metallic powders of spherical
shape by gas atomization. How much total surface area is added by the process if the diameter of
each particle is 100 microns (assume that all particles are the same size)?
Solution: Area of starting cube A = 6(1 m)2 = 6 m2
Volume of starting cube V = (1 m)3 = 1 m3
D = 100 m = 0.1 mm = 0.1 x 10-3 m
Surface area of a sphere of D = 0.1 x 10-3 m is A = D2 = (0.1 x 10-3)2
= 3.142 x 10-8 m3/particle
Volume of a sphere of D = 0.1 x 10-3 m is V = D3/6 = (0.1 x 10-3)3/6
= 0.5236 x 10-12 m3/particle
Number of particles in 1 m3 = 1.0/0.5236 x 10-12 = 1.91 x 1012
Total surface area = (1.91 x 1012)( 0.5236 x 10-12 m3) = 5.9958 x 104 = 59,958 m2
Added surface area = 59,958 - 6 = 59,952 m2
16.10 Given a large volume of metallic powders, all of which are perfectly spherical and having the same
exact diameter, what is the maximum possible packing factor that the powders can take?
Solution: The maximum packing factor is achieved when the spherical particles are arranged as a
face-centered cubic unit cell, similar to the atomic structure of FCC metals; see Figure 2.8(b). The
87
unit cell of the FCC structure contains 8 spheres at the corners of the cube and 6 spheres on each
face. Our approach to determine the packing factor will consist of: (1) finding the volume of the
spheres and portions thereof that are contained in the cell, and (2) finding the volume of the unit
cell cube. The ratio of (1) over (2) is the packing factor.
(1) Volume of whole and/or partial spheres contained in the unit cell. The unit cell contains 6 half
spheres in the faces of the cube and 8 one-eighth spheres in corners. The equivalent number of
whole spheres = 6(.5) + 8(.125) = 4 spheres. Volume of 4 spheres = 4D3/6 = 2.0944 D3 where D
= diameter of a sphere.
(2) Volume of the cube of one unit cell. Consider that the diagonal of any face of the unit cell
contains one full diameter (the sphere in the center of the cube face) and two half diameters (the
spheres at the corners of the face). Thus, the diagonal of the cube face = 2D. Accordingly, the
face is a square with each edge = D2 = 1.414D. The volume of the unit cell is therefore
(1.414D)3 = 2.8284 D3.
The packing factor = 2.0944/2.8284 = 0.7405 = 74.05%
Compaction and Design Considerations
16.11 In a certain pressing operation, the metallic powder fed into the open die has a packing factor of
0.5. The pressing operation reduces the powder to 2/3 of its starting volume. In the subsequent
sintering operation, shrinkage amounts to 10% on a volume basis. Given that these are the only
factors that affect the structure of the finished part, determine its final porosity.
Solution: Packing factor = bulk density / true density
Density = (specific volume)-1
Packing factor = true specific volume / bulk specific volume
Pressing reduces bulk specific volume to 2/3 = 0.667
Sintering further reduces the bulk specific volume to 0.90 of value after pressing.
Let true specific volume = 1.0
Thus for a packing factor of 0.5, bulk specific volume = 2.0.
Packing factor after pressing and sintering = 1.0/(2.0 x .667 x .90) = 1.0/1.2 = 0.833
By Eq. (18.7), porosity = 1 - 0.833 = 0.167
16.12 A bearing of simple geometry is to be pressed out of bronze powders, using a compacting pressure
of 207 MPa. The outside diameter = 44 mm, the inside diameter = 22 mm, and the length of the
bearing = 25 mm. What is the required press tonnage to perform this operation?
Solution: Projected area of part Ap = 0.25(Do
2 - Di
2) = 0.25(442 - 222) = 1140.4 mm2
F = Appc = 1140.4(207) = 236,062 kN
16.13 The part shown in Figure P16.13 is to be pressed of iron powders using a compaction pressure of
75,000 lb/in2. Dimensions are inches. Determine: (a) the most appropriate pressing direction, (b) the
required press tonnage to perform this operation, and (c) the final weight of the part if the porosity
is 10%. Assume shrinkage during sintering can be neglected.
Solution: (a) Most appropriate pressing direction is parallel to the part axis.
(b) Press tonnage F = Appc
Projected area of part Ap = 0.25(Do
2 - Di
2) = 0.25(2.82 - 0.8752) = 5.556 in2
F = Appc = 5.556(75,000) = 416,715 lb = 208 tons.
(c) V = 0.25(2.82 - 0.8752)(0.5) + 0.25(2.82 - 1.52)(1.25 - 0.5) = 0.25(3.5372 + 4.1925)
= 6.071 in3
88
From Table 4.1, density of iron = 0.284 lb/in3.
At 10% porosity, part weight W = 6.071(0.284)(0.90) = 1.55 lb.
16.14 For each of the four part drawings in Figure P16.14, indicate which PM class the parts belong to,
whether the part must be pressed from one or two directions, and how many levels of press control
will be required. Dimensions are mm.
Solution: (a) Class II, 2 directions because of axial thickness, one level of press control.
(b) Class I, one direction part is relatively thin, one level of press control.
(c) Class IV, 2 directions of pressing, 3 levels of press control required.
(d) Class IV, 2 directions of pressing, 4 or 5 levels of press control due to multiple steps in part
design.
89
17 PROCESSING OF CERAMICS AND CERMETS
Review Questions
17.1 What is the difference between the traditional ceramics and the new ceramics, as far as raw
materials are concerned?
Answer. The traditional ceramics are based on hydrous aluminum silicates (clay), whereas the
new ceramics are based on man-made simpler compounds such as oxides, nitrides, and carbides.
17.2 List the basic steps in the traditional ceramics processing sequence.
Answer. The sequence is: (1) preparation of raw materials, (2) shaping, (3) drying, and (4) firing.
17.3 What is the technical difference between crushing and grinding in the preparation of traditional
ceramic raw materials?
Answer. Crushing is performed to reduce large lumps of mineral to smaller size. Grinding is a
secondary process which further reduces the particle size to fine powder.
17.4 Describe the slip casting process in traditional ceramics processing.
Answer. In slip casting a slurry of clay is poured into a plaster of Paris mold, whereupon water is
absorbed from the slurry into the plaster to form a clay layer against the mold wall. The remaining
slurry is usually poured out to leave a hollow part.
17.5 List and briefly describe some of the plastic forming methods used to shape traditional ceramics
products.
Answer. The plastic forming methods include: (1) hand modeling, molding, and throwing; (2)
jiggering, which is a mechanized extension of hand throwing used to manufacture bowls and plates;
(3) plastic pressing, in which a clay slug is pressed in a mold; and (4) extrusion, in which the clay is
compressed through a die opening to make long sections of uniform cross-sectional shape.
17.6 What is the process of jiggering?
Answer. Jiggering is a clay forming process that uses a convex mold on a potters wheel. The clay
is first pressed into rough shape and then rotated and formed with a jigger tool to final shape. It is
suited to the manufacture of flatware.
17.7 What is the difference between dry pressing and semi-dry pressing of traditional ceramics parts?
Answer. The difference is in the starting clay. For semi- dry pressing, the clay has a typical water
content of 10% to 15%. For dry pressing, the water content is usually less than 5%. Dry clay has
virtually no plasticity, and so this imposes certain limitations on part geometry in dry pressing.
17.8 What happens to a ceramic material when it is sintered?
Answer. Sintering of green ceramics (or powdered metals) causes bonding between the ceramic
grains, which is accompanied by densification and reduction of porosity.
17.9 What is the name given to the furnace used to fire ceramic ware?
Answer. Kiln.
17.10 What is glazing in traditional ceramics processing?
90
Answer. Glazing refers to the process of putting a ceramic coating on the surface of the ceramic
piece. The coating, usually consisting of ceramic oxides, is referred to as a glaze.
17.11 Why is the drying step, so important in the processing of traditional ceramics, usually not required in
processing of new ceramics?
Answer. Because water is usually not one of the ingredients in the new ceramics during forming.
Drying is only needed when the green piece contains water.
17.12 Why is raw material preparation more important in the processing of new ceramics than for
traditional ceramics?
Answer. Because the requirements on the strength of the finished product are usually more
demanding for new ceramics.
17.13 What is the freeze drying process used to make certain new ceramic powders?
Answer. In freeze drying, salts are dissolved in water and sprayed into small droplets which are
immediately frozen; the water is then removed from the droplets in a vacuum chamber, and the
freeze-dried salt is decomposed by heating to form the ceramic powders.
17.14 Describe the doctor-blade process.
Answer. In the doctor-blade process, a ceramic slurry is flowed onto a moving film which flows
under a wiper blade, so that the resulting ceramic is in the form of a thin green sheet which is dried
and reeled onto a spool for subsequent shaping and sintering.
17.15 Liquid phase sintering is used for WC-Co compacts, even though the sintering temperatures are
below the melting points of either WC or Co. How is this possible?
Answer. The melting point of cobalt is reduced when WC is dissolved in it. At the sintering
temperatures used for WC-Co, WC gradually dissolves in the cobalt, reducing its melting point to
the sintering temperature. Thus does liquid phase sintering occur in the WC-Co system.
17.16 What are some design recommendations for ceramic parts?
Answer. The guidelines include: (1) subject ceramic parts to compressive, not tensile loads; (2)
ceramics are brittle, so avoid impact loading; (3) part geometries should be simple; (4) use large
radii on inside and outside corners; (5) take into account shrinkage; (6) no screw threads.
Multiple Choice Quiz
There are a total of 15 correct answers in the following multiple choice questions (some questions have
multiple answers that are correct). To attain a perfect score on the quiz, all correct answers must be
given, since each correct answer is worth 1 point. For each question, each omitted answer or wrong
answer reduces the score by 1 point, and each additional answer beyond the number of answers required
reduces the score by 1 point. Percentage score on the quiz is based on the total number of correct
answers.
17.1 The following equipment is used for crushing and grinding of minerals in the preparation of
traditional ceramics raw materials. Which one of the pieces listed is used for grinding? (a) ball mill,
(b) hammer mill, (c) jaw crusher, or (d) roll crusher.
Answer. (a)
17.2 Which one of the following compounds becomes a plastic and formable material when mixed with
suitable proportions of water? (a) aluminum oxide, (b) hydrogen oxide, (c) hydrous aluminum
silicate, or (d) silicon dioxide.
91
Answer. (c)
17.3 At which one of the following water contents does clay become a suitably plastic material for the
traditional ceramics plastic forming processes? (a) 5%, (b) 10%, (c) 20%, or (d) 40%.
Answer. (c)
17.4 Which of the following processes are not plastic forming methods used in the shaping of traditional
ceramics (more than one)? (a) extrusion, (b) jangling, (c) jiggering, (d) jolleying, or (e) spinning.
Answer. (b) and (c)
17.5 The term green piece in ceramics refers to a part that has been shaped but not yet fired: (a) true,
or (b) false.
Answer. (a)
17.6 In the final product made of a polycrystalline new ceramic material, strength increases with grain
size: (a) true, or (b) false.
Answer. (b)
17.7 Which one of the following processes for the new ceramic materials accomplishes shaping and
sintering simultaneously? (a) doctor-blade process, (b) freeze drying, (c) hot pressing, (d) injection
molding, or (e) isostatic pressing.
Answer. (c)
17.8 Which of the following are not the purposes of finishing operations used for parts made of the new
ceramics (more than one)? (a) apply a surface coating, (b) improve surface finish, (c) increase
dimensional accuracy, (d) remove material, or (e) work harden the surface.
Answer. (a) and (e)
17.9 Which one of the following terms best describes what a cemented carbide is? (a) ceramic, (b)
cermet, (c) composite, or (d) metal.
Answer. (b)
17.10 Which of the following geometric features should be avoided if possible in the design of structural
components made of new ceramics (more than one)? (a) complicated shapes, (b) rounded inside
corners, (c) sharp edges, (d) thin sections, or (e) threads.
Answer. (a), (c), (d), and (e)
92
18 FUNDAMENTALS OF METAL FORMING
Review Questions
18.1 What are the characteristics that distinguish bulk deformation processes from sheet metal
processes?
Answer. In bulk deformation, the shape changes are significant, and the workparts have a low
area-to-volume ratio. In sheet metal processes, the area-to-volume ratio is high.
18.2 Extrusion is a fundamental shaping process. Describe it.
Answer. Extrusion is a compression process in which the work material is forced to flow through
a die orifice, thereby forcing its cross-section to assume the profile of the orifice.
18.3 Why is the term pressworking often used for sheet metal processes?
Answer. Because these operations are generally performed on presses.
18.4 What is the difference between deep drawing and bar drawing?
Answer. Deep drawing is a sheet metal forming process used to fabricate cup-shaped parts; bar
drawing is a bulk deformation process used to reduce the diameter of a cylindrical workpart.
18.5 Indicate the mathematical equation for the flow curve.
Answer. The flow curve is defined in Eq. (18.1) as Yf = Ken.
18.6 How does increasing temperature affect the parameters in the flow curve equation?
Answer. Increasing temperature decreases both K and n in the flow curve equation.
18.7 Indicate some of the advantages of cold working relative to warm and hot working.
Answer. Advantages of cold working are: (1) better accuracy, (2) better surface finish, (3)
increased strength due to work hardening, (4) possible directional properties due to grain flow, and
(5) no heating of work required.
18.8 What is isothermal forming?
Answer. An isothermal forming operation is performed in such a way as to eliminate surface
cooling and thermal gradients in the workpart. This is accomplished by preheating the forming
tools.
18.9 Describe the effect of strain rate in metal forming.
Answer. Increasing strain rate tends to increase the resistance to deformation. The tendency is
especially prominent in hot forming operations.
18.10 Why is friction generally undesirable in metal forming operations?
Answer. Reasons why friction is undesirable in metal forming: (1) inhibits metal flow during
deformation, causing residual stresses and product defects; (2) increases forces and power
required; and (3) wearing of the tools.
18.11 What is sticking friction in metalworking?
Answer. Sticking friction is when the work surface adheres to the surface of the tool rather than
slides against it; it occurs when the friction stress is greater than the shear flow stress of the metal.
93
Multiple Choice Quiz
There are a total of 13 correct answers in the following multiple choice questions (some questions have
multiple answers that are correct). To attain a perfect score on the quiz, all correct answers must be
given, since each correct answer is worth 1 point. For each question, each omitted answer or wrong
answer reduces the score by 1 point, and each additional answer beyond the number of answers required
reduces the score by 1 point. Percentage score on the quiz is based on the total number of correct
answers.
18.1 Which of the following are bulk deformation processes (more than one)? (a) bending, (b) deep
drawing, (c) extrusion, (d) forging, and (e) rolling.
Answer. (c), (d), and (e).
18.2 Which of the following is typical of the work geometry in sheet metal processes? (a) high
volume-to-area ratio, or (b) low volume-to-area ratio.
Answer. (b)
18.3 The flow curve expresses the behavior of a metal in which of the following regions of the
stress-strain curve? (a) elastic region, or (b) plastic region.
Answer. (b)
18.4 The average flow stress is the flow stress multiplied by which of the following factors? (a) n, (b)
(1+n), (c) 1/n, or (d) 1/(1+n), where n is the strain hardening exponent.
Answer. (d)
18.5 Hot working of metals refers to which one of the following temperature regions relative to the
melting point of the given metal on an absolute temperature scale? (a) room temperature, (b)
0.2Tm, (c) 0.4Tm, or (d) 0.6Tm.
Answer. (d)
18.6 Which of the following are advantages and characteristics of hot working relative to cold working
(more than one)? (a) fracture of workpart less likely, (b) increased strength properties, (c) isotropic
mechanical properties, (d) less overall energy required, (e) lower deformation forces required, and
(f) more significant shape changes are possible.
Answer. (a), (c), (e), and (f).
18.7 Increasing strain rate tends to have which one of the following effects on flow stress during hot
forming of metal? (a) decreases flow stress, (b) has no effect, or (c) increases flow stress.
Answer. (c)
18.8 The coefficient of friction between the part and the tool in cold working tends to be which of the
following relative to its value in hot working? (a) higher, (b) lower, or (c) no effect.
Answer. (b).
Problems
Flow Curve in Forming
18.1 K = 600 MPa and n = 0.20 for a certain metal. During a forming operation, the final true strain that
the metal experiences = 0.73. Determine the flow stress at this strain and the average flow stress
that the metal experienced during the operation.
94
Solution: Flow stress Yf = 600(0.73)0.2 = 563.4 MPa.
Average flow stress Y
f
= 600(0.73)0.2/1.2 = 469.5 MPa.
18.2 A metal has a flow curve with parameters: K = 850 MPa and strain hardening exponent n = 0.30.
A tensile specimen of the metal with gage length = 100 mm is stretched to a length = 157 mm.
Determine the flow stress at the new length and the average flow stress that the metal has been
subjected to during the deformation.
Solution: = ln (157/100) = ln 1.57 = 0.451
Flow stress Yf = 850(0.451)0.30 = 669.4 MPa.
Average flow stress Y
f
= 850(0.451)0.30/1.30 = 514.9 MPa.
18.3 A particular metal has a flow curve with parameters: strength coefficient K = 35,000 lb/in2 and
strain hardening exponent n = 0.26. A tensile specimen of the metal with gage length = 2.0 in is
stretched to a length = 3.3 in. Determine the flow stress at this new length and the average flow
stress that the metal has been subjected to during deformation.
Solution: = ln (3.3/2.0) = ln 1.65 = 0.501
Flow stress Yf = 35,000(0.501)0.26 = 29,240 lb/in2.
Average flow stress Y
f
= 35,000(0.501)0.26/1.26 = 23,206 lb/in2.
18.4 The strength coefficient and strain hardening exponent of a certain test metal are K = 40,000 lb/in2
and n = 0.19. A cylindrical specimen of the metal with starting diameter = 2.5 in and length = 3.0 in
is compressed to a length of 1.5 in. Determine the flow stress at this compressed length and the
average flow stress that the metal has experienced during deformation.
Solution: = ln (1.5/3.0) = ln 0.5 = -0.69315
Flow stress Yf = 40,000(0.69315)0.19 = 37,309 lb/in2.
Average flow stress Y
f
= 40,000(0.69315)0.19/1.19 = 31,352 lb/in2.
18.5 Derive the equation for average flow stress, Eq. (18.2) in the text.
Solution: Flow stress equation [Eq. (18.1)]: Yf = Kn
Y
f
over the range = 0 to = is given by Kn d= K n d= Kn+1/(n+1) = Ke n/(n+1)
18.6 For a certain metal, K = 700 MPa and n = 0.27. Determine the average flow stress that the metal
experiences if it is subjected to a stress that is equal to its strength coefficient K.
Solution: Yf = K = 700 = Kn = 700.27
must be equal to 1.0.
Y
f
= 700(1.0).27/1.27 = 700/1.27 = 551.2 MPa
18.7 Determine the value of the strain hardening exponent for a metal that will cause the average flow
stress to be 3/4 of the final flow stress after deformation.
Solution: Y
f
= 0.75 Yf
Kn/(1+n) = 0.75 Kn
1/(1+n) = 0.75
1 = 0.75(1+n) = 0.75 + 0.75n
0.25 = 0.75n n = 0.333
95
18.8 K = 35,000 lb/in2 and n = 0.40 for a metal used in a forming operation in which the workpart is
reduced in cross-sectional area by stretching. If the average flow stress on the part is 20,000 lb/in2,
determine the amount of reduction in cross-sectional area experienced by the part.
Solution: Y
f
= Kn/(1+n)
20,000 = 35,000 .4/(1.4)
1.4(20,000) = 35,000 .4
28,000/35,000 = 0.8 = .4
0.4 ln = ln (0.8) = -0.22314
ln = -0.22314/0.4 = -0.55786
= 0.5724
= ln(Ao/Af) = 0.5724
Ao/Af = 1.7726
Af = Ao/1.7726 = 0.564Ao
Strain Rate
18.9 The gage length of a tensile test specimen = 150 mm. It is subjected to a tensile test in which the
grips holding the end of the test specimen are moved with a relative velocity = 0.1 m/s. Construct a
plot of the strain rate as a function of length as the specimen is pulled to a length = 200 mm.
Solution: The following values are calculated for the plot:
At L = 150 mm, strain rate = 0.1/0.15 = 0.667 s-1
At L = 160 mm, strain rate = 0.1/0.16 = 0.625 s-1
At L = 170 mm, strain rate = 0.1/0.17 = 0.588 s-1
At L = 180 mm, strain rate = 0.1/0.18 = 0.555 s-1
At L = 190 mm, strain rate = 0.1/0.19 = 0.526 s-1
At L = 200 mm, strain rate = 0.1/0.20 = 0.500 s-1
18.10 A specimen with 6.0 in starting gage length is subjected to a tensile test in which the grips holding
the end of the test specimen are moved with a relative velocity = 1.0 in/sec. Construct a plot of the
strain rate as a function of length as the specimen is pulled to a length = 8.0 in.
Solution: The following values are calculated for the plot:
At L = 6.0 in., strain rate = 1/6.0 = 0.1667 sec-1
At L = 6.5 in., strain rate = 1/6.5 = 0.1538 sec-1
At L = 7.0 in., strain rate = 1/7.0 = 0.1429 sec-1
At L = 7.5 in., strain rate = 1/7.5 = 0.1333 sec-1
At L = 8.0 in., strain rate = 1/8.0 = 0.1250 sec-1
18.11 A workpart with starting height h = 100 mm is compressed to a final height of 50 mm. During the
deformation, the relative speed of the plattens compressing the part = 200 mm/s. Determine the
strain rate at (a) h = 100 mm, (b) h = 75 mm, and (c) h = 51 mm.
Solution: (a) strain rate = 200/100 = 2.0 s-1
(b) strain rate = 200/75 = 2.667 s-1
(c) strain rate = 200/51 = 3.922 s-1
18.12 A hot working operation is carried out at various speeds. The strength constant C = 30,000 lb/in2
and the strain-rate sensitivity exponent m = 0.15. Determine the flow stress if the strain rate is: (a)
0.01/sec (b) 1.0/sec, (c) 100/sec.
96
Solution: (a) Yf = C(strain rate)m = 30,000(0.01).15 = 15,036 lb/in2
(b) Yf = 30,000(1.0).15 = 30,000 lb/in2
(c) Yf = 30,000(100).15 = 59,858 lb/in2
18.13 A tensile test is performed to determine the parameters C and m in Eq. (18.4) for a certain metal.
The temperature at which the test is performed = 500C. At a strain rate = 12/s, the stress is
measured at 160 MPa; and at a strain rate = 250/s, the stress = 300 MPa. (a) Determine C and m.
(b) If the temperature were 600C, what changes would you expect in the values of C and m?
Solution: (a) Two equations: (1) 160 = C(12)m and (2) 300 = C(250)m
(1) ln 160 = ln C + m ln 12 or ln 160 - m ln 12 = ln C
(2) ln 300 = ln C + m ln 250 or ln 300 - m ln 250 = ln C
(1) and (2): ln 160 - m ln 12 = ln 300 - m ln 250
5.0752 – 2.4849 m = 5.7038 – 5.5215 m
(5.5215 – 2.4849)m = 5.7038 – 5.0752
3.0366 m = 0.6286 m = 0.207
(1) C = 160/(12).207 = 160.1.6726 = 95.658
(2) C = 300/(250).207 = 300/3.1361 = 95.660
Averaging these values, C = 95.659
(b) If temperature were 600C, the strength constant C would decrease and the strain-rate
sensitivity exponent m would increase.
18.14 A tensile test is carried out to determine the strength constant C and strain-rate sensitivity exponent
m for a certain metal at 1000F. At a strain rate = 10/sec, the stress is measured at 23,000 lb/in2;
and at a strain rate = 300/sec, the stress = 45,000 lb/in2. (a) Determine C and m. (b) If the
temperature were 900F, what changes would you expect in the values of C and m?
Solution: (a) Two equations: (1) 23,000 = C(10)m and (2) 45,000 = C(300)m
45,000/23,000 = 1.9565 = (300/10)m = (30)m
ln 1.9656 = m ln 30
0.67117 = 3.4012 m m = 0.1973
(1) C = 23000/100.1973 = 23000/1.5752 = 14,601.4
(2) C = 45000/3000.1973 = 45000/3.0819 = 14,601.4 C = 14,601.4
(b) If temperature were decreased to 900F, the strength constant C would increase and the strainrate
sensitivity exponent m would decrease.
97
19 BULK DEFORMATION PROCESSES IN
METALWORKING
Review Questions
19.1 What are the reasons why the bulk deformation processes are important commercially and
technologically?
Answer. Reasons why the bulk deformation processes are important include: (1) they are capable
of significant shape change when hot working is used, (2) they have a positive effect on part
strength when cold working is used, and (3) most of the processes produce little material waste;
some are net shape processes.
19.2 List some of the products produced on a rolling mill.
Answer. Rolled products include flat sheet and plate stock, round bar and rod stock, rails,
structural shapes such as I-beams and channels.
19.3 Identify some of the ways in which force in flat rolling can be reduced.
Answer. Ways to reduce force in flat rolling include: (1) use hot rolling, (2) reduce draft in each
pass, and (3) use smaller diameter rolls.
19.4 What is a two-high rolling mill?
Answer. A two-high rolling mill consists of two opposing rolls between which the work is
compressed.
19.5 What is a reversing mill in rolling?
Answer. A reversing mill is a two-high rolling mill in which the direction of rotation of the rolls can
be reversed to allow the work to pass through from either side.
19.6 Besides flat rolling and shape rolling, identify some additional bulk forming processes that use rolls
to effect the deformation.
Answer. Some other processes that use rolls are ring rolling, thread rolling, gear rolling, roll
piercing, and roll forging.
19.7 One way to classify forging operations is by the degree to which the work is constrained in the die.
By this classification, name the three basic types.
Answer. The three basic types are: (1) open die forging, (2) impression die forging, and (3) closed
die forging.
19.8 Why is flash desirable in impression die forging?
Answer. Because its presence constrains the metal in the die to fill the details of the die cavity.
19.9 What are the two basic types of forging equipment?
Answer. The two types of forging machines are hammers, which impact the workpart, and
presses, which apply a gradual pressure to the work.
19.10 What is isothermal forging?
98
Answer. Isothermal forging is a hot forging operation in which the die surfaces are heated to
reduce heat transfer from the work into the tooling.
19.11 Distinguish between direct and indirect extrusion.
Answer. See Article 19.3.1.
19.12 Name some products that are produced by extrusion.
Answer. Products produced by continuous extrusion include: structural shapes (window frames,
shower stalls, channels), tubes and pipes, and rods of various cross-section. Products made by
discrete extrusion include: toothpaste tubes, aluminum beverage cans, and battery cases.
19.13 What does the centerburst defect in extrusion have in common with the roll piercing process?
Answer. They are both examples of how compressive stresses applied to the outside surface of a
solid cylindrical cross-section can create high tensile stresses in the interior of the cylinder.
19.14 In a wire drawing operation, why must the drawing stress never exceed the yield strength of the
work metal?
Answer. Because if the drawing stress exceeded the yield strength, the metal on the exit side of
the draw die would stretch rather than force metal to be drawn through the die opening.
Multiple Choice Quiz
There are a total of 22 correct answers in the following multiple choice questions (some questions have
multiple answers that are correct). To attain a perfect score on the quiz, all correct answers must be
given, since each correct answer is worth 1 point. For each question, each omitted answer or wrong
answer reduces the score by 1 point, and each additional answer beyond the number of answers required
reduces the score by 1 point. Percentage score on the quiz is based on the total number of correct
answers.
19.1 The maximum possible draft in a rolling operation depends on which of the following parameters
(more than one)? (a) coefficient of friction between roll and work, (b) roll diameter, (c) roll
velocity, (d) stock thickness, (e) strain, and (f) strength coefficient of the work metal.
Answer. (a) and (b).
19.2 Which of the following rolling mill types are associated with relatively small diameter rolls in
contact with the work (more than one)? (a) cluster mill, (b) continuous rolling mill, (c) four-high
mill, (d) reversing mill, or (e) three-high configuration.
Answer. (a) and (c).
19.3 Production of pipes and tubes is associated with which of the following bulk deformation processes
(more than one)? (a) extrusion, (b) hobbing, (c) ring rolling, (d) roll forging, (e) roll piercing, (f) tube
sinking, or (g) upsetting.
Answer. (a), (e), and (f).
19.4 Which of the four basic bulk deformation processes use compression to effect shape change (more
than one)? (a) bar and wire drawing, (b) extrusion, (c) forging, and (d) rolling.
Answer. (a), (b), (c), and (d). Bar and wire drawing (a) is the only tricky answer; although tensile
stresses are applied to the work, the work is squeezed through the die opening by compression -
the term indirect compression is sometimes used.
99
19.5 Flash in impression die forging serves no useful purpose and is undesirable because it must be
trimmed from the part after forming: (a) true or (b) false?
Answer. (b). Flash causes build-up of pressure inside the die which causes the work metal to fill
the cavity.
19.6 Which of the following are classified as forging operations (more than one)? (a) coining, (b)
fullering, (c) impact extrusion, (d) roll forging, (e) thread rolling, and (f) upsetting.
Answer. (a), (b), (d), and (f).
19.7 The production of tubing is possible in indirect extrusion but not in direct extrusion: (a) true or (b)
false?
Answer. (b). Tube and pipe cross-sections can be produced by either direct or indirect extrusion.
19.8 Theoretically, the maximum reduction possible in a wire drawing operation, under the assumptions
of a perfectly plastic metal, no friction, and no redundant work, is which of the following (one
answer)? (a) zero, (b) 0.63, (c) 1.0, or (d) 2.72.
Answer. (b)
19.9 Which of the following bulk deformation processes are involved in the production of nails for
lumber construction (more than one)? (a) bar and wire drawing, (b) extrusion, (c) forging, and (d)
rolling.
Answer. (a), (c), and (d). Bar stock is rolled, drawn into wire stock, and upset forged to form the
nail head.
19.10 Johnson's formula is associated with which of the four bulk deformation processes (one answer)?
(a) bar and wire drawing, (b) extrusion, (c) forging, and (d) rolling.
Answer. (b)
Problems
Rolling
19.1 A 40 mm thick plate is to be reduced to 30 mm in one pass in a rolling operation. Entrance speed =
16 m/min. Roll radius = 300 mm, and rotational speed = 18.5 rev/min. Determine: (a) the minimum
required coefficient of friction that would make this rolling operation possible, (b) exit velocity
under the assumption that the plate widens by 2% during the operation, and (c) forward slip.
Solution: (a) Maximum draft dmax
= 2 R
Given that d = to - tf = 40 - 30 = 10 mm,
2 = 10/300 = 0.0333
= (0.0333)0.5 = 0.1826
(b) Plate widens by 2%.
towovo = tfwfvf
wf = 1.02 wo
40(wo)(16) = 30(1.02wo)vf
vf = 40(wo)(16)/ 30(1.02wo) = 640/30.6 = 20.915 m/min
(c) s = (vf - vr)/vr = (20.915 - 18.5)/18.5 = 0.13
19.2 A 2.0 in thick slab is 10.0 in wide and 12.0 ft long. Thickness is to be reduced in three steps in a
hot rolling operation. Each step will reduce the slab to 75% of its previous thickness. It is expected
100
that for this metal and reduction, the slab will widen by 3% in each step. If the entry speed of the
slab in the first step is 40 ft/min, and roll speed is the same for the three steps, determine: (a) length
and (b) exit velocity of the slab after the final reduction.
Solution: (a) After three passes, tf = (0.75)(0.75)(0.75)(2.0) = 0.844 in.
wf = (1.03)(1.03)(1.03)(10.0) = 10.927 in.
towoLo = tfwfLf
(2.0)(10.0)(12 x 12) = (0.844)(10.927)Lf
Lf = (2.0)(10.0)(12 x 12)/(0.844)(10.927) = 312.3 in. = 26.025 ft
(b) Given that roll speed is the same at all three stands and that towovo = tfwfvf ,
Step 1: vf = (2.0)(10.0)(40)/(0.75 x 2.0)(1.03 x 10.0) = 51.78 ft/min.
Step 2: vf = (0.75 x 2.0)(1.03 x 10.0)(40)/(0.752 x 2.0)(1.032 x 10.0) = 51.78 ft/min.
Step 3: vf = (0.752 x 2.0)(1.032 x 10.0)(40)/(0.753 x 2.0)(1.033 x 10.0) = 51.78 ft/min.
19.3 A series of cold rolling operations are to be used to reduce the thickness of a plate from 50 mm
down to 25 mm in a reversing two-high mill. Roll diameter = 700 mm and coefficient of friction
between rolls and work = 0.15. The specification is that the draft is to be equal on each pass.
Determine: (a) minimum number of passes required, and (b) draft for each pass?
Solution: (a) Maximum draft dmax = 2 R = (0.15)2 (350) = 7.875 mm
Minimum number of passes = (to - tf)/dmax = (50 - 25)/7.875 = 3.17 ® 4 passes
(b) Draft per pass d = (50 - 25)/4 = 6.25 mm
19.4 In the previous problem, suppose that the percent reduction were specified to be equal for each
pass, rather than the draft. (a) What is the minimum number of passes required? (b) What is the
draft for each pass?
Solution: (a) Maximum possible draft occurs on first pass: dmax = 2 R = (0.15)2 (350) = 7.875 mm
This converts into a maximum possible reduction x = 7.875/50 = 0.1575
Let x = fraction reduction per pass, and n = number of passes. The number of passes must be an
integer. To reduce from to = 50 mm to to = 25 mm in n passes, the following relationship must be
satisfied:
50(1 - x)n = 25
(1 - x)n = 25/50 = 0.5
(1 - x) = 0.51/n
Try n = 4: (1 - x) = (.5)1/4 = 0.8409
x = 1 - 0.8409 = 0.1591, which exceeds the maximum possible reduction of 0.1575.
Try n = 5: (1 - x) = (.5)1/5 = 0.87055
x = 1 - 0.87055 = 0.12945, which is within the maximum possible reduction of 0.1575.
(b) Pass 1: d = 50(0.12945) = 6.47 mm, tf = 50 - 6.47 = 43.53 mm
Pass 2: d = 43.53(0.12945) = 5.63 mm, tf = 43.53 - 5.63 = 37.89 mm
Pass 3: d = 37.89(0.12945) = 4.91 mm, tf = 37.89 - 4.91 = 32.98 mm
Pass 4: d = 32.98(0.12945) = 4.27 mm, tf = 32.98 - 4.27 = 28.71 mm
Pass 5: d = 28.71(0.12945) = 3.71 mm, tf = 28.71 - 3.71 = 25.00 mm
19.5 A continuous hot rolling mill has two stands. Thickness of the starting plate = 25 mm and width =
300 mm. Final thickness is to be 13 mm. Roll radius at each stand = 250 mm. Rotational speed at
the first stand = 20 rev/min. Equal drafts of 6 mm are to be taken at each stand. The plate is wide
enough relative to its thickness that no increase in width occurs. Under the assumption that the
forward slip is equal at each stand, determine: (a) speed vr at each stand, and (b) forward slip s.
101
(c) Also, determine the exiting speeds at each rolling stand, if the entering speed at the first stand =
26 m/min.
Solution: (a) Let to = entering plate thickness at stand 1. to = 25 mm. Let t1 = exiting plate
thickness at stand 1 and entering thickness at stand 2. t1 = 25 - 6 = 19 mm.
Let t2 = exiting plate thickness at stand 2. t2 = 19 - 6 = 13 mm.
Let vo = entering plate speed at stand 1.
Let v1 = exiting plate speed at stand 1 and entering speed at stand 2.
Let v2 = exiting plate speed at stand 2.
Let vr1 = roll speed at stand 1. vr1 = DNr = (2 x 250)(10-3)(20) = 31.42 m/min.
Let vr2 = roll speed at stand 2. vr2 = ?
Forward slip s = (vf - vr)/vr
svr = vf - vr
(1 + s)vr = vf
At stand 1, (1 + s)vr1 = v1 (Eq. 1)
At stand 2, (1 + s)vr2 = v2 (Eq. 2)
By constant volume, towovo = t1w1v1 = t2w2v2
Since there is no change in width, wo = w1 = w2
Therefore, tovo = t1v1 = t2v2
1.0vo = 0.75v1 = 0.50v2
v2 = 1.5v1 (Eq. 3)
Combining (Eqs. 2 and 3), (1 + s)vr2 = v2 = 1.5v1
Substituting (Eq. 1), (1 + s)vr2 = 1.5(1 + s)vr1 , thus vr2 = 1.5vr1
vr2 = 1.5(31.42) = 47.1 m/min.
(b) 25vo = 19v1
v1 = 25(26)/19 = 34.2 m/min
(Eq. 1): (1 + s)vr1 = v1
(1 + s)(31.4) = 34.2
(1 + s) = 34.2/31.4 = 1.089
s = 0.089
(c) v1 = 34.2 m/min, previously calculated in (b)
v2 = 1.5v1 = 1.5(34.2) = 51.3 m/min.
19.6 A continuous hot rolling mill has eight stands. The dimensions of the starting slab are: thickness =
3.0 in, width = 15.0 in, and length = 10 ft. The final thickness is to be 0.3 in. Roll diameter at each
stand = 36 in, and rotational speed at stand number 1 = 30 rev/min. It is observed that the speed of
the slab entering stand 1 = 240 ft/min. Assume that no widening of the slab occurs during the
rolling sequence. Percent reduction in thickness is to be equal at all stands, and it is assumed that
the forward slip will be equal at each stand. Determine: (a) percent reduction at each stand, (b)
rotational speed of the rolls at stands 2 through 8, and (c) forward slip. (d) What is the draft at
stands 1 and 8? (e) What is the length and exit speed of the final strip exiting stand 8?
Solution: (a) To reduce from to = 3.0 in. to tf = 0.3 in. in 8 stands, 3.0(1 - x)8 = 0.3
(1 - x)8 = 0.3/3.0 = 0.10
(1 - x) = (0.10)1/8 = 0.74989
x = 1 - 0.74989 = r = 0.2501 = 25.01% at each stand.
(b) Forward slip s = (vf - vr)/vr
svr = vf - vr
(1 + s)vr = vf
102
At stand 1: (1 + s)vr1 = v1 , where vr1 = roll speed, v1 = exit speed of slab.
At stand 2: (1 + s)vr2 = v2 , where vr2 = roll speed, v2 = exit speed of slab.
Etc.
At stand 8: (1 + s)vr8 = v8, where vr8 = roll speed, v8 = exit speed of slab.
By constant volume, towovo = t1w1v1 = t2w2v2 = . . . = t8w8v8
Since there is no change in width, wo = w1 = w2 = . . . w8
Therefore, tovo = t1v = t2v2 = . . . = t8v8
to = 3.0,
3vo = 3(1 - r)v1 = 3(1 - r)2v2 = . . . 3(1 - r)8v8 , where r = 0.2501 as determined in part (a).
Since s is a constant, vr1 : vr2 : . . . : vr8 = v1 : v2 : . . . : v8
Given that Nr1 = 30 rev/min, vr1 = DNr1 = (2x 18/12)(30) = 282.78 ft/min
In general Nr = (30/282.78) = 0.10609vr
Nr2 = 0.10609 x 282.78/(1-r) = 0.10609 x 282.78/(1-0.2501) = 40 rev/min
Nr3 = 0.10609 x 282.78/(1-r)2 = 53.3 rev/min
Nr4 = 0.10609 x 282.78/(1-r)3 = 71.1 rev/min
Nr5 = 0.10609 x 282.78/(1-r)4 = 94.9 rev/min
Nr6 = 0.10609 x 282.78/(1-r)5 = 126.9.3 rev/min
Nr7 = 0.10609 x 282.78/(1-r)6 = 168.5 rev/min
Nr8 = 0.10609 x 282.78/(1-r)7 = 224.9 rev/min
(c) Given vo = 240 ft/min
v1 = 240/(1-r) = 240/0.74989 = 320 ft/min
v2 = 320/0.74989 = 426.8 ft/min
From equations for forward slip, (1 + s)vr1 = v1
(1 + s)(282.78) = 320
(1 + s) = 320/282.78 = 1.132 s = 0.132
Check with stand 2: given v2 = 426.8 ft/min from above
Nr2 = 0.10609vr2
Rearranging, vr2 = Nr2/0.10609 = 9.426Nr2 = 0.426(40) = 377.04 ft/min
(1 + s)(377.04) = 426.8
(1 + s) = 426.8/377.14 = 1.132 s = 0.132, as before
(d) Draft at stand 1 d1 = 3.0(.2501) = 0.7503 in.
Draft at stand 8 d8 = 3.0(1 - .2501)7(.2501) = 0.10006 in.
(e) Length of final strip Lf = L8
towoLo = t8w8L8
Given that wo = w8, toLo = t8L8
3.0(10 ft) = 0.3L8 L8 = 100 ft
towovo = t8w8v8
tovo = t8v8
v8 = 240(3/0.3) = 2400 ft/min.
19.7 A plat that is 250 mm wide and 25 mm thick is to be reduced in a single pass in a two-high rolling
mill to a thickness of 20 mm. The roll has a radius = 500 mm, and its speed = 30 m/min. The work
material has a strength coefficient = 240 MPa and a strain hardening exponent = 0.2. Determine:
(a) roll force, (b) roll torque, and (c) power required to accomplish this operation.
Solution: (a) Draft d = 25 - 20 = 5 mm,
Contact length L = (500 x 5).5 = 50 mm
True strain = ln(25/20) = ln 1.25 = 0.223
103
Y
f
= 240(0.223)0.20/1.20 = 148.1 MPa
Rolling force F = 148.1(250)(50) = 1,851,829 N
(b) Torque T = 0.5(1,851,829)( 50 x 10-3) = 46,296 N-m
(c) N = (30 m/min)/(2x 0.500) = 9.55 rev/min = 0.159 rev/s
Power P = 2(0.159)(1,851,829)(50 x 10-3) = 92,591 N-m/s = 92,591 W
19.8 Solve Problem 19.7 using a roll radius = 250 mm.
Solution: (a) Draft d = 25 - 20 = 5 mm,
Contact length L = (250 x 5).5 = 35.35 mm
True strain = ln(25/20) = ln 1.25 = 0.223
Y
f
= 240(0.223)0.20/1.20 = 148.1 MPa
Rolling force F = 148.1(250)(35.35) = 1,311,095 N
(b) Torque T = 0.5(1,311,095)(35.35 x 10-3) = 23,174 N-m
(c) N = (30 m/min)/(2x 0.250) = 19.1 rev/min = 0.318 rev/s
Power P = 2(0.318)(1,311,095)(35.35 x 10-3) = 92,604 N-m/s = 92,604 W
Note that the force and torque are reduced as roll radius is reduced, but that the power remains
the same (within calculation error) as in the previous problem.
19.9 Solve Problem 19.7, only assume a cluster mill with working rolls of radius = 50 mm. Compare the
results with the previous two problems, and note the important effect of roll radius on force, torque
and power.
Solution: (a) Draft d = 25 - 20 = 5 mm,
Contact length L = (50 x 5).5 = 15.81 mm
True strain = ln(25/20) = ln 1.25 = 0.223
Y
f
= 240(0.223)0.20/1.20 = 148.1 MPa
Rolling force F = 148.1(250)(15.81) = 585,417 N
(b) Torque T = 0.5(585,417)(15.81 x 10-3) = 4,628 N-m
(c) N = (30 m/min)/(2x 0.050) = 95.5 rev/min = 1.592 rev/s
Power P = 2(1.592)(585,417)(15.81 x 10-3) = 92,554 N-m/s = 92,554 W
Note that this is the same power value (within calculation error) as in Problems 19.7 and 19.8. In
fact, power would probably increase because of lower mechanical efficiency in the cluster type
rolling mill.
19.10 A 3.0 in thick slab that is 9 in wide is to be reduced in a single pass in a two-high rolling mill to a
thickness of 2.50 in. The roll has a radius = 15 in, and its speed = 30 ft/min. The work material has
a strength coefficient = 25,000 lb/in2 and a strain hardening exponent = 0.16. Determine: (a) roll
force, (b) roll torque, and (c) power required to accomplish this operation.
Solution: (a) Draft d = 3.0 - 2.5 = 0.5 in.,
Contact length L = (15 x 0.5).5 = 2.74 in.
True strain = ln(3.0/2.5) = ln 1.20 = 0.1823
Y
f
= 25,000(0.1823)0.16/1.16 = 16,414 lb/in2
Rolling force F = 16,414(2.74)(9.0) = 404,771 lb
104
(b) Torque T = 0.5(401,771)(2.74) = 554,536 in-lb.
(c) N = (30 ft/min)/(2x 15/12) = 3.82 rev/min.
Power P = 2(3.82)(404,771)(2.74) = 26,617,741 in-lb/min
HP = (26,617,741 in-lb/min)/(396,000) = 67.2 hp
19.11 A single-pass rolling operation reduces a 20 mm thick plate to 18 mm. The starting plate is 200 mm
wide. Roll radius = 250 mm and rotational speed = 12 rev/min. The work material has a strength
coefficient = 600 MPa and a strength coefficient = 0.22. Determine: (a) roll force, (b) roll torque,
and (c) power required for this operation.
Solution: (a) Draft d = 20 - 18 = 2.0 mm,
Contact length L = (250 x 2).5 = 11.18 mm = 0.0112 m
True strain = ln(20/18) = ln 1.111 = 0.1054
Y
f
= 600(0.1054)0.22/1.22 = 300 MPa
Rolling force F = 300(0.0112)(0.2) = 0.672 MN = 672,000 N
(b) Torque T = 0.5(672,000)(0.0112) = 3,720 N-m
(c) Given that N = 12 rev/min
Power P = 2(12/60)(672,000)(0.0112) = 37,697 W
19.12 A hot rolling mill has rolls of diameter = 24 in. It can exert a maximum force = 400,000 lb. The mill
has a maximum horsepower = 100 hp. It is desired to reduce a 1.5 in thick plate by the maximum
possible draft in one pass. The starting plate is 10 in wide. In the heated condition, the work
material has a strength coefficient = 20,000 lb/in2 and a strain hardening exponent = zero.
Determine: (a) maximum possible draft, (b) associated true strain, and (c) maximum speed of the
rolls for the operation.
Solution: (a) Assumption: maximum possible draft is determined by the force capability of the
rolling mill and not by coefficient of friction between the rolls and the work.
Draft d = 1.5 - tf
Contact length L = (12d)0.5
Y
f
= 20,000()0/1.0 = 20,000 lb/in2
Force F = 20,000(10) (12d)0.5 = 400,000 (the limiting force of the rolling mill)
(12d)0.5 = 400,000/200,000 = 2.0
12 d = 2.02 = 4
d = 4/12 = 0.333 in.
(b) True strain = ln(1.5/tf)
tf = to -d = 1.5 - 0.333 = 1.167 in.
= ln(1.5/1.167) = ln 1.285 = 0.251
(c) Given maximum possible power HP = 100 hp = 100 x 396000 (in-lb/min)/hp = 39,600,000
in-lb/min
Contact length L = (12 x 0.333)0.5 = 2.0 in.
P = 2N(400,000)(2.0) = 5,026,548N in-lb/min
5,026,548N = 39,600,000
N = 7.88 rev/min
vr = 2RN = 2(12/12)(7.88) = 49.5 ft/min
19.13 Solve Problem 19.12 except that the operation is warm rolling and the strain hardening exponent n
= 0.15. Assume the strength coefficient remains K = 20,000 lb/in2.
105
Solution: (a) Assumption (same as in previous problem): maximum possible draft is determined by
the force capability of the rolling mill and not by coefficient of friction between the rolls and the
work.
Draft d = 1.5 - tf
Contact length L = (12d)0.5
= ln(1.5/tf)
Y
f
= 20,000()0.15/1.15 = 17,391.15
F = Y
f
(10)(12d)0.5 = 34.641 Y
f
(d)0.5 = 400,000 (as given)
Y
f
(d)0.5 = 400,000/34.641 = 11,547
Now use trial-and-error to values of Y
f
and d that fit this equation.
Try d = 0.3 in., tf = 1.5 - 0.3 = 1.2 in.
= ln(1.5/1.2) = ln 1.25 = 0.223
Y
f
= 17,391(0.223).15 = 13,887 lb/in2.
(d)0.5 = 11,547/13,887 = 0.8315
d = 0.691, which does not equal the initial trial value of d = 0.3
Try d = 0.5 in., tf = 1.5 - 0.5 = 1.0 in.
= ln(1.5/1.0) = ln 1.50 = 0.4055
Y
f
= 17,391(0.4055).15 = 15,189 lb/in2.
(d)0.5 = 11,547/15,189 = 0.7602
d = 0.578, which does not equal the trial value of d = 0.5
Try d = 0.55 in., tf = 1.5 - 0.55 = 0.95 in.
= ln(1.5/0.95) = ln 1.579 = 0.457
Y
f
= 17,391(0.457).15 = 15,462 lb/in2.
(d)0.5 = 11,547/15,462 = 0.747
d = 0.558, which is close to the trial value of d = 0.55
Try d = 0.555 in., tf = 1.50 - 0.555 = 0.945 in.
= ln(1.5/0.945) = ln 1.5873 = 0.462
Y
f
= 17,391(0.462).15 = 15,489 lb/in2.
(d)0.5 = 11,547/15,489 = 0.745
d = 0.556, which is very close to the trial value of d = 0.555.
(b) True strain = ln(1.5/0.945) = 0.462
(c) Given maximum possible power HP = 100 hp = 100 x 396000 (in-lb/min)/hp
= 39,600,000 in-lb/min
Contact length L = (12 x 0.555)0.5 = 2.58 in.
P = 2N(400,000)(2.58) = 6,486,000N in-lb/min
6,486,000N = 39,600,000
N = 6.11 rev/min
vr = 2RN = 2(12/12)(6.11) = 38.4 ft/min
Forging
19.14 A cylindrical part is warm upset forged in an open die. Do = 50 mm and ho = 40 mm. Final height =
20 mm. Coefficient of friction at the die -work interface = 0.20. The work material has a flow
106
curve defined by: K = 600 MPa and n = 0.12. Determine the force in the operation (a) just as the
yield point is reached (yield at strain = 0.002), (b) at h = 30 mm, and (c) at h = 20 mm.
Solution: (a) V = D2L/4 = (50)2(40)/4 = 78,540 mm3
Given = 0.002, Yf = 600(0.002)0.12 = 284.6 MPa, and h = 40 - 40(0.002) = 39.92
A = V/h = 78,540/39.92 = 1963.5 mm2
Kf = 1 + 0.4(.2)(50)/39.92 = 1.1
F = 1.1(284.6)(1963.5) = 614,693 N
(b) Given h = 30, = ln(40/30) = ln 1.333 = 0.287
Yf = 600(0.287)0.12 = 516.6 MPa
V = 78,540 mm3 from part (a) above.
At h = 30, A = V/h = 78,540/30 = 2618 mm2
Corresponding D = 57.7 mm (from A = D2/4)
Kf = 1 + 0.4(.2)(57.7)/30 = 1.154
F = 1.154(516.6)(2618) = 1,560,557 N
(c) Given h = 20, = ln(40/20) = ln 2.0 = 0.693
Yf = 600(0.693)0.12 = 574.2 MPa
V = 78,540 mm3 from part (a) above.
At h = 20, A = V/h = 78,540/20 = 3927 mm2
Corresponding D = 70.7 mm (from A = D2/4)
Kf = 1 + 0.4(.2)(70.7)/20 = 1.283
F = 1.283(574.2)(3927) = 2,892,661 N
19.15 A cylindrical workpart with D = 2.5 in and h = 2.5 in is upset forged in an open die to a height =
1.5 in. Coefficient of friction at the die -work interface = 0.10. The work material has a flow curve
defined by: K = 40,000 lb/in2 and n = 0.15. Determine the instantaneous force in the operation (a)
just as the yield point is reached (yield at strain = 0.002), (b) at height h = 2.3 in, (c) h = 1.9 in, and
(d) h = 1.5 in.
Solution: (a) V = D2L/4 = (2.5)2(2.5)/4 = 12.273 in3
Given = 0.002, Yf = 40,000(0.002)0.15 = 15,748 lb/in2 and h = 2.5 - 2.5(0.002) = 2.495
A = V/h = 12.273/2.495 = 4.92 in2
Kf = 1 + 0.4(.1)(2.5)/2.495 = 1.04
F = 1.04(15,748)(4.92) = 80,579 lb
(b) Given h = 2.3, = ln(2.5/2.3) = ln 1.087 = 0.0834
Yf = 40,000(0.0834)0.15 = 27,556 lb/in2
V = 12.273 in3 from part (a) above.
At h = 2.3, A = V/h = 12.273/2.3 = 5.34 in2
Corresponding D = 2.61 (from A = D2/4)
Kf = 1 + 0.4(.1)(2.61)/2.3 = 1.045
F = 1.045(27,556)(4.34) = 153,822 lb
(c) Given h = 1.9, = ln(2.5/1.9) = ln 1.316 = 0.274
Yf = 40,000(0.274)0.15 = 32,948 lb/in2
V = 12.273 in3 from part (a) above.
At h = 1.9, A = V/h = 12.273/1.9 = 6.46 in2
Corresponding D = 2.87 (from A = D2/4)
Kf = 1 + 0.4(.1)(2.87)/1.9 = 1.060
F = 1.060(32,948)(6.46) = 225,695 lb
107
(d) Given h = 1.5, = ln(2.5/1.5) = ln 1.667 = 0.511
Yf = 40,000(0.511)0.15 = 36,166 lb/in2
V = 12.273 in3 from part (a) above.
At h = 1.5, A = V/h = 12.273/1.5 = 8.182 in2
Corresponding D = 3.23 (from A = D2/4)
Kf = 1 + 0.4(.1)(3.23)/1.5 = 1.086
F = 1.086(36,166)(8.182) = 321,379 lb
19.16 A cylindrical workpart has a diameter = 2.0 in and a height = 4.0 in. It is upset forged to a height =
2.5 in. Coefficient of friction at the die -work interface = 0.10. The work material has a flow curve
with strength coefficient = 25,000 lb/in2 and strain hardening exponent = 0.22. Determine the plot
of force vs. work height.
Solution: Volume of cylinder V = D2L/4 = (2.5)2(4.0)/4 = 19.635 in3
We will compute the force F at selected values of height h: h = (a) 4.0, (b) 3.75, (c) 3.5, (d) 3.25,
(e) 3.0, (f) 2.75, and (g) 2.5. These values can be used to develop the plot. The shape of the plot
will be similar to Figure 21.13 in the text.
(a) At h = 4.0, we assume yielding has just occurred and the height has not changed significantly.
Use = 0.002 (the approximate yield point of metal).
At = 0.002, Yf = 25,000(0.002)0.22 = 6,370 lb/in2
Adjusting the height for this strain, h = 4.0 - 4.0(0.002) = 3.992
A = V/h = 19.635/3.992 = 4.92 in2
Kf = 1 + 0.4(.1)(2.5)/3.992 = 1.025
F = 1.025(6,370)(4.92) = 32,125 lb
(b) At h = 3.75, = ln(4.0/3.75) = ln 1.0667 = 0.0645
Yf = 25,000(0.0645)0.22 = 13,680 lb/in2
V = 19.635 in3 calculated above.
At h = 3.75, A = V/h = 19.635/3.75 = 5.236 in2
Corresponding D = 2.582 (from A = D2/4)
Kf = 1 + 0.4(.1)(2.582)/3.75 = 1.028
F = 1.028(13,680)(5.236) = 73,601 lb
(c) At h = 3.5, = ln(4.0/3.5) = ln 1.143 = 0.1335
Yf = 25,000(0.1335)0.22 = 16,053 lb/in2
At h = 3.5, A = V/h = 19.635/3.5 = 5.61 in2
Corresponding D = 2.673 (from A = D2/4)
Kf = 1 + 0.4(.1)(2.673)/3.5 = 1.031
F = 1.031(16,053)(5.61) = 92,808 lb
(d) At h = 3.25, = ln(4.0/3.25) = ln 1.231 = 0.2076
Yf = 25,000(0.2076)0.22 = 17,691 lb/in2
At h = 3.25, A = V/h = 19.635/3.25 = 6.042 in2
Corresponding D = 2.774 (from A = D2/4)
Kf = 1 + 0.4(.1)(2.774)/3.25 = 1.034
F = 1.034(17,691)(6.042) = 110,538 lb
(e) At h = 3.0, = ln(4.0/3.0) = ln 1.333 = 0.2874
Yf = 25,000(0.2874)0.22 = 19,006 lb/in2
At h = 3.0, A = V/h = 19.635/3.0 = 6.545 in2
Corresponding D = 2.887 (from A = D2/4)
108
Kf = 1 + 0.4(.1)(2.887)/3.0 = 1.038
F = 1.038(19,006)(6.545) = 129,182 lb
(f) At h =2.75, = ln(4.0/2.75) = ln 1.4545 = 0.3747
Yf = 25,000(0.3747)0.22 = 20,144 lb/in2
V = 19.635 in3 calculated above.
At h = 2.75, A = V/h = 19.635/2.75 = 7.140 in2
Corresponding D = 3.015 (from A = D2/4)
Kf = 1 + 0.4(.1)(3.015)/2.75 = 1.044
F = 1.044(20,144)(7.140) = 150,136 lb
(g) At h = 2.5, = ln(4.0/2.5) = ln 1.60 = 0.470
Yf = 25,000(0.470)0.22 = 21,174 lb/in2
At h = 2.5, A = V/h = 19.635/2.5 = 7.854 in2
Corresponding D = 3.162 (from A = D2/4)
Kf = 1 + 0.4(.1)(3.162)/2.5 = 1.051
F = 1.051(21,174)(7.854) = 174,715 lb
19.17 A cold heading operation is performed to produce the head on a steel nail. The strength coefficient
for this steel is K = 550 MPa, and the strain hardening exponent n = 0.24. Coefficient of friction at
the die-work interface = 0.10. The wire stock out of which the nail is made is 4.75 mm in diameter.
The head is to have a diameter = 9.5 mm and a thickness = 1.5 mm. (a) What length of stock must
project out of the die in order to provide sufficient volume of material for this upsetting operation?
(b) Compute the maximum force that the punch must apply to form the head in this open-die
operation.
Solution: (a) Volume of nail head V = Df
2hf/4 = (9.5)2(1.5)/4 = 106.3 mm3.
Ao = Do
2/4 = (4.75)2/4 = 17.7 mm2
ho = V/Ao = 106.3/17.7 = 6.0 mm
(b) = ln(6.0/1.5) = ln 4 = 1.3863
Yf = 550(1.3863)0.24 = 595 MPa
Af = (9.5)2/4 = 70.9 mm2
Kf = 1 + 0.4(.1)(9.5/1.5) = 1.25
F = 1.25(595)(70.9) = 52,872 N
19.18 Obtain a large common nail (flat head). Measure the head diameter and thickness, as well as the
diameter of the nail shank. (a) What stock length must project out of the die in order to provide
sufficient material to produce the nail? (b) Using appropriate values for strength coefficient and
strain hardening exponent for the metal out of which the nail is made (Table 3.5), compute the
maximum force in the heading operation to form the head.
Solution: Student exercise. Calculations similar to those above for the data developed by the
student.
19.19 A hot upset forging operation is performed in an open die. The initial size of the workpart is: Do =
25 mm, and ho = 50 mm. The part is upset to a diameter = 50 mm. The work metal at this elevated
temperature yields at 85 MPa (n = 0). Coefficient of friction at the die -work interface = 0.40.
Determine: (a) final height of the part, and (b) maximum force in the operation.
Solution: (a) V = Do
2ho/4 = (25)2(50)/4 = 24,544 mm3.
Af = Df
2/4 = (50)2/4 = 1963.5 mm2.
hf = V/Af = 24,544 /1963.5 = 12.5 mm.
109
(b) = ln(50/12.5) = ln 4 = 1.3863
Yf = 85(1.3863)0 = 85 MPa
Force is maximum at largest area value, Af = 1963.5 mm2
D = (4 x 1963.5/)0.5 = 50 mm
Kf = 1 + 0.4(.4)(50/12.5) = 1.64
F = 1.64(85)( 1963.5) = 273,712 N
19.20 A hydraulic forging press is capable of exerting a maximum force = 1,000,000 N. A cylindrical
workpart is to be cold upset forged. The starting part has diameter = 30 mm and height = 30 mm.
The flow curve of the metal is defined by K = 400 MPa and n = 0.2. Determine the maximum
reduction in height to which the part can be compressed with this forging press, if the coefficient of
friction = 0.1.
Solution: Volume of work V = Do
2ho/4 = (30)2(30)/4 = 21,206 mm3.
Final area Af = 21,206/hf
= ln(30/hf)
Yf = 4000.2 = 400(ln 30/hf)0.2)
Kf = 1 + 0.4(Df/hf) = 1 + 0.4(0.1)(Df/hf)
Forging force F = KfYfAf = (1 + 0.04Df/hf)( 400(ln 30/hf)0.2)( 21,206/hf)
Requires trial and error solution to find the value of hf that will match the force of 1,000,000 N.
(1) Try hf = 20 mm
Af = 21,206/20 = 1060.3 mm2
= ln(30/20) = ln 1.5 = 0.405
Yf = 400(0.405)0.2 = 333.9 MPa
Df = (4 x 1060.3/)0.5 = 36.7 mm
Kf = 1 + 0.04(36.7/20) = 1.073
F = 1.073(333.9)(1060.3) = 380,050 N
Too low. Try a smaller value of hf to increase F.
(2) Try hf = 10 mm.
Af = 21,206/10 = 2120.6 mm2
= ln(30/10) = ln 3.0 = 1.099
Yf = 400(1.099)0.2 = 407.6 MPa
Df = (4 x 2120.6/)0.5 = 51.96 mm
Kf = 1 + 0.04(51.96/10) = 1.208
F = 1.208(407.6)(2120.6) = 1,043,998 N
Slightly high. Need to try a value of hf between 10 and 20, closer to 10.
(3) Try hf = 11 mm
Af = 21,206/11 = 1927.8 mm2
= ln(30/11) = ln 2.7273 = 1.003
Yf = 400(1.003)0.2 = 400.3 MPa
Df = (4 x 1927.8/)0.5 = 49.54 mm
Kf = 1 + 0.04(51.12/11) = 1.18
F = 1.18(400.3)(1927.8) = 910,653 N
(4) By linear interpolation, try hf = 10 + (44/133) = 10.33 mm
Af = 21,206/10.33 = 2052.8 mm2
= ln(30/10.33) = ln 2.9042 = 1.066
Yf = 400(1.066)0.2 = 405.16 MPa
110
Df = (4 x 2052.8/)0.5 = 51.12 mm
Kf = 1 + 0.04(51.12/10.33) = 1.198
F = 1.198(405.16)(2052.8) = 996,364 N
(5) By further linear interpolation, try hf = 10 + (44/48)(0.33) = 10.30
Af = 21,206/10.30 = 2058.8 mm2
= ln(30/10.30) = ln 2.913 = 1.069
Yf = 400(1.069)0.2 = 405.38 MPa
Df = (4 x 2058.8/)0.5 = 51.2 mm
Kf = 1 + 0.04(51.2/10.3) = 1.199
F = 1.199(405.38)(2058.8) = 1,000,553 N
Close enough! Maximum height reduction = 30.0 - 10.3 = 19.7 mm
19.21 A part is designed to be hot forged in an impression die. The projected area of the part, including
flash, is 15 in2. After trimming, the part has a projected area = 10 in2. Part geometry is relatively
simple. As heated the work material yields at 9,000 lb/in2, and has no tendency to strain harden.
Determine the maximum force required to perform the forging operation.
Solution: Since the work material has no tendency to work harden, n = 0.
From Table 21.1, choose Kf = 6.0.
F = 6.0(9,000)(15) = 810,000 lb.
19.22 A connecting rod is designed to be hot forged in an impression die. The projected area of the part
is 6,500 mm2. The design of the die will cause flash to form during forging, so that the area,
including flash, will be 9,000 mm2. The part geometry is considered to be complex. As heated the
work material yields at 75 MPa, and has no tendency to strain harden. Determine the maximum
force required to perform the operation.
Solution: Since the work material has no tendency to work harden, n = 0.
From Table 21.1, choose Kf =8.0.
F = 8.0(75)(9,000) = 5,400,000 N.
Extrusion
19.23 A cylindrical billet that is 100 mm long and 40 mm in diameter is reduced by indirect (backward)
extrusion to a 15 mm diameter. Die angle = 90. If the Johnson equation has a = 0.8 and b = 1.5,
and the flow curve for the work metal has K = 750 MPa and n = 0.15, determine: (a) extrusion
ratio, (b) true strain (homogeneous deformation), (c) extrusion strain, (d) ram pressure, and (e) ram
force.
Solution: (a) rx = Ao/Af = Do
2/Df
2 = (40)2/(15)2 = 7.111
(b) = ln rx = ln 7.111 = 1.962
(c) x = a + b ln rx = 0.8 + 1.5(1.962) = 3.742
(d) Y
f
= 750(1.962)0.15/1.15 = 721.5 MPa,
p = 721.5(3.742) = 2700 MPa
(e) Ao = Do
2/4 = (40)2/4 = 1256.6 mm2
F = 2700(1256.6) = 3,392,920 N.
19.24 A 3.0-in long cylindrical billet whose diameter = 1.5 in is reduced by indirect extrusion to a
diameter = 0.375 in. Die angle = 90. In the Johnson equation, a = 0.8 and b = 1.5. In the flow
111
curve for the work metal, K = 75,000 lb/in2 and n = 0.25. Determine: (a) extrusion ratio, (b) true
strain (homogeneous deformation), (c) extrusion strain, (d) ram pressure, (e) ram force, and (f)
power if the ram speed = 20 in/min.
Solution: (a) rx = Ao/Af = Do
2/Df
2 = (1.5)2/(0.375)2 = 42 = 16.0
(b) = ln rx = ln 16 = 2.773
(c) x = a + b ln rx = 0.8 + 1.5(2.773) = 4.959
(d) Y
f
= 75,000(2.773)0.25/1.25 = 77,423 lb/in2
p = 77,423(4.959) = 383,934 lb/in2
(e) Ao = Do
2/4 = (1.5)2/4 = 1.767 in2
F = (383,934)(1.767) = 678,411 lb.
(f) P = 678,411(20) = 13,568,228 in-lb/min
HP = 13,568,228/396,000 = 34.26 hp
19.25 A billet that is 75 mm long with diameter = 35 mm is direct extruded to a diameter of 20 mm. The
extrusion die has a die angle = 75. For the work metal, K = 600 MPa and n = 0.25. In the Johnson
extrusion strain equation, a = 0.8 and b = 1.4. Determine: (a) extrusion ratio, (b) true strain
(homogeneous deformation), (c) extrusion strain, and (d) ram pressure at L = 70, 40, and 10 mm.
Solution: (a) rx = Ao/Af = Do
2/Df
2 = (35)2/(20)2 = 3.0625
(b) = ln rx = ln 3.0625 = 1.119
(c) x = a + b ln rx = 0.8 + 1.4(1.119) = 2.367
(d) Y
f
= 750(1.119)0.25/1.25 = 493.7 MPa
It is appropriate to determine the volume of metal contained in the cone of the die at the start of the
extrusion operation, to assess whether metal has been forced through the die opening by the time
the billet has been reduced from L = 75 mm to L = 70 mm. For a cone-shaped die with angle =
75, the height h of the frustum is formed by metal being compressed into the die opening: The two
radii are: R1 = 0.5Do = 17.5 mm and R2 = 0.5Df = 10 mm, and h = (R1 - R2)/tan 75 = 7.5/tan 75 =
2.01 mm
Frustum volume V = 0.333h(R1
2 + R1R2 + R2
2) = 0.333(2.01)(17.52 + 10 x 17.5 + 102) = 1223.4
mm3. Compare this with the volume of the portion of the cylindrical billet between L = 75 mm and
L = 70 mm.
V = Do
2 h/4 = 0.25(35)2(75 - 70) = 4810.6 mm3
Since this volume is greater than the volume of the frustum, this means that the metal has extruded
through the die opening by the time the ram has moved forward by 5 mm.
L = 70 mm: pressure p = 493.7(2.367 + 2 x 70/35) = 3143.4 MPa
L = 40 mm: pressure p = 493.7(2.367 + 2 x 40/35) = 2297.0 MPa
L = 10 mm: pressure p = 493.7(2.367 + 2 x 10/35) = 1450.7 MPa
19.26 A 2.0-in long billet with diameter = 1.25 in is direct extruded to a diameter of 0.50 in. The extrusion
die angle = 90. For the work metal, K = 45,000 lb/in2, and n = 0.20. In the Johnson extrusion strain
equation, a = 0.8 and b = 1.5. Determine: (a) extrusion ratio, (b) true strain (homogeneous
deformation), (c) extrusion strain, and (d) ram pressure at L = 2.0, 1.5, 1.0, 0.5 and zero in.
Solution: (a) rx = Ao/Af = Do
2/Df
2 = (1.25)2/(0.5)2 = 6.25
112
(b) = ln rx = ln 6.25 = 1.8326
(c) x = a + b ln rx = 0.8 + 1.5(1.8326) = 3.549
(d) Y
f
= 45,000(1.8326)0.05/1.20 = 42,330 lb/in2
Unlike the previous problem, the die angle = 90, so metal is forced through the die opening as
soon as the billet starts to move forward in the chamber.
L = 2.0 in.: pressure p = 42,330(3.549 + 2 x 2.0/1.25) = 285,677 lb/in2
L = 1.5 in.: pressure p = 42,330(3.549 + 2 x 1.5/1.25) = 251,813 lb/in2
L = 1.0 in.: pressure p = 42,330(3.549 + 2 x 1.0/1.25) = 217,950 lb/in2
L = 0.5 in.: pressure p = 42,330(3.549 + 2 x 0.5/1.25) = 184,086 lb/in2
L = 0.0 in.: pressure p = 42,330(3.549 + 2 x 0.0/1.25) = 150,229 lb/in2
19.27 A direct extrusion operation is performed on a cylindrical billet with Lo = 3.0 in and Do = 2.0 in. Die
angle = 45and orifice diameter = 0.50 in. In the Johnson extrusion strain equation, a = 0.8 and b =
1.3. The operation is carried out hot and the hot metal yields at 15,000 lb/in2 (n = 0). (a) What is
the extrusion ratio? (b) Determine the ram position at the point when the metal has been
compressed into the cone of the die and starts to extrude through the die opening. (c) What is the
ram pressure corresponding to this position? (d) Also determine the length of the final part if the
ram stops its forward movement at the start of the die cone.
Solution: (a) rx = Ao/Af = Do
2/Df
2 = (2.0)2/(0.5)2 = 16.0
(b) The portion of the billet that is compressed into the die cone forms a frustum with R1 = 0.5Do =
1.0 in and R2 = 0.5Df = 0.25 in. The height of the frustum h = (R1 - R2)/tan 45 = 1.0 - 0.25 = 0.75
in. The volume of the frustum is
V = 0.333h(R1
2 + R1R2 + R2
2) = 0.333(0.75)(1.02 + 1.0 x 0.25 + 0.252) = 1.031 in3
The billet has advanced a certain distance by the time this frustum is completely filled and extrusion
through the die opening is therefore initiated. The volume of billet compressed forward to fill the
frustum is given by:
V = R1
2(Lo - L1) = (1.0)2(Lo - L1)
Setting this equal to the volume of the frustum, we have
(Lo - L1) = 1.031 in3
(Lo - L1) = 1.031/= 0.328 in
L1 = 3.0 - 0.328 = 2.672 in.
(c) = ln rx = ln 16 = 2.7726
x = a + b ln rx = 0.8 + 1.3(2.7726) = 4.404
Y
f
= 15,000(2.7726)0/1.0 = 15,000 lb/in2
p = 15,000(4.404 + 2 x 2.672/2.0) = 106,140 lb/in2
(d) Length of extruded portion of billet = 2.672 in. With a reduction rx = 16, the final part length,
excluding the cone shaped butt remaining in the die is L = 2.672(16) = 42.75 in.
19.28 An indirect extrusion process starts with an aluminum billet with diameter = 2.0 in and length = 3.0
in. Final cross-section after extrusion is a square with 1.0 in on a side. The die angle = 90. The
operation is performed cold and the strength coefficient of the metal K = 26,000 lb/in2 and strain
hardening exponent n = 0.20. In the Johnson extrusion strain equation, a = 0.8 and b = 1.2. (a)
Compute the extrusion ratio, true strain, and extrusion strain. (b) What is the shape factor of the
product? (c) If the butt left in the container at the end of the stroke is 0.5 in thick, what is the
length of the extruded section? (d) Determine the ram pressure in the process.
113
Solution: (a) rx = Ao/Af
Ao = Do
2/4 = (2)2/4 = 3.142 in2
Af = 1.0 x 1.0 = 1.0 in2
rx = 3.142/1.0 = 3.142
= ln 3.142 = 1.145
x = 0.8 + 1.3(1.145) = 2.174
(b) To determine the die shape factor we need to determine the perimeter of a circle whose area is
equal to that of the extruded cross-section, A = 1.0 in2. The radius of the circle is R = (1.0/)0.5 =
0.5642 in., Cc = 2(0.5642) = 3.545 in.
The perimeter of the extruded cross-section Cx = 4(1.0) = 4.0 in.
Kx = 0.98 + 0.02(4.0/3.545)2.25 = 1.006
(c) Given that the butt thickness = 0.5 in.
Original volume V = (3.0)(x 22/4) = 9.426 in3
The final volume consists of two sections: (1) butt, and (2) extrudate. The butt volume V1 = (0.5)(
x 22/4) = 1.571 in3. The extrudate has a cross-sectional area Af = 1.0 in2. Its volume V2 = LAf =
9.426 - 1.571 = 7.855 in3. Thus, length L = 7.855/1.0 = 7.855 in.
(d) Y
f
= 26,000(1.145)0.2/1.2 = 22,261 lb/in3
p = 1.006(22,261)(2.174) = 48,698 lb/in2
19.29 An L-shaped structural section is direct extruded from an aluminum billet in which Lo = 250 mm
and Do = 88 mm. Dimensions of the cross-section are given in Figure P19.29. Die angle = 90.
Determine: (a) extrusion ratio, (b) shape factor, and (c) length of the extruded section if the butt
remaining in the container at the end of the ram stroke is 25 mm.
Solution: (a) rx = Ao/Af
Ao = (88)2/4 = 6082.1 mm2
Af = 2 x (12 x 50) = 1200 mm2
rx = 6082.1/1200= 5.068
(b) To determine the die shape factor we need to determine the perimeter of a circle whose area is
equal to that of the extruded cross-section, A = 1200 mm2. The radius of the circle is R =
(1200/)0.5 = 19.54 mm, Cc = 2(19.54) = 122.8 mm.
The perimeter of the extruded cross-section Cx = 62 + 50 + 12 + 38 + 50 + 12 = 224 mm
Kx = 0.98 + 0.02(224/122.8)2.25 = 1.057
(c) Total original volume V = 0.25(88)2(250) = 1,520,531 mm3
The final volume consists of two sections: (1) butt, and (2) extrudate. The butt volume V1 =
0.25(88)2(25) = 152,053 mm3. The extrudate has a cross-sectional area Af = 1200 mm2. Its
volume V2 = LAf = 1,520,531 - 152,053 = 1,368,478 mm3.
Thus, length L = 1,368,478/1200 = 1140.4 mm
19.30 The flow curve parameters for the aluminum alloy of Problem 19.29 are: K = 240 MPa and n =
0.16. If the die angle in this operation = 90, and the corresponding Johnson strain equation has
constants a = 0.8 and b = 1.5, compute the maximum force required to drive the ram forward at
the start of extrusion.
Solution: From Problem 19.29, rx = 5.068
= ln 5.068= 1.623
x = 0.8 + 1.5(1.623) = 3.234
114
Y
f
= 240(1.623)0.16/1.16 = 223.6 MPa
Maximum ram force occurs at beginning of stroke when L is maximum at L = 250 mm
p = Kx Y
f
(x + 2L/Do) = 1.057(223.6)( 3.234+ 2 x 250/88) = 2107.2 MPa
F = pAo = 2107.2 (6082.1) = 12,816,267 N
19.31 A cup-shaped part is backward extruded from an aluminum slug that is 50 mm in diameter. The
final dimensions of the cup are: OD = 50 mm, ID = 40 mm, height = 100 mm, and thickness of base
= 5 mm. Determine: (a) extrusion ratio, (b) shape factor, and (c) height of starting slug required to
achieve the final dimensions. (d) If the metal has flow curve parameters K = 400 MPa and n =
0.25, and the constants in the Johnson extrusion strain equation are: a = 0.8 and b = 1.5, determine
the extrusion force.
Solution: (a) rx = Ao/Af
Ao = 0.25(50)2 = 1963.75 mm2
Af = 0.25(502 - 402) = 706.86 mm2
rx = 1963.75/706.86 = 2.778
(b) To determine the die shape factor we need to determine the perimeter of a circle whose area is
equal to that of the extruded cross-section, A = 706.86 mm2. The radius of the circle is R =
(706.86/)0.5 = 15 mm, Cc = 2(15) = 94.25 mm.
The perimeter of the extruded cross-section Cx = (50 + 40) = 90= 282.74 mm.
Kx = 0.98 + 0.02(282.74/94.25)2.25 = 1.217
(c) Volume of final cup consists of two geometric elements: (1) base and (2) ring.
(1) Base t = 5 mm and D = 50 mm. V1 = 0.25(50)2(5) = 9817.5 mm3
(2) Ring OD = 50 mm, ID = 40 mm, and h = 95 mm.
V2 = 0.25(502 - 402)(95) = 0.25(2500 - 1600)(95) = 67,151.5 mm3
Total V = V1 + V2 = 9817.5 + 67,151.5 = 76,969 mm3
Volume of starting slug must be equal to this value V = 76,969 mm3
V = 0.25(50)2(h) = 1963.5h = 76,969 mm3
h = 39.2 mm
(d) = ln 2.778 = 1.0218
x = 0.8 + 1.5(1.0218) = 2.33
Y
f
= 400(1.0218)0.25/1.25 = 321.73 MPa
p = Kx Y
f
x = 1.217(321.73)(2.33) = 912.3 MPa
Ao = 0.25(40)2 = 1256.6 mm2
F = 912.3(1256.6) = 1,146,430 N
19.32 Determine the shape factor for each of the extrusion die orifice shapes in Figure P19.32.
Solution: (a) Ax = 20 x 60 = 1200 mm, Cx = 2(20 + 60) = 160 mm
Ao = R2 = 1200
R2 = 1200/= 381.97, R = 19.544 mm, Cc = 2R = 2(19.544) = 122.8 mm
Kx = 0.98 + 0.02(160/122.8)2.25 = 1.016
(b) Ax = Ro
2 - Ri
2 = (252 - 22.52) = 373.06 mm2
Cx = Do + Di = (50 + 45) = 298.45 mm
R2 = 373.06/= 118.75, R = 10.897 mm, Cc = 2R = 2(10.897) = 68.47 mm
Kx = 0.98 + 0.02(298.45/68.47)2.25 = 1.53
115
(c) Ax = 2(5)(30) + 5(60 - 10) = 300 + 250 = 550 mm2
Cx = 30 + 60 + 30 + 5 + 25 + 50 + 25 + 5 = 230 mm
Ao = R2 = 550, R2 = 550/= 175.07, R = 13.23 mm
Cc = 2R = 2(13.23) = 83.14 mm
Kx = 0.98 + 0.02(230/83.14)2.25 = 1.177
(d) Ax = 5(55)(5) + 5(85 - 5x5) = 1675 mm2
Cx = 2 x 55 + 16 x 25 + 8 x 15 + 10 x 5 = 680 mm
Ao = R2 = 1675, R2 = 1675/= 533.17, R = 23.09 mm
Cc = 2R = 2(23.09) = 145.08 mm
Kx = 0.98 + 0.02(680/145.08)2.25 = 1.626
Drawing
19.33 Wire of starting diameter = 3.0 mm is drawn to 2.5 mm in a die with entrance angle = 15degrees.
Coefficient of friction at the work-die interface = 0.07. For the work metal, K = 500 MPa and n =
0.30. Determine: (a) area reduction, (b) draw stress, and (c) draw force required for the operation.
Solution: (a) r = (Ao - Af)/Ao
Ao = 0.25(3.0)2 = 9.0695 mm2
Af = 0.25(2.5)2 = 4.9094 mm2
r = (9.0695 - 4.9094)/9.0695 = 0.3056
(b) Draw stress d:
= ln(7.0695/4.9094) = ln 1.44 = 0.365
Y
f
= 500(0.365)0.30/1.30 = 284.2 MPa
= 0.88 + 0.12(D/Lc)
D = 0.5(3.0 + 2.5) = 2.75
Lc = 0.5(3.0 - 2.5)/sin 15 = 0.966
= 0.88 + 0.12(2.75/0.966) = 1.22
d = Y
f
(1 + /tan )(ln Ao/Af) = 284.2(1 + 0.07/tan 15)(1.22)(0.365) = 159.6 MPa
(c) Draw force F:
F = Af d = 4.9094(159.6) = 783.5 N
19.34 Rod stock is drawn through a draw die with an entrance angle of 12. Starting diameter = 0.50 in
and final diameter = 0.35 in. Coefficient of friction at the work-die interface = 0.1. The metal has a
strength coefficient = 45,000 lb/in2 and a strain hardening exponent = 0.22. Determine: (a) area
reduction, (b) draw force for the operation, and (c) horsepower to perform the operation if the exit
velocity of the stock = 2 ft/sec.
Solution: (a) r = (Ao - Af)/Ao
Ao = 0.25(0.50)2 = 0.1964 in2
Af = 0.25(0.35)2 = 0.0962 in2
r = (0.1964 - 0.0962)/0.1964 = 0.51
(b) Draw force F:
= ln(0.1964/0.0962) = ln 2.0416 = 0.7137
Y
f
= 45,000(0.7137)0.22/1.22 = 34,247 lb/in2
= 0.88 + 0.12(D/Lc)
D = 0.5(.50 + 0.35) = 0.425
116
Lc = 0.5(0.50 - 0.35)/sin 12 = 0.3607
= 0.88 + 0.12(0.425/0.3607) = 1.021
F = Af Y
f
(1 + /tan )(ln Ao/Af)
F = 0.0962(34,247)(1 + 0.1/tan 12)(1.021)(0.7137) = 3530 lb
(c) P = 3530(2 ft/sec x 60) = 423,600 ft/lb/min
HP = 423,600/33,000 = 12.84 hp
19.35 Bar stock of initial diameter = 90 mm is drawn with a draft = 15 mm. The draw die has an
entrance angle = 18, and the coefficient of friction at the work-die interface = 0.08. The metal
behaves as a perfectly plastic material with yield stress = 105 MPa. Determine: (a) area reduction,
(b) draw stress, (c) draw force required for the operation, and (d) power to perform the operation
if exit velocity = 1.0 m/min.
Solution: (a) r = (Ao - Af)/Ao
Ao = 0.25(90)2 = 6361.7 mm2
Df = Do - d = 90 - 15 = 75 mm,
Af = 0.25(75)2 = 4417.9 mm2
r = (6361.7 - 4417.9)/ 6361.7 = 0.3056
(b) Draw stress d:
= ln(6361.7 /4417.9) = ln 1.440 = 0.3646
Y
f
= k = 105 MPa
= 0.88 + 0.12(D/Lc)
D = 0.5(90 + 75) = 82.5 mm
Lc = 0.5(90 - 75)/sin 18 = 24.3 mm
= 0.88 + 0.12(82.5/24.3) = 1.288
d = Y
f
(1 + /tan )(ln Ao/Af) = 105(1 + 0.08/tan 18)(1.288)(0.3646) = 61.45 MPa
(c) F = Af d = 4417.9 (61.45) = 271,475 N
(d) P = 271,475(1 m/min) = 271,475 N-m/min = 4524.6 N-m/s = 4524.6 W
19.36 Wire stock of initial diameter = 0.125 in is drawn through two dies each providing a 0.20 area
reduction. The starting metal has a strength coefficient = 40,000 lb/in2 and a strain hardening
exponent = 0.15. Each die has an entrance angle of 12, and the coefficient of friction at the
work-die interface is estimated to be 0.10. The motors driving the capstans at the die exits can
each deliver 1.50 hp at 90% efficiency. Determine the maximum possible speed of the wire as it
exits the second die.
Solution: First draw: Do = 0.125 in., Ao = 0.25(0.125)2 = 0.012273 in2 009819 in2
= ln(0.012273/0.009819) = ln 1.250 = 0.2231
r = (Ao - Af)/Ao , Af = Ao(1 - r) = 0.012773(1 - 0.2) = 0.
Y
f
= 40,000(0.2231)0.15/1.15 = 27,775 lb/in2
= 0.88 + 0.12(D/Lc)
Df = 0.125(1 - r)0.5 = 0.125(.8).5 = 0.1118 in
D = 0.5(.125 + 0.1118) = 0.1184
Lc = 0.5(0.125 - 0.1118)/sin 12 = 0.03173
= 0.88 + 0.12(0.1184/0.03173) = 1.33
F = Af Y
f
(1 + /tan )(ln Ao/Af)
117
F = 0.09819(27,775)(1 + 0.1/tan 12)(1.33)(0.2231) = 119 lb
1.5 hp at 90% efficiency = 1.5 x 0.90(33,000 ft-lb/min)/60 = 742.5 ft-lb/sec
P = Fv = 119v = 742.5
v = 742.5/119 = 6.24 ft/sec
Second draw: Do = 0.1118 in., Ao = 0.25(0.1118)2 = 0.009819 in2
r = (Ao - Af)/Ao , Af = Ao(1 - r) = 0.009819(1 - 0.2) = 0.007855 in2
= ln(0.009819/0.007855) = ln 1.250 = 0.2231
Total strain experienced by the work metal is the sum of the strains from the first and second
draws:
= 1 + 2 = 0.2231 + 0.2231 = 0.4462
Y
f
= 40,000(0.4462)0.15/1.15 = 30,818 lb/in2
= 0.88 + 0.12(D/Lc)
Df = 0.1118(1 - r)0.5 = 0.1118(.8).5 = 0.100 in
D = 0.5(0.1118 + 0.100) = 0.1059
Lc = 0.5(0.1118 - 0.100)/sin 12 = 0.0269
= 0.88 + 0.12(0.1059/0.0269) = 1.35
F = Af Y
f
(1 + /tan )(ln Ao/Af)
F = 0.007855(30,818)(1 + 0.1/tan 12)(1.35)(0.4462) = 214 lb.
1.5 hp at 90% efficiency = 742.5 ft-lb/sec as before in the first draw.
P = Fv = 214v = 742.5
v = 742.5/214 = 3.47 ft/sec
Note: The calculations indicate that the second draw die is the limiting step in the drawing
sequence. The first operation would have to be operated at well below its maximum possible
speed; or the second draw die could be powered by a higher horsepower motor; or the reductions
to achieve the two stages could be reallocated to achieve a higher reduction in the first drawing
operation.
118
20 SHEET METALWORKING
Review Questions
20.1 Identify the three basic types of sheet metalworking operations.
Answer. The three operations are: (1) cutting, (2) bending, and (3) drawing.
20.2 In blanking of a round sheet metal part, indicate how the clearance should be applied to the punch
and die diameters.
Answer. For a blanking operation, die size = blank size, and the punch is smaller by twice the
clearance.
20.3 What is the difference between a cutoff operation and a parting operation?
Answer. A cutoff operation separates parts from a strip by shearing one edge of each part in
sequence. A parting operation cuts a slug between adjacent parts in the strip. See Figure 20.9.
20.4 Describe V-bending and edge bending.
Answer. In V-bending, a simple punch and die which have the included angle are used to bend
the part. In edge bending, the punch force a cantilevered sheet metal section over a die edge to
obtain the desired bend angle. See Figure 20.14.
20.5 What is springback in sheet metal bending?
Answer. Springback is the elastic recovery of the sheet metal after bending; it is usually
measured as the difference between the final included angle of the bent part and the angle of the
tooling used to make the bend, divided by the angle of the tooling.
20.6 What are some of the simple measures used to assess the feasibility of a proposed cup drawing
operation?
Answer. Measures of drawing feasibility include: (1) drawing ratio, DR = D/Dp; (2) reduction, r =
(D - Dp)/D; and (3) thickness-to-diameter ratio, t/D; where t = stock thickness, D = blank
diameter, and Dp = punch diameter.
20.7 Distinguish between redrawing and reverse drawing.
Answer. In redrawing, the shape change is significant enough (e.g., drawing ratio greater than
2.0) that it must be carried out in two drawing steps, probably with annealing between the steps.
In reverse drawing, two draws are accomplished on the part, one in one direction, the second in
the opposite direction.
20.8 What are some of the possible defects in drawn sheet metal parts?
Answer. Drawing defects include: (1) wrinkling, (2) tearing, (3) earing, and (4) surface scratches.
See Article 20.3.4.
20.9 What is stretch forming?
Answer. Stretch forming of sheet metal involves stretching and simultaneous bending of the
workpart to achieve shape change.
20.10 Identify the principal components of a stamping die that performs blanking.
119
Answer. See Article 20.5.1.
20.11 What are the two basic categories of structural frames used in stamping presses?
Answer. Two press frame types are: (1) gap frame, also called C-frame, and (2) straight-sided
frame. See Article 20.6.1.
20.12 What are the relative advantages and disadvantages of mechanical versus hydraulic presses in
sheet metalworking?
Answer. Advantage of mechanical presses: faster cycle rates. Advantages of hydraulic presses:
longer ram strokes and uniform force throughout stroke.
20.13 What is the Guerin process?
Answer. The Guerin process is a sheet metal forming process that uses a rubber die which flexes
to force the sheet metal to take the shape of a form block (punch).
20.14 Identify a major technical problem in tube bending?
Answer. A major technical problem in tube bending is collapse of the tube walls during the
process.
20.15 Distinguish between roll bending and roll forming.
Answer. Roll bending involves the forming of large sheet and plate metal sections into curved
forms. Roll forming involves feeding a lone strip or coil through rotating rolls so that the shape
of the rolls is imparted to the strip. See Section 20.10.
Multiple Choice Quiz
There are a total of 17 correct answers in the following multiple choice questions (some questions have
multiple answers that are correct). To attain a perfect score on the quiz, all correct answers must be
given, since each correct answer is worth 1 point. For each question, each omitted answer or wrong
answer reduces the score by 1 point, and each additional answer beyond the number of answers required
reduces the score by 1 point. Percentage score on the quiz is based on the total number of correct
answers.
20.1 As sheet metal stock hardness increases, the clearance between punch and die should: (a) be
decreased, (b) be increased, or (c) be unaffected.
Answer. (b)
20.2 A round sheet metal slug produced in a hole punching operation will have the same diameter as
which of the following? (a) die opening, or (b) punch.
Answer. (a)
20.3 The cutting force in a blanking operation depends on which mechanical property of the sheet
metal (one best answer)? (a) compressive strength, (b) modulus of elasticity, (c) shear strength,
(d) tensile strength, or (e) yield strength.
Answer. (c)
20.4 Sheet metal bending involves which of the following stresses and strains (may be more than one)?
(a) compressive, (b) shear, and (c) tensile.
Answer. (a) and (c).
120
20.5 Which one of the following is the best definition of bend allowance? (a) amount by which the die
is larger than the punch, (b) amount of elastic recovery experienced by the metal after bending,
(c) safety factor used in calculating bending force, or (d) length before bending of the straight
sheet metal section to be bent.
Answer. (d)
20.6 Which of the following are variations of sheet metal bending operations (more than one)? (a)
coining, (b) flanging, (c) hemming, (d) ironing, (e) notching, (f) shear spinning, (g) trimming, (h)
tube bending, and (i) tube forming.
Answer. (b), (c), and (i).
20.7 The following are measures of feasibility for several proposed cup drawing operations; which of
the operations are likely to be feasible? (a) DR = 1.7, (b) DR = 2.7, (c) r = 0.35, (d) r = 65%, and
(e) t/D = 2%.
Answer. (a), (c), and (e).
20.8 Holding force in drawing is most likely to be which of the following relative to maximum drawing
force? (a) less than, (b) equal to, or (c) greater than.
Answer. (a)
20.9 Which one of the following stamping dies is the most complicated? (a) blanking die, (b)
combination die, (c) compound die, (d) wiping die for edge bending, (e) progressive die, or (f)
V-die.
Answer. (e)
20.10 Which one of the following press types is usually associated with the highest production rates in
sheet metal stamping operations? (a) adjustable bed, (b) open back inclinable, (c) press brake, (d)
solid gap, and (e) straight-sided.
Answer. (b)
20.11 Which of the following processes are classified as high- energy-rate forming processes (more
than one)? (a) electrochemical machining, (b) electromagnetic forming, (c) electron beam cutting,
(d) explosive forming, (e) Guerin process, (f) hydroforming, (g) redrawing, and (h) shear spinning.
Answer. (b) and (d).
Problems
Cutting Operations
20.1 A power shears is used to cut soft cold rolled steel that is 4.75 mm thick. At what clearance
should the shears be set to yield an optimum cut?
Solution: From Table 20.1, a = 0.060. Thus, c = at = 0.060(4.75) = 0.285 mm
20.2 A blanking operation is to be performed on 2.0 mm thick cold rolled steel (half hard). The part is
circular with diameter = 75.0 mm. Determine the appropriate punch and die sizes for this
operation.
Solution: From Table 20.1, a = 0.075. Thus, c = 0.075(2.0) = 0.15 mm.
Punch diameter = Db - 2c = 75.0 - 2(0.15) = 74.70 mm.
Die diameter = Db = 75.0 mm.
121
20.3 A compound die will be used to blank and punch a large washer out of aluminum alloy sheet stock
3.2 mm thick. The outside diameter of the washer = 65 mm and the inside diameter = 30 mm.
Determine: (a) the punch and die sizes for the blanking operation, and (b) the punch and die sizes
for the punching operation.
Solution: From Table 20.1, a = 0.045. Thus, c = 0.045(3.2) = 0.144 mm
(a) Blanking punch diameter = Db - 2c = 65 - 2(0.144) = 64.71 mm
Blanking die diameter = Db = 65 mm
(b) Punching punch diameter = Dh = 30 mm
Punching die diameter = Dh + 2c = 30 + 2(0.144) = 30.29 mm
20.4 A blanking die is to be designed to blank the part outline shown in Figure P20.4. The material is
5/32 inch thick stainless steel (half hard). Determine the dimensions of the blanking punch and the
die opening.
Solution: From Table 20.1, a = 0.075. Thus, c = 0.075(5/32) = 0.0117 in.
Blanking die: dimensions are the same as for the part in Figure P20.4.
Blanking punch: 3.500 inch length dimension = 3.500 - 2(0.0117) = 3.4766 in.
2.000 inch width dimension = 2.000 - 2(0.0117) = 1.9766 in.
top and bottom 1.00 inch extension widths = 1.0 - 2(0.0117) = 0.9766 in.
1.000 inch inset dimension remains the same.
20.5 Determine the blanking force required in Problem 20.2, if the steel has a shear strength = 350
MPa.
Solution: F = StL
t = 2.0 mm from Problem 20.2.
L = D = 75= 235.65 mm
F = 350(2.0)(235.65) = 164,955 N
20.6 Determine the minimum tonnage press to perform the blanking and punching operation in Problem
20.3, if the aluminum sheetmetal has a tensile strength = 290 MPa. Assume that blanking and
punching occur simultaneously.
Solution: F = 0.7(TS)tL
t =3.2 mm from Problem 20.3.
L = 65+ 30= 95= 298.5 mm
F = 0.7(290)(3.2)(298.5) = 193,874 N
20.7 Determine the tonnage requirement for the blanking operation in Problem 20.4, given that the
stainless steel has a shear strength = 62,000 lb/in2.
Solution: F = StL
t = 5/32 in. from Problem 20.4.
L = 3.5 + 2.0 + 1.0 + 1.0 + 1.5 + 1.0 + 1.0 + 2.0 = 13.0 in.
F = 62,000(5/32)(13.0) = 125,938 lb = 62.97 tons
20.8 The foreman in the pressworking section comes to you with the problem of a blanking operation
that is producing parts with excessive burrs. What are the possible reasons for the burrs, and what
can be done to correct the condition?
122
Solution: Reasons for excessive burrs: (1) clearance between punch and die is too large for the
material and stock thickness. (2) Punch and die cutting edges are worn (rounded) which has the
same effect as excessive clearance.
To correct the problem: (1) Check the punch and die cutting edges to see if they are worn. If
they are, regrind the faces to sharpen the cutting edges. (2) If the die is not worn, measure the
punch and die clearance to see if it equals the recommended value. If not, die maker must
rebuild the punch and die.
Bending
20.9 A bending operation is to be performed on 4.75 mm thick cold rolled steel. The part drawing is
given in Figure P20.9. Determine the blank size required.
Solution: From drawing, A’ = 45, R = 4.75 mm
A = 180 - A’ = 135.
BA = 2(A/360)(R + Kbat)
R/t = (9.5)/(4.75) = 2.0; therefore, Kba = 0.5
BA = 2(135/360)(9.5 + 0.5 x 4.75) = 27.98 mm
Dimensions of starting blank: w = 32 mm, L = 50 + 27.98 + 37.5 = 115.48 mm
20.10 Solve Problem 20.9 except that the bend radius R = 6.35 mm.
Solution: From drawing, A’ = 45, R = 6.35 mm
A = 180 - A’ = 135.
BA = 2(A/360)(R + Kbat)
R/t = (6.35)/(4.75) = 1.337; therefore, Kba = 0.333
BA = 2(135/360)(6.35 + 0.333 x 4.75) = 18.37 mm
Dimensions of starting blank: w = 32 mm, L = 50 + 18.37 + 37.5 = 105.87 mm
20.11 An L-shaped part is to be bent in a V-bending operation on a press brake from a flat blank 4.0
inches by 1.5 inches that is 5/32 inch thick. The bend of 90is to be made in the middle of the
4-inch length. (a) Determine the dimensions of the two equal sides that will result after the bend, if
the bend radius = 3/16 inch. For convenience, these sides should be measured to the beginning of
the bend radius. (b) Also, determine the length of the part's neutral axis after the bend. (c) Where
should the machine operator set the stop on the press brake relative to the starting length of the
part?
Solution: (a) R/t = (3/16)/(5/32) = 1.2. Therefore, Kba = 0.33
B = 2(90/360)(0.1875 + 0.33 x 0.15625) = 0.3756 in.
Dimensions (lengths) of each end = 0.5(4.0 - 0.3756) = 1.8122 in.
(b) Since the metal stretches during bending, its length will be greater after the bend than before.
Its length before bending = 4.000 in. The stretched length of the bend along the neutral axis will
be:
B = 2(90/360)(0.1875 + 0.5 x 0.15625) = 0.4173 in.
Therefore, the length of the neutral axis of the part will be 2(1.8122) + 0.4173 = 4.0417 in.
(c) The operator should set the stop so that the tip of the V-punch contacts the starting blank at
a distance = 2.000 in. from the end.
20.12 Determine the bending force required in Problem 20.9 if the bend is to be performed in a V-die
with a die opening width = 38 mm. The material has a tensile strength = 620 MPa.
Solution: For V-bending, Kbf = 1.33.
123
F = Kbf(TS)wt2/D = 1.33(620)(32)(4.75)2/38 = 15,667 N
20.13 Solve Problem 20.12 except that the operation is performed using a wiping die with die opening W
= 25 mm.
Solution: For edge-bending in a wiping die, Kbf = 0.33.
F = Kbf(TS)wt2/D = 0.33(620)(32)(4.75)2/25 = 5,909 N
20.14 Determine the bending force required in Problem 20.11 if the bend is to be performed in a V-die
with a die opening width W = 1.25 inches. The material has a tensile strength = 70,000 lb/in2.
Solution: For V-bending, Kbf = 1.33.
F = Kbf(TS)wt2/D = 1.33(70,000)(1.5)(5/32)2/1.25 = 2728 lb.
20.15 Solve Problem 20.14 except that the operation is performed using a wiping die with die opening W
= 0.75 inch.
Solution: For edge-bending in a wiping die, Kbf = 0.33.
F = Kbf(TS)wt2/D = 0.33(70,000)(1.5)(5/32)2/0.75 = 1128 lb.
20.16 A sheetmetal part 3.0 mm thick and 20.0 mm long is bent to an included angle = 60and a bend
radius = 7.5 mm in a V-die. The metal has a tensile strength = 340 MPa. Compute the required
force to bend the part, given that the die opening = 15 mm.
Solution: For V-bending, Kbf = 1.33.
F = Kbf(TS)wt2/D = 1.33(340)(20)(3)2/15 = 5426 N
Drawing Operations
20.17 Derive an expression for the reduction r in drawing as a function of drawing ratio DR.
Solution: Reduction r = (D - Dp)/D
Drawing ratio DR = D/Dp
r = D/D - Dp/D = 1 - Dp/D = 1 - 1/DR
20.18 A cup is to be drawn in a deep drawing operation. The height of the cup is 75 mm and its inside
diameter = 100 mm. The sheetmetal thickness = 2 mm. If the blank diameter = 225 mm,
determine: (a) drawing ratio, (b) reduction, and (c) thickness-to-diameter ratio. (d) Does the
operation seem feasible?
Solution: (a) DR = D/Dp = 225/100 = 2.25
(b) r = (D - Dp)/D = (225 - 100)/225 = 0.555 = 55.5%
(c) t/D = 2/225 = 0.0089 = 0.89%
(d) Feasibility? No! DR is too large (greater than 2.0), r is too large (greater than 50%), and t/D
is too small (less than 1%).
20.19 Solve Problem 20.18 except that the starting blank size diameter = 175 mm.
Solution: (a) DR = D/Dp = 175/100 = 1.75
(b) r = (D - Dp)/D = (175 - 100)/175 = 0.429 = 42.9%
(c) t/D = 2/175 = 0.0114 = 1.14%
(d) Feasibility? DR < 2.0, r < 50%, and t/D > 1%. However, the operation is not feasible
because the 175 mm diameter blank size does not provide sufficient metal to draw a 75 mm cup
124
height. The actual cup height possible with a 175 mm diameter blank can be determined by
comparing surface areas (one side only for convenience) between the cup and the starting blank.
Blank area = D2/4 = (175)2/4 = 24,053 mm2. To compute the cup surface area, let us divide the
cup into two sections: (1) walls, and (2) base, assuming the corner radius on the punch has a
negligible effect in our calculations and there is no earing of the cup. Thus, Cup area = Dph +
Dp
2/4 = 100h + (100)2/4 = 100h + 2500= 314.16h + 7854. Set surface area of cup =
surface are of starting blank:
314.16h + 7854= 24,053
314.16h = 16,199
h = 51.56 mm. This is less than the specified 75 mm height.
20.20 A deep drawing operation is performed in which the inside of the cylindrical cup has a diameter =
4.0 inches and a height = 2.5 inches. The stock thickness = 1/8 inch, and the starting blank
diameter = 7.5 inches. Punch and die radii = 5/32 inch. The metal has a tensile strength = 60,000
lb/in2 and a yield strength = 30,000 lb/in2. Determine: (a) drawing ratio, (b) reduction, (c) drawing
force, and (d) blankholder force.
Solution: (a) DR = 7.5/4.0 = 1.875
(b) t/D = 0.125/7.5 = 0.01667 = 1.667%
(c) F = Dpt(TS)(D/Dp - 0.7) = (4)(0.125)(60,000)(7.5/4 - 0.7) = 110,756 lb.
(d) Fh = 0.015Y(D2 - (Dp + 2.2t + 2Rd)2)
Fh = 0.015(30,000)(7.52 - (4 + 2.2 x 0.125 + 2 x 0.15625)2) = 0.015(30,000)(7.52 - 4.58752)
Fh = 49,770 lb
20.21 Solve Problem 20.20 except that the stock thickness t = 3/16 inch.
Solution: (a) DR = 7.5/4.0 = 1.875 (same as previous problem)
(b) t/D = 0.1875/7.5 = 0.025 = 2.5%
(c) F = Dpt(TS)(D/Dp - 0.7) = (4)(0.1875)(60,000)(7.5/4 - 0.7) = 166,133 lb.
(d) Fh = 0.015(30,000)(7.52 - (4 + 2.2 x 0.125 + 2 x 0.15625)2) = 49,770 lb (same as previous
problem)
20.22 A cup drawing operation is performed in which the inside diameter = 80 mm and the height = 50
mm. The stock thickness = 3.0 mm, and the starting blank diameter = 150 mm. Punch and die radii
= 4 mm. Tensile strength = 400 MPa and a yield strength = 180 MPa for this sheetmetal.
Determine: (a) drawing ratio, (b) reduction, (c) drawing force, and (d) blankholder force.
Solution: (a) DR = 150/80 = 1.875
(b) r = (D – Dp)/D = )150 – 80)/80 = 70/150 = 0.46
(c) F = Dpt(TS)(D/Dp - 0.7) = (80)(3)(400)(150/80 - 0.7) = 354,418 N.
(d) Fh = 0.015Y(D2 - (Dp + 2.2t + 2Rd)2)
Fh = 0.015(180)(1502 - (80 + 2.2 x 3 + 2 x 4)2) = 0.015(180)(1502 - 94.62)
Fh = 114,942 N
20.23 A deep drawing operation is to be performed on a sheetmetal blank that is 1/8 inch thick. The
height (inside dimension) of the cup = 3.8 inches and the diameter (inside dimension) = 5.0 inches.
Assuming the punch radius = 0, compute the starting diameter of the blank to complete the
125
operation with no material left in the flange. Is the operation feasible (ignoring the fact that the
punch radius is too small)?
Solution: Use surface area computation, assuming thickness t remains constant.
Cup area = wall area + base area = Dph + Dp
2/4 = 5(3.8) + 0.25(5)2 = 25.25in2
Blank area = D2/4 = 0.25D2
Setting blank area = cup area: 0.25D2 = 25.25
D2 = 25.25/0.25 = 101.0
D = 10.050 in.
Test for feasibility: DR = D/Dp = 10.050/5.0 = 2.01. Because DR > 2.0, this operation may not
be feasible. Of course, the zero punch radius makes this operation infeasible anyway. With a
rounded punch radius, the blank size would be slightly smaller, which would reduce DR.
20.24 Solve Problem 20.23 except use a punch radius = 0.375 inch.
Solution: Use surface area computation, assuming thickness t remains constant. The surface area
of the cup will be divided into three sections: (1) straight walls, whose height = 3.80 - 0.375 =
3.425 in., (2) quarter toroid formed by the 0.375 radius at the base of the cup, and (3) base, which
has a diameter = 5.0 - 2 x 0.375 = 4.25 in.
A1 = Dph = (5.0)(3.425) = 53.807 in2
A2 = length of the quarter circle at the base multiplied by the circumference of the circle
described by the centroid (Pappus-Guldin Theorem): length of quarter circle = D/4 = 0.25(2 x
0.375) = 0.589 in. The centroid is located at the center of the arc which is 0.375 sin 45 = 0.265
beyond the center of the 0.375 in. radius. Thus, the diameter of the circle described by the
centroid is 4.25 + 2 x 0.265 = 4.780 in.
A2 = 4.78(0.589) = 8.847 in2
A3 = (4.25)2/4 = 14.188 in2
Total area of cup = 53.807 + 8.847 + 14.188 = 76.842 in2
Blank area = D2/4 = 0.7855D2
Setting blank area = cup area: 0.7855D2 = 76.842
D2 = 76.842/0.7855 = 97.825
D = 9.890 in.
Test for feasibility: DR = D/Dp = 9.89/5.0 = 1.978, which is less than the limiting ratio of 2.0. The
thickness to diameter ratio t/D = 0.125/9.89 = 0.0126 = 1.26%, which is above the value of 1%
used as a criterion of feasibility in cup drawing. Whereas the operation in Problem 20.23 was not
feasible, the operation in the present problem seems feasible.
20.25 A drawing operation is performed on 3.0 mm stock. The part is a cylindrical cup with height = 50
mm and inside diameter = 70 mm. Assume the corner radius on the punch = zero. (a) Find the
required starting blank size Db. (b) Is the drawing operation feasible?
Solution: Use surface area computation, assuming thickness t remains constant.
Cup area = wall area + base area = Dph + Dp
2/4 = (70)(50) + 0.25(70)2 = 14,846 mm2.
Blank area = D2/4 = 0.7855D2
Setting blank area = cup area: 0.7855D2 = 14,846
D2 = 14,846/0.7855 = 18,900
D = 137.48 mm.
Test for feasibility: DR = D/Dp = 137.48/70 = 1.964; t/D = 3/137.48 = 0.0218 = 2.18%. These
criteria values indicate that the operation is feasible; however, with a punch radius Rp = 0, this
126
shape would be difficult to draw because the drawing punch would act on the metal like a blanking
punch.
20.26 Solve Problem 20.25 except that the height = 60 mm.
Solution: Cup area = wall area + base area
Cup area = Dph + Dp
2/4 = (70)(60) + 0.25(70)2 = 17,045 mm2.
Blank area = D2/4 = 0.7855D2
Setting blank area = cup area: 0.7855D2 = 17,045
D2 = 17,045/0.7855 = 21,700
D = 147.31 mm.
Test for feasibility: DR = D/Dp = 147.31/70 = 2.10; t/D = 3/147.31 = 0.0204 = 2.04%. Since the
DR is greater than 2.0, this operation is considered infeasible. Also, as in the previous problem,
the punch radius Rp = 0 would render this operation difficult if not infeasible.
20.27 Solve Problem 20.26 except that the corner radius on the punch = 10 mm.
Solution: Use surface area computation, assuming thickness t remains constant. The surface area
of the cup will be divided into three sections: (1) straight walls, whose height = 60 - 10 = 50 mm,
(2) quarter toroid formed by the 0.375 radius at the base of the cup, and (3) base, which has a
diameter = 70 - 2 x 10 = 50 mm.
A1 = Dph = (70)(50) = 10,995.6 mm2
A2 = length of the quarter circle at the base multiplied by the circumference of the circle
described by the centroid (Pappus-Guldin Theorem): length of quarter circle = 2Rp/4 = 0.25(2 x
10) = 15.71 mm. The centroid is located at the center of the arc which is 10 sin 45 = 7.071
beyond the center of the 0.375 in. radius. Thus, the diameter of the circle described by the
centroid is 50 + 2 x 7.071 = 64.142 mm.
A2 = 64.142(15.71) = 3166.1 mm2
A3 = (50)2/4 = 1963.8 mm2
Total area of cup = 10,995.6 + 3166.1 + 1963.8 = 16,125.5 mm2
Blank area = D2/4 = 0.7855D2
Setting blank area = cup area: 0.7855D2 = 16,125.5
D2 = 16,125.5/0.7855 = 20,529.0
D = 143.28 mm.
Test for feasibility: DR = D/Dp = 143.28/70 = 2.047. Since the DR is greater than 2.0, this
operation is considered infeasible.
20.28 The foreman in the drawing section of the shop brings to you several samples of parts that have
been drawn in the shop. The samples have various defects. One has ears, another has wrinkles,
and still a third has torn sections at its base. What are the causes of each of these defects and
what remedies would you propose?
Solution: (1) Ears are caused by sheet metal that has directional properties. The material is
anisotropic. One remedy is to anneal the metal to reduce the directionality of the properties.
(2) Wrinkles are caused by compressive buckling of the flange as it is drawn inward to form the
cup. There are several possible remedies: (a) increase the t/D ratio by using a thicker gage sheet
metal. This may not be possible since a design change is required. (b) Increase the blankholder
pressure against the work during drawing.
127
(3) Tearing occurs due to high tensile stresses in the walls of the cup near the base. A remedy
would be to provide a large punch radius. Tearing can also occur due to a die corner radius that is
too small.
20.29 A cup-shaped part is to be drawn without a blankholder from sheetmetal whose thickness = 0.25
inches. The inside diameter of the cup = 2.5 inches, its height = 1.5 inches, and the corner radius
at the base = 0.375 inch. (a) What is the minimum starting blank diameter that can be used,
according to Eq. (20.14)? (b) Does this blank diameter provide sufficient material to complete the
cup?
Solution: (a) According to Eq. (22.14), D - Dp < 5t D < 5t + Dp = 5(0.25) + 2.5 = 3.75 in. (b) Because the sheet metal is rather thick, let us use volume rather than area to determine whether there is sufficient metal in a 3.75 inch blank diameter. The drawn cup consists of three sections: (1) cup walls, (2) toroid at base, (3) base. V1 = (1.5 - 0.375)[(2.5 + 2 x 0.25)2] - (2.5)2)/4 = 1.125(2.75)/4 = 2.430 in3 V2 = (cross-section of quarter toroid) x (circle made by sweep of centroid) Cross-section of quarter toroid = 0.25[(0.375 + 0.25)2 - (0.375)2] = 0.1964 in2 Circle made by centroid sweep has diameter = (2.5 - 2 x 0.25) + 2(0.375 + 0.25/2)sin 45 = 2.457 in. V2 = 2.457(0.1964) = 1.516 in3 V3 = (2.5 - 2 x 0.375)2(0.25)/4 = 0.601 in3 Total V = V1 + V2 + V3 = 2.430 + 1.516 + 0.601 = 4.547 in3 Volume of blank = D2t/4 = (0.25)D2/4 = 0.1963D2 Setting blank volume = cup volume: 0.1963D2 = 4.547 D2 = 4.547/0.1963 = 23.16 D = 4.81 in. The diameter of 3.75 in. computed in (a) does not provide sufficient metal to complete the drawing. Other Operations 20.30 A 20 inch long sheetmetal workpiece is stretched in a stretch forming operation to the dimensions shown in Figure P20.30. The thickness of the beginning stock t = 0.125 inch and the width = 10 inches. The metal has a flow curve defined by K = 70,000 lb/in2 and n = 0.25. (a) Find the stretching force F required near the beginning of the operation when yielding first occurs. Determine: (b) true strain experienced by the metal, (c) stretching force F, and (d) die force Fdie at the very end when the part is formed as indicated in Figure P20.30(b). Solution: (a) Use = 0.002 as start of yielding. F = LtYf Yf = 70,000(0.002)0.25 = 14,803 lb/in2 F = (10)(0.12)(14,803) = 17,764 lb. (b) After stretching, the length of the piece is increased from 20.0 in. to 2(102 + 52)0.5 = 22.361 in. = ln(22.361/20) = ln 1.118 = 0.1116 (c) At the final length of 22.361 in., the thickness of the sheet metal has been reduced to maintain constant volume, assuming width L = 10 in. remains the same during stretching. tf = 0.12(20/22.361) = 0.1073 in. Yf = 70,000(0.1116)0.25 = 40,459 lb/in2 F = 10(0.1073)(40,459) = 43,413 lb. 128 (d) Fdie = 2F sin A A = tan-1(5/10) = 26.57 Fdie = 2( 43,413) sin 26.57 = 38,836 lb. 20.31 Determine the starting disk diameter required to spin the part in Figure P20.31 using a conventional spinning operation. The starting thickness = 2.4 mm. Solution: From part drawing, radius = 25 + (100 - 25)/sin 30 = 25 + 75/0.5 = 175 mm Starting diameter = 2(175) = 350 mm 20.32 If the part illustrated in Figure P20.31 were made by shear spinning, determine: (a) the wall thickness along the cone-shaped portion, and (b) the spinning reduction r. Solution: (a) tf = t sin = (2.4)sin 30 = 2.4(0.5) = 1.2 mm (b) r = (t - tf)/t = (2.4 – 1.2)/2.4 = 0.50 = 50% 20.33 Determine the shear strain that is experienced by the material that is shear spun in Problem 20.32. Solution: Based on sidewise displacement of metal through a shear angle of 30, Shear strain = cot 30 = 1.732. 20.34 A 75 mm diameter tube is bent into a rather complex shape with a series of simple tube bending operations. The wall thickness on the tube = 4.75 mm. The tubes will be used to deliver fluids in a chemical plant. In one of the bends where the bend radius is 125 mm, the walls of the tube are flattening badly. What can be done to correct the condition? Solution: Possible solutions: (1) Use a mandrel to prevent collapsing of tube wall. (2) Request the designer to increase the bend radius to 3D = 225 mm. (3) Pack sand into the tube. The sand will act as an internal flexible mandrel to support the tube wall. 129 21 THEORY OF METAL MACHINING Review Questions 21.1 What distinguishes machining from other manufacturing processes? Answer. In machining, material is removed from the workpart so that the remaining material is the desired part geometry. 21.2 Identify some of the reasons why machining is commercially and technologically important. Answer. The reasons include: (1) its applicability to most materials; (2) its capability to produce a variety of geometries to a part; (3) it can achieve closer tolerances than most other processes; and (4) it can create good surface finishes. 21.3 Name the three most common machining processes. Answer. The three common machining processes are: (1) turning, (2) drilling, and (3) milling. 21.4 What are the two basic categories of cutting tools in machining? Give an example of a machining operation that uses each of the tooling types. Answer. The two categories are: (1) single -point tools, used in operations such as turning and boring; and (2) multiple-edge cutting tools, used in operations such as milling and drilling. 21.5 Identify the parameters of a machining operation that are included within the scope of cutting conditions. Answer. Cutting conditions include: speed, feed, depth of cut, and whether or not a cutting fluid is used. 21.6 Define the difference between roughing and finishing operations in machining. Answer. A roughing operation is used to remove large amounts of material rapidly and to produce a part geometry close to the desired shape. A finishing operation follows roughing and is used to achieve the final geometry and surface finish. 21.7 What is a machine tool? Answer. A machine tool can be defined as a power-driven machine that positions and moves a tool relative to the work to accomplish machining or other metal shaping process. 21.8 What is an orthogonal cutting operation? Answer. Orthogonal cutting involves the use of a wedge- shaped tool in which the cutting edge is perpendicular to the direction of speed motion into the work material. 21.9 Name and briefly describe the three types of chips that occur in metal cutting. Answer. The three types are: (1) discontinuous, in which the chip is formed into separated segments; (2) continuous, in which the chip does not segment and is formed from a ductile metal; and (3) continuous with built-up edge, which is the same as (2) except that friction at the tool-chip interface causes adhesion of the work material to the tool rake face. 21.10 Describe in words what the Merchant equation tells us. 130 Answer. The Merchant equation states that the shear plane angle increases when rake angle is increased and friction angle is decreased. 21.11 What is the specific energy in metal machining? Answer. Specific energy is the amount of energy required to remove a unit volume of the work material. 21.12 What does the term size effect mean in metal cutting? Answer. The size effect refers to the fact that the specific energy increases as the cross-section area of the chip (to x w) decreases. 21.13 What is a tool-chip thermocouple? Answer. A tool-chip thermocouple is comprised of the tool and chip as the two dissimilar (metallic) materials forming the thermocouple junction; as the tool-chip interface heats up during cutting, an emf is emitted from the junction which can be measured to indicate cutting temperature. Multiple Choice Quiz There are a total of 11 correct answers in the following multiple choice questions (some questions have multiple answers that are correct). To attain a perfect score on the quiz, all correct answers must be given, since each correct answer is worth 1 point. For each question, each omitted answer or wrong answer reduces the score by 1 point, and each additional answer beyond the number of answers required reduces the score by 1 point. Percentage score on the quiz is based on the total number of correct answers. 21.1 A lathe is used to perform which of the following machining operations (one best answer)? (a) broaching, (b) drilling, (c) milling, or (d) turning. Answer. (d) 21.2 With which one of the following geometric forms is the drilling operation most closely associated? (a) external cylinder, (b) flat plane, (c) round hole, (d) screw threads, or (e) sphere. Answer. (c) 21.3 If the cutting conditions in a turning operation are v = 300 ft/min, f = 0.010 in/rev, and d = 0.100 inch, which one of the following is the material removal rate? (a) 0.3 in3/min, (b) 0.025 in3/min, (c) 3.0 in3/min, or (d) 3.6 in3/min. Answer. (d) 21.4 A roughing operation generally involves which one of the following combinations of cutting conditions? (a) high v, f, and d; (b) high v, low f and d; (c) low v, high f and d; or (d) low v, f, and d. Answer. (c) 21.5 The chip thickness ratio is which one of the following? (a) tc/to, (b) to/tc, (c) f/d, or (d) to/w. Answer. (b) 21.6 Which of the three types of chip would be expected in a turning operation conducted at low cutting speeds on a brittle work material (one answer)? (a) continuous, (b) continuous with built-up edge, or (c) discontinuous. 131 Answer. (c) 21.7 According to the Merchant equation, an increase in rake angle would have which of the following results, all other factors remaining the same (more than one)? (a) decrease in friction angle, (b) decrease in power requirements, (c) decrease in shear plane angle, (d) increase in cutting temperature, or (e) increase in shear plane angle. Answer. (b) and (e). 21.8 Which of the following metals would usually have the lowest unit horsepower (one answer)? (a) aluminum, (b) brass, (c) cast iron, or (d) steel. Answer. (a) 21.9 For which one of the following values of chip thickness before the cut to would you expect the specific energy to be the greatest? (a) 0.010 inch, (b) 0.025 inch, or (c) 0.12 mm. Answer. (c) 21.10 Which of the following cutting conditions has the strongest effect on cutting temperature? (a) feed or (b) speed. Answer. (b) Problems Chip Formation and Forces in Machining 21.1 In an orthogonal cutting operation, the tool has a rake angle = 15. The chip thickness before the cut = 0.30 mm and the cut yields a deformed chip thickness = 0.65 mm. Calculate (a) the shear plane angle and (b) the shear strain for the operation. Solution: (a) r = to/tc = 0.30/0.65 = 0.4615 = tan-1(.4615 cos 15/(1 - .4615 sin 15)) = tan-1(.5062) = 26.85° (b) Shear strain = cot 26.85 + tan (26.85 - 15) = 1.975 + 0.210 = 2.185 21.2 In Problem 21.1, suppose the rake angle were changed to a = 0. Assuming that the friction angle remains the same, determine (a) the shear plane angle, (b) the chip thickness, and (c) the shear strain for the operation. Solution: From Problem 21.1, = 15and = 26.85. Using the Merchant Equation, Eq. (21.16): = 45 + /2 - /2; rearranging, = 2(45) + - 2 = 90 + 15 – 2(26.85) = 51.3 Now, with = 0 and remaining the same at 51.3, = 45 + 0/2 – 51.3/2 = 19.35° (b) Chip thickness at = 0: tc = to/tan = 0.30/tan 19.35 = 0.854 mm (c) Shear strain = cot 19.35 + tan (19.35 - 0) = 2.848 + 0.351 = 3.199 21.3 In an orthogonal cutting operation, the tool has a rake angle = -5. The chip thickness before the cut = 0.012 in and the cut yields a deformed chip thickness = 0.028 in. Calculate (a) the shear plane angle and (b) the shear strain for the operation. Solution: (a) r = to/tc = 0.012/0.028 = 0.4286 = tan-1(.4286 cos -5/(1 - .4286 sin -5)) = tan-1(.4116) = 22.37° (b) Shear strain = cot 22.37 + tan (22.37 – (-5)) = 2.430 + 0.518 = 2.948 132 21.4 The cutting conditions in a turning operation are: v = 2 m/s, f = 0.25 mm, and d = 3.0 mm. The tool rake angle = 10which produces a deformed chip thickness tc = 0.54 mm. Determine: (a) shear plane angle, (b) shear strain, and (c) material removal rate. Use the orthogonal cutting model as an approximation of the turning process. Solution: (a) r = to/tc = 0.25/0.54 = 0.463 = tan-1(.463 cos 10/(1 - .463 sin 10)) = tan-1(.4959) = 26.4° (b) = cot 26.4 + tan (26.4 - 10) = 2.017 + 0.294 = 2.311 (c) MRR = (2 m/s x 103 mm/m)(0.25)(3.0) = 1500 mm3/s 21.5 The cutting force and thrust force in an orthogonal cutting operation are: Fc = 1470 N and Ft = 1589 N. The rake angle = 5, the width of the cut = 5.0 mm, the chip thickness before the cut = 0.6, and the chip thickness ratio = 0.38. Determine (a) the shear strength of the work material and (b) the coefficient of friction in the operation. Solution: (a) = tan-1(0.38 cos 5/(1 - 0.38 sin 5)) = tan-1(0.3916) = 21.38 Fs = 1470 cos 21.38 – 1589 sin 21.38 = 789.3 N As = (0.6)(5.0)/sin 21.38 = 3.0/.3646 = 8.23 mm2 S = 789.3/8.23 = 95.9 N/mm2 = 95.9 MPa (b) = 45 + /2 - /2; rearranging, = 2(45) + - 2 = 90 + 5 – 2(21.38) = 52.24 = tan 52.24 = 1.291 21.6 The cutting force and thrust force have been measured in an orthogonal cutting operation: Fc = 300 lb and Ft = 291 lb. The rake angle = 10, the width of the cut = 0.200 in, the chip thickness before the cut = 0.015, and the chip thickness ratio = 0.4. Determine (a) the shear strength of the work material and (b) the coefficient of friction in the operation. Solution: = tan-1(0.4 cos 10/(1 - 0.4 sin 10)) = tan-1(.4233) = 22.94 Fs = 300 cos 22.94 - 291sin 22.94 = 162.9 lb. As = (0.015)(0.2)/sin 22.94 = 0.0077 in2 S = 162.9/0.0077 = 21,167 lb/in2. = 2(45) + 10 - 2(22.94) = 54.1 = tan 54.1 = 1.38 21.7 An orthogonal cutting operation is performed using a rake angle of 15, to = 0.012 in and w = 0.100 in. The chip thickness ratio is measured after the cut to be 0.55. Determine: (a) the chip thickness after the cut, (b) the shear angle, (c) the friction angle, (d) the coefficient of friction, and (e) the shear strain. Solution: (a) r = to/tc, tc = to/r = 0.012/.55 = 0.022 in. (b) = tan-1(.55 cos 15/(1 - .55 sin 15)) = tan-1(.6194) = 31.8° (c) = 2(45) + 15 - 2(31.8) = 41.5° (d) = tan 41.5 = 0.88 (e) = cot 31.8 + tan (31.8 - 15) = 1.615 + 0.301 = 1.92 21.8 The orthogonal cutting operation described in previous Problem 21.7 involves a work material whose shear strength is 40,000 lb/in2. Based on your answers to the previous problem, compute: (a) the shear force, (b) the cutting force, (c) the thrust force, and (d) the friction force. 133 Solution: (a) As = (0.012)(0.10)/sin 31.8 = 0.00228 in2. Fs = AsS = 0.0028(40,000) = 91.2 lb. (b) Fc = 91.2 cos (41.5 - 15)/cos (31.8 + 41.5 -15) = 155 lb. (c) Ft = 91.2 sin (41.5 - 15)/cos (31.8 + 41.5 -15) = 77.2 lb. (d) F = 155 sin 15 - 77.2 cos 15 = 115 lb. 21.9 In an orthogonal cutting operation, the rake angle = -5, to = 0.2 mm and w = 4.0 mm. The chip ratio r = 0.4. Determine: (a) the chip thickness after the cut, (b) the shear angle, (c) the friction angle, (d) the coefficient of friction, and (e) the shear strain. Solution: (a) r = to/tc, tc = to/r = 0.2/.4 = 0.5 mm (b) = tan-1(.4 cos -5/(1 - .4 sin -5)) = tan-1(0.3851) = 21.1° (c) = 2(45) + (-5) - 2(21.8) = 42.9° (d) = tan 42.9 = 0.93 (e) = cot 31.8 + tan (31.8 - 15) = 2.597 + 0.489 = 3.09 21.10 The shear strength of a certain work material = 50,000 lb/in2. An orthogonal cutting operation is performed using a tool with a rake angle = 20at the following cutting conditions: Speed = 100 ft/min, chip thickness before the cut = 0.015 in, and width of cut = 0.150 in. The resulting chip thickness ratio = 0.50. Determine: (a) the shear plane angle; (b) the shear force; (c) cutting force and thrust force, and (d) friction force. Solution: (a) = tan-1(.5 cos 20/(1 - .5 sin 20)) = tan-1(0.5668) = 29.5° (b) As = (0.015)(0.15)/sin 29.5 = 0.00456 in2. Fs = AsS = 0.00456(50,000) = 228 lb. (c) = 2(45) + 20 - 2(29.5) = 50.9 Fc = 228 cos (50.9 - 20)/cos (29.5 + 50.9 -20) = 397 lb. Ft = 228 sin (50.9 - 20)/cos (29.5 + 50.9 -20) = 238 lb. (d) F = 397 sin 20 - 238 cos 20 = 359 lb. 21.11 Solve the previous problem except that the rake angle has been changed to -5and the resulting chip thickness ratio = 0.35. Solution: (a) = tan-1(.35 cos -5/(1 - .35 sin -5)) = tan-1(0.3384) = 18.7° (b) As = (0.015)(0.15)/sin 18.7 = 0.00702 in2. Fs = AsS = 0.00702(50,000) = 351 lb. (c) = 2(45) + (-5) - 2(18.7) = 47.6 Fc = 351 cos (47.6 - (-5))/cos (18.7 + 47.6 - (-5)) = 665 lb. Ft = 351 sin (47.6 - (-5))/cos (18.7 + 47.6 - (-5)) = 870 lb. (d) F = 665 sin (-5) - 870 cos (-5) = 808 lb. 21.12 A turning operation is performed using the following cutting conditions: v = 300 ft/min, f = 0.010 in/rev, and d = 0.100 in. The rake angle on the tool in the direction of chip flow = 10, resulting in a chip ratio = 0.42. The shear strength of the work material = 40,000 lb/in2. Using the orthogonal model as an approximation of turning, determine: (a) the shear plane angle; (b) the shear force; (c) cutting force and feed force. 134 Solution: (a) = tan-1(.42 cos 10/(1 - .42 sin 10)) = tan-1(0.4462) = 24.0° (b) As = (0.010)(0.10)/sin 24.0 = 0.00245 in2. Fs = AsS = 0.00245(40,000) = 98.2 lb. (c) = 2(45) + 10 - 2(24.0) = 51.9 Fc = 98.2 cos (51.9 - 10)/cos (24.0 + 51.9 - 10) = 179 lb. Ft = 98.2 sin (51.9 - 10)/cos (24.0 + 51.9 - 10) = 161 lb. 21.13 Turning is performed on a work material with shear strength of 250 MPa. The following conditions are used: v = 3.0 m/s, f = 0.20 mm/rev, d = 3.0 mm, and rake angle = 7in the direction of chip flow. The resulting chip ratio = 0.5. Using the orthogonal model as an approximation of turning, determine: (a) the shear plane angle; (b) the shear force; (c) cutting force and feed force. Solution: (a) = tan-1(.5 cos 7/(1 - .5 sin 7)) = tan-1(0.5285) = 27.9° (b) As = (0.2)(3.0)/sin 27.9 = 1.284 mm2. Fs = AsS = 1.284(250) = 321 N. (c) = 2(45) + 7 - 2(27.9) = 41.2 Fc = 321 cos (41.2 - 7)/cos (27.9 + 41.2 - 7) = 568 N. Ff = 321 sin (41.2 - 7)/cos (27.9 + 41.2 - 7) = 387 N. 21.14 A turning operation is made with a rake angle of 10, a feed of 0.010 in/rev and a depth of cut = 0.100 in. The shear strength of the work material is known to be 50,000 lb/in2, and the chip thickness ratio is measured after the cut to be 0.40. Determine the cutting force and the feed force. Use the orthogonal cutting model as an approximation of the turning process. Solution: = tan-1(0.4 cos 10/(1 - 0.4 sin 10)) = tan-1(0.4233) = 22.9° As = (0.010)(0.10)/sin 22.9 = 0.00257 in2. Fs = AsS = 0.00256(50,000) = 128 lb. = 2(45) + 10 - 2(22.9) = 54.1 Fc = 128 cos (54.1 - 10)/cos (22.9 + 54.1 - 10) = 236 lb. Ft = 128 sin (54.1 - 10)/cos (22.9 + 54.1 - 10) = 229 lb. 21.15 Show how Eq. (21.3) is derived from the definition of chip ratio, Eq. (21.2) and Figure 21.5(b). Solution: Begin with the definition of the chip ratio, Eq. (21.2): r = to/tc = sin /cos (- ) Rearranging, r cos (- ) = sin  Using the trigonometric identity cos(- ) = cos cos + sin sin  r (cos cos + sin sin ) = sin  Dividing both sides by sin , we obtain r cos /tan + r sin = 1 r cos /tan = 1 - r sin  Rearranging, tan = r cos /(1 - r sin ) Q.E.D. 21.16 Show how Eq. (21.4) is derived from Figure 21.6. Solution: In the figure, = AC/BD = (AD + DC)/BD = AD/BD + DC/BD AD/BD = cot and DC/BD = tan (- ) Thus, = cot + tan (- ) Q.E.D. 21.17 Derive the force equations for F, N, Fs, and Fn (Eqs. (21.9) through (21.12) in the text) using the force diagram of Figure 21.11. 135 Solution: Eq. (21.9): In Figure 23.11, construct a line starting at the intersection of Ft and Fc that is perpendicular to the friction force F. The constructed line is at an angle with Fc. The vector F is divided into two line segments, one of which = Fc sin and the other = Ft cos . Thus, F = Fc sin + Ft cos . Q.E.D. Eq. (21.10): In Figure 23.11, translate vector N vertically upward until it coincides with the previously constructed line, whose length = Fc cos . Next, translate vector Ft to the right and downward at an angle until its base is at the arrowhead of F. Ft now makes an angle with F. The arrowhead of Ft will now be at the base of the translated base of N. The distance along the previously constructed line between the Ft arrowhead (base of translated N vector) and F is Ft sin. Hence, N = Fc cos - Ft sin Q.E.D. Eq. (21.11): In Figure 23.11, extend vector Fs in the opposite direction of its arrowhead, and from the intersection of Ft and Fc construct a line that is perpendicular to vector Fs. A right triangle now exists in which Fc is the hypotenuse and the two sides are (1) the extended Fs vector and (2) the constructed line that runs between Fs and the intersection of Fc and Ft. The extended Fs vector is related to Fc as Fc cos . The length difference between the extended Fs vector and the original Fs vector is Ft sin . Thus Fs (original) = Fc cos - Ft sin Q.E.D. Eq. (21.12): In Figure 23.11, construct a line from the intersection of Ft and Fc that is perpendicular to and intersects with vector Fn. Vector Fn is now divided into two line segments, one of which = Ft cos and the other = Fc sin . Hence, Fn = Fc sin + Ft cos Q.E.D. Power and Energy in Machining 21.18 In a turning operation on stainless steel with hardness = 200 HB, the cutting speed = 200 m/min, feed = 0.25 mm/rev, and depth of cut = 7.5 mm. How much power will the lathe draw in performing this operation if its mechanical efficiency = 90%. Use Table 21.3 to obtain the appropriate specific energy value. Solution: From Table 21.3, U = 2.8 N-m/mm3 = 2.8 J/mm3 MRR = vfd = (200 m/min)(103 mm/m)(0.25 mm)(7.5 mm) = 375,000 mm3/min = 6250 mm3/s Pc = (6250 mm3/s)(2.8 J/mm3) = 17,500 J/s = 17,500 W = 17.5 kW Accounting for mechanical efficiency, Pg = 17.5/0.90 = 19.44 kW 21.19 In previous Problem 21.18, compute the lathe power requirements if feed = 0.50 mm/rev. Solution: This is the same basic problem as the previous, except that a correction must be made for the “size effect.” Using Figure 21.14, for f = 0.50 mm, correction factor = 0.85. From Table 21.3, U = 2.8 J/mm3. With the correction factor, U = 2.8(0.85) = 2.38 J/mm3. MRR = vfd = (200 m/min)(103 mm/m)(0.50 mm)(7.5 mm) = 750,000 mm3/min = 12,500 mm3/s Pc = (12,500 mm3/s)(2.38 J/mm3) = 29,750 J/s = 29,750 W = 29.75 kW Accounting for mechanical efficiency, Pg = 29.75/0.90 = 33.06 kW 21.20 In a turning operation on aluminum, cutting conditions are as follows: v = 900 ft/min, f = 0.020 in/rev, and d = 0.250 in. What horsepower is required of the drive motor, if the lathe has a mechanical efficiency = 87%? Use Table 21.3 to obtain the appropriate unit horsepower value. 136 Solution: From Table 21.3, HPu = 0.25 hp/(in3/min) for aluminum. Since feed is greater than 0.010 in/rev in the table, a correction factor must be applied from Figure 21.14. For f = 0.020 in/rev = to, correction factor = 0.9. HPc = HPu x MRR, HPg = HP/E MRR = vfd = 900 x 12(.020)(0.250) = 54 in3/min HPc = 0.9(0.25)(54) = 12.2 hp HPg = 12.2/0.87 = 14.0 hp 21.21 In a turning operation on plain carbon steel whose Brinell hardness = 275 HB, the cutting speed is set at 200 m/min and depth of cut = 6.0 mm. The lathe motor is rated at 25 kW, and its mechanical efficiency = 90%. Using the appropriate specific energy value from Table 21.3, determine the maximum feed that can be set for this operation. Solution: From Table 21.3, U = 2.8 N-m/mm3 = 2.8 J/mm3 MRR = vfd = (200 m/min)(103 mm/m)(6 mm)f = 1200(103)f mm3/min = 20(103)f mm3/s Available power Pc = Pg E = 25(103)(0.90) = 22.5 (103) = 22,500W Required power Pc = (2.8 N-m/mm3)( 20 x 103) f = 56,000 f Setting available power = required power, 22,500 = 56,000 f f = 22,500/56,000 = 0.402 mm (this should be interpreted as mm/rev for a turning operation) However, for this feed, correction factor in Figure 21.14 = 0.9; thus U = 2.8(0.90) = 2.52 N-m/mm3 and an iterative calculation procedure is required to match the unit power value with the feed, taking the correction factor into account. Required Pc = (2.52)(20 x 103) f = 50,400 f Again setting available power = required power, 22,500 = 50,400 f f = 22,500/50,400 = 0.446 mm/rev One more iteration using the correction factor yields a value around f = 0.45 mm/rev. 21.22 A turning operation is to be performed on a 20 hp lathe with efficiency = 90%. The work material is an alloy steel whose hardness is in the range 360 to 380 HB. Cutting conditions are: v = 400 ft/min., feed = 0.010 in/rev, and depth of cut = 0.150 in. Based on these values, can the job be performed on the 20 hp lathe? Use Table 21.3 to obtain the appropriate unit horsepower value. Solution: From Table 21.3, HPu = 1.6 hp/(in3/min) MRR = vfd = 400 ft/min(12 in./ft)(0.01 in.)(0.150 in.) = 7.2 in.3/min. HPc = (7.2 in3/min)(1.6 hp/(in3/min)) = 11.52 hp required. At efficiency E = 90%, available horsepower = 0.90(20) = 18 hp Since available horsepower exceeds required horsepower, the job can be accomplished on the 20 hp lathe. 21.23 Suppose the cutting speed in Problems 21.7 and 21.8 is v = 200 ft/min. From your answers to those problems, find: (a) the horsepower consumed in the operation, (b) the metal removal rate in in.3/min, (c) the unit horsepower (hp-min/(in3), and (d) the specific energy (in-lb/in3). Solution: (a) From Problem 21.8, Fc = 155 lb. HPc = 155(200)/33,000 = 0.94 hp. (b) MRR = vfd = (200 x 12)(0.012)(0.100) = 2.88 in.3/min. (c) HPu = 0.94/2.88 = 0.326 hp/(in.3/min) (d) U = 155(200)/2.88 = 10,764 ft-lb/in3 = 129,167 in.-lb/in3. 21.24 For Problem 21.12, the lathe has a mechanical efficiency = 0.80. Determine: (a) the horsepower consumed by the turning operation; (b) the horsepower that must be generated by the lathe; (c) the unit horsepower and specific energy for the work material in this operation. 137 Solution: (a) From Problem 21.12, Fc = 179 lb. HPc = 179(300)/33,000 = 1.63 hp. (b) HPg = 1.63/0.8 = 2.04 hp (c) MRR = (300 x 12)(0.01)(0.100) = 3.6 in3/min. HPu = 1.63/3.6 = 0.453 hp/(in3/min) U = 179(300 x 12)/3.6 = 179,000 in-lb/in3. 21.25 In a turning operation on low carbon steel (175 BHN), the cutting conditions are: v = 400 ft/min, f = 0.010 in/rev, and d = 0.075 in. The lathe has a mechanical efficiency = 0.85. Based on the unit horsepower values in Table 21.3, determine: (a) the horsepower consumed by the turning operation; (b) the horsepower that must be generated by the lathe. Solution: (a) From Table 21.3, HPu = 0.6 hp/(in3/min) for low carbon steel. HPc = HPu x MRR MRR = vfd = 400 x 12(.010)(0.075) = 3.6 in3/min HPc = 0.6(3.6) = 2.16 hp (b) HPg = 2.16/0.85 = 2.54 hp 21.26 Solve Problem 21.24 except that the feed f = 0.005 in/rev and the work material is stainless steel (Brinell Hardness = 225 HB). Solution: (a) From Table 21.3, HPu = 1.0 hp/(in3/min) for stainless steel. Since feed is lower than 0.010 in/rev in the table, a correction factor must be applied from Figure 21.14. For f = 0.005 in/rev = to, correction factor = 1.25. HPc = HPu x MRR MRR = 400 x 12(.005)(0.075) = 1.8 in3/min HPc = 1.25(1.0)(1.8) = 2.25 hp (b) HPg = 2.25/0.85 = 2.65 hp 21.27 A turning operation is carried out on aluminum (100 BHN), the cutting conditions are: v = 5.6 m/s, f = 0.25 mm/rev, and d = 2.0 mm. The lathe has a mechanical efficiency = 0.85. Based on the specific energy values in Table 21.3, determine: (a) the cutting power and (b) the gross power in the turning operation, in Watts. Solution: (a) From Table 21.3, U = 0.7 N-m/mm3 for aluminum. MRR = vfd = 5.6(103)(.25)(2.0) = 2.8(103) mm3/s. Pc = U x MRR = 0.7(2.8)(103) = 1.96(103) n-m/s = 1960 W. (b) Gross power Pg = 1960/0.8 = 2450 W. 21.28 Solve Problem 21.27 but with the following changes: v = 1.3 m/s, f = 0.75 mm/rev, and d = 4.0 mm. Note that although the power used in this operation is virtually the same as in the previous problem, the metal removal rate is about 40% greater. Solution: (a) From Table 21.3, U = 0.7 N-m/mm3 for aluminum. Since feed is greater than 0.25 mm/rev in the table, a correction factor must be applied from Figure 21.14. For f = 0.75 mm/rev = to, correction factor = 0.80. MRR = vfd = 1.3(103)(.75)(4.0) = 3.9(103) mm3/s. Pc = U x MRR = 0.8(0.7)(3.9)(103) = 1.97(103) n-m/s = 1970 W. (b) Gross power Pg = 1970/0.8 = 2460 W. 21.29 A turning operation is performed on an engine lathe using a tool with zero rake angle in the direction of chip flow. The work material is an alloy steel with hardness = 325 Brinell hardness. 138 The feed is .015 in/rev, the depth of cut is 0.125 in and the cutting speed is 300 ft/min. After the cut, the chip thickness ratio is measured to be 0.45. (a) Using the appropriate value of specific energy from Table 21.3, compute the horsepower at the drive motor, if the lathe has an efficiency = 85%. (b) Based on horsepower, compute your best estimate of the cutting force for this turning operation. Use the orthogonal cutting model as an approximation of the turning process. Solution: (a) From Table 21.3, HPu = 1.3 hp/(in3/min) for alloy steel of the specified hardness. Since feed is greater than 0.010 in/rev in the table, a correction factor must be applied from Figure 21.14. For f = 0.015 in/rev = to, correction factor = 0.95. HPc = HPu x MRR, HPg = HP/E MRR = 300 x 12(.015)(0.125) = 6.75 in3/min HPc = 0.95(1.3)(6.75) = 8.3 hp HPg = 8.3/0.85 = 9.8 hp (b) HPc = vFc/33,000. Rearranging, Fc = 33,000 (HP/v) = 33,000 (8.3/300) = 913 lb. 21.30 A lathe performs a turning operation on a workpiece of 6.0 in diameter. The shear strength of the work = 40,000 lb/in2. The rake angle of the tool = 10. The machine settings are: rotational speed = 500 rev/min, feed = 0.0075 in/rev, and depth = 0.075 in. The chip thickness after the cut is 0.015 in. Determine: (a) the horsepower required in the operation, (b) the unit horsepower for this material under these conditions, and (c) the unit horsepower as it would be listed in Table 21.3 for a to of 0.010 in. Use the orthogonal cutting model as an approximation of the turning process. Solution: (a) Must find Fc and v to determine HP. r = 0.0075/0.015 = 0.5 = tan-1(0.5 cos 10/(1 - 0.5 sin 10)) = tan-1 (0.5392) = 28.3 = 2(45) + 10 -2(28.3) = 43.4 As = 0.0075(0.075)/sin 28.3 = 0.00119 in2 Fs = 40,000(0.00119) = 47.5 lb. Fc = 47.5 cos(43.4 - 10)/cos(28.3 + 43.4 - 10) = 83.6 lb. v = 500 rev/min(6/12 ft/rev) = 785 ft/min. HP = 83.6(785)/33,000 = 1.99 hp. (b) MRR = 785 x 12(0.0075)(0.075) = 5.3 in3/min. HPu = 1.99/5.3 = 0.375 hp/(in3/min) (c) Correction factor = 1.15 from Fig. 21.14 to account for the fact that f = 0.0075 in/rev instead of 0.010 in/rev. Taking this correction factor into account, HPu = 0.375/1.15 = 0.326 hp/(in3/min) as it would appear in Table 21.3 for a feed f = 0.010 in/rev. Cutting Temperature 21.31 Orthogonal cutting is performed on a metal whose mass specific heat = 1.1 J/g-C, density = 2.7 g/cm3, and thermal diffusivity = 0.9 cm2/s. The following cutting conditions are used: v = 4.0 m/s, to = 0.3 mm, and w = 2.0 mm. The cutting force is measured at Fc = 1100 N. Using Cook's equation, determine the cutting temperature if the ambient temperature = 20C. Solution: C = (2.7 g/cm3)(1.1 J/g-C) = 2.97 J/cm3-C = 2.97(10-3) J/mm3-C K = 0.9 cm2/s = 90 mm2/s U = 1100 N x 4 m/s/(4000 mm/s x 0.3 mm x 2 mm) = 1.833 N-m/mm3 T = 20 + (0.4 x 1.833 N-m/mm3/2.97(10-3) J/mm3-C)(4000 mm/s x 0.3 mm/90 mm2/s)0.333 T = 20 + (0.2469 x 103 C)(13.33).333 = 20 + 246.9(2.37) = 20+ 585= 605°C 139 21.32 Consider a turning operation performed on steel whose hardness = 225 HB at a speed = 3.0 m/s, feed = 0.25 mm, and depth = 4.0 mm. Using values of thermal properties found in the tables and definitions of Section 4.1 and the appropriate specific energy value from Table 21.3, compute an estimate of cutting temperature using the Cook equation. Assume ambient temperature = 20C. Solution: From Table 21.3, U = 2.8 N-m/mm3 = 2.8 J/mm3 From Table 4.1, = 7.87 g/cm3 = 7.87(10-3) g/mm3 From Table 4.1, C = 0.11 Cal/g-C. From note “a” at the bottom of the table, 1 cal = 4.186 J. Thus, C = 0.11(4.186) = 0.460 J/ g-C C = (7.87 g/cm3)(0.46 J/g-C) = 3.62(10-3) J/mm3-C From Table 4.2, thermal conductivity k = 0.046 J/s-mm-C From Eq. (4.3), thermal diffusivity K = k/C K = 0.046 J/s-mm-C /[(7.87 x 10-3 g/mm3)(0.46 J/g-C) = 12.7 mm2/s Using Cook’s equation, to = f = 0.25 mm T = 0.4(2.8)/3.62(10-3)(3(103)(0.25)/12.7) = 0.309(103)(59.06)0.333 = 309(3.89) = 1201 C Final temperature, taking ambient temperature in account T = 20 + 1201 = 1221°C 21.33 An orthogonal cutting operation is performed on a certain metal whose volumetric specific heat = 110 in-lb/in3-F, and thermal diffusivity = 0.140 in2/sec. The following cutting conditions are used: v = 350 ft/min, to = 0.008 in, and w = 0.100 in. The cutting force is measured at Fc = 200 lb. Using Cook's equation, determine the cutting temperature if the ambient temperature = 70F. Solution: v = 350 ft/min x 12 in/ft/60 sec/min. = 70 in/sec. U = Fcv/vtow = 200(70)/(70 x 0.008 x 0.100) = 250,000 in-lb/in3. T = 70 + (0.4U/C)(vto/K)0.333 = T = 70 + (0.4 x 250,000/110)(70 x 0.008/0.14)0.333 = 70 + (909)(4)0.333 = 70 + 1436 = 1506° F 21.34 It is desired to estimate the cutting temperature for a certain alloy steel whose hardness = 275 Brinell. Use the appropriate value of specific energy from Table 21.3 and compute the cutting temperature by means of the Cook equation for a turning operation in which the following cutting conditions are used: speed v = 300 ft/min, feed f = 0.0075 in/rev, and depth d = 0.100 in. The thermal properties of the work material are: volumetric specific heat = 200 in lb/in3 F, and thermal diffusivity = 0.14 in2/sec. Assume ambient temperature = 70F. Solution: From Table 21.3, U for alloy steel (275 BHN) = 400,000 in-lb/in3. Since f = 0.0075 in/rev., correction factor = 1.10. Therefore U = 400,000(1.1) = 440,000 in-lb/in3. v = 300 ft/min x 12 in/ft/60 sec/min. = 60 in/sec. T = 70 + (0.4U/C)(vto/K)0.333 = 70 + (0.4 x 440,000/200)(60 x 0.0075/0.14)0.333 = 70 + (872)(3.214)0.333 = 70 + 1282 = 1352° F 21.35 An orthogonal machining operation removes metal at 1.8 in3/min. The cutting force in the process = 300 lb. The work material has a thermal diffusivity = 0.18 in2/sec and a volumetric specific heat = 124 in-lb/in3 F. If the feed f = to = 0.010 in and width of cut = 0.100 in, use the Cook formula to compute the cutting temperature in the operation given that ambient temperature = 70F. Solution: MRR = vtow, v = MRR/tow = 1.8/(0.01 x 0.100) = 1800 in/min = 30 in/sec U = Fcv/vtow = 300(30)/(30 x 0.010 x 0.100) = 300,000 in-lb/in3. T = 70 + (0.4U/C)(vto/K)0.333 = 70 + (0.4 x 300,000/124)(30 x 0.010/0.18)0.333 = 70 + (968)(1.667)0.333 = 70 + 1147 = 1217° F 140 21.36 A turning operation uses a cutting speed = 200 m/min, feed = 0.25 mm/rev, and depth of cut = 4.00 mm. The thermal diffusivity of the work material = 20 mm2/s and the volumetric specific heat = 3.5 (10-3) J/mm3-C. If the temperature increase above ambient temperature (20F) is measured by a tool-chip thermocouple to be 700C, determine the specific energy for the work material in this operation. Solution: Rearranging the Cook equation, U = T(C/0.4)(K/vto)0.333 U = (700 – 2-)(3.5 x 10-3/0.4)(20/{(200/60)(103)(0.25)})0.333 U = 680(8.75 x 10-3)(0.024)0.333 = 5.95(0.2888) = 1.72 N-m/mm3 21.37 During a turning operation, a tool-chip thermocouple was used to measure cutting temperature. The following temperature data were collected during the cuts at three different cutting speeds (feed and depth were held constant): (1) v = 100 m/min, T = 505C, (2) v = 130 m/min, T = 552C, (3) v = 160 m/min, T = 592C. Determine an equation for temperature as a function of cutting speed that is in the form of the Trigger equation, Eq. (21.23). Solution: Trigger equation T = Kvm Choose points (1) and (3) and solve simultaneous equations using T = Kvm as the model. (1) 505 = K(100)m and (3) 592 = K(160)m (1) ln 505 = ln K + m ln 100 and (3) ln 592 = ln K + m ln 160 Combining (1) and (3): ln 505 - m ln 100 = ln 592 - m ln 160 6.2246 – 4.6052 m = 6.3835 – 5.0752 m 0.47 m = 0.1589 m = 0.338 (1) K = 505/1000.338 = 505/4.744 = 106.44 (2) K = 592/1600.338 = 592/5.561 = 106.45 Use K = 106.45 Check equation with data point (2): T = 106.45(130)0.338 = 551.87C (pretty close to the given value of 552C). 141 22 MACHINING OPERATIONS AND MACHINE TOOLS Review Questions 22.1 Discuss the differences between rotational parts and prismatic parts in machining. Answer. Rotational parts are cylindrical or disk-shaped and are machined on a turning machine; prismatic parts are block-shaped or flat and are generally produced on a milling machine, shaper, or planer. 22.2 Distinguish between generating and forming when machining workpart geometries. Answer. Generating refers to the creation of work geometry due to the feed trajectory of the cutting tool; examples include straight turning, taper turning, and profile milling. Forming involves the creation of work geometry due to the shape of the cutting tool; common examples include form turning and drilling. 22.3 Give two examples of machining operations in which generating and forming are combined to create workpart geometry. Answer. Two examples are thread cutting on a lathe and slot milling; both are described in Article 25.6.1. 22.4 Describe the turning process. Answer. Turning is a machining process in which a single point tool removes material from the surface of a rotating cylindrical workpiece, the tool being fed in a direction parallel to the axis of work rotation. 22.5 What is the difference between threading and tapping? Answer. A threading operation is performed on a turning machine and produces an external thread, while tapping is normally performed on a drilling machine and produces an internal thread. 22.6 How does a boring operation differ from a turning operation? Answer. Boring produces an internal cylindrical shape from an existing hole, while turning produces an external cylindrical shape. 22.7 What is meant by the designation 12 x 36 inch lathe? Answer. A 12 x 36 lathe has a 12 inch swing (maximum work diameter that can be accommodated) and a 36 inch distance between centers (indicating the maximum work length that can be held between centers). 22.8 Name the various ways in which a workpart can be held in a lathe. Answer. Methods of holding the work in a lathe include: (1) between centers, (2) chuck, (3) collet, and (4) face plate. See Article 25.1.4. 22.9 What is the difference between a live center and a dead center, when these terms are used in the context of workholding in a lathe? 142 Answer. A center holds the work during rotation at the tailstock end of the lathe. A live center is mounted in bearings and rotates with the work, while a dead center does not rotate - the work rotates about it. 22.10 How does a turret lathe differ from an engine lathe? Answer. A turret lathe has a toolholding turret in place of a tailstock; the tools in the turret can be brought to work to perform multiple cutting operations on the work without the need to change tools as in operating a conventional engine lathe. 22.11 What is a blind hole? Answer. A blind hole does not exit the work; by comparison, a through hole exits the opposite side of the workpart. 22.12 What is the distinguishing feature of a radial drill press? Answer. A radial drill has a long radial arm along which the drill head can be positioned to allow the drilling of large workparts. 22.13 What is the difference between peripheral milling and face milling? Answer. In peripheral milling, cutting is accomplished by the peripheral teeth of the milling cutter and the tool axis is parallel to the work surface; in face milling, cutting is accomplished by the flat face of the cutter whose axis is perpendicular to the work surface. 22.14 Describe profile milling. Answer. Profile milling generally involves the milling of the outside periphery of a flat part. 22.15 What is pocket milling? Answer. Pocket milling uses an end milling cutter to machine a shallow cavity (pocket) into a flat workpart. 22.16 Describe the difference between up milling and down milling? Answer. In up milling, the cutter speed direction is opposite the feed direction; in down milling, the direction of cutter rotation is the same as the feed direction. 22.17 How does a universal milling machine differ from a conventional knee-and-column machine? Answer. The universal milling machine has a worktable that can be rotated about a vertical axis to present the part at any specified angle to the cutter spindle. 22.18 What is a machining center? Answer. A machining center is a CNC machine tool capable of performing multiple types of cutting operations involving rotating spindles (e.g., milling, drilling); the machine is typically equipped with automatic tool-changing, pallet shuttles to speed workpart changing, and automatic workpart positioning. 22.19 What is the difference between a machining center and a turning center? Answer. A machining center is generally confined to rotating spindle operations (e.g., milling, drilling); while a turning center performs turning type operations, generally with single point tools. 22.20 What can a mill-turn center do that a conventional turning center cannot do? 143 Answer. The mill-turn center has the capacity to position a rotational workpart at a specified angular location, permitting milling or drilling to be performed at a location on the periphery of the part. 22.21 How do shaping and planing differ? Answer. In shaping, the work is stationary during the cut, and the speed motion is performed by the cutting tool; while in planing, the cutting tool is stationary, and the workpart is moved past the tool in the speed motion. 22.22 What is the difference between internal broaching and external broaching? Answer. Internal broaching is accomplished on the inside surface (hole) of a workpart; while external broaching is performed on one of the outside surfaces of the part. 22.23 Identify the three basic forms of sawing operation? Answer. The three forms of sawing are: (1) hacksawing, (2) bandsawing, and (3) circular sawing. Multiple Choice Questions There are a total of 20 correct answers in the following multiple choice questions (some questions have multiple answers that are correct). To attain a perfect score on the quiz, all correct answers must be given, since each correct answer is worth 1 point. For each question, each omitted answer or wrong answer reduces the score by 1 point, and each additional answer beyond the number of answers required reduces the score by 1 point. Percentage score on the quiz is based on the total number of correct answers. 22.1 Which of the following are examples of generating the workpart geometry in machining, as compared to forming the geometry (more than one)? (a) broaching, (b) contour turning, (c) drilling, and (d) profile milling. Answer. (b) and (d). 22.2 In a turning operation, the change in diameter of the workpart is equal to which one of the following? (a) 1 x depth of cut, (b) 2 x depth of cut, (c) 1 x feed, or (d) 2 x feed. Answer. (b) 22.3 A lathe can be used to perform which of the following machining operations (more than one)? (a) boring, (b) broaching, (c) drilling, (d) milling, (e) planing, or (f) turning. Answer. (a), (c), and (f) 22.4 A facing operation is normally performed on which of the following machine tools (one best answer)? (a) drill press, (b) lathe, (c) milling machine, (d) planer, or (e) shaper. Answer. (b) 22.5 Knurling is performed on a lathe, but it is a metal forming operation rather than a metal removal operation: (a) true or (b) false? Answer. (a) 22.6 Which of the following cutting tools can be used on a turret lathe (more than one)? (a) broach, (b) cut-off tool, (c) drill bit, (d) single point turning tool, or (e) threading tool. Answer. (b), (c), (d), and (e). 144 22.7 Which of the following turning machines permits very long bar stock to be used (one best answer)? (a) chucking machine, (b) engine lathe, (c) screw machine, (d) speed lathe, or (e) turret lathe. Answer. (c) 22.8 Reaming is used for which of the following functions (more than one)? (a) accurately locate a hole position, (b) enlarge a drilled hole, (c) improve surface finish on a hole, (d) improve tolerance on hole diameter, and (e) provide an internal thread. Answer. (b), (c), and (d). 22.9 End milling is most similar to which one of the following? (a) face milling, (b) peripheral milling, (c) plain milling, or (d) slab milling. Answer. (a) 22.10 The basic milling machine is which one of the following: (a) bed type, (b) knee-and-column, (c) profiling mill, (d) ram mill, and (e) universal milling machine. Answer. (b) 22.11 A broaching operation is best described by which one of the following: (a) a rotating tool moves past a stationary workpart, (b) a tool with multiple teeth moves linearly past a stationary workpart, (c) a workpart is fed past a rotating cutting tool, or (d) a workpart moves linearly past a stationary single point tool. Answer. (b) 22.12 A planing operation is best described by which one of the following: (a) a single point tool moves linearly past a stationary workpart, (b) a tool with multiple teeth moves linearly past a stationary workpart, (c) a workpart is fed linearly past a rotating cutting tool, or (d) a workpart moves linearly past a single-point tool. Answer. (d) Problems Turning and Related Operations 22.1 A cylindrical workpart 125 mm in diameter and 900 mm long is to be turned in an engine lathe. Cutting conditions are: v = 2.5 m/s, f = 0.3 mm/rev, and d = 2.0 mm. Determine: (a) cutting time, and (b) metal removal rate. Solution: (a) N = (2.5 m/s)/.125= 6.366 rev/s. fr = 6.366(.3) = 1.91 mm/s Tm = 900/1.91 = 471.2 s = 7.85 min. (b) MRR = vfd = (2.5 m/s)(103)(.3 mm)(2.0 mm) = 1500 mm3/s 22.2 In a production turning operation, the foreman has decreed that the single pass must be completed on the cylindrical workpiece in 5.0 min. The piece is 400 mm long and 150 mm in diameter. Using a feed = 0.30 mm/rev and a depth of cut = 4.0 mm, what cutting speed must be used to meet this machining time requirement? Solution: Starting with Eq. (22.4): Tm = L/fr Substitute Eq. (22.3) (fr = Nf) into the denominator to obtain Tm = L/Nf 145 Then substituting for N from Eq. (22.1) we get Tm = DoL/vf (this equation is later used in Chapter 24). Rearranging to determine cutting speed: v = DoL/fTm Tm = (0.4)(0.15)/(0.30)(10-3)(5.0) = 0.1257(103) m/min = 125.7 m/min 22.3 A tapered surface is to be turned on an automatic lathe. The workpiece is 750 mm long with minimum and maximum diameters of 100 mm and 200 mm at opposite ends. The automatic controls on the lathe permit the surface speed to be maintained at a constant value of 200 m/min by adjusting the rotational speed as a function of workpiece diameter. Feed = 0.25 mm/rev and depth of cut = 3.0 mm. The rough geometry of the piece has already been formed, and this operation will be the final cut. Determine (a) the time required to turn the taper and (b) the rotational speeds at the beginning and end of the cut. Solution: (a) MRR = vfd = (200 m/min)(103 mm/m)(0.25 mm)(3.0 mm) = 150,000 mm3/min Area of frustrum of cone A = (R1 + R2){h2 + (R1 – R2)2}0.5 Given R1 = 100 mm, R2 = 50 mm, and h = 750 mm, A = (100 + 50){7502 + (100 – 50)2}0.5 = 150(565,000)0.5 = 354,214 mm2 Given depth of cut d = 3.0 mm, volume cut V = Ad = (354,214 mm2)(3.0 mm) = 1,062,641 mm3 Tm = V/MRR = (1,062,641 mm3)/(150,000 mm3/min) = 7.084 min (b) At beginning of cut (D1 = 100 mm), N = v/D = 200,000/100= 636.6 rev/min At end of cut (D2 = 200 mm), N = 200,000/200= 318.3 rev/min 22.4 In the taper turning job of previous Problem 22.3, suppose that the automatic lathe with surface speed control is not available and a conventional lathe must be used. Determine the rotational speed that would be required to complete the job in exactly the same time as your answer to part (a) of that problem. Solution: At a constant rotational speed and feed, feed rate fr is constant and Eqs. (22.3) and (22.4) can be used. Combining, Tm = L/Nf and then rearranging to obtain rotational speed N = L/fTm Given L = 750 mm, f = 0.25 mm/rev, and Tm = 7.084 min from Problem 22.3, N = 750/(0.25)(7.084) = 423.5 rev/min 22.5 A workbar with 5.0 in diameter and 48 in length is chucked in an engine lathe and supported at the opposite end using a live center. A 40.0 in portion of the length is to be turned to a diameter of 4.75 in one pass at a speed = 400 ft/min and a feed = 0.012 in/rev. Determine: (a) the required depth of cut, (b) cutting time, and (c) metal removal rate. Solution: (a) depth d = (5.00 - 4.75)/2 = 0.125 in. (b) N = 400 x 12/5= 305.5 rev/min. fr = 305.5(0.012) = 3.67 in/min Tm = 40/3.67 = 10.91 min. (c) MRR = (400 x 12)(0.125)(0.012) = 7.2 in3/min. 22.6 A 4.00 in diameter workbar that is 25 in long is to be turned down to 3.50 in diameter in two passes on an engine lathe using the following cutting conditions: v = 300 ft/min, f = 0.015 in/rev, and d = 0.125 in. The bar will be held in a chuck and supported on the opposite end in a live center. With this workholding setup, one end must be turned to diameter; then the bar must be reversed to turn the other end. Using an overhead crane available at the lathe, the time required to load and unload the bar is 5.0 minutes, and the time to reverse the bar is 3.0 minutes. For each turning cut an allowance must be added to the cut length for approach and overtravel. The total 146 allowance (approach plus overtravel) = 0.50 in. Determine the total cycle time to complete this turning operation. Solution: First end: cut 15 in. of 25 in. length. N = 300 x 12/4= 286.4 rev/min., fr = 286.4(0.015) = 4.297 in/min. Tm = (15 + 0.5)/4.297 = 3.61 min.; this reduces diameter to 3.75 in. N = 300 x 12/3.75= 305.5 rev/min., fr = 305.5(0.015) = 4.583 in/min. Tm = 15.5/4.583 = 3.38 min’ this reduces diameter to 3.50 in. Reverse bar which takes 3.0 min. cut remaining 10 in. of 25 in. length. N = 300 x 12/4= 286.4 rev/min., fr = 286.4(0.015) = 4.297 in/min. Tm = (10 + 0.5)/4.297 = 2.44 min.; this reduces diameter to 3.75 in. N = 300 x 12/3.75= 305.5 rev/min., fr = 305.5(0.015) = 4.583 in/min. Tm = 10.5/4.583 = 2.29 min’ this reduces diameter to 3.50 in. Loading and unloading bar takes 5.0 min. Total cycle time = 5.0 + 3.61 + 3.38 + 3.0 + 2.44 + 2.29 = 19.72 min. 22.7 The end of a large tubular workpart is to be faced on a NC vertical boring mill. The part has an outside diameter = 45.0 in and inside diameter = 25 in. If the facing operation is performed at a rotational speed = 30 rev/min, feed = 0.020 in/rev, and depth = 0.150 in, determine: (a) the cutting time to complete the facing operation, (b) the cutting speeds and metal removal rates at the beginning and end of the cut. Solution: (a) Distance traveled L = (Do - Di)/2 = (45 - 25)/2 = 10 in. fr = (30 rev/min)(0.020 in/rev) = 0.6 in/rev Tm = 10/0.6 = 16.67 min (b) at Do = 45 in., v = (30 rev/min)(45/12) = 353.5 ft/min. MRR = (353.5 x 12)(0.020)(0.15) = 12.73 in3/min at Di = 25 in., v = (30 rev/min)(25/12) = 196.4 ft/min. MRR = (196.4 x 12)(0.020)(0.15) = 7.07 in3/min 22.8 Solve previous Problem 22.7 except that the machine tool controls operate at a constant cutting speed by continuously adjusting rotational speed for the position of the tool relative to the axis of rotation. The rotational speed at the beginning of the cut = 30 rev/min, and is continuously increased thereafter to maintain a constant cutting speed. Solution: (a) Total metal removed = 0.25d(Do 2 - Di 2) = 0.25(0.15)(452 - 252) = 164.96 in3 MRR is constant throughout cutting if v is constant. v = (30 rev/min)(45/12) = 353.5 ft/min MRR = (353.5 x 12)(0.020)(0.15) = 12.73 in3/min Tm = (total metal removed)/MRR = 164.96/12.73 = 12.96 min. Drilling 22.9 A drilling operation is to be performed with a 25.4 mm diameter twist drill in a steel workpart. The hole is a blind-hole at a depth = 50 mm, and the point angle = 118. Cutting conditions are: speed = 25 m/min, feed = 0.25 mm/rev. Determine: (a) the cutting time to complete the drilling operation, and (b) metal removal rate during the operation, after the drill bit reaches full diameter. Solution: (a) A = 0.5(25.4) tan(90 – 118/2) = 12.7 tan 31 = 7.65 mm 147 N = 25(103)/25.4= 313.3 rev/min fr = 313.3(0.25) = 78.3 mm/min Tm = (50 + 7.63)/78.3 = 0.736 min (b) MRR = 0.25(25.4)2(78.3) = 39,675.2 mm3/min 22.10 A NC drill press is to perform a series of through-hole drilling operations on a 1.75 in thick aluminum plate that is a component in a heat exchanger. Each hole is 3/4 in diameter. There are 100 holes in all, arranged in a 10 by 10 matrix pattern, and the distance between adjacent hole centers (along the square) = 1.5 in. The cutting speed = 300 ft/min, the penetration feed (z-direction) = 0.015 in/rev, and the feed rate between holes (x-y plane) = 15.0 in/min. Assume that x-y moves are made at a distance of 0.5 in above the work surface, and that this distance must be included in the penetration feed rate for each hole. Also, the rate at which the drill is retracted from each hole is twice the penetration feed rate. The drill has a point angle = 100 degrees. Determine the time required from the beginning of the first hole to the completion of the last hole, assuming the most efficient drilling sequence will be used to accomplish the job. Solution: Time to drill each hole: N = 300 x 12/0.75= 1527.7 rev/min. fr = 1527.7(0.015) = 22.916 in/min. Distance per hole = 0.5 + A + 1.75 A = 0.5(0.75) tan(90 - .5 x 100) = 0.315 in. Tm = (0.5 + 0.315 + 1.75)/22.916 = 0.112 min. Time to retract drill from hole = 0.112/2 = 0.056 min. All moves between holes are at a distance = 1.5 in. using a back and forth path between holes. Time to move between holes = 1.5/15 = 0.1 min. With 100 holes, the number of moves between holes = 99. Total cycle time to drill 100 holes = 100(0.112 + 0.056) + 99(0.1) = 26.7 min. 22.11 A gundrilling operation is used to drill a 7/16-in diameter hole to a certain depth. It takes 4.5 minutes to perform the drilling operation using high pressure fluid delivery of coolant to the drill point. The cutting conditions are: N = 3000 rev/min at a feed = 0.002 in/rev. In order to improve the surface finish in the hole, it has been decided to increase the speed by 20% and decrease the feed by 25%. How long will it take to perform the operation at the new cutting conditions? Solution: fr = 3000 rev/min(0.001 in/rev) = 3.0 in/min. Hole depth d = 4.5 min(3.0 in/min.) = 13.5 in. New speed v = 3000(1 + 0.20) = 3600 rev/min. New feed f = 0.001(1- 0.25) = 0.00075 in/min. New feed rate fr = 3600(0.00075) = 2.7 in/min. New drilling time Tm = 13.5 in/2.7 in/min = 5.0 min. Milling 22.12 A peripheral milling operation is performed on the top surface of a rectangular workpart which is 300 mm long by 100 mm wide. The milling cutter, which is 75 mm in diameter and has four teeth, overhangs the width of the part on both sides. Cutting conditions are: v = 80 m/min, f = 0.2 mm/tooth, and d = 7.0 mm. Determine: (a) the time to make one pass across the surface, and (b) the material removal rate during the cut. Solution: N = 80,000 mm/75= 339.5 rev/min. 148 fr = 339.5(4)(0.2) = 271.6 mm/min. A = (d(D-d)).5 = (7(75-7)).5 = 21.82 mm Tm = (300 + 21.82)/271.6 = 1.185 min. MRR = 100(7)(271.6) = 190,120 mm3/min. 22.13 A face milling operation is used to machine 5 mm from the top surface of a rectangular piece of aluminum 400 mm long by 100 mm wide. The cutter has four teeth (cemented carbide inserts) and is 150 mm in diameter. Cutting conditions are: v = 3 m/s, f = 0.27 mm/tooth, and d = 5.0 mm. Determine: (a) time to make one pass across the surface, and (b) metal removal rate during cutting. Solution: (a) N = (3000 mm/s)/150= 6.366 rev/s fr = 6.366(4)(.27) = 6.875 mm/s A = D/2 = 150/2 = 75 mm Tm = (400 + 75)/6.875 = 80 s = 1.333 min. (b) MRR = 100(5)(6.875) = 3437.5 mm3/s 22.14 A slab milling operation is performed to finish the top surface of a steel rectangular workpiece 10.0 in long by 3.0 in wide. The helical milling cutter, which has a 2.5 in diameter and eight teeth, is set up to overhang the width of the part on both sides. Cutting conditions are: v = 100 ft/min, f = 0.009 in/tooth, and d = 0.250 in. Determine: (a) the time to make one pass across the surface, and (b) the metal removal rate during the cut. Solution: N = 100 x 12/2.5= 152.8 rev/min. fr = 152.8(8)(0.009) = 11.0 in/min. A = (d(D-d)).5 = (.25(2.5-.25)).5 = 0.75 in Tm = (10.0 + 0.75)/11 = 0.98 min. MRR = 3.0(.25)(11.0) = 8.25 in3/min. 22.15 A face milling operation is performed to finish the top surface of a steel rectangular workpiece 12.0 in long by 2.0 in wide. The milling cutter has four teeth (cemented carbide inserts) and a 3.0 in diameter. Cutting conditions are: v = 500 ft/min, f = 0.010 in/tooth, and d = 0.150 in. Determine: (a) the time to make one pass across the surface, and (b) the metal removal rate during the cut. Solution: N = 500 x 12/3= 636.5 rev/min. fr = 636.5(4)(0.010) = 25.46 in/min. A = O = 3/2 = 1.5 in. Tm = (12.0 + 2 x 1.5)/25.46 = 0.59 min. MRR = 2.0(.150(25.46) = 7.64 in3/min. 22.16 Solve previous Problem 22.15 except that the workpiece is 5.0 in wide and the cutter is offset to one side so that the swath cut by the cutter = 1.0 in wide. Solution: N = 500 x 12/3= 636.5 rev/min. fr = 636.5(4)(0.010) = 25.46 in/min. A = O = (1(3-1)).5 = 1.414 in Tm = (12.0 + 2 x 1.414)/25.46 = 0.58 min. MRR = 1.0(.150(25.46) = 3.82 in3/min. Other Operations 22.17 An open side planer is to be used to plane the top surface of a rectangular workpart, 25.0 in by 40.0 in. Cutting conditions are: v = 25 ft/min, f = 0.020 in/pass, and d = 0.200 in. The length of the 149 stroke across the work must be set up so that 10 in are allowed at both the beginning and end of the stroke for approach and overtravel. The return stroke, including an allowance for acceleration and deceleration, takes 75% of the time for the forward stroke. How long will it take to complete the job, assuming that the part is oriented in such a way as to minimize the time? Solution: Orient work so that its length (L = 40 in.) is in direction of stroke. This will minimize the number of passes required which will minimize time in this case. Time per forward stroke = (10 + 40 + 10)/(25 x 12) = 0.2 min. Time per reverse stroke = 0.75(.2) = 0.15 min. Total time per pass = 0.20 + 0.15 = 0.35 min. Number of passes = 25.0/0.020 = 1250 passes Total time = 1250(0.35) = 437.5 min. Check: orient work so that its width (w = 25 in.) is in direction of stroke. Time per forward stroke = (10 + 25 + 10)/(25 x 12) = 0.15 min. Time per reverse stroke = 0.75(.15) = 0.1125 min. Total time per pass = 0.15 + 0.1125 = 0.2625 min. Number of passes = 40.0/0.020 = 2000 passes Total time = 2000(0.2625) = 525.0 min. 150 23 CUTTING TOOL TECHNOLOGY Review Questions 23.1 What are the two principal aspects of cutting tool technology? Answer. The two main aspects of cutting tool technology are: (1) tool material and (2) tool geometry. 23.2 Name the three modes of tool failure in machining. Answer. The three tool failure modes are: (1) fracture failure, (2) temperature failure, and (3) gradual wear. 23.3 What are the two principal locations on a cutting tool where tool wear occurs? Answer. Wear occurs on the top face of the cutting tool as crater wear and on the side or flank of the tool, called flank wear. Portions of flank wear are often identified separately as notch wear, corresponding to the surface of the work; and nose radius wear, corresponding to the tool point. 23.4 Identify the mechanisms by which cutting tools wear during machining. Answer. The important tool wear mechanisms are: (1) abrasion, (2) adhesion, (3) diffusion, and (4) plastic deformation of the cutting edge. 23.5 What is meant by the parameter C in the Taylor tool life equation? Answer. The parameter C is the cutting speed corresponding to a one-minute tool life. C is the speed-axis intercept on the log-log plot of the tool life data. 23.6 In addition to cutting speed, what other cutting variables are included in the expanded version of the Taylor tool life equation? Answer. The expanded version of the Taylor equation can include any of the following: feed, depth of cut, and/or work material hardness. 23.7 What are some of the tool life criteria used in production machining operations? Answer. Production tool life criteria include: (1) complete failure of the tool, (2) visual observation of flank or crater wear, (3) fingernail test to feel flank wear, (4) sound of the tool, (5) chip disposal problems, (6) degradation of finish, (7) power increase, (8) workpiece count, and (9) length of cutting time for the tool. 23.8 Identify three desirable properties of a cutting tool material. Answer. Three desirable properties are: (1) toughness to resist fracture failure, (2) hot hardness to resist temperature failure, and (3) wear resistance to prolong the life of the tool during gradual wear. 23.9 What are the principal alloying ingredients in high speed steel? Answer. Principal alloying ingredients in HSS are: (1) either tungsten or a combination of tungsten and molybdenum, (2) chromium, (3) vanadium, and (4) carbon. Some grades of HSS also contain cobalt. 151 23.10 What is the difference in ingredients between steel cutting grades and non-steel cutting grades of cemented carbides? Answer. The non-steel cutting grades contain only WC and Co. The steel cutting grades contain TiC and/or TaC in addition to WC-Co. 23.11 Identify some of the common compounds that form the thin coatings on the surface of coated carbide inserts. Answer. The common coatings are: TiN, TiC, and Al2O3. 23.12 Name the seven elements of tool geometry for a single point cutting tool. Answer. The seven elements of single point tool geometry are: (1) back rake angle, (2) side rake angle, (3) end relief angle, (4) side relief angle, (5) end cutting edge angle, (6) side cutting edge angle, and (7) nose radius. 23.13 Why are ceramic cutting tools generally designed with negative rake angles? Answer. Ceramics possess low shear and tensile strength but good compressive strength. During cutting, this combination of properties is best exploited by giving the tool a negative rake angle to tend to load the tool in compression. 23.14 Identify the alternative ways by which a cutting tool is held in place during machining. Answer. There are three principal ways: (1) solid shank, in which the cutting edge is an integral part of the tool shank, an example being high speed; (2) brazed inserts, used for some cemented carbides; and (3) mechanically clamped inserts, used for most hard tool materials including cemented carbides and coated carbides, ceramics, cermets, SPD, and CBN. 23.15 Name the two main categories of cutting fluid according to function. Answer. The two functional categories of cutting fluids are: (1) coolants and (2) lubricants. 23.16 Name the three main types of cutting fluid according to chemistry. Answer. The three types of cutting fluids are: (1) cutting oils, (2) emulsified oils, and (3) chemical and semi- chemical fluids. 23.17 What is the principal lubricating mechanism by which cutting fluids work? Answer. There are two lubricating mechanisms that are believed to be effective in metal cutting: (1) boundary lubrication, which involves the formation of a thin fluid film to help separate and protect the contacting surfaces; and (2) extreme pressure lubrication, in which a thin solid layer of a salt such as iron sulfide is formed on the tool surface to provide lubrication. 23.18 What are the methods by which cutting fluids are applied in a machining operation? Answer. The most common method of application is flooding, in which a steady stream of fluid is direct at the operation. Other methods include: mist application, fluid- hole delivery, and manual application. 23.19 Why are cutting fluid filter systems becoming more common and what are their advantages? Answer. Cutting fluid filter systems are becoming more common due to the environmental protection laws and the need to prolong the life of the fluid before disposal. Advantages of filter systems include: longer fluid life, reduced disposal costs, better hygiene, lower machine tool maintenance, and longer cutting tool life. 152 23.20 Dry machining is being considered by machine shops because of certain problems inherent in the use of cutting fluids. (a) What are those problems associated with the use of cutting fluids? (b) What are some of the new problems introduced by machining dry? Answer. (a) Cutting fluids become contaminated over time with a variety of contaminants, including tramp oil, garbage, small chips, molds, fungi, and bacteria. In addition to causing odors and health hazards, contaminated cutting fluids do not perform their lubricating function as well as when they are fresh and clean. (b) Problems with dry machining include: (1) overheating the tool, (2) operating at lower cutting speeds and production rates to prolong tool life, and (3) absence of chip removal benefits in grinding and milling. Multiple Choice Quiz There are a total of 18 correct answers in the following multiple choice questions (some questions have multiple answers that are correct). To attain a perfect score on the quiz, all correct answers must be given, since each correct answer is worth 1 point. For each question, each omitted answer or wrong answer reduces the score by 1 point, and each additional answer beyond the number of answers required reduces the score by 1 point. Percentage score on the quiz is based on the total number of correct answers. 23.1 Of the following cutting conditions, which one has the greatest effect on tool wear? (a) cutting speed, (b) depth of cut, or (c) feed. Answer. (a) 23.2 As an alloying ingredient in high speed steel, tungsten serves which of the following functions (more than one)? (a) forms hard carbides to resist abrasion, (b) improves strength and hardness, (c) increases corrosion resistance, and (d) increases hot hardness. Answer. (a), (b), and (d). 23.3 Cast cobalt alloys typically contain which of the following main ingredients (more than one)? (a) aluminum, (b) cobalt, (c) chromium, (d) nickel, and (e) tungsten. Answer. (b), (c), and (e). 23.4 Which of the following is not a common ingredient in cemented carbide cutting tools (more than one)? (a) Al2O3, (b) Co, (c) CrC, (d) TiC, and (e) WC. Answer. (a) and (c). 23.5 An increase in cobalt content has which of the following effects on WC-Co cemented carbides (one best answer)? (a) decreases transverse rupture strength, (b) increases hardness, (c) increases toughness. Answer. (c) 23.6 Steel cutting grades of cemented carbide are typically characterized by which of the following ingredients (more than one)? (a) Co, (b) Ni, (c) TiC, (d) TaC, and (e) WC. Answer. (c) and (d). 23.7 If you had to select a cemented carbide for an application involving finish turning of steel, which C-grade would you select (one best answer)? (a) C1, (b) C3, (c) C5, or (d) C7. Answer. (d) 153 23.8 Which of the following processes are used to provide the thin coatings on the surface of coated carbide inserts (more than one)? (a) chemical vapor deposition, (b) electroplating, (c) physical vapor deposition, or (d) pressing and sintering. Answer. (a) and (c). 23.9 Which of the following materials has the highest hardness? (a) aluminum oxide, (b) cubic boron nitride, (c) high speed steel, (d) titanium carbide, or (e) tungsten carbide. Answer. (b) 23.10 Which of the following are the two main functions of a cutting fluid in machining (two answers only)? (a) improve surface finish on the workpiece, (b) reduce forces and power, (c) reduce friction at the tool-chip interface, (d) remove heat from the process, and (e) wash away chips. Answer. (c) and (d). Problems Tool Life and the Taylor Equation 23.1 The following flank wear data were collected in a series of turning tests using a coated carbide tool on hardened alloy steel. The feed rate was 0.30 mm/rev and the depth was 4.0 mm. The last wear data value in each column is when final tool failure occurred. (a) On a single piece of linear graph paper, plot flank wear as a function of time. Using 0.75 mm of flank wear as the criterion of tool failure, determine the tool lives for the two cutting speeds. (b) On a piece of natural log-log paper, plot your results determined in the previous part. From the plot, determine the values of n and C in the Taylor Tool Life Equation. (c) As a comparison, calculate the values of n and C in the Taylor equation solving simultaneous equations. Are the resulting n and C values the same? Cutting time, min. 1 3 5 7 9 11 13 15 20 25 Flank wear, mm. at v = 125 m/min 0.12 0.20 0.27 0.33 0.40 0.45 0.50 0.58 0.73 0.97 Flank wear, mm. at v = 165 m/min 0.22 0.35 0.47 0.57 0.70 0.80 0.99 Solution: At v = 125 m/min, T = 20.4 min using criterion FW = 0.75 mm. At v = 165 m/min, T = 10.0 min using criterion FW = 0.75 mm. (a) and (b) Student exercises. Values of C and n may vary in part (b) due to variations in the plots. The values should be approximately the same as those obtained in part (c) below. (c) Two equations: (1) 125(20.4)n = C, and (2) 165(10.0)n = C (1) and (2) 125(20.4)n = 165(10.0)n ln 125 + n ln 20.4 = ln 165 + n ln 10.0 4.8283 + 3.0155 n = 5.1059 + 2.3026 n 0.7129 n = 0.2776 n = 0.3894 (1) C = 125(20.4)0.3894 = 404.46 (2) C = 165(10.0)0.3894 = 404.46 C = 404.46 23.2 Solve Problem 23.1 except that the tool life criterion is 0.50 mm of flank land wear. Solution: At v = 125 m/min, T = 13.0 min using criterion FW = 0.50 mm. At v = 165 m/min, T = 5.6 min using criterion FW = 0.50 mm. (a) and (b) Student exercises. Values of C and n may vary in part (b) due to variations in the plots. The values should be approximately the same as those obtained in part (c) below. 154 (c) Two equations: (1) 125(13.0)n = C, and (2) 165(5.6)n = C (1) and (2) 125(13.0)n = 165(5.6)n ln 125 + n ln 13.0 = ln 165 + n ln 5.6 4.8283 + 2.5649 n = 5.1059 + 1.7228 n 0.8421 n = 0.2776 n = 0.3296 (1) C = 125(13.0)0.3894 = 291.14 (2) C = 165(5.6)0.3894 = 291.15 C = 291.15 23.3 Tool life tests on a lathe have resulted in the following data: (1) v = 350 ft/min, T = 7 min; (2) v = 250 ft/min, T = 50 min. (a) Determine the parameters n and C in the Taylor tool life equation. (b) Based on the n and C values, what is the likely tool material used in this operation? (c) Using your equation, compute the tool life that corresponds to a cutting speed v = 300 ft/min. (d) Compute the cutting speed that corresponds to a tool life T = 10 min. Solution: (a) Two equations: (1) 350(7)n = C and (2) 250(50)n = C 350(7)n = 250(50)n 350/250 = (50/7)n 1.4 = (7.14286)n ln 1.4 = n ln 7.14286 0.3365 = 1.9661 n n = 0.171 C = 350(7).171 = 350(1.3948) C = 488 Check: C = 250(50).171 = 250(1.9522) = 488 (b) Comparing these values of n and C with those in Table 24.2, the likely tool material is high speed steel. (c) At v = 300 ft/min., T = (C/v)1/n = (488/300)1/.171 = (1.6267)5.848 = 17.2 min. (d) For T = 10 min., v = C/Tn = 488/10.171 = 488/1.4825 = 329 ft/min. 23.4 Tool life tests in turning yield the following data: (1) v = 100 m/min, T = 10 min; (2) v = 75 m/min, T = 30 min. (a) Determine the n and C values in the Taylor tool life equation. Based on your equation, compute (b) the tool life for a speed of 90 m/min, and (c) the speed corresponding to a tool life of 20 min. Solution: (a) Two equations: (1) 100(10)n = C and (2) 75(30)n = C. 100(10)n =75(30)n ln 100 + n ln 10 = ln 75 + n ln 30 4.6052 + 2.3026 n = 4.3175 + 3.4012 n 4.6052 - 4.3175 = (3.4012 - 2.3026) n 0.2877 = 1.0986 n n = 0.2619 C = 100(10).2619 = 100(1.8277) C = 182.77 Check: C = 75(30).2619 = 75(2.437) = 182.77 (b) 90 T.2619 = 182.77 T.2619 = 182.77/90 = 2.037 T = 2.0371/.2619 = 2.0373.818 = 15.13 min. (c) v (20).2619 = 182.77 v = 182.77/(20).2619 = 182.77/2.1914 = 83.4 m/min. 23.5 Turning tests have resulted in 1-min tool life for a cutting speed v = 4.0 m/s and a 20-min tool life at a speed v = 2.0 m/s. (a) Find the n and C values in the Taylor tool life equation. (b) Project how long the tool would last at a speed v = 1.0 m/s. 155 Solution: (a) For data (1) T = 1.0 min, then C = 4.0 m/s = 240 m/min. For data (2) v = 2 m/s = 120 m/min. 120(20)n = 240 20n = 240/120 = 2.0 n ln 20 = ln 2.0 2.9957 n = 0.6931 n = 0.2314 (b) At v = 1.0 m/s = 60 m/min. 60(T)0.2314 = 240 (T)0.2314 = 240/60 = 4.0 T = (4.0)1/0.2314 = (4)4.3215 = 400 min. 23.6 In a production turning operation, the workpart is 125 mm in diameter and 300 mm long. A feed rate of 0.225 mm/rev is used in the operation. If cutting speed = 3.0 m/s, the tool must be changed every 5 workparts; but if cutting speed = 2.0 m/s, the tool can be used to produce 25 pieces between tool changes. Determine the Taylor tool life equation for this job. Solution: (1) Tm = (125 mm)(0.3 m)/(3.0 m/s)(0.225 mm) = 174.53 s = 2.909 min. T = 5(2.909) = 14.54 min. (2) Tm = (125 mm)(0.3 m)/(20 m/s)(0.225 mm) = 261.80s = 4.363 min. T = 25(4.363) = 109.08 min. (1) v = 3 m/s = 180 m/min. (2) v = 2 m/s = 120 m/min. (1) 180(14.54)n = C (2) 120(109.08)n = C 180(14.54)n = 120(109.08)n ln 180 + n ln(14.54) = ln 120 + n ln(109.08) 5.1929 + 2.677 n = 4.7875 + 4.692 n 5.1929 - 4.7875 = (4.692 - 2.677) n 0.4054 = 2.0151 n n = 0.2012 C = 180 (14.54).2012 C = 308.43 23.7 For the tool life plot of Figure 23.5, show that the middle data point (v = 130 m/min, T = 12 min) is consistent with the Taylor equation determined in Example Problem 23.1. Solution: Taylor equation calculated in Example 23.1 is: vT0.223 = 229. Consistency would be demonstrated by using the values from the middle data point (T = 12 min at v = 130 ft/min) in the equation and obtaining the same value of C as above (C = 229). 130(12).223 = 130(1.7404) = 226.3 This represents a difference of less than 1.2%, which is close enough and well within expected random variation in typical tool life data. 23.8 In the tool wear plots of Figure 23.4, complete failure of the cutting tool is indicated by the end of each wear curve. Using complete failure as the criterion of tool life instead of 0.50 mm flank wear, the resulting data would be: (1) v = 160 m/min, T = 5.75 min; (2) v = 130 m/min, T = 14.25 min; (3) v = 100 m/min, T = 47 min. Determine the parameters n and C in the Taylor tool life equation for this data. Solution: Let us use the two extreme data points to calculate the values of n and C, then check the resulting equation against the middle data point. (1) 160(5.75)n = C and (3) 100(47)n = C 160(5.75)n = 100(47)n 156 ln 160 + n ln 5.75 = ln 100 + n ln 47 5.0752 + 1.7492 n = 4.6052 + 3.8501 n 0.4700 = 2.1009 n n = 0.224 (1) C = 160(5.75)0.224 = 236.7 (3) C = 100(47)0.224 = 236.9 use average: C = 236.8 Check against data set (2): 130(14.25)0.224 = 235.7. This represents a difference of less than 0.5%, which would be considered good agreement for experimental data. Better results on determining the Taylor equation would be obtained by using regression analysis on all three data sets to smooth the variations in the tool life data. Note that the n value is very close to the value obtained in Example 23.1 (n = 0.224 here vs. n = 0.223 in Example 23.1), and that the C value is higher here (C = 236.8 here vs. C = 229 in Example 23.1). The higher C value here reflects the higher wear level used to define tool life (complete failure of cutting edge here vs. a flank wear level of 0.50 mm in Example 23.1). 23.9 The Taylor equation for a certain set of test conditions is vT.25 = 1000, where the U.S. customary units are used: ft/min for v and min for T. Convert this equation to the equivalent Taylor equation in the International System of units (metric), where v is in m/sec and T is in seconds. Validate the metric equation using a tool life = 16 min. That is, compute the corresponding cutting speeds in ft/min and m/sec using the two equations. Solution: vT0.25 = 1000(Tref)0.25 C = 1000 ft/min for a 1.0 min tool life; ft/min converts to m/s as (1000 ft/min)(0.3048m/ft)(1 min/60 s) = 5.08 m/s Tref = 1 min = 60 s. (Tref)0.25 = (60)0.25 = 2.78316 The converted value of C = 5.08(2.78316) = 14.14 The converted equation is: vT0.25 = 14.14, where v = m/s and T = s. Check both equations at T = 16 min = 960 s. USCU: v = 1000/160.25 = 1000/2 = 500 ft/min. SI: v = 14.14/9600.25 = 14.14/5.566 = 2.54 m/s Check: (500 ft/min)( 0.3048m/ft)(1 min/60 s) = 2.54 m/s Q.E.D. 23.10 A series of turning tests are performed to determine the parameters n, m, and K in the expanded version of the Taylor equation, Eq. (23.4). The following data were obtained during the tests: (1) v = 2.0 m/s, f = 0.20 mm/rev, T = 12 min; (2) v = 1.5 m/s, f = 0.20 mm/rev, T = 40 min; and (3) v = 2.0 m/s, f = 0.3 mm/rev, T = 10 min. (a) Determine n, m, and K. (b) Using your equation, compute the tool life when v = 1.5 m/s and f = 0.3 mm/rev. Solution: Three equations to be solved simultaneously: (1) (2 x 60)(12)n(0.2)m = K (2) (1.5 x 60)(40)n(0.2)m = K (3) (2 x 60)(10)n(0.3)m = K (1) and (2): ln 120 + n ln 12 + m ln 0.2 = ln 90 + n ln 40 + m ln 0.2 ln 120 + n ln 12 = ln 90 + n ln 40 4.7875 + 2.4849 n = 4.4998 + 3.6889 n 0.2877 = 1.204 n n = 0.239 (1) and (3): ln 120 + 0.239 ln 12 + m ln 0.2 = ln 120 + 0.239 ln 10 + m ln 0.3 0.5939 + m (-1.6094) = 0.5503 + m (-1.2040) -0.4054 m = -0.0436 m = 0.108 157 (1) K = 120(12)0.239(0.2)0.108 =120(1.811)(0.8404) = K = 182.65 (b) v = 1.5 m/s, f = 0.3 mm/rev (1.5 x 60)(T)0.239(0.3)0.108 = 182.65 90(T)0.239(0.8781) = 182.65 (T)0.239 = 2.311 T = 2.3111/.239 = 33.3 min. 23.11 Eq. (23.4) in the text relates tool life to speed and feed. In a series of turning tests conducted to determine the parameters n, m, and K, the following data were collected: (1) v = 400 ft/min, f = 0.010 in/rev, T = 10 min; (2) v = 300 ft/min, f = 0.010 in/rev, T = 35 min; and (3) v = 400 ft/min, f = 0.015 in/rev, T = 8 min. Determine n, m, and K. What is the physical interpretation of the constant K? Solution: Three equations to be solved simultaneously: (1) 400(10)n(0.010)m = K (2) 300(35)n(0.010)m = K (3) 400(8)n(0.015)m = K (1) and (2): ln 400 + n ln 10 + m ln 0.010 = ln 300 + n ln 35 + m ln 0.010 ln 400 + n ln 10 = ln 300 + n ln 35 5.9915 + 2.3026 n = 5.7038 + 3.5553 n 0.2877 = 1.2527 n n = 0.2297 (1) and (3): ln 400 + n ln 10 + m ln 0.010 = ln 400 + n ln 8 + m ln 0.015 n ln 10 + m ln 0.010 = n ln 8 + m ln 0.015 0.2297(2.3026) + m (-4.6052) = 0.2297(2.0794) + m (-4.1997) 0.2297(2.3026 - 2.0794) = m(-4.1997 + 4.6052) 0.05127 = 0.4055 m m = 0.1264 (1) 400(10)0.2297(0.010)0.1264 = 400(1.6971)(0.5587) = 379.3 (2) 300(35)0.2297(0.010)0.1264 = 300(2.2629)(0.5587) = 379.3 (3) 400(8)0.2297(0.015)0.1264 = 400(1.6123)(0.5881) = 379.3 K = 379.3 The constant K represents the cutting speed (ft/min) for a 1.0 minute tool life at a feed rate of 1.0 in/rev. This feed is of course an extrapolation and not a real possible feed value. 23.12 The n and C values in Table 23.2 are based on a feed rate of 0.25 mm/rev and a depth of cut = 4.0 mm. Determine how many cubic mm of steel would be removed for each of the following tool materials, if a 10-min tool life were required in each case: (a) plain carbon steel, (b) high speed steel, (c) cemented carbide, (d) ceramic, and (e) coated carbide. Solution: (a) Plain carbon steel: n = 0.10, C = 60 ft/min. v = 60/100.1 = 60/1.259 = 47.6 ft/min. MRR = (47.6 x 12)(0.01)(0.10) = 0.572 in3/min. For 10 min, metal removed = 10(0.572) = 5.72 in3 (b) HSS: n = 0.125, C = 200 ft/min. v = 200/100.125 = 200/1.333 = 150 ft/min. MRR = (150 x 12)(0.01)(0.10) = 1.80 in3/min. For 10 min, metal removed = 10(1.8) = 18.0 in3 (c) Cemented carbide: n = 0.25, C = 1500 ft/min. v = 1500/100.25 = 1500/1.778 = 843.5 ft/min. MRR = (843.5 x 12)(0.01)(0.10) = 10.12 in3/min. For 10 min, metal removed = 10(10.12) = 101.2 in3 158 (d) Ceramic: n = 0.60, C = 10,000 ft/min. v = 12000/100.6 = 12000/3.981 = 2512 ft/min. MRR = (2512 x 12)(0.01)(0.10) = 30.14 in3/min. For 10 min, metal removed = 10(30.14) = 301.4 in3 (e) Coated carbide: n = 0.25, C = 2200 ft/min. v = 2200/100.25 = 2200/1.778 = 1237 ft/min. MRR = (1237 x 12)(0.01)(0.10) = 14.85 in3/min. For 10 min, metal removed = 10(14.85) = 148.5 in3 23.13 A drilling operation is performed in which 0.5 in diameter holes are drilled through cast iron plates that are 1.0 in thick. Sample holes have been drilled to determine the tool life at two cutting speeds. At 80 surface ft/min, the tool lasted for exactly 50 holes. At 120 surface ft/min, the tool lasted for exactly 5 holes. The feed rate of the drill was 0.003 in/rev. (Ignore effects of drill entrance and exit from the hole. Consider the depth of cut to be exactly 1.00 in, corresponding to the plate thickness.) Determine the values of n and C in the Taylor tool life equation for the above sample data, where cutting speed v is expressed in ft/min, and tool life T is expressed in min. Solution: (1) v = 80 ft/min, N = (80)/(.5/12) = 611 rev/min. feed rate fr = (0.003)(611) = 1.833 in/min. time per hole = 1.0 in/(1.833 in/min) = 0.545 min. for 50 holes, T = 50(0.545 min) = 27.25 min. Formulating the data as vTn = C, we have: 80(27.25)n = C (2) v = 120 ft/min, N = (120)/(.5/12) = 917 rev/min. feed rate fr = (0.003)(917) = 2.75 in/min. time per hole = 1.0 in/(2.75 in/min) = 0.364 min. for 5 holes, T = 5(0.364 min) = 1.82 min. Formulating the data as vTn = C, we have: 120(1.82)n = C Setting (1) = (2): 80(27.25)n = 120(1.82)n ln 80 + n ln 27.25 = ln 120 + n ln 1.82 4.382 + 3.3051 n = 4.7875 + 0.5978 n 2.7073 n = 0.4055 n = 0.15 C = 80(27.25)0.15 = 80(1.6417) = 131.34 C = 120(1.82)0.15 = 120(1.094) = 131.29C = 131.32 23.14 The outside diameter of a cylinder made of titanium alloy is to be turned. The starting diameter = 500 mm and the length = 1000 mm. Cutting conditions are: f = 0.4 mm/rev, and d = 3.0 mm. The cut will be made with a cemented carbide cutting tool whose Taylor tool life parameters are: n = 0.23 and C = 400. Units for the Taylor equation are min. for tool life and m/min for cutting speed. Compute the cutting speed that will allow the tool life to be just equal to the cutting time for this part. Solution: In this problem we want Tm = T, where Tm = machining time per piece and T = tool life. Both of these times must be expressed in terms of cutting speed. Tm = DL/fv and T = (C/v)1/n Tm = (500)(1000)(10-6)/0.4(10-3)v = 3926.99/v = 3926.99 (v)-1 T = (400/v)1/.23 = (400/v)4.3478 = 4004.3478(v)-4.3478 = 2057.33(108) (v)-4.3478 Setting Tm = T: 3926.99 v-1 = 2057.33(108) (v)-4.3478 v3.3478 = 0.52389(108) v = {0.52389(108)}1/3.3478 = {0.52389(108)}0.2987 = 202.18 m/min Check: Tm = 3926.99 (202.18)-1 = 19.423 min 159 T = (400/202.18)1/.23 = (400/202.18)4.3478 = 19.423 min 23.15 The outside diameter of a roll for a steel rolling mill is to be turned. In the final pass, the starting diameter = 26.25 in and the length = 48.0 in. The cutting conditions will be: feed = 0.0125 in/rev, and depth of cut = 0.125 in. A cemented carbide cutting tool is to be used and the parameters of the Taylor tool life equation for this setup are: n = 0.25 and C = 1300. Units for the Taylor equation are min. for tool life and ft/min for cutting speed. It is desirable to operate at a cutting speed so that the tool will not need to be changed during the cut. Determine the cutting speed which will make the tool life equal to the time required to complete the turning operation. Solution: In this problem we want Tm = T, where Tm = machining time per piece and T = tool life. Both of these times must be expressed in terms of cutting speed. Tm = DL/12fv and T = (C/v)1/n Tm = (26.25)(48.0)/12(0.0125)v = 26,389.38/v = 26,389.38 (v)-1 T = (1300/v)1/.25 = (1300/v)4.0 = 13004.0(v)-4.0 = 2.8561(1012) (v)-4.0 Setting Tm = T: 26,389.38 (v)-1= 2.8561(1012) (v)-4.0 v3.0 = 1.08229(108) v = {1.08229(108)}1/3 = {1.08229(108)}0.3333 = 476.56 ft/min Check: Tm = 26,389.38 (476.56)-1= 55.375 min T = (1300/476.56)1/.25 = (1300/476.56)4.0 = 55.375 min Tooling Applications 23.16 A certain machine shop uses a limited number of cemented carbide grades in its operations. These grades are listed below by chemical composition. (a) Which grade should be used for finish turning of unhardened steel? (b) Which grade should be used for rough milling of aluminum? (c) Which grade should be used for finish turning of brass? (d) Which of the grades listed would be suitable for machining cast iron? For each case, explain your recommendation. Grade %WC %Co %TiC 1 95 5 0 2 82 4 14 3 80 10 10 4 89 11 0 Solution: (a) Finish turning of unhardened steel. Specify a steel-cutting grade suitable for finishing. This is a grade with TiC and low cobalt. Choose grade 2. (b) Rough milling of aluminum. Specify a non-steel roughing grade. This is a grade with no TiC and high cobalt. Choose grade 4. (c) Finish turning of brass. Specify a non-steel finishing grade. This is a grade with no TiC and low cobalt. Choose grade 1. (d) Machining cast iron. Cast iron is included with the non-steel grades. Specify grade 1 for finishing and grade 4 for roughing. 23.17 A turning operation is performed on a steel shaft with diameter = 5.0 in and length = 32 in. A slot or keyway has been milled along its entire length. The turning operation reduces the shaft diameter. For each of the following tool materials, indicate whether or not it is a reasonable candidate to use in the operation: (a) plain carbon steel, (b) high speed steel, (c) cemented carbide, (d) ceramic, and (e) sintered polycrystalline diamond. For each material that is not a good candidate, give the reason why it is not. 160 Solution: The slot will result in an interrupted cut, so toughness is important in the tool material. (a) Plain carbon steel: not economical because of low cutting speeds. (b) HSS: this is a reasonable candidate; it has good toughness for the interrupted cut. (c) Cemented carbide: this is a reasonable candidate; it must be a steel cutting grade with high toughness (high cobalt content). (d) Ceramic: this is not a good candidate because of its low toughness; it is likely to fracture during interrupted cutting. (e) Sintered polycrystalline diamond: this is not suitable for cutting steel. Cutting Fluids 23.18 In a turning operation using high speed steel tooling, a cutting speed v = 90 m/min is used. The Taylor tool life equation has parameters n = 0.120 and C = 130 (m/min) when the operation is conducted dry. When a coolant is used in the operation, the value of C is increased by 10%. Determine the percent increase in tool life that results if the cutting speed is maintained at v = 90 m/min. Solution: Dry: 90(T)0.12 = 130 T = (130/90)1/.12 = (1.444)8.3333 = 21.42 min. With coolant: 90(T)0.12 = 130(1 + 10%) = 130(1.1) = 143 T = (143/90)1/.12 = (1.5889)8.3333 = 47.40 min. Increase = (47.40 - 21.42)/21.42 = 1.21 = 121% 23.19 A production turning operation on a steel workbar normally operates at a cutting speed of 125 ft/min using high speed steel tooling with no cutting fluid. The appropriate n and C values in the Taylor equation are given in Table 23.2 in the text. It has been found that the use of a coolant type cutting fluid will allow an increase of 25 ft/min in the speed without any effect on tool life. If it can be assumed that the effect of the cutting fluid is simply to increase the constant C by 25, what would be the increase in tool life if the original cutting speed of 125 ft/min were used in the operation? Solution: From Table 23.2, n = 0.125 and C = 200 for dry cutting. With cutting fluid, C = 200 + 25 = 225. Dry: at v = 125 ft/min, T = (200/125)1/.125 = (1.6)8 = 42.95 min. With cutting fluid: at v = 125 ft/min, T = (225/125)1/.125 = (1.8)8 = 110.2 min. Increase = (110.2 - 42.95) = 67.25 min = 156.6% 23.20 A high speed steel 6.0 mm twist drill is being used in a production drilling operation on mild steel. A cutting oil is applied by the operator by brushing the lubricant onto the drill point and flutes prior to each hole. The cutting conditions are: speed = 25 m/min, and feed = 0.10 mm/rev, and hole depth = 40 mm. The foreman says that the "speed and feed are right out of the handbook" for this work material. Nevertheless, he says, "the chips are clogging in the flutes, resulting in friction heat, and the drill bit is failing prematurely due to overheating." What's the problem? What do you recommend to solve it? Solution: There are several problems here. First, the depth-to-diameter ratio is 1.75:0.25 = 7:1, which is greater than the 4:1 which is usually recommended. As a consequence the chips produced in the hole are having difficulty exiting, thus causing overheating of the drill. Second, the 161 manual method of applying the cutting oil may not be particularly effective. Third, with overheating as a problem, the cutting oil may not be removing heat from the operation effectively. The 7:1 depth-to-diameter ratio is a given, a requirement of the drilling operation, and we assume it cannot be changed. The twist drill might be operated in a peck-drilling mode to solve the chip clogging problem. Peck-drilling means drilling for a distance approximately equal to one drill diameter, then retract the drill, then drill some more, etc. A twist drill with a fluid hole could be used to more effectively deliver the cutting fluid to the drill point to help extract the chips. Finally, an emulsified oil might be tried in the operation, one with good lubricating qualities, as a substitute for the cutting oil. Since overheating is a problem, it makes sense to try a coolant. 162 24 ECONOMIC AND PRODUCT DESIGN CONSIDERATIONS IN MACHINING Review Questions 24.1 Define machinability. Answer. Machinability can be defined as the relative ease with which a material can be machined using an appropriate cutting tool under appropriate cutting conditions. 24.2 What are the criteria by which machinability is commonly assessed in a production machining operation? Answer. The machinability criteria include: (1) tool wear and tool life, (2) forces and power, (3) surface finish, and (4) ease of chip disposal. 24.3 Name some of the important mechanical and physical properties that affect the machinability of a work material. Answer. The properties mentioned in the text include: hardness, strength, and thermal diffusivity. 24.4 Why do costs tend to increase when better surface finish is required on a machined part? Answer. Because additional operations such as grinding, lapping, or similar finishing processes must be included in the manufacturing sequence at higher cost. 24.5 What are the basic factors that affect surface finish in machining? Answer. The factors that affect surface finish are: (1) geometric factors such as type of operation, feed, and tool shape (nose radius in particular); (2) work material factors such as built-up edge effects, and tearing of the work surface when machining ductile materials, which factors are affected by cutting speed; and (3) vibration and machine tool factors such as setup and workpart rigidity, and backlash in the feed mechanism. 24.6 What are the parameters that have the greatest influence in determining the ideal surface roughness Ri ? Answer. The ideal surface roughness is determined by geometric parameters of the machining operation. These parameters are: (1) the type of machining operation, e.g., whether turning or milling, (2) tool nose radius, and (3) feed or chip load, and (4) in some cases the end cutting edge of the tool. 24.7 Name some of the steps that can be taken to reduce or eliminate vibrations in machining. Answer. Steps to reduce vibration in machining include: (1) increase stiffness or damping in the setup; (2) operating at speeds away from the natural frequency of the machine tool system; and (3) reducing forces in machining through changes in feed or depth and changes in cutter design. 24.8 What are the factors on which the selection of feed in a machining operation should be based? Answer. The factors are: (1) type of tooling, (2) roughing or finishing operation, (3) limiting cutting forces, and (4) surface roughness requirements. 24.9 The unit cost in a machining operation is the sum of four cost terms. The first three terms are: (1) part load/unload cost, (2) cost of time the tool is actually cutting the work, and (3) the cost of the time to change the tool. What is the fourth term? 163 Answer. The fourth term is the cost of purchasing (and grinding, if applicable) the tool. 24.10 Which cutting speed is always lower for a given machining operation, cutting speed for minimum cost or cutting speed for maximum production rate? Why? Answer. Cutting speed for minimum cost. The fourth term in the unit cost equation, dealing with the actual cost of the cutting edge, tends to push the U-shaped function toward a lower value in the case of cutting speed for minimum cost. Multiple Choice Quiz There are a total of 14 correct answers in the following multiple choice questions (some questions have multiple answers that are correct). To attain a perfect score on the quiz, all correct answers must be given, since each correct answer is worth 1 point. For each question, each omitted answer or wrong answer reduces the score by 1 point, and each additional answer beyond the number of answers required reduces the score by 1 point. Percentage score on the quiz is based on the total number of correct answers. 24.1 Which of the following criteria are generally recognized to indicate good machinability (more than one)? (a) all of the following, (b) ease of chip disposal, (c) high value of Ra, (d) long tool life, (e) low cutting forces, (f) low value of Ra, (g) zero shear plane angle. Answer. (b), (d), (e), and (f) 24.2 Of the various methods for testing machinability, which of the following seems to be the most important (one answer)? (a) cutting forces, (b) cutting temperature, (c) horsepower consumed in the operation, (d) surface roughness, (e) tool life, or (f) tool wear. Answer. (e) 24.3 A machinability rating of greater than 1.0 indicates that the work material is which of the following relative to the defined base material, whose rating = 1.0? (a) easier to machine than the base or (b) more difficult to machine than the base. 24.4 In general, which of the following materials has the highest machinability (one best answer)? (a) aluminum, (b) cast iron, (c) copper, (d) low carbon steel, (e) stainless steel, (f) titanium alloys, or (g) unhardened tool steel. Answer. (a) 24.5 Which one of the following operations is generally capable of the closest tolerances (one best answer)? (a) broaching, (b) drilling, (c) end milling, (d) planing, or (e) sawing. Answer. (a) 24.6 When cutting a ductile work material, an increase in cutting speed will generally have which effect on surface finish? (a) degrade surface finish, which means high value of Ra or (b) improve surface finish, which means lower value of Ra. Answer. (b) 24.7 Which one of the following operations is generally capable of the best surface finishes (lowest value of Ra) (one best answer)? (a) broaching, (b) drilling, (c) end milling, (d) planing, or (e) turning. Answer. (a) 164 24.8 Which of the following time components in the average production machining cycle is affected by cutting speed (more than one)? (a) part loading and unloading time, and (b) setup time for the machine tool, (c) time the tool is engaged in cutting, and (d) tool change time. Answer. (c) and (d) 24.9 Which cutting speed is always lower for a given machining operation? (a) cutting speed for maximum production rate, or (b) cutting speed for minimum cost. Answer. (b) 24.10 A high tooling cost and/or tool change time will tend to have which of the following effects on vmax or vmin? (a) decrease or (b) increase. Answer. (a) Problems Machinability 24.1 A machinability rating is to be determined for a new work material using the cutting speed for a 60 min tool life as the basis of comparison. For the base material (B1112 steel), test data resulted in Taylor equation parameter values of n = 0.29 and C = 500, where speed is in m/min and tool life is min. For the new material, the parameter values were n = 0.21 and C = 400. These results were obtained using cemented carbide tooling. (a) Compute a machinability rating for the new material. (b) Suppose the machinability criterion were the cutting speed for a 10 min tool life rather than the present criterion. Compute the machinability rating for this case. (c) What do the results of the two calculations show about the difficulties in machinability measurement? Solution: (a) Base material: v60 = 500/60.29 = 152.5 m/min New material: v60 = 400/60.21 = 169.3 m/min MR = 169.3/152.5 = 1.11 = 111% (b) (a) Base material: v10 = 500/10.29 = 256.4 m/min New material: v10 = 400/10.21 = 246.6 m/min MR = 246.6/256.4 = 0.96 = 96% (c) Different test conditions often result in different machinability results. 24.2 A machinability rating is to be determined for a new work material. For the base material (B1112), test data resulted in a Taylor equation with parameters n = 0.27 and C = 450. For the new material, the Taylor parameters were n = 0.22 and C = 420. Units in both cases are: speed in m/min and tool life in min. These results were obtained using cemented carbide tooling. (a) Compute a machinability rating for the new material using cutting speed for a 30 min tool life as the basis of comparison. (b) If the machinability criterion were tool life for a cutting speed of 150 m/min, what is the machinability rating for the new material? Solution: (a) Base material: v30 = 450/30.27 = 179.6 m/min New material: v30 = 420/30.22 = 198.7 m/min MR = 198.7/179.6 = 1.107 = 110.7% (b) (a) Base material: T150 = (450/150)1/.27 = (3.0)3.704= 58.5 min New material: v10 = (420/150)1/.22 = (2.8)4.545 = 107.8 min MR = 107.8/58.5 = 1.84 = 184% 165 24.3 Tool life turning tests have been conducted on B1112 steel with high speed steel tooling, and the resulting parameters of the Taylor equation are: n = 0.13 and C = 225. The feed and depth during these tests were: f = 0.010 in/rev and d = 0.100 in. Based on this information, and machinability data given in Table 24.1, determine the cutting speed you would recommend for the following work materials, if the tool life desired in operation is 30 min: (a) C1008 low carbon steel with 150 Brinell hardness, (b) 4130 alloy steel with 190 Brinell hardness, and (c) B1113 steel with 170 Brinell hardness. Assume that the same feed and depth of cut are to be used. Solution: First determine v30 for the base material: v30 = 225/30.13 = 225/1.556 = 144.6 ft/min. (a) From Table 24.1, MR for C1008 = 0.50. Recommended v30 = 0.50(144.6) = 72 ft/min. (b) From Table 24.1, MR for 4130 = 0.65. Recommended v30 = 0.65(144.6) = 94 ft/min. (c) From Table 24.1, MR for B1113 = 1.35. Recommended v30 = 1.35(144.6) = 195 ft/min. Surface Roughness 24.4 In a turning operation on cast iron, the nose radius on the tool = 1.0 mm, feed rate = 0.2 mm/rev, and speed = 2 m/s. Compute an estimate of the surface roughness for this cut. Solution: Ri = f2/32NR = (0.20)2/(32 x 1) = 0.00125 mm. = 1.25 m. From Fig. 24.2, rai = 1.3 Ra = 1.3 x 1.25 = 1.63 mm. 24.5 A turning operation uses a 2/64 in nose radius cutting tool on a free machining steel with a feed rate = 0.010 in/rev and a cutting speed = 300 ft/min. Determine the surface roughness for this cut. Solution: Ri = f2/32NR = (0.010)2/(32 x 2/64) = 0.0001 in. = 100 in. From Fig. 24.2, rai = 1.02 Ra = 1.02 x 100 = 102 min. 24.6 A single-point HSS tool with a 3/64 in nose radius is used in a shaping operation on a ductile steel workpart. Cutting speed = 100 ft/min, feed = 0.015 in/pass, and depth of cut = 0.125 in. Determine the surface roughness for this operation. Solution: Ri = f2/32NR = (0.015)2/(32 x 3/64) = 0.00015 in. = 150 in. From Fig. 24.2, rai = 1.9 Ra = 1.9 x 150 = 285 min. 24.7 A part to be turned in an engine lathe must have a surface finish of 1.6 m. The part is made of a free-machining aluminum alloy. Cutting speed = 150 m/min, and depth of cut = 4.0 mm. The nose radius on the tool = 0.75 mm. Determine the feed that will achieve the specified surface finish. Solution: For free-machining aluminum at 150 m/min, from Figure 24.2 ratio rai = 1.0 in Eq. (24.3), so Ra = Ri Ra = Ri = f2/32NR Rearranging, f2 = Ri(32NR) = 1.6(10-6)(32)(0.75)(10-3) = 38.4(10-9) = 3.84(10-8) m2 f = (3.84(10-8) m2)0.5 = 1.96(10-4) m = 0.196 mm (here, mm is interpreted mm/rev) 24.8 Solve previous Problem 24.4 except that the part is made of cast iron instead of aluminum and the cutting speed is reduced to 100 m/min. Solution: For cast iron at 150 m/min, extrapolating Figure 24.2 ratio rai = 1.2 in Eq. (24.3), so Ra = 1.2 Ri = 1.2f2/32NR Rearranging, f2 = Ri(32NR)/1.2 = 1.6(10-6)(32)(0.75)(10-3)/1.2 = 31.96(10-9) = 3.196(10-8) m2 166 f = 3.196(10-8) m2)0.5 = 1.79(10-4) m = 0.179 mm (here, mm is interpreted mm/rev) 24.9 A part to be turned in an engine lathe must have a surface finish of 1.6 m. The part is made of a free-machining steel. Cutting conditions: v = 1.5 m/s and d = 3.0 mm. The nose radius on the tool = 1.2 mm. Determine the feed that will achieve the specified surface finish. Solution: For free-machining steel at 90 m/min, from Figure 24.2 ratio rai = 1.0 in Eq. (24.3), so Ra = Ri = f2/32NR Rearranging, f2 = Ri(32NR) = 1.6(10-6)(32)(1.2)(10-3) = 61.44(10-9) = 6.144(10-8) m2 f = (6.144(10-8) m2)0.5 = 2.48(10-4) m = 0.248 mm (here, mm is interpreted mm/rev) 24.10 The surface finish specification in a turning job is 0.8 m. The work material is cast iron. The cutting conditions have been selected as follows: v = 75 m/min, f = 0.3 mm/rev, and d = 4.0 mm. The nose radius of the cutting tool must be selected. Determine the minimum nose radius that will obtain the specified finish in this operation. Solution: For cast iron at 75 m/min, from Figure 24.2 ratio rai = 1.35 in Eq. (24.3), so Ra = 1.35Ri = 1.35f2/32NR Rearranging, NR = 1.35f2/(32Ra) NR = 1.35(0.3 x 10-3)2/(32)(0.8)(10-6) = 0.00475 m = 4.75 mm 24.11 A face milling operation is to be performed on a cast iron part at 400 ft/min to finish the surface to 32 -in. The cutter uses four inserts and its diameter is 3.0 in. To obtain the best possible finish, a type of carbide insert with 4/64 in nose radius is to be used. Determine the required feed rate (in/min) that will achieve the 32 -in finish. Solution: For cast iron at 400 ft/min, from Figure 24.2 ratio rai = 1.27, so Ra = 1.27 Ri Ri = Ra/1.27 = 32/1.27 = 25.2 in. Ri = f2/32NR Rearranging, f2 = 32(Ra)(NR) = 32(25.2 x 10-6)(4/64) = 50.4 x 10-6 in2 f = (50.4 x 10-6).5 = 7.1 x 10-3 = 0.0071 in/rev. N = v/D = (400 x 12)/3= 509.3 rev/min. fr = Nntf = 509.3(4)(0.0071) = 14.46 in/min. 24.12 A face milling operation is not yielding the required surface finish on the work. The cutter is a four-tooth insert type face milling cutter. The machine shop foreman thinks the problem is that the work material is too ductile for the job, but this property tests well within the ductility range for the material specified by the designer. Without knowing any more about the job, what changes in cutting conditions and tooling would you suggest to improve the surface finish? Solution: Changes in cutting conditions: (1) decrease chip load f, (2) increase cutting speed v, (3) use cutting fluid. Changes in tooling: (1) increase nose radius NR, (2) increase rake angle, and (3) increase relief angle. Items (2) and (3) will have a marginal effect. 24.13 A turning operation is to be performed on C1010 steel, which is a ductile grade. It is desired to achieve a surface finish of 64 -in (AA), while at the same time maximizing the metal removal rate. It has been decided that the speed should be in the range 200 ft/min to 400 ft/min, and that the depth of cut will be 0.080 in. The tool nose radius = 3/64 in. Determine the speed and feed combination that meets these criteria. Solution: Increasing feed will increase both MRR and Ra. Increasing speed will increase MRR and reduce Ra. Therefore, it stands to reason that we should operate at the highest possible v. 167 Try v = 400 ft/min. From Fig. 25.45, rai = 1.15. Ra = 1.15 Ri Ri = Ra/1.15 = 64/1.15 = 55.6 in. Ri = f2/32NR f2 = 32(Ra)(NR) = 32(55.6 x 10-6)(3/64) = 83.4 x 10-6 in2 f = (83.4 x 10-6).5 = 0.0091 in/rev. MRR = 3.51 in3/min. Compare at v = 300 ft/min. From Fig. 25.45, rai = 1.26. Ra = 1.26 Ri Ri = Ra/1.26 = 64/1.26 = 50.8 in. Ri = f2/32NR f2 = 32(Ra)(NR) = 32(50.8 x 10-6)(3/64) = 76.2 x 10-6 in2 f = (76.2 x 10-6).5 = 0.0087 in/rev. MRR = 2.51 in3/min. Optimum cutting conditions are: v = 400 ft/min and f = 0.0091 in/rev, which maximizes MRR = 3.51 in3/min. 24.14 Plain milling is performed to finish a cast iron workpart prior to plating. The milling cutter has four equally spaced teeth and the diameter = 60 mm. The chip load f = 0.35 mm/tooth, and cutting speed v = 1.0 m/s. Estimate the surface roughness for (a) up-milling, and (b) down-milling. Solution: (a) Up milling: Ri = 0.125f2/(.5D + fnt/) = 0.125(0.35)2/(30 + 0.35x4/) = 0.503 m. From Fig. 25.45, rai = 1.43 Ra = 1.43(0.503) = 0.719 mm. (b) Down milling: Ri = 0.125f2/(.5D - fnt/) = 0.125(0.35)2/(30 - 0.35x4/) = 0.518 m. From Fig. 25.45, rai = 1.43 Ra = 1.43(0.518) = 0.741 mm. 24.15 A peripheral milling operation is performed using a slab milling cutter with four teeth and a 2.50 in diameter. Feed = 0.015 in/tooth, and cutting speed = 150 ft/min. Assuming first that the teeth are equally spaced around the cutter, and that each tooth projects an equal distance from the axis of rotation, determine the theoretical surface roughness for (a) up-milling, and (b) down-milling. Solution: (a) Up milling: Ri = 0.125f2/(0.5D+fnt/) = 0.125(0.015)2/(1.25+0.015x4/) = 22.2 min. (b) Down milling: Ri = 0.125f2/(0.5D - fnt/) = 0.125(0.015)2/(1.25-0.015x4/) = 22.8 min. Machining Economics 24.16 A HSS tool is used to turn a steel workpart that is 300 mm long and 80 mm in diameter. The parameters in the Taylor equation are: n = 0.13 and C = 75 (m/min) for a feed of 0.4 mm/rev. The operator and machine tool rate = $30.00/hr, and the tooling cost per cutting edge = $4.00. It takes 2.0 min to load and unload the workpart and 3.50 min to change tools. Determine: (a) cutting speed for maximum production rate, (b) tool life in min of cutting, and (c) cycle time and cost per unit of product. Solution: (a) Co = $30/hr = $0.50/min vmax = 75/[(1/.13 - 1)(3.5)].13 = 75/[6.692 x 3.5].27 = 49.8 m/min. (b) Tmax = (75/49.8)1/.13 = (1.506)7.692 = 23.42 min. (c) Tm = DL/fv = (80)(300)/(.4 x 49.8 x 103) = 3.787 min. 168 np = 23.42/3.787 = 6.184 pc/tool life Use np = 6 pc/tool life Tc = Th + Tm + Tt/np = 2.0 + 3.787 + 3.5/6 = 6.37 min/pc. Cc = 0.50(6.37) + 4.00/6 = $3.85/pc 24.17 Solve previous Problem 24.16 except that in part (a), determine cutting speed for minimum cost. Solution: (a) Co = $30/hr = $0.50/min vmin = 75[.50/((1/.13 - 1)(.50 x 3.5 + 4.00))].13 = 75[.50/(6.692 x 5.75)].13 = 42.6 m/min. (b) Tmin = (75/42.6)1/.13 = (1.76)7.692 = 76.96 min. (c) Tm = DL/fv = (80)(300)/(.4 x 42.6 x 103) = 4.42 min/pc. np = 76.96/4.42 = 17.41 pc/tool life Use np = 17 pc/tool life Tc = Th + Tm + Tt/np = 2.0 + 4.42 + 3.5/17 = 6.63 min/pc. Cc = 0.50(6.63) + 4.0/17 = $3.55/pc 24.18 A cemented carbide tool is used to turn a part with length = 18.0 in and diameter = 3.0 in. The parameters in the Taylor equation are: n = 0.27 and C = 1200. The rate for the operator and machine tool = $33.00/hr, and the tooling cost per cutting edge = $2.00. It takes 3.0 min to load and unload the workpart and 1.50 min to change tools. The feed = 0.013 in/rev. Determine: (a) cutting speed for maximum production rate, (b) tool life in min of cutting, and (c) cycle time and cost per unit of product. Solution: (a) vmax = 1200/[(1/.27 - 1)(1.5)].27 = 1200/[2.704 x 1.5].27 = 822 ft/min. (b) Tmax = (1200/822)1/.27 = (1.460)3.704 = 4.06 min. (c) Tm = DL/fv = (3)(18)/(.013 x 822 x 12) = 1.323 min. np = 4.055/1.323 = 3.066 pc/tool Use np = 3 pc/tool life Tc = Th + Tm + Tt/np = 3.0 + 1.323 + 1.5/3 = 4.823 min/pc. Co = $33/hr = $0.55/min Cc = 0.55(5.823) + 2.0/3 = $3.32/pc 24.19 Solve previous Problem 24.18 except that in part (a), determine cutting speed for minimum cost. Solution: (a) Co = $33/hr = $0.55/min vmin = 1200[.55/((1/.27 - 1)(.55 x 1.5 + 2.00))].27 = 1200[.55/(2.704 x 2.825)].27 = 590 ft/min. (b) Tmin = (1200/590)1/.27 = (2.034)3.704 = 13.89 min. (c) Tm = DL/fv = (3)(18)/(.013 x 590 x 12) = 1.843 min. np = 13.89/1.843 = 7.54 pc/tool Use np = 7 pc/tool life Tc = Th + Tm + Tt/np = 3.0 + 1.843 + 1.5/7 = 5.06 min/pc. Cc = 0.55(5.06) + 2.0/7 = $3.07/pc 24.20 Compare disposable and regrindable tooling. The same grade of cemented carbide tooling is available in two forms for turning operations in a certain machine shop: disposable inserts and brazed inserts. The parameters in the Taylor equation for this grade are: n = 0.25 and C = 300 (m/min) under the cutting conditions considered here. For the disposable inserts, price of each insert = $6.00, there are four cutting edges per insert, and the tool change time = 1.0 min (this is an average of the time to index the insert and the time to replace it when all edges have been used). For the brazed insert, the price of the tool = $30.00 and it is estimated that it can be used a total of 15 times before it must be scrapped. The tool change time for the regrindable tooling = 3.0 min. The standard time to grind or regrind the cutting edge is 5.0 min, and the grinder is paid at a rate = $20.00/hr. Machine time on the lathe costs $24.00/hr. The workpart to be used in the 169 comparison is 375 mm long and 62.5 mm in diameter, and it takes 2.0 min to load and unload the work. The feed = 0.30 mm/rev. For the two tooling cases, compare: (a) cutting speeds for minimum cost, (b) tool lives, (c) cycle time and cost per unit of production. Which tool would you recommend? Solution: Disposable inserts: (a) Co = $24/hr = $0.40/min, Ct = $6/4 = $1.50/edge vmin = 300[0.40/((1/0.25 - 1)(0.40 x 1.0 + 1.50))].25 = 300[0.40/(3 x 1.9)].25 = 154.4 m/min. (b) Tmin = (1/0.25 - 1)(0.4 + 1.5)/0.4 = 3(1.9/0.4) = 14.25 min. (c) Tm = (62.5)(375)/(0.30)(10-3)(154.4) = 1.59 min/pc np = 14.25/1.59 = 8.96 pc/tool life Use np = 8 pc/tool Tc = 2.0 + 1.59 + 1.0/8 = 3.72 min/pc. Cc = 0.40(3.72) + 1.50/8 = $1.674/pc Regrindable tooling: (a) Co = $24/hr = $0.40/min, Ct = $30/15 + 5($20/60) = $3.67/edge vmin = 300[0.40/((1/0.25 - 1)(0.40 x 3.0 + 3.67))].25 = 300[0.40/(3 x 4.87)].25 = 122.0 m/min. (b) Tmin = (1/0.25 - 1)(0.4 x 3 + 3.67)/0.4 = 3(4.87/0.4) = 36.5 min. (c) Tm = (62.5)(375)/(0.30)(10-3)(122) = 2.01 min/pc np = 36.5/2.01 = 18.16 pc/tool life Use np = 18 pc/tool Tc = 2.0 + 2.01 + 3.0/18 = 4.18 min/pc. Cc = 0.40(4.18) + 3.67/18 = $1.876/pc Disposable inserts are recommended. Cycle time and cost per piece are less. 24.21 Solve previous Problem 24.20 except that in part (a), determine the cutting speeds for maximum production rate. Solution: Disposable inserts: (a) Co = $24/hr = $0.40/min, Ct = $6/4 = $1.50/edge vmax = 300[1.0/((1/0.25 - 1)(1.0)].25 = 300[1.0/(3 x 1.0)].25 = 228.0 m/min. (b) Tmax = (1/0.25 - 1)(1.0) = 3(1.0) = 3.0 min. (c) Tm = (62.5)(375)/(0.30)(10-3)(228) = 1.08 min/pc np = 3.0/1.08 = 2.78 pc/tool life Use np = 2 pc/tool Tc = 2.0 + 1.08 + 1.0/2 = 3.58 min/pc. Cc = 0.40(3.58) + 1.50/2 = $2.182/pc Regrindable tooling: (a) Co = $24/hr = $0.40/min, Ct = $30/15 + 5($20/60) = $3.67/edge vmax = 300[1.0/((1/0.25 - 1)(3.0))].25 = 300[1.0/(3 x 3.0)].25 = 173.2 m/min. (b) Tmax = (1/0.25 - 1)(3) = 3(3.0) = 9.0 min. (c) Tm = (62.5)(375)/(0.30)(10-3)(173.2) = 1.42 min/pc np = 9.0/1.42 = 6.34 pc/tool life Use np = 6 pc/tool Tc = 2.0 + 1.42 + 3.0/6 = 3.92 min/pc. Cc = 0.40(3.92) + 3.67/6 = $2.180/pc Disposable inserts are recommended. Cycle time and cost per piece are less. Comparing the results in this problem with those of the previous problem, note that with the maximum production rate objective here, cycle times are less, but that unit costs are less in the previous problem where the objective is minimum cost per piece. 24.22 Three tool materials are to be compared for the same finish turning operation on a batch of 100 steel parts: high speed steel, cemented carbide, and ceramic. For the high speed steel tool, the 170 Taylor equation parameters are: n = 0.125 and C = 70. The price of the HSS tool is $15.00 and it is estimated that it can be ground and reground 15 times at a cost of $1.50. Tool change time = 3 min. Both carbide and ceramic tools are in insert form and can be held in the same mechanical toolholder. The Taylor equation parameters for the cemented carbide are: n = 0.25 and C = 500; and for the ceramic: n = 0.6 and C = 3,000. The cost per insert for the carbide = $6.00 and for the ceramic = $8.00. Number of cutting edges per insert in both cases = 6. Tool change time = 1.0 min for both tools. Time to change parts = 2.0 min. Feed = 0.25 mm/rev, and depth = 3.0 mm. The cost of machine time = $30/hr. The part dimensions are: diameter = 56.0 mm and length = 290 mm. Setup time for the batch is 2.0 hr. For the three tooling cases, compare: (a) cutting speeds for minimum cost, (b) tool lives, (c) cycle time, (d) cost per production unit, (e) total time to complete the batch and production rate. (f) What is the proportion of time spent actually cutting metal for each tooling? Solution: HSS tooling: (a) Ct = $15/15 + 1.50 = $2.50/edge. Co = $30/hr = $0.50/min. vmin = 70[0.50/((1/.125 - 1)(0.50 x 3.0 + 2.50))].125 = 70[0.50/(7 x 4.0)].125 = 42.4 m/min. (b) Tmin = (1/.125 - 1)(0.50 x 3 + 2.50)/.50 = 7(4.0/.50) = 56 min. (c) Tm = (56)(290(10-6))/(0.25(10-3)42.4) = 4.81min/pc np = 56/4.81 = 11.6 pc/tool life Use np = 11 pc/tool life Tc = 2.0 + 4.81 + 3.0/11 = 7.08 min/pc. (d) Cc = 0.50(7.08) + 2.50/11 = $3.77/pc (e) Time to complete batch = 2.0(60) + 100(7.08) = 828 min = 13.8 hr. Production rate Rp = 100 pc/13.8 hr = 7.25 pc/hr. (f) Proportion of time spent cutting = 100(4.81)/828 = 0.581 = 58.1% Cemented carbide tooling: (a) Ct = $6/6 = $1.00/edge. Co = $30/hr = $0.50/min. vmin = 500[.50/((1/.25 - 1)(.50 x 1.0 + 1.00))].25 = 500[.50/(3 x 1.50)].25 = 289 m/min. (b) Tmin = (1/.25 - 1)(.50 x 1 + 1.0)/.50 = 3(1.50/.50) = 9 min. (c) Tm = (56)(290(10-6))/(0.25(10-3)289) = 0.706min/pc np = 9/0.706 = 12.7 pc/tool life Use np = 12 pc/tool life Tc = 2.0 + 0.706 + 1.0/12 = 2.79 min/pc. (d) Cc = 0.50(2.79) + 1.00/12 = $1.48/pc (e) Time to complete batch = 2.0(60) + 100(2.79) = 399 min = 6.65 hr. Production rate Rp = 100 pc/6.783 hr = 14.74 pc/hr. (f) Proportion of time spent cutting = 100(0.706)/399 = 0.177 = 17.7% Ceramic tooling: (a) Ct = $8/6 = $1.33/edge. Co = $30/hr = $0.50/min. vmin = 3,000[.50/((1/.6 - 1)(.50 x 1.0 + 1.33))].6 = 3,000[.50/(.667 x 1.83)].6 = 1756 m/min. (b) Tmin = (1/0.6 - 1)(0.50 x 1 + 1.33)/.50 = 0.667(1.83/.50) = 2.44 min. (c) Tm = (56)(290(10-6))/(0.25(10-3)1756) = 0.116 min/pc np = 2.44/0.116 = 21 pc/tool life Tc = 2.0 + 0.116 + 1.0/21 = 2.16 min/pc. (d) Cc = 0.50(2.16) + 1.33/21 = $1.15/pc 171 (e) Time to complete batch = 2.0(60) + 100(2.16) = 336 min = 5.60 hr. Production rate Rp = 100 pc/5.6 hr = 17.86 pc/hr. (f) Proportion of time spent cutting = 100(0.116)/336 = 0.035 = 3.5% Comment: One might conclude that such a low proportion of time spent cutting would argue against the use of the calculated cutting speed for ceramic tooling. However, note that ceramic tooling provides a significant advantage in terms of unit cost, batch time, and production rate compared to HSS tooling and even carbide tooling. The very small cutting time Tm and resulting low proportion of time spent cutting for ceramic tooling focuses attention on the nonproductive work elements in the batch time, specifically, setup time and workpart handling time; and puts pressure on management to seek ways to reduce these nonproductive elements. 24.23 Solve previous Problem 24.22 except that in parts (a) and (b), determine the cutting speeds and tool lives for maximum production rate. Solution: HSS tooling: (a) Ct = $15/15 + 1.50 = $2.50/edge. Co = $30/hr = $0.50/min. vmax = 70/[(1/.125 - 1)(3.0)].125 = 70/[7 x 3)].125 = 48 m/min. (b) Tmax = (1/0.125 - 1)(3) = 7(3) = 21 min. (c) Tm = (c) Tm = (56)(290(10-6))/(0.25(10-3)484) = 4.25min/pc np = 21/4.25 = 4.9 pc/tool life Use np = 4 pc/tool life Tc = 2.0 + 4.25 + 3.0/4 = 7.00 min/pc. (d) Cc = 0.50(7.00) + 2.50/4 = $4.13/pc (e) Time to complete batch = 2.0(60) + 100(7.00) = 820 min = 13.67 hr. Production rate Rp = 100 pc/13.67 hr = 7.32 pc/hr. (f) Proportion of time spent cutting = 100(4.25)/820 = 0.518 = 51.8% Cemented carbide tooling: (a) Ct = $6/6 = $1.00/edge. Co = $30/hr = $0.50/min. vmax = 500/[(1/.25 - 1)(1.0)].25 = 500/[(3 x 1.0)].25 = 380 m/min. (b) Tmax = (1/0.25 - 1)(1.0) = 3(1.0) = 3.0 min. (c) Tm = (56)(290(10-6))/(0.25(10-3)380) = 0.537 min/pc np = 3/0.537 = 5.6 pc/tool life Use np = 5 pc/tool life Tc = 2.0 + 0.537 + 1.0/5 = 2.74 min/pc. (d) Cc = 0.50(2.74) + 1.00/5 = $1.57/pc (e) Time to complete batch = 2.0(60) + 100(2.74) = 394 min = 6.57 hr. Production rate Rp = 100 pc/6.57 hr = 15.23 pc/hr. (f) Proportion of time spent cutting = 100(0.537)/394 = 0.136 = 13.6% Ceramic tooling: (a) Ct = $8/6 = $1.33/edge. Co = $30/hr = $0.50/min. vmax = 3,000/[(1/.6 - 1)(1.0)].6 = 3,000/[.667 x 1.0].6 = 3825 m/min. (b) Tmax = (1/0.6 - 1)(1) = 0.667(1) = .667 min. (c) Tm = (56)(290(10-6))/(0.25(10-3)3825) = 0.053 min/pc np = 0.667/0.053 = 12 pc/tool life Tc = 2.0 + 0.053 + 1.0/12 = 2.14 min/pc. 172 (d) Cc = 0.50(2.14) + 1.33/12 = $1.18/pc (e) Time to complete batch = 2.0(60) + 100(2.14) = 334 min = 5.57 hr. Production rate Rp = 100 pc/5.57 hr = 17.96 pc/hr. (f) Proportion of time spent cutting = 100(0.053)/334 = 0.016 = 1.6% Comment: One might conclude that such a low proportion of time spent cutting would argue against the use of the calculated cutting speed for ceramic tooling. However, note that ceramic tooling provides a significant advantage in terms of unit cost, batch time, and production rate compared to HSS tooling and even carbide tooling. The very small cutting time Tm and resulting low proportion of time spent cutting for ceramic tooling focuses attention on the nonproductive work elements in the batch time, specifically, setup time and workpart handling time; and puts pressure on management to seek ways to reduce these nonproductive elements. 24.24 A vertical boring mill is used to bore the inside diameter of a large batch of tube-shaped parts. The diameter = 28.0 in and the length of the bore = 14.0 in. Current cutting conditions are: speed = 200 ft/min, feed = 0.015 in/rev, and depth = 0.125 in. The parameters of the Taylor equation for the cutting tool in the operation are: n = 0.23 and C = 850 (ft/min). Tool change time = 3.0 min, and tooling cost = $3.50 per cutting edge. The time required to load and unload the parts = 12.0 min, and the cost of machine time on this boring mill = $42.00/hr. Management has decreed that the production rate must be increased by 25%. Is that possible? Assume that feed must remain unchanged in order to achieve the required surface finish. What is the current production rate and the maximum possible production rate for this job? Solution: At the current operating speed v = 200 ft/min: T = (850/200)1/.23 = 540 min. Tm = (28)(14)/(200 x 12 x 0.015) = 34.2 min/pc np = 540/34.2 = 15 pc/tool life Tc = 12 + 34.2 + 3/15 = 46.4 min. Rc = 60/46.4 = 1.293 pc/hr Find vmax to compare with current operating speed. vmax = 850/[(1/.23 - 1)(3.0)].23 = 850/[(3.348 x 3.0)].23 = 500 ft/min. Tmax = (1/.23 - 1)(3.0) = 3.348(3.0) = 10.0 min. Tm = (28)(14)/(500 x 12 x 0.015) = 13.7 min/pc np = 10/13.7 = 0.73 pc/tool life Tc = 12 + 13.7 + 3/.73 = 29.8 min. Rc = 60/29.8 = 2.01 pc/hr This is a 56% increase in production rate relative to the 200 ft/min cutting speed. 24.25 A NC lathe cuts two passes across a cylindrical workpiece under automatic cycle. The operator loads and unloads the machine. The starting diameter of the work is 3.00 in and its length = 10 in. The work cycle consists of the following steps (with element times given in parentheses where applicable): 1 - Operator loads part into machine, starts cycle (1.00 min); 2 - NC lathe positions tool for first pass (0.10 min); 3 - NC lathe turns first pass (time depends on cutting speed); 4 - NC lathe repositions tool for second pass (0.4 min); 5 - NC lathe turns second pass (time depends on cutting speed); and 6 - Operator unloads part and places in tote pan (1.00 min). In addition, the cutting tool must be periodically changed. This tool change time takes 1.00 min. The feed rate = 0.007 in/rev and the depth of cut for each pass = 0.100 in. The cost of the operator and machine = $39/hr and the tool cost = $2.00/cutting edge. The applicable Taylor tool life equation has parameters: n = 0.26 and C = 900 (ft/min). Determine: (a) the cutting speed for minimum cost per piece, (b) the average time required to complete one production cycle, (c) cost of the production 173 cycle. (d) If the setup time for this job is 3.0 hours and the batch size = 300 parts, how long will it take to complete the batch? Solution: (a) Co = $39/hr = $0.65/min. vmin = 900[.65/((1/.26 - 1)(.65 x 1.0 + 2.00))].26 = 900[.65/(2.846 x 2.65)].26 = 476 ft/min. (b) Tmin = (1/.26 - 1)(.65 x 1 + 2.0)/.65 = 2.846(2.65/.65) = 11.6 min. Tm = (3)(10)/(476 x 12 x 0.007) = 2.36 min/pc. Assume both passes have equal Tm. np = 11.6/2.36 = 4.9 passes/tool life Since there are two passes/workpiece, np = 2.45 pc/tool life Tc = 2.5 + 2 x 2.36 + 1.0/2.45 = 7.63 min/pc. (c) Cc = .65(2.5 + 2 x 2.36) + (0.65 x 1 + 2.00)/2.45 = $5.77/pc (d) Time to complete batch = 3.0(60) + 300(7.63) = 2469 min = 41.15 hr. 24.26 As indicated in Section 24.4, the effect of a cutting fluid is to increase the value of C in the Taylor tool life equation. In a certain machining situation using HSS tooling, the C value is increased from C = 200 to C = 225 due to the use of the cutting fluid. The n value is the same with or without fluid at n = 0.125. Cutting speed used in the operation is v = 125 ft/min. Feed = 0.010 in/rev and depth = 0.100 in. The effect of the cutting fluid can be to either increase cutting speed (at the same tool life) or increase tool life (at the same cutting speed). (a) What is the cutting speed that would result from using the cutting fluid if tool life remains the same as with no fluid? (b) What is the tool life that would result if the cutting speed remained at 125 ft/min. (c) Economically, which effect is better, given that tooling cost = $2.00 per cutting edge, tool change time = 2.5 min., and operator and machine rate = $30/hr. Justify you answer with calculations, using cost per cubic in of metal machined as the criterion of comparison. Ignore effects of workpart handling time. Solution: Cutting dry, the Taylor tool life equation parameters are n = 0.125 and C = 200. At v = 125 ft/min, tool life T = (200/125)1/.125 = (1.6)8 = 43 min. With a cutting fluid, the Taylor tool life equation parameters are n = 0.125 and C = 225. The corresponding cutting speed for a 43 min tool life v = 225/430.125 = 140.6 ft/min (b) Cutting at v = 125 ft/min with a cutting fluid gives a tool life T = (225/125)8.0 = 110 min (c) Which is better, (1) cutting at a speed of 140.6 ft/min to give a 43 min tool life, or (2) cutting at 125 ft/min to give a 110 min tool life. Use 1.0 in3 of metal cut as the basis of comparison, with cost and time parameters as follows: Ct = $2.00/cutting edge, Tt = 2.5 min, and Co = $30/hr = $0.50/min. (1) At 140.6 ft/min, Tm = 1.0 in3/MRR = 1.0/(140.6 x 12 x 0.010 x 0.100) = 0.5927 min. For T = 43 min, volume cut per tool life = 43/0.5927 = 72.5 in3 between tool changes. Ignoring work handling time, cost/in3 = 0.50(.5927) + (0.50 x 2.5 + 2.00)/72.5 = $0.341/in3. (2) At 125 ft/min, Tm = 1.0 in3/MRR = 1.0/(125 x 12 x 0.010 x 0.100) = 0.6667 min. For T = 110 min, volume cut per tool life = 110/0.6667 = 164.9 in3 between tool changes. Ignoring work handling time, cost/in3 = 0.50(.6667) + (0.50 x 2.5 + 2.00)/164.9 = $0.353/in3. Conclusion: it is better to take the benefit of a cutting fluid in the form of increased cutting speed. 24.27 In a turning operation on ductile steel, it is desired to obtain an actual surface roughness of 63 -in with a 2/64 in nose radius tool. The ideal roughness is given by Eq. (24.1) and an adjustment will have to be made using Figure 24.2 to convert the 63 -in actual roughness to an ideal roughness, taking into account the material and cutting speed. Disposable inserts are used at a cost of $1.75 per cutting edge (each insert costs $7.00 and there are four edges per insert). Time to index each 174 insert = 25 sec and to replace an insert every fourth index takes 45 sec. The workpiece length = 30.0 in and its diameter = 3.5 in. The machine and operator’s rate = $39.00 per hour including applicable overheads. The Taylor tool life equation for this tool and work combination is given by: vT0.23 f0.55 = 40.75, where T = tool life, min; v = cutting speed, ft/min; and f = feed, in/rev. Solve for (a) the feed in in/rev that will achieve the desired actual finish, (b) cutting speed for minimum cost per piece at the feed determined in (a). Hint: to solve (a) and (b) requires and iterative computational procedure. Solution: Cost and time parameters: Co = $39/hr = $0.65/min, Ct = $1.75/cutting edge, Tt = (3(25)+45)/4 = 30 sec = 0.5 min. Iteration 1: assume Ri = Ra = 63 -in. = 63 x 10-6 in. Rearranging Eq. (24.1), f2 = 32NR(Ri) = 32(2/64)(63 x 10-6) in.2 = 63(10-6) in.2 f = (63 x 10-6)0.5 = 0.00794 in. (interpreted as in./rev for turning) C = vT0.23 = 40.75/f0.55 = 40.75/0.00794 = 582.5 vmin = 582.5{(0.23/(1-0.23))(0.65/(0.65 x 0.5 + 1.75)}0.23 = 582.5{0.09357}0.23 = 337.8 ft/min Iteration 2: At v = 337.8 ft/min, the ratio from Figure 24.2 rai = 1.2. Thus, Ri = Ra/1.2 = 63/1.2 = 52.5 -in. = 52.5(10-6) in. f2 = 32NR(Ri) = 32(2/64)(52.5 x 10-6) in.2 = 52.5(10-6) in.2 f = (52.5 x 10-6)0.5 = 0.00725 in. C = vT0.23 = 40.75/f0.55 = 40.75/0.00725 = 612.5 vmin = 612.5{(0.23/(1-0.23))(0.65/(0.65 x 0.5 + 1.75)}0.23 = 612.5{0.09357}0.23 = 355.2 ft/min Iteration 3: At v = 355.2 ft/min, the ratio from Figure 24.2 rai = 1.18. Thus, Ri = Ra/1.2 = 63/1.18 = 53.4 -in. = 53.4(10-6) in. f2 = 32NR(Ri) = 32(2/64)(53.4 x 10-6) in.2 = 53.4(10-6) in.2 f = (53.4 x 10-6)0.5 = 0.00731 in. C = vT0.23 = 40.75/f0.55 = 40.75/0.00731 = 609.6 vmin = 609.6{(0.23/(1-0.23))(0.65/(0.65 x 0.5 + 1.75)}0.23 = 609.6{0.09357}0.23 = 353.5 ft/min Select v = 353.5 ft/min and f = 0.0073 in/rev. 24.28 Verify that the derivative of Eq. (24.8) results in Eq. (24.9). Solution: Starting with Eq. (24.8): Tc = Th + DL/fv + Tt(DLv1/n-1)/fC1/n Tc = Th + (DL/f)v-1 + (TtDL/fC1/n)v1/n-1 dTc/dv = 0 - (DL/f)v-2 + (1/n – 1)(TtDL/fC1/n)v1/n-2 = 0 (DL/f)v-2 = (1/n – 1)(TtDL/fC1/n)v1/n-2 = 0 (DL/f) = (1/n – 1)(Tt DL/fC1/n)v1/n 1 = (1/n – 1)(Tt /C1/n)v1/n v1/n = C1/n/[(1/n-1)Tt] vmax = C/[(1/n-1)Tt]n Q.E.D 24.29 Verify that the derivative of Eq. (24.14) results in Eq. (24.15). Solution: Starting with Eq. (24.14): Tc = Th + DL/fv + (CoTt + Ct)(DLv1/n-1)/fC1/n Tc = Th + (DL/f)v-1 + (CoTt + Ct)(DL/fC1/n)v1/n-1 dTc/dv = 0 - (DL/f)v-2 + (1/n – 1)(CoTt + Ct)(DL/fC1/n)v1/n-2 = 0 (DL/f)v-2 = (1/n – 1)(CoTt + Ct)(DL/fC1/n)v1/n-2 = 0 (DL/f) = (1/n – 1)(CoTt + Ct)DL/fC1/n)v1/n 1 = (1/n – 1)((CoTt + Ct)/C1/n)v1/n v1/n = C1/n/[(1/n-1)(CoTt + Ct)] 175 vmax = C/[(1/n-1)(CoTt + Ct)]n Q.E.D 176 25 GRINDING AND OTHER ABRASIVE PROCESSES Review Questions 25.1 Why are abrasive processes technologically and commercially important? Answer. Important reasons include: (1) applications on all types of materials, (2) very fine finishes, and (3) close tolerances. 25.2 What are the five principal parameters of a grinding wheel? Answer. The parameters are: (1) abrasive material, (2) grit size, (3) bonding material, (4) wheel structure, which refers to the relative spacing of grains, and (5) wheel grade, which refers to the bond strength of the wheel in retaining abrasive grains. 25.3 What are some of the principal abrasive materials used in grinding wheels? Answer. The principal abrasive grit materials include: (1) aluminum oxide, (2) silicon carbide, (3) cubic boron nitride, and (4) diamond. 25.4 Name some of the principal bonding materials used in grinding wheels. Answer. The bonding materials in grinding wheels are: (1) vitrified bond - clay and ceramics, (2) silicate, (3) rubber, (4) resinoid, (5) shellac, and (6) metallic. 25.5 What is wheel structure? Answer. Wheel structure indicates the relative spacing of the abrasive grains in the wheel. An open structure is one in which the grains are far apart, and a dense structure indicates that the grains are close together. 25.6 What is wheel grade? Answer. Wheel grade refers to the wheel's ability to retain abrasive grains during cutting. It indicates the bond strength of the bonding material used to shape the wheel. A soft grade indicates that the grains are released easily from the bonding material. A hard wheel is one which retains the abrasive grains. 25.7 Why are specific energy values so much higher in grinding than in traditional metal cutting processes? Answer. Reasons for higher specific energy in grinding: (1) size effect - smaller chip size means higher specific energy; (2) extremely negative rake angles on the abrasive particles in a grinding wheel; and (3) not all of the grains in the wheel surface are engaged in cutting; some are plowing or deforming while others are simply rubbing and creating friction. 25.8 Grinding creates high temperatures. How is temperature harmful in grinding? Answer. High temperatures in grinding create surface burns and cracks. High temperatures can also soften the surfaces of workparts that have been heat treated to obtain high hardness. 25.9 What are the three mechanisms of grinding wheel wear? Answer. The mechanisms are: (1) grain fracture, in which a portion of the grain breaks off during cutting; (2) attritious wear, in which the grains become dull during cutting; and (3) bond fracture, in which the grains are pulled out of the bonding material. 177 25.10 What is dressing, in reference to grinding wheels? Answer. Dressing is a procedure applied to worn grinding wheels to break off dull grits and expose fresh grits, and to remove chips of work material that have become clogged in the wheel. It uses a rotating disk or abrasive stick held against the wheel while it rotates. 25.11 What is truing, in reference to grinding wheels? Answer. Truing is similar to dressing, but it also restores the ideal cylindrical shape to the wheel. It uses a diamond-pointed tool fed slowly and precisely across the wheel while it rotates. 25.12 What abrasive material would one select for grinding a cemented carbide cutting tool? Answer. Choose a diamond wheel. 25.13 What are the functions of a grinding fluid? Answer. Functions of a grinding fluid are: (1) reduce friction, (2) remove heat, (3) wash away chips, and (4) reduce workpiece temperature. 25.14 What is centerless grinding? Answer. Centerless grinding is a grinding operation in which cylindrical workparts (e.g., rods) are fed between two rotating wheels: (1) a high speed grinding wheel and (2) a low speed regulating wheel which is tilted at a slight angle to control the feed-through rate. 25.15 How does creep feed grinding differ from conventional grinding? Answer. In creep feed grinding, the depth of cut is very high - several thousand times higher than conventional grinding - and the feed rates are lower by the same proportion. 25.16 How does abrasive belt grinding differ from a conventional surface grinding operation? Answer. Instead of a grinding wheel, abrasive belt grinding uses abrasive particles bonded to a flexible cloth belt loop which is moved through a pulley system to obtain the speed motion. Parts are pressed against the belt to accomplish grinding. 25.17 Name some of the abrasive operations available to achieve very good surface finishes. Answer. High finish abrasive processes include: honing, lapping, superfinishing, buffing, and polishing. Multiple Choice Quiz There are a total of 17 correct answers in the following multiple choice questions (some questions have multiple answers that are correct). To attain a perfect score on the quiz, all correct answers must be given, since each correct answer is worth 1 point. For each question, each omitted answer or wrong answer reduces the score by 1 point, and each additional answer beyond the number of answers required reduces the score by 1 point. Percentage score on the quiz is based on the total number of correct answers. 25.1 Which of the following conventional machining processes is closest to grinding (one answer)? (a) drilling, (b) milling, (c) shaping, or (d) turning. Answer. (b) 25.2 Of the following abrasive materials, which has the highest hardness? (a) aluminum oxide, (b) cubic boron nitride, or (c) silicon carbide. 178 Answer. (b) 25.3 Smaller grain size in a grinding wheel tends to do which one of the following? (a) improve surface finish, (b) have no effect on surface finish, or (c) degrade surface finish. Answer. (a) 25.4 Which of the following would tend to give higher material removal rates? (a) larger grain size, or (b) smaller grain size. Answer. (a) 25.5 Which of the following will improve surface finish in grinding (more than one)? (a) higher wheel speed, (b) larger infeed, (c) lower wheel speed, (d) lower work speed. Answer. (a) and (d). 25.6 Which of the following abrasive materials is most appropriate for grinding steel and cast iron (one best answer)? (a) aluminum oxide, (b) cubic boron nitride, (c) diamond, or (d) silicon carbide. Answer. (a) 25.7 Which of the following abrasive materials is most appropriate for grinding hardened tool steel (one best answer)? (a) aluminum oxide, (b) cubic boron nitride, (c) diamond, or (d) silicon carbide. Answer. (b) 25.8 Which of the following abrasive materials is most appropriate for grinding nonferrous metals (one best answer)? (a) aluminum oxide, (b) cubic boron nitride, (c) diamond, or (d) silicon carbide. Answer. (d) 25.9 Which of the following will help to reduce the incidence of heat damage to the work surface in grinding (more than one)? (a) frequent dressing or truing of the wheel, (b) higher infeeds, (c) higher work speeds, or (d) lower wheel speeds. Answer. (a), (c), and (d). 25.10 Which of the following abrasive processes achieves the best surface finish (one best answer)? (a) centerless grinding, (b) honing, (c) lapping, or (d) superfinishing. Answer. (d) 25.11 Which of the following abrasive processes could be used to finish a hole or internal bore (more than one)? (a) centerless grinding, (b) honing, (c) cylindrical grinding, (d) lapping, or (e) superfinishing. Answer. (a), (b), and (c). 25.12 The term deep grinding refers to which of the following (one best answer)? (a) alternative name for any creep feed grinding operation, (b) external cylindrical creep feed grinding, (c) grinding operation performed at the bottom of a hole, (d) surface grinding which uses a large crossfeed, or (e) surface grinding which uses a large infeed. Answer. (b) 179 Problems 25.1 In a surface grinding operation the wheel diameter = 150 mm and the infeed = 0.07 mm. The wheel speed = 1450 m/min, work speed = 0.25 m/s, and the cross-feed = 5 mm. The number of active grits per area of wheel surface C = 0.75 grits/mm2. Determine: (a) average length per chip, (b) metal removal rate, and (c) number of chips formed per unit time for the portion of the operation when the wheel is engaged in the work. Solution: (a) lc = (150 x 0.07)0.5 = 3.24 mm (b) MRR = vwwd = (0.25 m/s)(103 mm/m)(5.0 mm)(0.07 mm) = 87.5 mm3/s = 5250 mm3/min (c) nc = vwC = (1450 m/min)( 103 mm/m)(5.0 mm)(0.75 grits/mm2) = 5,437,500 chips/min. 25.2 The following conditions and settings are used in a certain surface grinding operation: wheel diameter = 6.0 in, infeed = 0.003 in, wheel speed = 4750 ft/min, work speed = 50 ft/min, and cross-feed = 0.20 in. The number of active grits per square inch of wheel surface C = 500. Determine: (a) the average length per chip, (b) the metal removal rate, and (c) the number of chips formed per unit time for the portion of the operation when the wheel is engaged in the work. Solution: (a) lc = (Dd)0.5 = (6.0 x 0.003)0.5 = (0.018)0.5 = 0.1342 in. (b) MRR = vwwd = (50 x 12)(0.20)(0.003) = 0.36 in3/min. (c) nc = vwC = (4750 x 12)(0.2)(500) = 5,700,000 chips/min. 25.3 An internal cylindrical grinding operation is used to finish an internal bore from an initial diameter of 250.00 mm to a final diameter of 252.5 mm. The bore is 125 mm long. A grinding wheel with an initial diameter of 150.00 mm and a width of 20.00 mm is used. After the operation, the diameter of the grinding wheel has been reduced to 149.75 mm. Determine the grinding ratio in this operation. Solution: GR = (volume of work material removed)/(volume of wheel removed) Volume of work material removed = (/4)(125)(252.52 – 250.02) = 123,332 mm2 Volume of wheel removed = (/4)(20)(1502 – 149.752) = 1177 mm2 GR = 123,332/1177 = 104.8 25.4 In a surface grinding operation performed on hardened plain carbon steel, the grinding wheel has a diameter = 200 mm and width = 25 mm. The wheel rotates at 2400 rev/min, with a depth of cut (infeed) = 0.05 mm/pass and a cross-feed = 3.50 mm. The reciprocating speed of the work is 6 m/min, and the operation is performed dry. Determine: (a) the length of contact between the wheel and the work, (b) the volume rate of metal removed. (c) If C = 0.64 active grits/mm2, estimate the number of chips formed per unit time. (d) What is the average volume per chip? (e) If the tangential cutting force on the work = 30 N, compute the specific energy in this operation? Solution: (a) lc = (200 x 0.05)0.5 = 3.16 mm (b) MRR = vwwd = (6 m/min)(103 mm/m)(3.5 mm)(0.05 mm) = 1050 mm3/min (c) nc = vwC v = ND = (2400 rev/min)(200mm/rev) = 1,507,964 mm/min nc = (1,507,964 mm/min)(3.5 mm)(0.64 grits/mm2) = 3,377,840 grits/min (= chips/min). (d) 3,377,840 grits/min. = 3,377,840 chips/min. Average volume per chip = (1050 mm3/min)/( 3,377,840 chips/min) = 0.00031 mm3/chip (e) U = Fcv/MRR 180 v = 1,507,964 mm/min = 1,508 m/min U = 30(1508)/1050 = 43.1 N-m/mm3 25.5 An 8-in diameter grinding wheel, 1.0 in wide, is used in a certain surface grinding job performed on a flat piece of heat-treated 4340 steel. The wheel is rotating to achieve a surface speed of 5000 ft/min, with a depth of cut (infeed) = 0.002 in per pass and a cross-feed = 0.15 in. The reciprocating speed of the work is 20 ft/min, and the operation is performed dry. (a) What is the length of contact between the wheel and the work? (b) What is the volume rate of metal removed? (c) If C = 300 active grits/in2, estimate the number of chips formed per unit time. (d) What is the average volume per chip? (e) If the tangential cutting force on the workpiece = 10 lbs, what is the specific energy calculated for this job? Solution: (a) lc = (Dd)0.5 = (8 x 0.002)0.5 = (0.016)0.5 = 0.1265 in. (b) MRR = vwwd = (20 x 12)(0.15)(0.002) = 0.072 in3/min (c) nc = vwC = (5000 x 12)(0.15)(300) = 2,700,000 chips/min. (d) Avg volume/chip = (0.072 in3/min)/(2,700,000 chips/min) = 0.000000026 in3 = 26 x 10-9 in3. (e) U = Fcv/MRR = 10(5000 x 12)/0.072 = 8,333,333 in-lb/in3 = 21 hp/(in3/min). 25.6 A surface grinding operation is being performed on a 6150 steel workpart (annealed, approximately 200 BHN). The designation on the grinding wheel is 51-C-24-D-5-V-23. The wheel diameter = 7.0 in and its width = 1.00 in. Rotational speed = 3000 rev/min. The depth (infeed) = 0.002 in per pass, and the cross-feed = 0.5 in. Workpiece speed = 20 ft/min. This operation has been a source of trouble right from the beginning. The surface finish is not as good as the 16 -in specified on the part print, and there are signs of metallurgical damage on the surface. In addition, the wheel seems to become clogged almost as soon as the operation begins. In short, nearly everything that can go wrong with the job has gone wrong. (a) Determine the rate of metal removal when the wheel is engaged in the work. (b) If the number of active grits per square inch = 200, determine the average chip length and the number of chips formed per time. (c) What changes would you recommend in the grinding wheel to help solve the problems encountered? Explain why you made each recommendation. Solution: (a) MRR = vwwd = (20 x 12)(0.5)(0.002) = 0.24 in3/min. (b) lc = (Dd)0.5 = (7.0 x .002)0.5 = 0.1183 in. v = DN = (7.0/12)(3000) = 5498 ft/min = 65,973 in/min nc = vwC = 65,973(0.5)(200) = 6,597,300 grits/min (c) Changes in wheel to help solve problems cited: (1) use Al2O3 oxide abrasive rather than silicon carbide; (2) use smaller grain size that 24; (3) use shellac bond rather than vitifried bond; (4) use more open structure than number 5 to reduce wheel clogging. 25.7 The grinding wheel in a centerless grinding operation has a diameter = 200 mm, and the regulating wheel diameter = 125 mm. The grinding wheel rotates at 3000 rev/min and the regulating wheel rotates at 200 rev/min. The inclination angle of the regulating wheel = 2.5. Determine the throughfeed rate of cylindrical workparts that are 25.0 mm in diameter and 175 mm long. Solution: From Eq. (25.11), fr = DrNr sin I fr = (125)(200) sin 2.5= 25,000(0.04362) = 3426 mm/min Parts throughfeed rate = (3426 mm/min)/(175 mm/pc) = 19.58 pc/min 181 25.8 A centerless grinding operation uses a regulating wheel that is 150 mm in diameter and rotates at 500 rev/min. At what inclination angle should the regulating wheel be set, if it is desired to feed a workpiece with length = 3.5 m and diameter = 18 mm through the operation in exactly 45 sec. Solution: From Eq. (25.11), fr = DrNr sin I fr = 3.5 m per 45 sec = 0.077778 m/s = 4.1667 m/min fr = (150 x 10-3)(500 rev/min) sin I = 235.62 sin I (units are m/min) 4.1667 m/min = 235.62 sin I m/min sin I = 4.1667/235.62 = 0.0198 I = 1.135° 25.9 In a certain centerless grinding operation, the grinding wheel diameter = 8.5 in, and the regulating wheel diameter = 5.0 in. The grinding wheel rotates at 3500 rev/min and the regulating wheel rotates at 150 rev/min. The inclination angle of the regulating wheel = 3 degrees. Determine the throughfeed rate of cylindrical workparts that have the following dimensions: diameter = 1.25 in and length = 8.0 in. Solution: From Eq. (25.11), fr = DrNr sin I = (5.0)(150) sin 3= 123.33 in/min. Parts feed at (8.0 in/part)/(123.33 in/min) = 0.0649 min/part = 3.9 sec/part 25.10 It is desired to compare the cycle times required to grind a particular workpiece using traditional surface grinding and using creep feed grinding. The workpiece is 200 mm long, 30 mm wide, and 75 mm thick. To make a fair comparison, the grinding wheel in both cases is 250 mm in diameter, 35 mm in width, and rotates at 1500 rev/min. It is desired to remove 25 mm of material from the surface. When traditional grinding is used, the infeed is set at 0.025 mm, and the wheel traverses twice (forward and back) across the work surface during each pass before resetting the infeed. There is no cross-feed since the wheel width is greater than the work width. Each pass is made at a work speed of 12 m/min, but the wheel overshoots the part on both sides. With acceleration and deceleration, the wheel is engaged in the work for 50% of the time on each pass. When creep feed grinding is used, the depth is increased by 1000 and the forward feed is decreased by 1000. How long will it take to complete the grinding operation (a) with traditional grinding and (b) with creep feed grinding? Solution: (a) Conventional surface grinding: Time of engagement/pass = 200 x 10-3 m/(12 m/min) = 0.01667 min = 1 s Forward and backward stroke = 2(1 s)/50% = 4 s Number of passes to remove 25 mm = 25/0.025 = 1000 passes Time to complete 1000 passes = 1000(4) = 4000 s = 66.67 min. (b) Creep feed grinding: Total length of feed = 200 mm + approach = 200 + (d(D-d))0.5 Given D = 250 mm and d = 25 mm, Total feed length = 200 + (25(250-25))0.5 = 275 mm fr = (12 x 103 mm/min)/1000 = 12 mm/min Time to feed = 275/12 = 22.917 min. Note: Creep feed grinding requires about 1/3 the time of conventional surface grinding for the situation defined here. 25.11 In a certain grinding operation, the grade of the grinding wheel should be “M” (medium), but the only available wheel is grade “T” (hard). It is desired to make the wheel appear softer by making changes in cutting conditions. What changes would you recommend? Solution: A hard wheel means that the grains are not readily pulled from the wheel bond. The wheel can be made to appear softer by increasing the force on the individual grits as given by Eq. 182 (25.8). According to this equation, the force on the abrasive grains will be increased by increasing work speed vw, decreasing wheel speed v, and increasing infeed d. 25.12 An aluminum alloy is to be ground in an external cylindrical grinding operation to obtain a good surface finish. Specify the appropriate grinding wheel parameters and the grinding conditions for this job. Solution: Grinding wheel specification: Abrasive type: silicon carbide Grain size: small - high grit size number Bond material: shellac bond Wheel structure: dense Wheel grade: medium to hard Wheel specification: XX-S-150-E-5-B-XY Grinding conditions: Wheel speed: high speed, around 1800 m/min (6000 ft/min) Work speed: low, around 10 m/min (30 ft/min) Infeed (depth of cut): low, around 0.012 mm (0.0005 in.) Crossfeed: low, around 1/6 of wheel width. 25.13 A high speed steel broach (hardened) is to be resharpened to achieve a good finish. Specify the appropriate parameters of the grinding wheel for this job. Solution: Grinding wheel specification: Abrasive type: cubic boron nitride Grain size: small - high grit size number Bond material: vitrified bond Wheel grade: soft to medium Wheel specification: XX-B-150-P-XY-V-XZ-1/8, where XX, XY, and XZ are manufacturer’s symbols. 25.14 Based on equations in the text, derive an equation to compute the average volume per chip formed in the grinding process. Solution: From Eq. (25.3), MRR = vwwd (in3/min) From Eq. (25.6), nc = vwC (chips/min) Volume per chip = MRR/nc = vwwd/vwC = vwd/vC 183 26 NONTRADITIONAL MACHINING AND THERMAL CUTTING PROCESSES Review Questions 26.1 Why are the nontraditional material removal processes important? Answer. Reasons for importance are: (1) need to shape new metal alloys and non-metals that are difficult to machine by conventional processes; (2) unusual and complex workpart geometries; and (3) need to avoid surface damage which is often associated with conventional machining. 26.2 There are four categories of nontraditional machining processes, based on principal energy form. Name the four categories. Answer. The four categories are: (1) mechanical, but not including conventional machining; (2) electrical; (3) thermal; and (4) chemical. 26.3 How does the ultrasonic machining process work? Answer. In ultrasonic machining, abrasives contained in a slurry are driven at high velocity against the work by a tool vibrating at low amplitude and high frequency. The tool oscillates in a direction perpendicular to the work surface, and is fed slowly into the work, so that the shape of the tool is formed in the part. The abrasives, impinging against the work surface, perform the cutting. 26.4 Describe the water jet cutting process. Answer. Water jet cutting uses a high-pressure, high-velocity stream of water directed at the work surface to cut of the work. 26.5 What is the difference between water jet cutting, abrasive water jet cutting, and abrasive jet cutting? Answer. WJC cuts with a narrow, high velocity water stream; AWJC adds abrasive grits to the water stream; and AJM cuts with abrasive particles that have been added to a high velocity air stream. 26.6 Name the three main types of electrochemical machining. Answer. The three types are: electrochemical machining, deburring, and grinding. 26.7 Identify the significant disadvantages of electrochemical machining. Answer. Disadvantages of ECM include: (1) cost of electrical power to operate the process, and (2) cost of disposal of electrolyte sludge. 26.8 How does increasing discharge current affect metal removal rate and surface finish in electric discharge machining? Answer. As discharge current increases, MRR increases and surface finish is degraded. 26.9 What is meant by the term overcut in electric discharge machining? Answer. Overcut refers to the difference between the size of the electrode (tool) in EDM and the size of the machined hole, cavity, or kerf (in wire EDM). 26.10 Identify two major disadvantages of plasma arc cutting. 184 Answer. Two disadvantages are: (1) rough surface on cut edge, and (2) metallurgical damage to cut surface. 26.11 What are some of the fuels used in oxyfuel cutting. Answer. Principal fuels are acetylene, MAPP (methylacetylene-propadiene), propylene, propane, and natural gas 26.12 Name the four principal steps in chemical machining. Answer. The four steps are: (1) cleaning, (2) masking, (3) etching, and (4) demasking. 26.13 What are the three methods of performing the masking step in chemical machining. Answer. The three steps are: (1) cut and peel, (2) screen resist, and (3) photographic resist. 26.14 What is a photoresist in chemical machining? Answer. A photoresist is a masking material that is sensitive to light. When exposed, it chemically transforms and can be removed from the surface of the work, leaving the desired surface unprotected by the maskant. Multiple Choice Quiz There are a total of 18 correct answers in the following multiple choice questions (some questions have multiple answers that are correct). To attain a perfect score on the quiz, all correct answers must be given, since each correct answer is worth 1 point. For each question, each omitted answer or wrong answer reduces the score by 1 point, and each additional answer beyond the number of answers required reduces the score by 1 point. Percentage score on the quiz is based on the total number of correct answers. 26.1 Which of the following processes use mechanical energy as the principal energy source (may be more than one)? (a) grinding, (b) laser beam machining, (c) milling, (d) ultrasonic machining, (e) water jet cutting, and (f) wire EDM. Answer. (a), (c), (d), and (e). 26.2 Ultrasonic machining can be used to machine both metallic and nonmetallic materials: (a) true or (b) false. Answer. (a) 26.3 Applications of electron beam machining are limited to metallic work materials due to the need for the work to be electrically conductive: (a) true or (b) false. Answer. (b) 26.4 Which one of the following is closest to the temperatures used in plasma arc cutting? (a) 2750C (5000F), (b) 5500C (10,000F), (c) 8300C (15,000F), (d) 11,000C (20,000F), (e) 16,500C (30,000F). Answer. (d) 26.5 Chemical milling is used in which of the following (may be more than one)? (a) drilling holes with high depth-to- diameter ratio, (b) making intricate patterns in sheet metal, (c) removing material to make shallow pockets in metal, (d) removing metal from aircraft wing panels, and (e) cutting of plastic sheets. Answer. (c) and (d). 185 26.6 Etch factor is which of the following in chemical machining (more than one)? (a) A, (b) 1/A, (c) C I t, (d) d/u, and (e) u/d; where A = degree of anisotropy, C = specific removal rate, d = depth of cut, I = current, t = time, and u = undercut. Answer. (a) and (d). 26.7 Of the following processes, which one is noted for the highest material removal rates? (a) electric discharge machining, (b) electrochemical machining, (c) laser beam machining, (d) oxyfuel cutting, (e) plasma arc cutting, (f) ultrasonic machining, and (g) water jet cutting. Answer. (e) 26.8 Which of the following processes would be appropriate to drill a hole with a square cross-section, 0.25 inch on a side and 1-inch deep in a steel workpiece (one best answer)? (a) abrasive jet machining, (b) chemical milling, (c) EDM, (d) laser beam machining, (e) oxyfuel cutting, (f) water jet cutting, and (g) wire EDM. Answer. (c) 26.9 Which of the following processes would be appropriate for cutting a narrow slot, less than 0.015 inch wide, in a 3/8- inch thick sheet of fiber-reinforced plastic (more than one)? (a) abrasive jet machining, (b) chemical milling, (c) EDM, (d) laser beam machining, (e) oxyfuel cutting, (f) water jet cutting, and (g) wire EDM. Answer. (d) and (f). 26.10 Which of the following processes would be appropriate for cutting a hole of 0.003 inch diameter through a plate of aluminum that is 1/16-inch thick (one best answer)? (a) abrasive jet machining, (b) chemical milling, (c) EDM, (d) laser beam machining, (e) oxyfuel cutting, (f) water jet cutting, and (g) wire EDM. Answer. (d) 26.11 Which of the following processes could be used to cut a large piece of 1/2-inch plate steel into two sections (more than one)? (a) abrasive jet machining, (b) chemical milling, (c) EDM, (d) laser beam machining, (e) oxyfuel cutting, (f) water jet cutting, and (g) wire EDM. Answer. (e) and (g). Problems General 26.1 For each of the following applications, identify one or more nontraditional machining processes that might be used, and present arguments to support your selection. Assume that either the part geometry or the work material (or both) preclude the use of conventional machining. (a) A matrix of 0.1 mm (0.004 in) diameter holes in a plate of 3.2 mm (0.125 in) thick hardened tool steel. The matrix is rectangular, 75 by 125 mm (3.0 by 5.0 in) with the separation between holes in each direction = 1.6 mm ( 0.0625 in). (b) An engraved aluminum printing plate to be used in an offset printing press to make 275 by 350 mm (11 by 14 in) posters of Lincoln's Gettysburg address. (c) A through-hole in the shape of the letter L in a 12.5 mm (0.5 in) thick plate of glass. The size of the "L" is 25 by 15 mm (1.0 by 0.6 in) and the width of the hole is 3 mm (1/8 in). (d) A blind-hole in the shape of the letter G in a 50 mm (2.0 in) cube of steel. The overall size of the "G" is 25 by 19 mm (1.0 by 0.75 in), the depth of the hole is 3.8 mm (0.15 in), and its width is 3 mm (1/8 in). 186 Solution: (a) Application: matrix of holes in 0.125 inch thick hardened steel, hole diameter = 0.004 in., separation between holes = 0.0625 in. Possible processes: EBM and LBM can make holes of this size with depth-to-diameter ratios as large as 0.125/0.004 = 31.25. (b) Application: engraved aluminum printing press plate for 11 in. by 14 in posters. Possible process: photochemical engraving; making a negative of the speech and transferring this to either a silk screen or directly to the photoresist would seem to be the most straightforward methods. (c) Application: through-hole in the shape of the letter “L” drilled through 0.5 inch thick plate glass. Possible process: USM works on glass and other brittle non-metallic materials. This is probably the best process. LBM might also work. (d) Application: the letter “G” drilled to a depth of 0.15 in. in block of steel. Possible processes: ECM and EDM would be useful for pocketing operations such as this. 26.2 Much of the work at the Cut-Anything Company involves cutting and forming of flat sheets of fiber-glass for the pleasure boat industry. Manual methods based on portable saws are currently used to perform the cutting operation, but production is slow and scrap rates are high. The foreman says the company should invest in a plasma arc cutting machine, but the plant manager thinks it would be too expensive. What do you think? Justify your answer by indicating the characteristics of the process that make PAC attractive or unattractive in this application. Solution: In plasma arc cutting, the workpart must be an electrically conductive material. Fiber glass is not electrically conductive. PAC is therefore not an appropriate process for this application. 26.3 A furniture company that makes upholstered chairs and sofas must cut large quantities of fabrics. Many of these fabrics are strong and wear-resistant, which properties make them difficult to cut. What nontraditional process(es) would you recommend to the company for this application? Justify your answer by indicating the characteristics of the process that make it attractive. Solution: Water jet cutting would be an ideal process for this application. WJC cuts through fabrics quickly and cleanly, and the process could be readily automated. Electrochemical Machining 26.4 The frontal working area of the electrode is 2000 mm2 in a certain ECM operation in which the applied current = 1800 amps and the voltage = 12 volts. The material being cut is nickel (valence = 2), whose specific removal rate C is given in Table 26.1. (a) If the process is 90% efficient, determine the rate of metal removal in mm3/min. (b) If the resistivity of the electrolyte = 140 ohm-mm, determine the working gap? Solution: (a) From Table 26.1, C = 3.42 x 10-2 mm3/A-s From Eq. (26.6) MRR = frA = (CI/A)A = CI = (3.42 x 10-2 mm3/A-s)(1800 A) = 6156 x 10-2 mm3/s = 61.56 mm3/s = 3693.6 mm3/min At 90% efficiency MRR = 0.9(3693.6 mm3/min) = 3324.2 mm3/min (b) Given resistivity r = 140 ohm-mm, I = EA/gr in Eq. (26.2). Rearranging, g = EA/Ir g = (12 V)(2000 mm2)/(1800 A)(140 ohm-mm) = 0.095 mm 26.5 In an electrochemical machining operation, the frontal working area of the electrode is 2.5 in2. The applied current = 1500 amps, and the voltage = 12 volts. The material being cut is pure aluminum, whose specific removal rate C is indicated in Table 26.1. (a) If the ECM process is 90 percent 187 efficient, determine the rate of metal removal in in3/hr. (b) If the resistivity of the electrolyte = 6.2 ohm-in, determine the working gap? Solution: (a) From Table 26.1, C = 0.000126 in3/A-min. MRR = fr x A = (CI/A)(A) = CI MRR = CI = 0.000126(1500) = 0.189 in3/min at 100% efficiency. At 90% efficiency MRR = 0.189(0.90) = 0.1701 in3/min = 10.206 in3/hr. (b) I = EA/gr; Rearranging, g = EA/Ir = 12(2.5)/(1500 x 6.2) = 0.0032 in. 26.6 A square hole is to be cut using ECM through a plate of pure copper (valence = 1) that us 20 mm thick. The hole is 25 mm on each side, but the electrode that is used to cut the hole is slightly less that 25 mm on its sides to allow for overcut, and its shape includes a hole in its center to permit the flow of electrolyte and to reduce the area of the cut. This tool design results in a frontal area of 200 mm2. The applied current = 1000 amps. Using an efficiency of 95%, determine how long it will take to cut the hole. Solution: From Table 26.1, C = 7.35 x 10-2 mm3/A-s From Eq. (26.6) fr = CI/A = (7.35 x 10-2 mm3/A-s)(1000 A)/(200 mm2) = 0.3675 mm/s At 95% efficiency, fr = 0.95(0.3675 mm/s) = 0.349 mm/s Time to machine = (20 mm)/(0.349 mm/s) = 57.3 s 26.7 A 3.5 in diameter through-hole is to be cut in a block of pure iron (Valence = 2) by electrochemical machining. The block is 2.0 in thick. To speed the cutting process, the electrode tool will have a center hole of 3.0 in which will produce a center core that can be removed after the tool breaks through. The outside diameter of the electrode is undersized to allow for overcut. The overcut is expected to be 0.005 in on a side. If the efficiency of the ECM operation is 90%, what current will be required to complete the cutting operation in 20 minutes? Solution: Electrode frontal gap area A = 0.25(3.52 - 3.02) = 2.553 in2 From Table 26.1, C = 0.000135 in3/A-min. fr = CI/A = 0.000135 I/2.553 = 0.0000529 I in/min at 100% efficiency. At 90% efficiency fr = 0.9(0.0000529 I) = 0.0000476 I in/min. To cut through a 2.0 inch thickness in 20 minutes requires a feed rate fr = 2.0/20 = 0.1 in/min. fr = 0.1 = 0.0000476 I I = 0.1/0.0000476 = 2101 A. Electric Discharge Machining 26.8 An electric discharge machining operation is being performed on tungsten. (a) Determine the amount of metal removed in the operation after one hour at a discharge amperage = 20 amps. (b) If the work material were tin, determine the amount of material removed in the same time. Use metric units and express the answer in mm3. Solution: (a) From Table 4.1, Tm = 3410C for tungsten From Eq. (26.7), MRR = KI/Tm 1.23 = 664(20)/(34101.23) = 13,280/22,146 = 0.5997 mm3/s = 2159 mm3 (b) From Table 4.1, Tm = 232C for tin From Eq. (26.7), MRR = KI/Tm 1.23 = 664(20)/(2321.23) = 13,280/812 = 16.355 mm3/s = 58,878 mm3 26.9 Same as Problem 26.8, except the new material to be compared with tungsten is zinc. Use U.S. Customary units and express the answer in in3. 188 Solution: (a) From Table 4.1, Tm = 6170F for tungsten From Eq. (26.7), MRR = KI/Tm 1.23 = 5.08(20)/(61701.23) = 101.6/45,925 = 0.00221 in.3/s = 0.1327 in.3 (b) From Table 4.1, Tm = 420F for Zinc From Eq. (26.7), MRR = KI/Tm 1.23 = 5.08(20)/(4201.23) = 101.6/1685 = 0.0603 in.3/s = 3.62 in.3 26.10 Suppose the hole in Problem 26.7 were to be cut using EDM rather than ECM. Using a discharge current = 20 amps (which would be typical for EDM), how long would it take to cut the hole? Solution: For EDM, using Eq. (26.7), MRR = 5.08 I/Tm 1.23 (USCS). From Table 4.1, Tm = 2802F for iron. MRR = 5.08(20)/28021.23 = 101.6/17,393 = 0.00584 in3/min Cross-sectional area of tool from previous problem A = 2.553 in2 fr = MRR/A = 0.00584/2.553 = 0.002293 in/min. Time to machine the 2.0 inch thickness = 2.0/0.002293 = 874.3 min = 14.57 hr. 26.11 A metal removal rate of 0.01 in3/min is achieved in a certain EDM operation on a pure iron workpart. What metal removal rate would be achieved on nickel in this EDM operation, if the same discharge current were used? Solution: From Table 4.1, Tm = 2802F for iron. MRR = 5.08 I/28021.23 = 5.08 I/17,393 = 0.000292 I in3/min Given that MRR = 0.01 in3/min. 0.000292 I = 0.01 I = 0.01/0.000292 = 34.24 A. From Table 4.1, Tm = 2651 F for iron. MRR = 5.08(34.24)/26511.23 = 173.93/16,248 = 0.0107 in3/min 26.12 In a wire EDM operation performed on 7 mm thick C1080 steel using a tungsten wire electrode whose diameter = 0.125 mm. Past experience suggests that the overcut will be 0.02 mm, so that the kerf width will be 0.165 mm. Using a discharge current = 10 amps, what is the allowable feed rate that can be used in the operation? Estimate the melting temperature of 0.80% carbon steel from the phase diagram of Figure 6.4. Solution: From Figure 6.4, Tm = 1500C for 1080 steel Using Eq. (26.7), MRR = 664(10)/(15001.23) = 6640/8065 = 0.8233 mm3/s Frontal area of kerf = 0.165(7.0) = 1.155 mm2 fr = 49.4/1.155 = 42.79 mm/min 26.13 A wire EDM operation is to be performed on a slab of 3/4 in thick aluminum using a brass wire electrode whose diameter = 0.005 in. It is anticipated that the overcut will be 0.001 in, so that the kerf width will be 0.007 in. Using a discharge current = 7 amps, what is the expected allowable feed rate that can be used in the operation? Solution: From Table 4.1, Tm = 1220F for aluminum. Using Eq. (27.7), MRR = 5.08(7)/12201.23 = 35.56/6255 = 0.005685 in3/min Frontal area of kerf = 0.75(0.007) = 0.00525 in2 fr = 0.005685/0.00525 = 1.083 in/min. 26.14 A wire EDM operation is used to cut out punch and die components from 25 mm thick tool steel plates. However, in preliminary cuts, the surface finish on the cut edge is poor. What changes in discharge current and frequency of discharges should be made to improve the finish? 189 Solution: As indicated in Figure 26.9(a), surface finish in EDM could be improved by reducing discharge current and increasing frequency of discharges. Chemical Machining 26.15 Chemical milling is used in an aircraft plant to create pockets in wing sections made of an aluminum alloy. The starting thickness of one workpart of interest is 20 mm. A series of rectangular-shaped pockets 12 mm deep are to be formed with dimensions 200 mm by 400 mm. The corners of each rectangle are radiused to 15 mm. The part is an aluminum alloy and the etchant is NaOH. The penetration rate for this combination is 0.024 mm/min and the etch factor is 1.75. Determine: (a) metal removal rate in mm3/min, (b) time required to machine to the specified depth, (c) required dimensions of the opening in the cut and peel maskant to achieve the desired pocket size on the part. Solution: (a) Neglecting the fact that the initial area would be less than the given dimensions of 200 mm by 400 mm, and that the material removal rate (MRR) would therefore increase during the cut as the area increased, A = 200 x 400 – (30 x 30 - (15)2) = 80,000 – 193 = 79,807 mm2 MRR = (0.024 mm/min)(79,807 mm2) = 1915.4 mm3/min. (b) Time to machine (etch) = 12/0.024 = 500 min. = 8.33 hr. (c) Given Fe = 1.75, undercut u = d/Fe = 12/1.75 = 6.86 mm Maskant opening length = L – 2u = 400 – 2(6.86) = 386.28 mm Maskant opening width = W – 2u = 200 – 2(6.86) = 186.28 mm Radius on corners = R – u = 15 – 6.86 = 8.14 mm 26.16 In a chemical milling operation on a flat mild steel plate, it is desired to cut an ellipse-shaped pocket to a depth of 0.4 in. The semiaxes of the ellipse are: a = 9.0 in and b = 6.0 in. A solution of hydrochloric and nitric acids will be used as the etchant. Determine: (a) metal removal rate in in3/hr, (b) time required to machine to depth, (c) required dimensions of the opening in cut and peel maskant required to achieve the desired pocket size on the part. Solution: (a) Neglecting the fact that the initial area would be less than the given dimensions of 9 in. by 6 in., and that the material removal rate (MRR) would therefore increase during the cut as the area increased, area of an ellipse A = ab = (9.0)(6.0) = 54= 169.65 in.2 MRR = (0.001 in/min)(169.65 in2) = 0.16965 in3/min = 10.18 in3/hr (b) Time to machine (etch) = 0.4/0.001 = 400 min. = 6.67 hr. (c) Given Fe = 2.0, undercut u = d/Fe = 0.4/2.0 = 0.2 mm Maskant opening a’ = a – u = 9.0 – 0.2 = 8.8 in Maskant opening b’ = b – u = 6.0 – 0.2 = 5.8 in 26.17 In a certain chemical blanking operation, a sulfuric acid etchant is used to remove material from a sheet of magnesium alloy. The sheet is 0.25 mm thick. The screen resist method of masking was used to permit high production rates to be achieved. As it turns out, the process is producing a large proportion of scrap. Specified tolerances of 0.025 mm are not being achieved. The foreman in the CHM department complains that there must be something wrong with the sulfuric acid. "Perhaps the concentration is incorrect," he suggests. Analyze the problem and recommend a solution. Solution: The problem in this chemical blanking operation is that the screen resist method of masking cannot achieve the tolerances specified. The photoresist method should have been used, and the process should be changed over to adopt this method. 190 26.18 In a chemical blanking operation, stock thickness of the aluminum sheet is 0.015 in. The pattern to be cut out of the sheet is a hole pattern, consisting of a matrix of 0.100 in diameter holes. If photochemical machining is used to cut these holes, and contact printing is used to make the resist (maskant) pattern, determine the diameter of the holes that should be used in the pattern. Solution: From Table 26.2, Fe = 1.75. In chemical blanking, etching will occur on both sides of the part. Therefore, the effective hole depth on each side = one-half of the stock thickness = 0.015/2 = 0.0075 in. Undercut u = 0.0075/1.75 = 0.0043 in. Diameter of opening = 0.100 - 2(0.0043) = 0.0914 in. 191 27 HEAT TREATMENT OF METALS Review Questions 27.1 Why are metals heat treated? Answer. Metals are heat treated to effect metallurgical changes that beneficially alter properties. 27.2 Identify the important reasons why metals are annealed. Answer. The purposes of annealing include: (1) control properties, (2) reduce brittleness and improve toughness, (3) recrystallize cold-worked metals, and (4) relieve stresses from prior metal working. 27.3 What is the most important heat treatment for hardening steels? Answer. Martensite formation by heating steel into the austenite region and quenching. 27.4 What is the mechanism by which carbon strengthens steel during heat treatment? Answer. If the steel is heat treated, martensite is formed which depends on the presence of carbon to create the nonequilibrium structure of this phase. 27.5 What information is conveyed by the TTT curve? Answer. The TTT curve indicates what phases in the iron- carbon phase diagram will be produced under various conditions of cooling. 27.6 What function is served by tempering? Answer. Tempering involves heating and soaking of martensite for about one hour, followed by slow cooling to reduce brittleness, relieve stresses, and increase toughness and ductility. 27.7 Define hardenability? Answer. Hardenability is the relative capacity of a steel to be hardened by transformation to martensite. 27.8 Name some of the elements which have the greatest effect on the hardenability of steel. Answer. Important hardenability elements are: chromium, manganese, molybdenum, and nickel. 27.9 Indicate how the hardenability alloying elements in steel affect the TTT curve. Answer. The hardenability alloying elements operate by pushing the nose of the TTT curve to the right, thereby permitting slower cooling rates for conversion of austenite to martensite. 27.10 Define precipitation hardening? Answer. Precipitation hardening is a heat treatment in which very fine particles (precipitates) are formed so that dislocation movement is blocked and the metal is thus strengthened and hardened. 27.11 How does carburizing work? Answer. Carburizing adds carbon to the surface of low-C steel, thereby transforming the surface into high-C steel for grater hardening potential. 27.12 Identify the selective surface hardening methods. 192 Answer. The selective surface hardening methods include: flame hardening, induction hardening, high-frequency (HF) resistance heating, electron beam (EB) heating, and laser beam (LB) heating. Multiple Choice Quiz There are a total of 14 correct answers in the following multiple choice questions (some questions have multiple answers that are correct). To attain a perfect score on the quiz, all correct answers must be given, since each correct answer is worth 1 point. For each question, each omitted answer or wrong answer reduces the score by 1 point, and each additional answer beyond the number of answers required reduces the score by 1 point. Percentage score on the quiz is based on the total number of correct answers. 27.1 Which of the following are the usual objectives of heat treatment (more than one)? (a) increase hardness, (b) increase toughness, (c) recrystallization of the metal, (d) reduce brittleness, (e) reduce density, or (f) relieve stresses. Answer. (a), (b), (c), (d), and (f). 27.2 Of the following quenching media, which one produces the most rapid cooling rate? (a) air, (b) brine, (c) oil, or (d) pure water. Answer. (b) 27.3 On which one of the following metals can the treatment called austenitizing be performed? (a) aluminum alloys, (b) brass, (c) copper alloys, or (d) steel. Answer. (d) 27.4 The treatment in which the brittleness of martensite is reduced is called which one of the following? (a) aging, (b) annealing, (c) austenitizing, (d) normalizing, (e) quenching, or (f) tempering. Answer. (f) 27.5 The Jominy end-quench test is designed to indicate which one of the following? (a) cooling rate, (b) ductility, (c) hardenability, (d) hardness, or (e) strength. Answer. (c) The reader might be tempted to select (d) because the Jominy test indicates hardness; however, the reason for measuring hardness in the Jominy test is to measure hardenability. 27.6 In precipitation hardening, the hardening and strengthening of the metal occurs in which one of the following steps (one answer only)? (a) aging, (b) quenching, or (c) solution treatment. Answer. (a) 27.7 Which one of the following surface hardening treatments is the most common? (a) boronizing, (b) carbonitriding, (c) carburizing, (d) chromizing, or (e) nitriding. Answer. (c) 27.8 Which of the following are selective surface hardening methods (more than one)? (a) electron beam heating, (b) fluidized bed furnaces, (c) induction heating, (d) laser beam heating, or (e) vacuum furnaces. Answer. (a), (c), and (d). 193 28 CLEANING AND SURFACE TREATMENTS Review Questions 28.1 What are some of the important reasons why manufactured parts must be cleaned? Answer. The reasons include: (1) to prepare the surface for subsequent industrial processing, (2) to improve hygiene conditions, (3) to remove contaminants which might chemically react with the surface; and (4) to enhance product appearance and performance. 28.2 Mechanical surface treatments are often performed for reasons other than or in addition to cleaning. What are the reasons? Answer. Reasons for mechanical surface treatments include: deburring, improving smoothness, adding luster, and enhancing surface properties. 28.3 What are the basic types of contaminants that must be cleaned from metallic surfaces in manufacturing? Answer. Basic contaminant types are: (1) oil and grease, (2) solid particles, such as metal chips, abrasive grits, shop dirt, and dust, (3) buffing and polishing compounds, and (4) oxide films, rust, and scale. 28.4 Identify some of the mechanical cleaning methods. Answer. Mechanical cleaning methods include: belt grinding, honing, lapping, polishing, buffing, and mass finishing (such as barrel tumbling). These method also accomplish other functions such as deburring and surface finish improvement. 28.5 In addition to surface cleaning, what is the main function performed by shot peening? Answer. Shot peening is primarily used to improve the fatigue strength of metals by introducing cold working the metallic surface. 28.6 Name some of the important chemical cleaning methods. Answer. The chemical cleaning methods can be categorized as follows: (1) alkaline cleaning, (2) emulsion cleaning, (3) solvent cleaning, (4) acid cleaning and pickling, and (5) ultrasonic cleaning. 28.7 What is meant by the term mass finishing? Answer. In mass finishing, parts are mechanically cleaned and deburred in bulk, usually in a barrel by the mixing action of an abrasive media. 28.8 What is the difference between diffusion and ion implantation? Answer. Diffusion coating is a diffusion process in which atoms or molecules move across a boundary between two contacting materials. Ion implantation produces a similar result, but the process involves penetration of high- velocity ions into the surface of a substrate material. 28.9 What is calorizing? Answer. Calorizing is the diffusion of aluminum into carbon steel, alloy steels, and the alloys of nickel and cobalt. The process is also known as aluminizing. 194 Multiple Choice Quiz There are a total of 16 correct answers in the following multiple choice questions (some questions have multiple answers that are correct). To attain a perfect score on the quiz, all correct answers must be given, since each correct answer is worth 1 point. For each question, each omitted answer or wrong answer reduces the score by 1 point, and each additional answer beyond the number of answers required reduces the score by 1 point. Percentage score on the quiz is based on the total number of correct answers. 28.1 Reasons why workparts must be cleaned include which of the following (more than one)? (a) for better appearance, (b) to enhance mechanical properties of the surface, (c) to improve hygiene conditions for worker, (d) to prepare the surface for subsequent processing, or (e) to remove contaminants that might chemically attack the surface. Answer. (a), (c), (d), and (e). 28.2 Which of the following chemicals is associated with alkaline cleaning (more than one)? (a) borax, (b) sodium hydroxide, (c) sulfuric acid, or (d) trichlorethylene. Answer. (a), (b). 28.3 Shot peening is a mechanical cleaning method used primarily to remove surface scale from metallic parts: (a) true or (b) false. Answer. (b) Principal function is to cold work the surface to improve fatigue strength. 28.4 In sand blasting, which one of the following abrasives is used? (a) Al2O3, (b) crushed nut shells, (c) nylon beads, (d) SiC, or (e) SiO2. Answer. (e) 28.5 The abrasive media used in mass finishing, such as barrel tumbling, include which of the following (more than one)? (a) Al2O3, (b) corundum, (c) emery, (d) limestone, and (e) SiC. Answer. (a), (b), (c), (d), and (e). 28.6 Which of the following processes generally produces a deeper penetration of atoms in the impregnated surface? (a) diffusion or (b) ion implantation. Answer. (a) 28.7 Calorizing is the same as which of the following? (a) aluminizing, (b) doping, (c) hot sand blasting, or (d) siliconizing. Answer. (a) 28.8 Carburizing involves which one of the following? (a) acid pickling, (b) blast finishing, (c) diffusion, (d) tumbling, or (e) vapor degreasing. Answer. (c) 195 29 COATING AND DEPOSITION PROCESSES Review Questions 29.1 Why are metals coated? Answer. (1) Corrosion protection, (2) enhance appearance, (3) provide a specified color, and (4) prepare surface for subsequent processing. 29.2 Identify the most common types of coating processes. Answer. The common coating processes are: (1) plating, (2) chemical surface treatments, such as anodizing, (3) vapor deposition processes such as PVD and CVD, (4) organic coating - painting, (5) porcelain enameling, and (6) thermal and mechanical treatments. 29.3 What are the many reasons why a metallic surface is plated? Answer. The reasons are: (1) corrosion protection of the substrate metal, (2) attractive appearance, (3) wear resistance, (4) increased electrical conductivity, (5) improved solderability, and (6) enhanced lubricity of the surface. 29.4 What is meant by the term cathode efficiency in electroplating? Answer. The cathode efficiency is the actual amount of metal deposited on the surface relative to the theoretical amount given by Eq. (33.1). 29.5 What are the two basic mechanisms of corrosion protection? Answer. The mechanisms are (1) barrier protection, in which the coating simply covers the substrate to protect it, and (2) sacrificial protection, in which the coating metal corrodes sacrificially to protect the substrate. 29.6 What is the most commonly plated substrate metal? Answer. Steel. 29.7 One of the mandrel types in electroforming is a solid mandrel. How is the part removed from a solid mandrel? Answer. A solid mandrel has certain geometric features, such as a taper, that permit the part to be removed. Parts are also sometimes removed by taking advantage of a difference in coefficient of thermal expansion. 29.8 How does electroless plating differ from electrochemical plating? Answer. Electroless plating uses only chemical reactions to form the plating; electroplating uses electrolysis. 29.9 What is a conversion coating? Answer. A conversion coating is a thin coating produced by chemical reaction of the metallic surface. The most common conversion coatings are phosphates, chromates, and oxides. 29.10 How does anodizing differ from other conversion coatings? Answer. Anodizing uses electrochemical processing methods to convert the metallic surface. Best example is aluminum anodizing. 196 29.11 What is physical vapor deposition? Answer. Physical vapor deposition (PVD) refers to a family of processes in which a material is converted to its vapor phase in a vacuum chamber and condensed onto a substrate surface as a very thin film. 29.12 What is the difference between physical vapor deposition (PVD) and chemical vapor deposition (CVD)? Answer. In PVD, the coating vapors are synthesized by heating the coating material and allowing it to condense as a thin film on the surface of the workpart. In CVD a coating is formed on a heated substrate by the chemical reaction or dissociation of vapors and/or gases; the reaction product nucleates and grows on the substrate surface. 29.13 What are some of the applications of PVD? Answer. PVC applications include: decorative coatings on trophies and automotive trim, antireflection coatings on optical lenses, deposition of metal in electronic connections, and cutting tool coatings (e.g., TiN). 29.14 Name the three basic types of PVD. Answer. The three types are: (1) vacuum evaporation, (2) sputtering, and (3) ion plating. 29.15 What is a commonly used coating material deposited by PVD onto cutting tools? Answer. Titanium nitride (TiN). Titanium carbide (TiC) and aluminum oxide (Al2O3) might also be mentioned. 29.16 Define sputtering yield? Answer. Sputtering yield is defined as the number of atoms ejected from the surface of a substance per ion bombarding the surface. 29.17 What are some of the advantages of chemical vapor deposition? Answer. Advantages include: (1) capability to deposit refractory materials at temperatures below their melting or sintering temperatures; (2) grain size control; (3) process is performed at atmospheric pressure; and (4) good bonding to substrate surface. 29.18 What are the two most common titanium compounds that are coated onto cutting tools by chemical vapor deposition? Answer. TiC and TiN. 29.19 Identify the four major types of ingredients in organic coatings? Answer. The major ingredients are: (1) binder, which are polymers, (2) dyes or pigments, which provide color, (3) solvents, and (4) additives such as surfactants and plasticizers. 29.20 What is meant by the term transfer efficiency in organic coating technology? Answer. Transfer efficiency indicates how much of the organic coating liquid reaches the target surface. 29.21 Describe the principal methods by which organic coatings are applied to a surface. Answer. The main methods include: brushing and rolling, spraying, immersion (dip coating), and flow coating. These methods are described in Article 33.4.2. 29.22 The terms drying and curing have different meanings; indicate the distinction. 197 Answer. Drying means evaporation of solvents in the organic coating liquid. Curing involves a chemical change in the organic resin (polymerization and/or cross-linking) which hardens the coating. 29.23 In porcelain enameling, what is frit? Answer. Frit is glassy porcelain prepared as fine particles (powders) by crushing and milling. 29.24 What does the term mechanical galvanizing refer to? Answer. Mechanical galvanizing refers to the mechanical plating of zinc onto a substrate, with thicknesses up to 0.003 in. Multiple Choice Quiz There are a total of 17 correct answers in the following multiple choice questions (some questions have multiple answers that are correct). To attain a perfect score on the quiz, all correct answers must be given, since each correct answer is worth 1 point. For each question, each omitted answer or wrong answer reduces the score by 1 point, and each additional answer beyond the number of answers required reduces the score by 1 point. Percentage score on the quiz is based on the total number of correct answers. 29.1 Which one of the following plate metals produces the hardest surface on a metallic substrate? (a) cadmium, (b) chromium, (c) copper, (d) nickel, or (e) tin. Answer. (b) 29.2 Which one of the following terms is used in connection with dip coating of lead onto a substrate such as sheet steel? (a) aluminizing, (b) anodizing, (c) conversion coating, (d) galvanizing, or (e) terneplating. Answer. (e) 29.3 Which one of the following plating metal is associated with the term galvanizing? (a) iron, (b) lead, (c) steel, (d) tin, or (e) zinc. Answer. (e) 29.4 Which of the following is most typical of the thickness of an electroplated coating (choose either of two acceptable answers)? (a) 0.0001 in, (b) 0.001 in, (c) 0.010 in, or (d) 0.100 in. Answer. either (a) or (b) is acceptable 29.5 Which of the following processes involves electrochemical reactions (more than one)? (a) anodizing, (b) chromate coatings, (c) electroless plating, (d) electroplating, or (e) phosphate coatings. Answer. (a) and (d). 29.6 With which one of the following metals is anodizing most commonly associated (one answer)? (a) aluminum, (b) magnesium, (c) steel, (d) titanium, or (e) zinc. Answer. (a) 29.7 Sputtering is a form of which one of the following? (a) chemical vapor deposition, (b) defect in arc welding, (c) diffusion, (d) ion implantation, or (e) physical vapor deposition. Answer. (e) 198 29.8 Which of the following gases is the most commonly used in sputtering and ion plating? (a) argon, (b) chlorine, (c) neon, (d) nitrogen, or (e) oxygen. Answer. (a) 29.9 The Mond process is used for which one of the following? (a) chemical vapor deposition of silicon nitride onto silicon, (b) an electroplating process, (c) physical vapor deposition for coating TiN onto cutting tools, or (d) reducing nickel carbonyl to metallic Ni. Answer. (d) 29.10 Which of the following thin film processes is most common in semiconductor processing? (a) chemical vapor deposition or (b) physical vapor deposition. Answer. (a) 29.11 The principal methods of applying powder coatings are which of the following (select two best answers)? (a) brushing, (b) electrostatic spraying, (c) fluidized bed, (d) immersion, and (e) roller coating. Answer. (b) and (c). 29.12 Porcelain enamel is applied to a surface in which one of the following forms? (a) liquid emulsion, (b) liquid solution, (c) molten liquid, or (d) powders. Answer. (d) 29.13 Which of the following are alternative names for thermal spraying (more than one answer)? (a) flexible overlay process, (b) hard facing, (c) metallizing, or (d) metal spraying. Answer. (c) and (d). 29.14 Hard facing utilizes which one of the following basic processes? (a) arc welding, (b) brazing, (c) dip coating, (d) electroplating, (e) mechanical deformation to work harden the surface. Answer. (a) Problems Electroplating 29.1 What volume (cm3) and weight (g) of zinc will be deposited onto a cathodic workpart if 10 amps of current are applied for one hour? Solution: From Table 29.1, C = 4.75 x 10-2 mm3/A-s, cathode efficiency E = 95%. Volume V = ECIt = 0.95(4.75 x 10-2 mm3/A-s)(10 A)(1 hr)(3600 s/hr) = 1624.5 mm3 = 1.6245 cm3 Density of zinc from Table 4.1 = 7.15 g/cm3. Weight W = 1.6245(7.15) = 11.615 g 29.2 A sheetmetal steel part with surface area A = 100 cm2 is to be zinc plated. What average plating thickness will result if 15 amps are applied for 12 minutes in a chloride electrolyte solution? Solution: From Table 29.1, C = 4.75 x 10-2 mm3/A-s, cathode efficiency E = 95%. Volume V = ECIt = 0.95(4.75 x 10-2 mm3/A-s)(15 A)(12 min)(60 s/min) = 487.35 mm3 Area A = 100 cm2 = 10,000 mm2 Plating thickness d = 487.35 mm3/10,000 mm2 = 0.049 mm 29.3 A sheetmetal steel part with surface area A = 15.0 in2 is to be chrome plated. What average plating thickness will result if 15 amps are applied for 10 minutes in a chromic acid-sulfate bath? 199 Solution: From Table 29.1, C = 0.92 x 10-4 in3/A-min, cathode efficiency E = 15%. Volume V = ECIt = 0.15(0.92 x 10-4)(15)(10) = 0.00207 in3. Plating thickness d = 0.00207/15 = 0.000138 in. 29.4 Twenty-five jewelry pieces, each with a surface area = 0.5 in2 are to be gold plated in a batch plating operation. (a) What average plating thickness will result if 8 amps are applied for 10 min in a cyanide bath? (b) What is the value of the gold that will be plated if one ounce of gold is valued at $300? The density of gold = 0.698 lb/in3. Solution: (a) From Table 29.1, C = 3.87 x 10-4 in3/A-min, cathode efficiency E = 80%. Volume V = ECIt = 0.80(3.87 x 10-4)(8)(10) = 0.02477 in3. With Q = 25 pieces and average area per piece = 0.5 in2, total area A = 25(0.5) = 12.5 in2 Plating thickness d = 0.02477/12.5 = 0.00198 in. (b) Given density for gold = 0.698 lb/in3 Weight of plated gold = (0.698 lb/in3)(0.02477 in3) = 0.01729 lb = 0.277 oz. At $300/oz, value of plated gold = $300(0.277) = $82.99 29.5 A part made of sheet steel is to be nickel plated. The part is a rectangular flat plate which is 0.075 cm thick and whose face dimensions are 14 cm by 19 cm. The plating operation is carried out in an acid sulfate electrolyte, using a current I = 20 amps for a duration t = 30 min. Determine the average thickness of the plated metal resulting from this operation. Solution: From Table 29.1, C = 3.42 x 10-2 mm3/A-s, cathode efficiency E = 95%. Volume V = ECIt = 0.95(3.42 x 10-2 mm3/A-s)(20 A)(30 min)(60 s/min) = 1169.6 mm3. Area A = 2(19 x 14) + 0.075 x 2(19 + 14) = 536.95 cm2 = 53,695 mm2 Plating thickness d = 1169.6/53,695 = 0.022 mm 29.6 A steel sheetmetal part has total surface area A = 36 in2. How long will it take to deposit a copper plating (assume valence = +1) of thickness = 0.001 in onto the surface if 15 amps of current are applied? Solution: From Table 29.1, C = 2.69 x 10-4 in3/A-min, cathode efficiency E = 98%. Required volume of plate metal = 36(0.001) = 0.036 in3 Plated volume V = ECIt = 0.98(2.69 x 10-4 in3/A-min)(15 A) t = 0.003954 t in3. 0.003954 t = 0.036 t = 0.036/0.003954 = 9.1 min. 29.7 Increasing current is applied to a workpart surface in an electroplating process according to the relation I = 12.0 + 0.2t, where I = current, amps; and t = time, min. The plating metal is chromium, and the part is submersed in the plating solution for a duration of 20 min. What volume of coating will be applied in the process? Solution: From Table 29.1, C = 0.92 x 10-4 in3/A-min, cathode efficiency E = 15%. Plated volume V = ECIdt = EC(12 + 0.2t)dt = EC(12t + 0.1t2) over the range 0 to 20 min. V = 0.15(0.92 x 10-4)(12 x 20 + 0.1(20)2) = 0.00386 in3 29.8 A batch of 100 parts are to be nickel plated in a barrel plating operation. The parts are identical, each with a surface area A = 7.8 in2. The plating process applies a current I = 120 amps, and the batch takes 40 minutes to complete. Determine the average plating thickness on the parts. Solution: From Table 29.1, C = 1.25 x 10-4 in3/A-min, cathode efficiency E = 95%. Volume V = ECIt = 0.95(1.25 x 10-4)(120)(40) = 0.57 in3. Area A = 100(7.8) = 780 in2 Plating thickness d = 0.57/780 = 0.00073 in. 200 29.9 A batch of 40 identical parts are to be chrome plated using racks. Each part has a surface are = 22.7 cm2. If it is desired to plate an average thickness = 0.010 mm on the surface of each part, how long should the plating operation be allowed to run at a current = 80 amps? Solution: From Table 29.1, C = 2.5 x 10-2 mm3/A-s, cathode efficiency E = 15%. Volume V = ECIt = 0.15(2.5 x 10-2 mm3/A-s)(80 A) t = 0.3 t mm3 With Q = 40 pieces and average area per piece = 22.7 mm2, total area A = 40(22.7) = 908 cm2 = 90,800 mm2 Plating thickness d = V/A = (0.3 t mm3)/(90,800 mm2) = 0.03304(10-4) t mm Given that d = 0.010 mm, 0.03304(10-4) t = 0.010 Thus, t = 0.010/0.03304(10-4) = 0.3027 x 104 = 3027 s = 50.44 min. 201 30 FUNDAMENTALS OF WELDING Review Questions 30.1 What are the advantages and disadvantages of welding compared to other types of assembly operations? Answer. Advantages: (1) provides a permanent joint, so the parts are joined permanently; (2) joint strength is typically as high as strength of base metals; (3) most economical in terms of material usage; (4) versatile in terms of where it can be accomplished. Disadvantages: (1) usually performed manually, so labor cost is high and the skilled labor to perform it is sometimes scarce; (2) welding is inherently dangerous; (3) difficult to disassemble; (4) quality defects sometimes difficult to detect. 30.2 What were the two discoveries of Sir Humphrey Davy that led to the development of modern welding technology? Answer. (1) electric arc and (2) acetylene gas. 30.3 What is meant by the term faying surface? Answer. The faying surfaces are the contacting surfaces in a welded joint. 30.4 Define the term fusion weld. Answer. A fusion weld is a weld in which the metal surfaces have been melted in order to cause coalescence. 30.5 What is the fundamental difference between a fusion weld and a solid state weld? Answer. In a fusion weld, the metal is melted. In a solid state weld, the metal is not melted. 30.6 What is an autogenous weld? Answer. It is a fusion weld made without the addition of filler metal. 30.7 Discuss the reasons why most welding operations are inherently dangerous. Answer. Most welding operations are carried out at high temperatures that can cause serious burns on skin and flesh. In gas welding, the fuels are a fire hazard. In arc welding and resistance welding, the high electrical energy can cause shocks which are fatal to the worker. In arc welding, the electric arc emits intense ultraviolet radiation which can cause blinding. Other hazards include sparks, smoke, fumes, and weld spatter. 30.8 What is the difference between machine welding and automatic welding? Answer. An automatic welding operation uses a weld cycle controller which regulates the arc movement and workpiece positioning; whereas in machine welding, a human worker must continuously control the arc and the relative movement of the welding head and the workpart. 30.9 Name and sketch the five joint types. Answer. Five joint types are: (1) butt, (2) corner, (3) lap, (4) tee, (5) edge. See Figure 28.3 in text for sketches. 30.10 Define and sketch a fillet weld? 202 Answer. A fillet weld is a weld joint of approximately triangular cross-section used to fill in the edges of corner, lap, and tee joints. See Figure 28.4 in text for sketch. 30.11 Define and sketch a groove weld? Answer. A groove weld is a weld joint used to fill in the space between the adjoining edges of butt and other weld types except lap. See Figure 28.5 in text for sketch. 30.12 Why is a surfacing weld different from the other weld types? Answer. Because it does not join to distinct parts, but instead adds only filler metal to a surface. 30.13 What is the difference between a continuous weld and an intermittent weld as the terms apply to a fillet weld of a lap joint? Answer. A continuous weld would be made along the entire length of the fillet weld, whereas an intermittent weld would only fill the joint along certain portions (usually equally spaced) of the total length. 30.14 Why is it desirable to use energy sources for welding that have high heat densities? Answer. Because the heat is concentrated in a small region for greatest efficiency and minimum metallurgical damage. 30.15 What is the unit melting energy in welding, and what are the factors on which it depends? Answer. The unit melting energy is the amount of heat energy required to melt one cubic inch or one cubic mm of metal. 30.16 Define and distinguish the two terms heat transfer efficiency and melting efficiency in welding. Answer. Heat transfer efficiency is the ratio of the actual heat received at the work surface divided by the total heat generated by the source. Melting efficiency is the ratio of heat required for melting divided by the heat received at the work surface. 30.17 What is epitaxial grain growth, and how is this form of solidification different from that which occurs in casting? Answer. Epitaxial grain growth occurs when atoms from the molten pool solidify on already existing lattice sites of the adjacent solid base metal. 30.18 What is the heat affected zone (HAZ) in a fusion weld? Answer. The HAZ is a region of base metal surrounding the fusion zone in which melting has not occurred, but temperatures from welding were high enough to cause solid state microstructural changes. Multiple Choice Quiz There are a total of 11 correct answers in the following multiple choice questions (some questions have multiple answers that are correct). To attain a perfect score on the quiz, all correct answers must be given, since each correct answer is worth 1 point. For each question, each omitted answer or wrong answer reduces the score by 1 point, and each additional answer beyond the number of answers required reduces the score by 1 point. Percentage score on the quiz is based on the total number of correct answers. 203 30.1 Welding can only be performed on metals that have the same melting point; otherwise, the metal with the lower melting temperature always melts while the other metal remains solid: (a) true, (b) false. Answer. (b) Welding can be accomplished between certain combinations of dissimilar metals. Both metals melt. 30.2 A fillet weld can be used to join which of the following joint types (more than one): (a) butt, (b) corner, (c) lap, (d) tee. Answer. (b), (c), and (d). 30.3 A fillet weld has a cross-sectional shape that is approximately which one of the following? (a) rectangular, (b) round, (c) square, or (d) triangular. Answer. (d) 30.4 Groove welds are most closely associated with which one of the following joint types: (a) butt, (b) corner, (c) edge, (d) lap, (e) tee. Answer. (a) 30.5 A flange weld is most closely associated with which one of the following joint types: (a) butt, (b) corner, (c) edge, (d) lap, (e) tee. Answer. (c) 30.6 For metallurgical reasons, it is desirable to melt the weld metal with minimum energy input. Which one of the following heat sources is most consistent with this objective? (a) high power, (b) high power density, (c) low power, or (d) low power density. Answer. (b) 30.7 The amount of heat required to melt a given volume of metal depends strongly on which of the following properties (more than one)? (a) coefficient of thermal expansion, (b) heat of fusion, (c) melting temperature, (d) modulus of elasticity, or (e) thermal conductivity. Answer. (b) and (c) 30.8 Weld failures always occur in the fusion zone of the weld joint, since this is the part of the joint that has been melted: (a) true, (b) false. Answer. (b) Failures also occur in the heat affected zone because metallurgical damage often occurs in this region. Problems Joint design 30.1 Prepare sketches showing how the part edges would be prepared and aligned with each other and also showing the weld cross-section for the following welds: (a) square groove weld, both sides, for a butt weld; (b) single fillet weld for a lap joint; (c) single fillet weld for tee joint; and (d) double U-groove weld for a butt weld. Solution: (a) Square groove weld as in Figure 30.5(a), but both sides as in Figure 30.5(f). (b) Similar to Figure 30.4(c) but one side only. (c) Same as Figure 30.4(d) but one side only. (d) U-groove weld as in Figure 30.5(d) but both sides as in Figure 30.5(f). 204 Power density 30.2 A heat source can transfer 3000 J/sec to a metal part surface. The heated area is circular, and the heat intensity decreases as the radius increases, as follows: 60% of the heat is concentrated in a circular area that is 3 mm in diameter. Is the resulting power density enough to melt metal? Solution: Area A = (3.0)2/4 = 7.0686 mm2 Power P = 0.60(3000) = 1800 J/s = 1800 W. Power density PD = 1800 W/7.0686 mm2 = 255 W/mm2. This power density is sufficient for welding. 30.3 A welding heat source is capable of transferring 150 Btu/min to the surface of a metal part. The heated area is approximately circular, and the heat intensity decreases with increasing radius as follows: 50% of the power is transferred within a circle of diameter = 0.1 inch, and 75% is transferred within a concentric circle of diameter = 0.25 inch. What is the power densities in: (a) the 0.1 inch diameter inner circle and (b) the 0.25 inch diameter ring that lies around the inner circle? (c) Are these power densities sufficient for melting metal? Solution: (a) Area A = (0.1)2/4 = 0.00785 in2 150 Btu/min = 2.5 Btu/sec. Power P = 0.50(2.5) = 1.25 Btu/sec Power density PD = (1.25 Btu/sec)/0.00785 in2 = 159 Btu/sec-in2 (b) A = (0.252 - 0.12)/4 = 0.0412 in2 Power P = (0.75 - 0.50)(2.5) = 0.625 Btu/sec Power density PD = (0.625 Btu/sec)/0.0412 in2 = 15.16 Btu/sec-in2 (c) Power densities are sufficient certainly in the inner circle and probably in the outer ring for welding. Unit melting energy 30.4 Compute the unit energy for melting for the following metals: (a) aluminum and (b) plain low carbon steel. Solution: (a) From Table 30.2, Tm for aluminum = 930 K (1680 R) Eq. (30.2) for SI units: Um = 3.33 x 10-6 Tm 2 Um = 3.33 x 10-6 (930)2 = 2.88 J/mm3 Eq. (30.2) for USCS units: Um = 1.467 x 10-5 Tm 2 Um = 1.467 x 10-5 (1680)2 = 41.4 Btu/in3 (b) From Table 30.2, Tm for plain low carbon steel = 1760 K (3160 R) Eq. (30.2) for SI units: Um = 3.33 x 10-6 Tm 2 Um = 3.33 x 10-6 (1760)2 = 10.32 J/mm3 Eq. (30.2) for USCS units: Um = 1.467 x 10-5 Tm 2 Um = 1.467 x 10-5 (3160)2 = 146.5 Btu/in3 30.5 Compute the unit energy for melting for the following metals:(a) copper and (b) titanium. Solution: (a) From Table 30.2, Tm for copper = 1350 K (2440 R) Eq. (30.2) for SI units: Um = 3.33 x 10-6 Tm 2 Um = 3.33 x 10-6 (1350)2 = 6.07 J/mm3 Eq. (30.2) for USCS units: Um = 1.467 x 10-5 Tm 2 Um = 1.467 x 10-5 (2440)2 = 87.3 Btu/in3 (b) From Table 30.2, Tm for titanium = 2070 K (3730 R) Eq. (30.2) for SI units: Um = 3.33 x 10-6 Tm 2 Um = 3.33 x 10-6 (2070)2 = 14.27 J/mm3 Eq. (30.2) for USCS units: Um = 1.467 x 10-5 Tm 2 Um = 1.467 x 10-5 (3730)2 = 204.1 Btu/in3 30.6 Make the calculations and plot on linearly scaled axes the relationship for unit melting energy as a function of temperature. Use temperatures as follows to construct the plot: 250C, 500C, 750C, 205 1000C, 1500C, and 2000C. On the plot, mark the positions of some of the welding metals in Table 30.2. Solution: Eq. (30.2) for SI units: Um = 3.33 x 10-6 Tm 2. The plot is based on the following calculated values. The plot is left as a student exercise. For Tm = 250°C = (250 + 273) = 523K: Um = 3.33 x 10-6 (523)2 = 0.91 J/mm3 For Tm = 500°C = (500 + 273) = 773K: Um = 3.33 x 10-6 (773)2 = 1.99 J/mm3 For Tm = 750° C = (750 + 273) = 1023K: Um = 3.33 x 10-6 (1023)2 = 3.48 J/mm3 For Tm = 1000°C = (1000 + 273) = 1273K: Um = 3.33 x 10-6 (1273)2 = 5.40 J/mm3 For Tm = 1500°C = (1500 + 273) = 1773K: Um = 3.33 x 10-6 (1773)2 = 10.47 J/mm3 For Tm = 2000°C = (2000 + 273) = 2273K: Um = 3.33 x 10-6 (2273)2 = 17.20 J/mm3 30.7 Make the calculations and plot on linearly scaled axes the relationship for unit melting energy as a function of temperature. Use temperatures as follows to construct the plot: 500F, 1000F, 1500F, 2000F, 2500F, 3000F, and 3500F. On the plot, mark the positions of some of the welding metals in Table 30.2. Solution: Eq. (30.2) for USCS units: Um = 1.467 x 10-5 Tm 2. The plot is based on the following calculated values. The plot is left as a student exercise. For Tm = 500°F = (500 + 460) = 960R: Um = 1.467 x 10-5 (960)2 = 13.5 Btu/in3 For Tm = 1000°F = (1000 + 460) = 1460R: Um = 1.467 x 10-5 (1460)2 = 31.3 Btu/in3 For Tm = 1500°F = (1500 + 460) = 1960R: Um = 1.467 x 10-5 (1960)2 = 56.4 Btu/in3 For Tm = 2000°F = (2000 + 460) = 2460R: Um = 1.467 x 10-5 (2460)2 = 88.8 Btu/in3 For Tm = 2500°F = (2500 + 460) = 2960R: Um = 1.467 x 10-5 (2960)2 = 128.5 Btu/in3 For Tm = 3000°F = (3000 + 460) = 3460R: Um = 1.467 x 10-5 (3460)2 = 175.6 Btu/in3 For Tm = 3500°F = (3500 + 460) = 3960R: Um = 1.467 x 10-5 (3960)2 = 230.0 Btu/in3 30.8 A fillet weld has a cross-sectional area Aw = 20.0 mm2 and is 200 mm long. (a) What quantity of heat (in joules) is required to accomplish the weld, if the metal to be welded is austenitic stainless steel? (b) How much heat must be generated at the welding source, if the heat transfer efficiency = 0.8 and the melting efficiency = 0.6? Solution: (a) Eq. (30.2) for SI units: Um = 3.33 x 10-6 Tm 2 From Table 30.2, Tm for austenitic stainless steel = 1670 K Um = 3.33 x 10-6 (1670)2 = 9.29 J/mm3 Volume of metal melted V = 20(200) = 4000 mm3 Hm = 9.29(4000) = 37,148 J at weld (b) Given f1 = 0.8 and f2 = 0.6. H = 37,148/(0.8 x 0.6) = 77,392 J at source. 30.9 A certain groove weld has a cross-sectional area Aw = 0.045 in2 and is 10 inches long. (a) What quantity of heat (in Btu) is required to accomplish the weld, if the metal to be welded is medium carbon steel? (b) How much heat must be generated at the welding source, if the heat transfer efficiency = 0.9 and the melting efficiency = 0.7? Solution: (a) Eq. (30.2) for USCS units: Um = 1.467 x 10-5 Tm 2 From Table 30.2, Tm for medium carbon steel = 3060 R Um = 1.467 x 10-5 (3060)2 = 137.4 Btu/in3 Volume of metal melted V = 0.045(10) = 0.45 in3 Hm = 137.4(0.45) = 61.8 Btu at weld (b) Given f1 = 0.9 and f2 = 0.7. H = 61.8/(0.9 x 0.7) = 98.1 Btu at source. 206 30.10 Solve the previous problem, except that the metal to be welded is aluminum, and the corresponding melting efficiency is half the value for steel. Solution: (a) Eq. (30.2) for USCS units: Um = 1.467 x 10-5 Tm 2 From Table 30.2, Tm for aluminum = 1680 R Um = 1.467 x 10-5 (1680)2 = 41.4 Btu/in3 Volume of metal melted V = 0.045(10) = 0.45 in3 Hm = 41.4(0.45) = 18.6 Btu at weld (b) Given f1 = 0.9 and f2 = 0.7. H = 18.6/(0.9 x 0.35) = 59.1 Btu at source. 30.11 Compute the unit melting energy for (a) aluminum and (b) steel as the sum of: (1) the heat required to raise the temperature of the metal from room temperature to its melting point, which is the product of the volumetric specific heat and the temperature rise; and (2) the heat of fusion, so that this value can be compared to the unit melting energy calculated by Eq. (30.2). Use either the U.S. Customary units or the International System. Find the values of the properties needed in these calculations either in this text or in other references. Are the values close enough to validate Eq. (30.2)? Solution: (a) Aluminum properties (from standard sources): heat of fusion Hf = 170 Btu/lb = 395,390 J/kg, melting temperature Tm = 1220F = 660C, density = 0.096 lb/in3 = 2700 kg/m3, specific heat C = 0.215 Btu/lb-F = 900 J/kg-C. In USCS, Um = C(Tm - 70) + Hf = 0.096(0.215)(1220 - 70) + 0.096(170) = 40.1 Btu/in3 This compares with Eq. (30.2): Um = 1.467 x 10-5 (1220 + 460)2 = 41.4 Btu/in3 This is about a 3% difference. In SI, Um = C(Tm - 21) + Hf Um = (2.7 x 10-6 kg/mm3)(900 J/kg-C))(660 - 21) + (2.7 x 10-6 kg/mm3)(395390 J/kg) Um = 2.62 J/mm3 This compares with Eq. (30.2): Um = 3.33 x 10-6 (660 + 273)2 = 2.90 J/mm3 This is about a 10% difference. These values for aluminum show good agreement. (b) Steel properties (from standard sources): heat of fusion Hf = 117 Btu/lb = 272,123 J/kg, melting temperature Tm = 2700F = 1480C, density = 0.284 lb/in3 = 7900 kg/m3, specific heat C = 0.11 Btu/lb-F = 460 J/kg-C. In USCS, Um = C(Tm - 70) + Hf = 0.284(0.11)(2700 - 70) + 0.284(117) = 115.4 Btu/in3 This compares with Eq. (30.2): Um = 1.467 x 10-5 (2700 + 460)2 = 146.5 Btu/in3 This is about a 27% difference. In SI, Um = C(Tm - 21) + Hf Um = (7.9 x 10-6 kg/mm3)(460 J/kg-C))(1480 - 21) + (7.9 x 10-6 kg/mm3)(272123 J/kg) Um = 7.45 J/mm3 This compares with Eq. (30.2): Um = 3.33 x 10-6 (1480 + 273)2 = 10.23 J/mm3 This is about a 37% difference. These values show a greater difference than for aluminum. This is at least partially accounted for by the fact that the specific heat of steel increases significantly with temperature, which would increase the calculated values based on Um = C(Tm - Tambient) + Hf . Energy balance in welding 30.12 The welding power generated in a particular arc welding operation = 3000 W. This is transferred to the work surface with a heat transfer efficiency f1 = 0.9. The metal to be welded is copper whose melting point is given in Table 30.2. Assume that the melting efficiency f2 = 0.25. A continuous fillet weld is to be made with a cross-sectional area Aw = 15.0 mm2. Determine the travel speed at which the welding operation can be accomplished. 207 Solution: From Table 30.2, Tm = 1350K for copper. Um = 3.33 x 10-6(1350)2 = 6.07 J/mm3 v = f1 f2 HR/UmAw = 0.9(0.25)(3000)/(6.07 x 15) = 7.4 mm/s. 30.13 Solve the previous problem except that the metal to be welded is high carbon steel, the cross-sectional area of the weld = 25.0 mm2, and the melting efficiency f2 = 0.6. Solution: From Table 30.2, Tm = 1650K for high carbon steel. Um = 3.33 x 10-6(1650)2 = 9.07 J/mm3 v = f1 f2 HR/UmAw = 0.9(0.6)(3000)/(9.07 x 25) = 7.15 mm/s. 30.14 In a certain welding operation to make a groove weld, Aw = 22.0 mm2 and v = 5 mm/sec. If f1 = 0.95, f2 = 0.5, and Tm = 1000C for the metal to be welded, determine the rate of heat generation required at the welding source to accomplish this weld. Solution: Um = 3.33 x 10-6(1000 + 273)2 = 5.40 J/mm3 f1 f2 HR = UmAw v HR = Um Aw v/f1 f2 = 5.40(22)(5)/(0.95 x 0.5) = 1250 J/s = 1250 W. 30.15 The power source in a particular welding operation generates 125 Btu/min which is transferred to the work surface with an efficiency f1 = 0.8. The melting point for the metal to be welded Tm = 1800F and its melting efficiency f2 = 0.5. A continuous fillet weld is to be made with a cross-sectional area Aw = 0.04 in2. Determine the travel speed at which the welding operation can be accomplished. Solution: Um = 1.467 x 10-5 (1800 + 460)2 = 74.9 Btu/in3 v = f1 f2 HR/UmAw = 0.8(0.5)(125)/(74.9 x 0.04) = 16.7 in/min. 30.16 In a certain welding operation to make a fillet weld, Aw = 0.025 in2 and v = 15 in/min. If f1 = 0.95 and f2 = 0.5, and Tm = 2000F for the metal to be welded, determine the rate of heat generation required at the welding source to accomplish this weld. Solution: Um = 1.467 x 10-5 (2000 + 460)2 = 88.8 Btu/in3 v = 15 = f1 f2 HR/UmAw = 0.95(0.5)HR/(88.8 x 0.025) = 0.214 HR HR = 15/.214 = 70.1 Btu/min. 30.17 A spot weld is to be made using an arc welding operation. The total volume of (melted) metal forming the weld = 0.005 in3, and the operation required the arc to be on for 4 sec. If f1 = 0.85, f2 = 0.5, and the metal to be welded was aluminum, determine the rate of heat generation that was required at the source to accomplish this weld. Solution: From Table 30.2, Tm = 1680R for aluminum. Um = 1.467 x 10-5 (1680)2 = 41.4 Btu/in3 Hw = 41.4(0.005) = 0.207 Btu H = 0.207/(0.85 x 0.5) = 0.487 Btu HR = 0.487/4 = 0.122 Btu/sec = 7.31 Btu/min. 30.18 A surfacing weld is to be applied to a rectangular low carbon steel plate which is 200 mm by 350 mm. The metal to be applied is a harder (alloy) grade of steel, whose melting point is assumed to be the same. A thickness of 2.0 mm will be added to the plate, but with penetration into the base metal, the total thickness melted during welding = 6.0 mm, on average. The surface will be applied by making a series of parallel, overlapped welding beads running lengthwise on the plate. The operation will be carried out automatically with the beads laid down in one long continuous operation at a travel speed v = 7.0 mm/s, using welding passes separated by 5 mm. Ignore the 208 minor complications of the turnarounds at the ends of the plate. Assuming the heat transfer efficiency = 0.8 and the melting efficiency = 0.6, determine: (a) the rate of heat that must be generated at the welding source, and (b) how long will it take to complete the surfacing operation. Solution: (a) From Table 30.2, Tm = 1760K for low carbon steel. Um = 3.33 x 10-6(1760)2 = 10.32 J/mm3 HR = Um Aw v/f1f2 = 10.32(6 x 5)(7)/(0.8 x 0.6) = 4515 J/s (b) Total length of cut = 350 x (200/5) = 14,000 mm Time to travel at v = 7 mm/s = 14,000/7 = 2000 s = 33.33 min. 209 31 WELDING PROCESSES Review Questions 31.1 Name the principal groups of processes included in fusion welding. Answer. Arc welding, resistance welding, oxyfuel welding, and "other." The other category includes EBW, LBW, thermit welding, and others. 31.2 What is the fundamental feature that distinguishes fusion welding from solid state welding? Answer. In fusion welding, melting occurs at the faying surfaces; in solid state welding, no melting occurs. 31.3 Define what an electrical arc is. Answer. An electrical arc is a discharge across a gap in a circuit. In arc welding, the arc is sustained by a thermally ionized column of gas through which the current can flow. 31.4 What do the terms arc-on time, arc time, and operating factor have in common? Provide a definition of these terms. Answer. The three terms mean the same thing: the proportion of the total time in a shift that the arc is actually on. 31.5 Electrodes in arc welding are divided into two categories. Name and define the two types. Answer. Consumable and nonconsumable. The consumable type, in addition to being the electrode for the process, also provide filler metal for the welding joint. The nonconsumable type are made of materials that resist melting, such as tungsten and carbon. 31.6 What are the two basic methods of arc shielding? Answer. (1) Shielding gas, such as argon and helium; and (2) flux, which covers the welding operation and protects the molten pool from the atmosphere. 31.7 Why is the heat transfer efficiency greater in arc welding processes that utilize consumable electrodes? Answer. Because molten metal from the electrode is transferred across the arc and contributes to the heating of the molten weld pool. 31.8 Describe the shielded metal arc welding (SMAW) process. Answer. SMAW is an arc welding process that uses a consumable electrode consisting of a filler metal rod coated with chemicals that provide flux and shielding. 31.9 Why is the shielded metal arc welding (SMAW) process difficult to automate? Answer. Because the stick electrodes must be changed frequently, which would be difficult to do automatically. It is much easier to automate the feeding of continuous filler wire, such as in GMAW, FCAW, SAW, or GTAW. 31.10 Describe submerged arc welding (SAW). Answer. SAW is an arc welding process that uses a continuous, consumable bare wire electrode, and arc shielding is provided by a cover of granular flux. 210 31.11 Describe electrogas welding (EGW) process and identify its major application. Answer. EGW is an arc welding process that uses a continuous consumable electrode, either flux-cored wire or bare wire with externally supplied shielding gas, and molding shoes to contain the molten pool. 31.12 Why are the temperatures much higher in plasma arc welding than in other AW processes? Answer. Because the arc is restricted in diameter, thus concentrating the energy into a smaller area, resulting in much higher power densities. 31.13 Define resistance welding. Answer. RW consists of a group of fusion welding processes that utilize a combination of heat and pressure to accomplish coalescence of the two faying surfaces. Most prominent in the group is resistance spot welding. 31.14 What are the desirable properties of a metal that would provide good weldability for resistance welding? Answer. High resistivity, low electrical and thermal conductivity, and low melting point. 31.15 Describe the sequence of steps in the cycle of a resistance spot welding operation. Answer. The steps are: (1) the parts are inserted between electrodes, (2) squeeze the parts between the electrodes, (3) weld, in which the current is switched on for a brief duration (0.1 to 0.4 sec), (4) hold, during which the weld nugget solidifies, and (5) the electrodes are opened and the parts removed. 31.16 What is resistance projection welding? Answer. RPW is a resistance welding process in which coalescence occurs at one or more relatively small points on the parts; the contact points are designed into the geometry of the parts as embossments or projections. 31.17 Describe cross-wire welding. Answer. Cross-wire welding is a form of resistance projection welding used to fabricate welded wire products such as shopping carts and stove grills. 31.18 Why is the oxyacetylene welding process favored over the other oxyfuel welding processes? Answer. Because acetylene and oxygen burn hotter than other oxyfuels. 31.19 Define pressure gas welding. Answer. PGW is a fusion welding process in which coalescence is obtained over the entire contact surfaces of the two parts by heating them with an appropriate fuel mixture and then applying pressure to bond the surfaces. 31.20 Electron beam welding has a significant disadvantage in high-production applications. What is that disadvantage? Answer. EBW is usually carried out in a vacuum for a high quality weld. The time to draw the vacuum adds significantly to the production cycle time. 31.21 Laser beam welding and electron beam welding are often compared because they both produce very high power densities. LBM has certain advantages over EBM. What are they? 211 Answer. (1) No vacuum chamber is required and (2) no x-rays are emitted in LBM; and (3) the laser beam can be focused and directed with conventional optical mirrors and lenses. 31.22 There are several modern-day variations of forge welding, the original welding process. Name the variations. Answer. (1) Cold welding, (2) roll welding, (3) and hot pressure welding. 31.23 There are two basic types of friction welding. Describe and distinguish the two types. Answer. The two types are: (1) continuous-drive friction welding and (2) inertia friction welding. In continuous-drive friction welding, one part is rotated at a constant speed and forced into contact with the stationary part with a certain force so that friction heat is generated at the interface; when the right temperature is reached, the rotating part is stopped abruptly and the two parts are forced together at forging pressures. In inertia friction welding, the rotating part is connected to a flywheel which is brought up to proper speed; then the flywheel is disengaged from the drive motor and the parts are forced together, so that the kinetic energy of the flywheel is converted to friction heat for the weld. 31.24 What is a sonotrode in ultrasonic welding? Answer. It is the actuator which is attached to one of the two parts to be welded with USW and which provides the oscillatory motion that results in coalescence of the two surfaces. 31.25 Distortion (warpage) is a serious problem in fusion welding, particularly arc welding. What are some of the measures that can be taken to reduce the incidence and extent of distortion? Answer. The following measures, explained in the text in Section 29.6, can be used to reduce warpage in arc welding: (1) welding fixtures, (2) presetting of the parts in relative orientations to compensate for warpage, (3) heat sinks, (4) tack welding at several points along the joint rather than continuous weld, (5) balance the weld about the neutral axis of the part, (6) selecting proper welding conditions, (7) preheating of base parts, (8) stress relief of the weldment, and (9) proper design of the weldment to minimize warpage. 31.26 What are some of the important welding defects? Answer. (1) cracks, (2) cavities, (3) solid inclusions, (4) incomplete fusion, (5) imperfect shape or contour of weld cross-section. 31.27 What are the three basic categories of inspection and testing techniques used for weldments? Name some typical inspections and/or tests in each category. Answer. The three categories are: (1) visual inspection, which includes dimensional checks and inspection for warpage, cracks, and other visible defects; (2) nondestructive evaluation, which includes dye-penetrant, magnetic particle, ultrasonic, and radiographic tests; and (3) destructive tests, which includes conventional mechanical tests adapted to weld joints, and metallurgical tests. 31.28 Identify the factors that affect weldability. Answer. Factors affect weldability include: (1) welding process, (2) metal properties (e.g., melting point, thermal conductivity, coefficient of thermal expansion), (3) whether the base metals are similar or dissimilar (dissimilar base metals are generally more difficult to weld), (4) surface condition (surfaces should be clean and free of oxides, moisture, etc.), and (5) filler metal and its composition relative to the base metals. 31.29 What are some of the design guidelines for weldments that are fabricated by arc welding? 212 Answer. The guidelines for weldments by arc welding include: (1) Good fit-up of parts to be welded is important to maintain dimensional control and minimize distortion. Machining is sometimes required to achieve satisfactory fit-up. (2) The design of the assembly must provide access room to allow the welding gun to reach the welding area. (3) Whenever possible, design of the assembly should allow flat welding to be performed, as opposed to horizontal, vertical, or overhead arc welding positions. Multiple Choice Quiz There are a total of 27 correct answers in the following multiple choice questions (some questions have multiple answers that are correct). To attain a perfect score on the quiz, all correct answers must be given, since each correct answer is worth 1 point. For each question, each omitted answer or wrong answer reduces the score by 1 point, and each additional answer beyond the number of answers required reduces the score by 1 point. Percentage score on the quiz is based on the total number of correct answers. 31.1 The feature that distinguishes fusion welding processes from solid state welding is that melting of the faying surfaces occurs during fusion welding: (a) true, (b) false. Answer. (a) 31.2 Which of the following processes is/are classified as fusion welding (more than one)? (a) electrogas welding, (b) electron beam welding, (c) explosive welding, (d) percussion welding. Answer. (a), (b), and (d) 31.3 Which of the following processes are classified as fusion welding (more than one)? (a) diffusion welding, (b) friction welding, (c) pressure gas welding, (d) RSW. Answer. (c) and (d) 31.4 Which of the following processes are classified as solid state welding? (a) friction welding, (b) resistance spot welding, (c) roll welding, (d) thermit welding, and (e) upset welding, Answer. (a) and (c) 31.5 Which of the following processes are classified as solid state welding (more than one)? (a) CW, (b) HPW, (c) LBW, and (d) OAW. Answer. (a) and (b) 31.6 An electric arc is a discharge of current across a gap in an electrical circuit. The arc is sustained in arc welding processes by the transfer of molten metal across the gap between the electrode and the work: (a) true, (b) false. Answer. (b) The arc is sustained, not by the transfer of molten metal, but by the presence of a thermally ionized column of gas through which the current flows. 31.7 Which one of the following arc welding processes uses a nonconsumable electrode? (a) FCAW, (b) GMAW, (c) GTAW, or (d) SMAW. Answer. (c) 31.8 MIG welding is a term sometimes applied when referring to which one of the following processes? (a) FCAW, (b) GMAW, (c) GTAW, or (d) SMAW. Answer. (b) 213 31.9 "Stick" welding is a term sometimes applied when referring to which one of the following processes? (a) FCAW, (b) GMAW, (c) GTAW, or (d) SMAW. Answer. (d) 31.10 Which of the following AW processes uses an electrode consisting of continuous consumable tubing containing flux and other ingredients in its core? (a) FCAW, (b) GMAW, (c) GTAW, or (d) SMAW. Answer. (a) 31.11 Which one of the following arc welding processes produces the highest temperatures? (a) CAW, (b) PAW, (c) SAW, or (a) TIG. Answer. (b) 31.12 Shielding gases used for welding do not include which of the following (more than one)? (a) argon, (b) carbon monoxide, (c) helium, (d) hydrogen, and (e) nitrogen. Answer. (b), (d), and (e) 31.13 Resistance welding processes make use of the heat generated by electrical resistance to achieve fusion of the two parts to be joined; no pressure is used in these processes, and no filler metal is added: (a) true, (b) false. Answer. (b) Pressure is applied in RW processes and is key to the success of these processes. 31.14 Metals that are easiest to weld in resistance welding are ones that have low resistivities since low resistivity assists in the flow of electrical current: (a) true, or (b) false. Answer. (b) Metals with low resistivities, such as aluminum and copper, are difficult to weld in RW. Higher resistance is required in the conversion of electrical power to heat energy; hence, metals with high resistivity are generally preferable. 31.15 Oxyacetylene welding is the most widely used oxyfuel welding process because acetylene mixed with an equal volume of air burns hotter than any other commercially available fuel: (a) true, (b) false. Answer. (a) 31.16 The term "laser" stands for "light actuated system for effective reflection": (a) true, (b) false. Answer. (b) Laser stands for "light amplification by stimulated emission of radiation." 31.17 Which of the following solid state welding processes applies heat from an external source (more than one)? (a) diffusion welding, (b) forge welding, (c) friction welding, (d) ultrasonic welding. Answer. (a) and (b) 31.18 The term weldability takes into account not only the ease with which a welding operation can be performed, but also the quality of the resulting weld: (a) true, (b) false. Answer. (a) 31.19 Copper is a relatively easy metal to weld because its thermal conductivity is high: (a) true, (b) false. Answer. (b) True that copper has a high thermal conductivity, one of the highest of any metal, but this is one of the main reasons why copper is generally difficult to weld. The heat readily flows 214 into the body of the parts that are to be welded, rather than remaining at the localized region where the joint is to be made. Problems Arc welding 31.1 A SMAW operation is accomplished in a work cell using a fitter and a welder. The fitter takes 5.5 min to place the unwelded components into the welding fixture at the beginning of the work cycle, and 2.5 min to unload the completed weldment at the end of the cycle. The total length of the several weld seams to be made is 2000 mm, and the travel speed used by the welder averages 400 mm/min. Every 750 mm of weld length, the welding stick must be changed, which takes 0.8 min. While the fitter is working, the welder is idle (resting); and while the welder is working, the fitter is idle. (a) Determine the average arc time in this welding cycle. (b) How much improvement in arc time would result if the welder used FCAW (manually operated), given that the spool of flux-cored weld wire must be changed every five weldments, and it takes the welder 5.0 min to accomplish the change. (c) What are the production rates for these two cases (weldments completed per hour)? Solution: (a) SMAW cycle time Tc = 5.5 + 2000/400 + (2000/750)(0.8) + 2.5 = 5.5 + 5.0 + 2.133 + 2.5 = 15.133 min. Arc time = 5.0/15.133 = 33.0% (b) FCAW cycle time Tc = 5.5 + 2000/400 + (1/5)(5.0) + 2.5 = 5.5 + 5.0 + 1.0 + 2.5 = 14.0 min. Arc time = 5.0/14.0 = 35.7% (c) SMAW Rp = 60/15.133 = 3.96 pc/hr FCAW Rp = 60/14.0 = 4.29 pc/hr. 31.2 In the previous problem, suppose an industrial robot cell were installed to replace the welder. The cell consists of the robot (using GMAW instead of SMAW or FCAW), two welding fixtures, and the fitter who loads and unloads the parts. With two fixtures, fitter and robot work simultaneously, the robot welding at one fixture while the fitter unloads and loads at the other. At the end of each work cycle, they switch places. The electrode wire spool must be changed every five workparts, which task requires 5.0 minutes and is accomplished by the fitter. Determine: (a) arc time and (b) production rate for this work cell. Solution: (a) Fitter: Tc = 5.5 + 2.5 + (1/5)(5.0) = 9.0 min. Robot: Tc = 2000/400 = 5.0 min. Limiting cycle is the fitter: arc time = 5.0/9.0 = 55.5% (b) Rp = 60/9.0 = 6.67 pc/hr. 31.3 A shielded metal arc welding operation is performed on steel at E = 30 volts and I = 225 amps. The heat transfer efficiency f1 = 0.85 and melting efficiency f2 = 0.75. The unit melting energy for steel = 10.2 J/mm3. Solve for: (a) the rate of heat generation at the weld and (b) the volume rate of metal welded. Solution: (a) HRw = f1f2EI = (0.85)(0.75)(30)(225) = 4303.1 W (b) WVR = (4303.1 W)/(10.2 J/mm3) = 421.9 mm3/sec. 31.4 A GTAW operation is performed on stainless steel, whose unit melting energy Um = 9.3 J/mm3. The conditions are: E = 25 volts, I = 125 amps, f1 = 0.65, and f2 = 0.70. If filler metal wire of 3.0 mm diameter is added to the operation, and the final weld bead is composed of equal volumes of 215 filler and base metal. If the travel speed in the operation v = 5 mm/sec, determine: (a) cross-sectional area of the weld bead, and (b) the feed rate (in mm/sec) at which the filler wire must be supplied. Solution: (a) HRw = f1f2EI = UmAwv 0.65(0.70)(25)(125) = 9.3(Aw)(5) 1421.9 = 46.5 Aw Aw = 1421.9/46.5 = 30.6 mm2 (b) Aw v = 30.6(5) = 153 mm3/s Filler wire A = D2/4 = (3)2/4 = 7.07 mm2 At 50% filler metal, feed rate of filler wire = 153(0.50)/7.07 = 10.82 mm/s. 31.5 A flux-cored arc welding operation is performed to butt weld two aluminum plates together, using the following conditions: E = 20 volts and I = 250 amps. The cross-sectional area of the weld seam = 80 mm2 and the melting efficiency of the aluminum is assumed to be f2 = 0.5. Using tabular data and equations given in this and the preceding chapter, determine the likely value for travel speed v in the operation. Solution: From Table 31.1, f2 = 0.9 for FCAW. From Table 30.2, Tm = 930K for aluminum. Um = 3.33 x 10-6 (930)2 = 2.88 J/mm3 f1f2EI = UmAwv v = f1f2EI/UmAw = 0.9(0.5)(20)(250)/(2.88 x 80) = 9.77 mm/s 31.6 A gas metal arc welding test is performed to determine the value of melting efficiency f2 for a certain metal and operation. The welding conditions are: E = 25 volts, I = 125 amps, and heat transfer efficiency is assumed to be f1 = 0.90, a typical value for GMAW. The rate at which the filler metal is added to the weld is 0.50 in3 per minute, and measurements indicate that the final weld bead consists of 57% filler metal and 43% base metal. The unit melting energy for the metal is known to be 75 Btu/in3. (a) Find f2. (b) What is the travel speed if the cross-sectional area of the weld bead = 0.05 in2? Solution: (a) f1f2EI = UmAwv Awv = welding volume rate = WVR = (0.50 in3/min)/0.57 = 0.877 in3/min. = 0.01462 in3/sec. Therefore, f1f2EI = Um(WVR) 1 Btu/sec = 1055 J/s = 1055 W, so 75 Btu/sec = 79,125 W f2 = Um(WVR)/ f1EI = 79,125(0.01462)/(0.9 x 25 x 125) = 0.41 (b) Given that Aw = 0.05 in2, v = (WVR)/Aw = 0.877/0.05 = 17.54 in/min. 31.7 A continuous weld is to be made around the circumference of a round steel tube of diameter = 6.0 ft, using a submerged arc welding operation under automatic control at a voltage of 25 volts and current of 300 amps. The tube is slowly rotated under a stationary welding head. The heat transfer efficiency for SAW is f1 = 0.95 and the assumed melting efficiency f2 = 0.7. The cross-sectional area of the weld bead is 0.12 in2. If the unit melting energy for the steel = 150 Btu/in3, determine: (a) the rotational speed of tube and (b) the time required to complete the weld. Solution: (a) f1f2EI = UmAwv v = f1f2EI/UmAw 1 Btu/sec = 1055 J/s = 1055 W, so 150 Btu/sec = 158,250 W v = 0.95(0.7)(25)(300)/(158,250 x 0.120) = 0.263 in/sec = 15.76 in/min. Circumference C = D = 12 x 6= 226.2 in/rev. Rotational speed N = (15.76 in/min)/(226.2 in/rev) = 0.06967 rev/min. 216 (b) Time to weld around circumference = C/v = (226.2 in/rev)/(15.76 in/min) = 14.35 min. Resistance welding 31.8 A RSW operation is used to make a series of spot welds between two pieces of aluminum, each 2.0 mm thick. The unit melting energy for aluminum Um = 2.90 J/mm3. Welding current I = 6,000 amps, time duration = 0.15 sec. Assume that the resistance = 75 micro-ohms. The resulting weld nugget measures 5.0 mm in diameter by 2.5 mm thick. How much of the total energy generated is used to form the weld nugget? Solution: H = I2Rt = (5000)2(75 x 10-6)(0.15) = 405 W-sec = 405 J Weld nugget volume V = D2d/4 = (5)2(2.5)/4 = 49.1 mm3 Heat required for melting = UmV = (2.9 J/mm3)(49.1 mm3) = 142.4 J Proportion of heat for welding =142.4/405 = 0.351 = 35.1% 31.9 The unit melting energy for a certain sheet metal to be spot welded is Um = 10.0 J/mm3. The thickness of each of the two sheets to be welded is 3.0 mm. To achieve required strength, it is desired to form a weld nugget that is 6.0 mm in diameter and 4.5 mm thick. The weld duration will be set at 0.2 sec. If it is assumed that the electrical resistance between the surfaces is 125 micro-ohms, and that only one-third of the electrical energy generated will be used to form the weld nugget (the rest being dissipated into the work), determine the minimum current level required in this operation. Solution: Hm = UmV V = D2d/4 = (6)2(4.5)/4 = 127.2 mm3 Hm = 10(127.2) = 1272 J Required heat for the RSW operation H = 1272/(1/3) = 3816 J H = I2Rt = I2(125 x 10-6)(0.2) = 25 x 10-6 I2 = 3816 J I2 = 3816/(25 x 10-6) = 152.64 x 106 A2. I = 12.35 x 103 = 12,350 A. 31.10 A resistance spot welding operation is performed on two pieces of 0.040 in thick sheet steel (low carbon). The unit melting energy for steel = 150 Btu/in3. Process parameters are: current = 9500 A and time duration = 0.17 sec. This results in a weld nugget of diameter = 0.19 in and thickness = 0.060 in. Assume the resistance = 100 micro-ohms. Determine: (a) the average power density in the interface area defined by the weld nugget, and (b) the proportion of energy generated that went into formation of the weld nugget. Solution: (a) PD = I2R/A A = D2/4 = (0.19)2/4 = 0.02835 in2 I2R = (9500)2 (100 x 10-6) = 9025 W 1 Btu/sec = 1055 W, so 9025 W = 8.554 Btu/sec PD = 8.554/0.02835 = 302 Btu/sec-in2 (b) H = I2Rt = (9500)2 (100 x 10-6)(0.17) = 1534 W-sec = 1.454 Btu Weld nugget volume V = D2d/4 = (0.19)2(0.060)/4 = 0.0017 in3 Heat required for melting = UmV = (150 Btu/in3)(0.0017) = 0.255 Btu Proportion of heat for welding = 0.255/1.454 = 0.175 = 17.5% 31.11 A resistance seam welding operation is performed on two pieces of 2.5 mm thick austenitic stainless steel to fabricate a container. The weld current in the operation is 10,000 amps, the weld duration t = 0.3 sec, and the resistance at the interface is 75 micro-ohms. Continuous motion 217 welding is used, with 200 mm diameter electrode wheels. The individual weld nuggets formed in this RSEW operation have dimensions: diameter = 6 mm and thickness = 3 mm (assume the weld nuggets are disc-shaped). These weld nuggets must be contiguous to form a sealed seam. The power unit driving the process requires an off-time between spot welds of 1.0 s. Given these conditions, determine: (a) the unit melting energy of stainless steel using the methods of the previous chapter, (b) the proportion of energy generated that goes into the formation of each weld nugget, and (c) the rotational speed of the electrode wheels. Solution: (a) From Table 30.2, Tm = 1670K for austenitic stainless steel. Um = 3.33 x 10-6 (1670)2 = 9.29 J/mm3. (b) Hm = UmV V = D2d/4 = (6.0)2(3.0)/4 = 84.82 mm3 Hm = (9.29 J/mm3)(84.82 mm3) = 788 J H = I2Rt = (10,000)2(75 x 10-6)(0.3) = 2225 J Proportion of heat for welding =788/2225= 0.354 (c) Total cycle time per weld = 0.3 + 1.0 = 1.3 sec. Distance moved per spot weld in order to have contiguous spot welds for the seam = D = 0.25 in. Therefore, surface speed of electrode wheel v = 6.00 mm/1.3 sec = 4.61 mm/s. = 276.9 mm/min. N = v/D = (276.9 mm/min)/(200mm/rev) = 0.441 rev/min. 31.12 Suppose in the previous problem that a roll spot welding operation is performed instead of seam welding. The interface resistance increases to 100 micro-ohms, and the center-to-center separation between weld nuggets is 25 mm. Given the conditions from the previous problem, with the changes noted here, determine: (a) the proportion of energy generated that goes into the formation of each weld nugget, and (b) the rotational speed of the electrode wheels. (c) At this higher rotational speed, how much does the wheel move during the current on-time, and might this have the effect of elongating the weld nugget (making it elliptical rather than round)? Solution: (a) Um = 3.33 x 10-6 (1670)2 = 9.29 J/mm3 from previous problem. Hm = (9.29 J/mm3)(84.82 mm3) = 788 J from previous problem. H = I2Rt = (10,000)2(100 x 10-6)(0.3) = 3000 J Proportion of heat for welding =788/3000 = 0.263 (b) Total cycle time per spot weld = 1.3 sec as in previous problem. Distance moved per spot weld = 25 mm as given. Surface speed of electrode wheel v = 25 mm/1.3 sec = 19.23 mm/s = 1153.8 mm/min. N = v/D = (1153.8 mm/min)/(200in/rev) = 1.836 rev/min. (c) Power-on time during cycle = 0.3 sec. Movement of wheel during 0.3 sec = (0.3 sec)(19.23 mm/s) = 5.77 mm. This movement is likely to have the effect of making the weld spot elliptical in shape. 31.13 An experimental power source for spot welding is designed to deliver current as a ramp function of time: I = 100,000 t, where I = amp and t = sec. At the end of the power-on time, the current is stopped abruptly. The sheet metal being spot welded is low carbon steel whose unit melting energy = 10 J/mm3. The resistance R = 85 micro-ohms. The desired weld nugget size is: diameter = 4 mm and thickness = 2 mm (assume a disc-shaped nugget). It is assumed that 1/4 of the energy generated from the power source will be used to form the weld nugget. Determine the power-on time the current must be applied in order to perform this spot welding operation. Solution: Hm = UmV V = D2d/4 = (4)2(2)/4 = 25.14 mm3 218 Hm = (10 J/mm3)(25.14 mm3) = 251.4 J H = 251.4/0.25 = 1005.6 J Power P = I2R dt = (100,000t)2 R dt = 100,000R t2 dt = (105)2(85 x 10-6)t3/3 evaluated between 0 and t. H = 850,000t3/3 = 31481.5 t3 = 1005.6 t3 = 1005.6/31481.5 = 0.031943 t = (0.031943)1/3 = 0.317 s. Oxyfuel Welding 31.14 Suppose in Example 31.3 in the text that the fuel used in the welding operation is MAPP instead of acetylene, and the proportion of heat concentrated in the 9 mm circle is 60% instead of 75 %. Compute: (a) rate of heat liberated during combustion, (b) rate of heat transferred to the work surface, and (c) average power density in the circular area. Solution: (a) Rate of heat generated by the torch HR = (0.3 m3/hr)(91.7 x 106 J/m3) = 27.5 x 106 J/hr = 7642 J/s (b) Rate of heat received at work surface = f1 HR = 0.25(7642) = 1910 J/s (c) Area of circle in which 60% of heat is concentrated A = D2/4 = (9.0)2/4 = 63.6 mm2 Power density PD = 0.60(1910)/63.6 = 18.0 W/mm2 31.15 An oxyacetylene torch supplies 10 ft3 of acetylene per hour and an equal volume rate of oxygen for an OAW operation on 3/16 in steel. Heat generated by combustion is transferred to the work surface with an efficiency f1 = 0.25. If 75% of the heat from the flame is concentrated in a circular area on the work surface whose diameter = 0.375 in, find: (a) rate of heat liberated during combustion, (b) rate of heat transferred to the work surface, and (c) average power density in the circular area. Solution: (a) Rate of heat generated by the torch HR = (10 ft3/hr)(1470 Btu/ft3) = 14,700 Btu/hr = 4.08 Btu/sec (b) Rate of heat received at work surface = f1 HR = 0.25(4.08 Btu/sec) = 1.02 Btu/sec (c) Area of circle in which 75% of heat is concentrated A = D2/4 = (0.375)2/4 = 0.1104 in.2 Power density PD = 0.75(1.02 Btu/sec)/(0.1104 in2) = 6.94 Btu/sec-in.2 Electron beam welding 31.16 The voltage in an EBW operation = 50 kV and the beam current = 65 milliamp. The electron beam is focused on a circular area that is 0.3 mm in diameter. The heat transfer efficiency f1 = 0.85. Calculate the average power density in the area in watt/mm2. Solution: Power density PD = f1EI/A Power P = f1EI = 0.85(50 x 103)(65 x 10-3) = 2762.5 W Area A = D2/4 = (0.30)2/4 = 0.0707 mm3 PD = 2762.5/0.0707 = 39,074 W/mm2 31.17 An electron beam welding operation is to be accomplished to butt weld two sheet metal parts that are 3.0 mm thick. The unit melting energy = 5.0 J/mm3. The weld joint is to be 0.35 mm wide, so that the cross-section of the fused metal is 0.35 mm by 3.0 mm. If accelerating voltage = 25 kV, beam current = 30 milliamp, heat transfer efficiency f1 = 0.85, and melting efficiency f2 = 0.75, determine the travel speed at which this weld can be made along the seam. 219 Solution: Available heat for welding HRw = f1f2EI = UmAwv Travel velocity v = f1f2EI/UmAw Cross sectional area of weld seam Aw = (0.35)(3.0) = 1.05 mm2 v = 0.85(0.75)(25 x 103)(30 x 10-3)/(5.0 x 1.05) = 91.05 mm/s 31.18 An electron beam welding operation uses the following process parameters: accelerating voltage = 25 kV, beam current = 100 milliamp, and the circular area on which the beam is focused has a diameter = 0.020 in. If the heat transfer efficiency f1 = 90%, determine the average power density in the area in Btu/sec in2. Solution: Power density PD = f1EI/A Area in which beam is focused A = D2/4 = (0.020)2/4 = 0.000314 in3 Power P = 0.90(25 x 103)(100 x 10-3)/1055 = 2.133 Btu/sec PD = 2.133/0.000314 = 6792 Btu/sec-in2 220 32 BRAZING, SOLDERING, AND ADHESIVE BONDING Review Questions 32.1 How do brazing and soldering differ from the fusion welding processes? Answer. In brazing and soldering, no melting of the base metal(s) occurs. 32.2 How do brazing and soldering differ from the solid state welding processes? Answer. In brazing and soldering filler metal is used, whereas in solid state welding no filler metal is added. 32.3 What is the technical difference between brazing and soldering? Answer. In brazing the filler metal melts at a temperature above 840 degrees F (450 degrees C). In soldering the filler metal melts at a temperature of 840 degrees F or below. 32.4 Under what circumstances would brazing or soldering be preferred over welding? Answer. Brazing or soldering might be preferred if: (1) the base metals have poor weldability, (2) the components cannot tolerate the higher heat and temperatures of welding, (3) production rates might be faster and less expensive than welding, (4) joint areas might be inaccessible for welding but brazing or soldering is possible, and (5) the high strength of a welded joint is not a requirement. 32.5 What are the two joint types most commonly used in brazing? Answer. Butt and lap joints. 32.6 Certain changes in joint configuration are usually made to improve the strength of brazed joints. What are some of these changes? Answer. In butt joints, the butting surface areas are increased in various ways such as scarfing or stepping the edges. In brazed or soldered lap joints, the overlap area is made as large as possible. Several of the adaptations are illustrated in the figures of this chapter. 32.7 The molten filler metal in brazing is distributed throughout the joint by capillary action. What is capillary action? Answer. Capillary action is the physical tendency of a liquid to be drawn into a small diameter tube or other narrow openings in spite of the force of gravity. It is caused by the adhesive attraction between the liquid molecules and the solid surfaces that define the narrow openings. 32.8 What are the desirable characteristics of a brazing flux? Answer. A brazing flux should do the following: (1) inhibit the formation of oxides and other unwanted byproducts during the brazing, (2) have a low melting temperature, (3) have a low viscosity when melted, (4) be a good wetting agent, and (5) protect the joint until solidification occurs. 32.9 What is dip brazing? Answer. The parts to be brazed are dipped into a molten salt or molten metal bath which supplies the heating for the operation. 32.10 Define braze welding. 221 Answer. Braze welding is used for adding braze metal to a more conventional geometry weld joint, such as a V-joint. It differs from the typical brazing operation in that no capillary action occurs. It differs from a conventional welding operation in that no melting of the base metals occurs. 32.11 What are some of the disadvantages and limitations of brazing? Answer. Disadvantages and limitations: (1) the strength of the brazed joint is generally less than that of a welded joint; (2) high service temperatures may weaken a brazed joint; (3) part sizes are limited; and (4) the color of the filler metal is often different than the color of the base metals. 32.12 What are the two most common alloying metals used in solders? Answer. Tin and lead. 32.13 Why are rosins as soldering fluxes losing favor in industry? Answer. Since natural rosin is insoluble in water, chemical solvents must be used to remove the flux after the joining operation. These chemical solvents are hazardous to the environment and to humans. 32.14 What are the functions served by the bit of a soldering iron in hand soldering? Answer. The functions include: (1) provide heat to the parts, (2) melt the solder, (3) convey solder to the joint, and (4) withdraw excess solder from the joint. 32.15 What is wave soldering? Answer. Wave soldering involves the flow of molten solder onto the underside of a printed circuit board to provide soldered connections between the component leads that project through holes in the boards. 32.16 List the advantages often attributed to soldering as an industrial joining process? Answer. Advantages of soldering: (1) lower heat energy required than brazing or welding, (2) various heating methods available, (3) good electrical and thermal conductivity of the joint, (4) capable of making air-tight and liquid-tight joints, and (5) ease of repair and rework. 32.17 What are the disadvantages and drawbacks of soldering? Answer. (1) low mechanical strength unless reinforced and (2) elevated service temperatures can weaken the joint. 32.18 What is meant by the term structural adhesive? Answer. A structural adhesive is capable of forming a strong permanent joint between strong, rigid components. 32.19 An adhesive must cure in order to bond. What is meant by the term curing? Answer. Curing is the chemical reaction in which the adhesive transforms from liquid to solid and in the process forms the surface attachment between the two adherends. 32.20 What are some of the methods used to cure adhesives? Answer. The curing methods include: chemical reaction between two components of the adhesive (e.g., epoxies), heating of the adhesive, use of ultraviolet light, and application of pressure. 32.21 Name the three basic categories of commercial adhesives. 222 Answer. The categories are: (1) natural adhesives (e.g., starch, collagen); (2) inorganic adhesives (e.g., sodium silicate); and (3) synthetic adhesives (e.g., thermoplastic and thermosetting polymers such as epoxies and acrylics). 32.22 What is an important precondition for the success of an adhesive bonding operation? Answer. The surfaces of the adherends must be very clean. Special surface preparation is sometimes required immediately prior to application of the adhesive in order to insure cleanliness. 32.23 What are some of the methods used to apply adhesives in industrial production operations? Answer. Methods include: (1) manual brushing, (2) use of manual rollers, (3) silk screening, (4) use of flow guns, (5) spraying, (6) automatic dispensers, and (7) roll coating. 32.24 Identify some of the advantages of adhesive bonding compared to alternative joining methods. Answer. Advantages of adhesive bonding: (1) applicable to a wide variety of materials - similar or dissimilar, (2) fragile parts can be joined, (3) bonding occurs over entire surface area of joint, (4) certain adhesives are flexible after curing, thus permitting them to tolerate strains encountered in service, (5) low curing temperatures, (6) some adhesives are suited to sealing as well as bonding, and (7) simplified joint design. 32.25 What are some of the limitations of adhesive bonding? Answer. Limitations of adhesive bonding: (1) adhesively bonded joints are generally not as strong as other joining techniques, (2) the adhesive must be compatible with the adherend materials, (3) service temperatures are limited, (4) surfaces to be bonded must be very clean, (5) curing times can limit production rates, (6) inspection of the bond is difficult. Multiple Choice Quiz There are a total of 24 correct answers in the following multiple choice questions (some questions have multiple answers that are correct). To attain a perfect score on the quiz, all correct answers must be given, since each correct answer is worth 1 point. For each question, each omitted answer or wrong answer reduces the score by 1 point, and each additional answer beyond the number of answers required reduces the score by 1 point. Percentage score on the quiz is based on the total number of correct answers. 32.1 In brazing, the base metals melt at temperatures above 840F (450C) while in soldering they melt at 840F (450C) or below: (a) true, or (b) false. Answer. (b) Neither brazing or soldering involve melting of the base metals. 32.2 The strength of a brazed joint is typically which one of the following relative to the filler metal out of which it is made: (a) equal to, (b) stronger than, or (c) weaker than. Answer. (b) 32.3 Scarfing in the brazing of a butt joint involves the wrapping of a sheath around the two parts to be joined to contain the molten filler metal during the heating process: (a) true, or (b) false. Answer. (b) Scarfing involves a preparation of the two edges to increase surface area for brazing. 32.4 Clearances between surfaces in brazing are which one of the following: (a) 0.0025 to 0.025 mm (0.0001 to 0.001 in.), (b) 0.025 to 0.250 mm (0.001 to 0.010 in.), (c) 0.250 to 2.50 mm (0.010 to 0.100 in.), or (d) 2.5 to 5.0 mm (0.10 to 0.20 in.). 223 Answer. (b) 32.5 Which of the following is an advantage of brazing (more than one): (a) dissimilar metals can be joined, (b) less heat and energy required than fusion welding, (c) multiple joints can be brazed simultaneously, (d) stronger joint than welding. Answer. (a), (b), and (c). 32.6 Which of the following soldering methods are not used for brazing (more than one)? (a) dip soldering, (b) infrared soldering, (c) soldering iron, (d) torch soldering, and (e) wave soldering. Answer. (c) and (e). 32.7 Which one of the following is not a function of a flux in brazing or soldering? (a) chemically etch the surfaces to increase roughness for better adhesion of the filler metal, (b) promote wetting of the surfaces, (c) protect the faying surfaces during the process, or (d) remove or inhibit formation of oxide films, Answer. (a) 32.8 Which type of soldering flux is preferred for electrical and electronics connections? (c) inorganic fluxes such as zinc chloride, (b) natural rosin fluxes, (c) water-soluble organic fluxes. Answer. (a) Preferred because it is water-soluble. 32.9 Which of the following metals is used in solder alloys (more than one)? (a) antimony, (b) gold, (c) lead, (d) silver, or (e) tin. Answer. (a), (c), (d), and (e). 32.10 A soldering gun is capable of injecting molten solder metal into the joint area: (a) true, or (b) false. Answer. (b) The trigger on a soldering gun is used to switch on the electric resistance heating elements. 32.11 In adhesive bonding, which one of the following is the term used for the parts that are joined: (a) adherend, (b) adherent, (c) adhesive, (d) adhibit, (e) ad infinitum. Answer. (a) 32.12 Weldbonding is an adhesive joining method in which heat is used to melt the adhesive: (a) true, or (b) false. Answer. (b) Weldbonding is a combination of adhesive bonding and spot welding. 32.13 Adhesively bonded joints are strongest under which type of stresses (pick two best answers): (a) cleavage, (b) peeling, (c) shear, and (d) tension. Answer. (c) and (d). 32.14 Which of the following are the mechanisms that operate in adhesive bonding (more than one)? (a) chemical bonding, in which a primary chemical bond is formed between the adhesive and the parts being joined, (b) mechanical interlocking, (c) secondary bonding forces between atoms of opposing surfaces, and (d) surface tension of the fluid adhesive. Answer. (a), (b), and (c). 32.15 Roughening of the faying surfaces tends to (a) increase, or (b) reduce the strength of an adhesively bonded joint because it increases the effective area of the joint and promotes mechanical interlocking. 224 Answer. (a) 225 33 MECHANICAL ASSEMBLY Review Questions 33.1 How does mechanical assembly differ from the other methods of assembly discussed in previous chapters (e.g., welding, brazing, etc.)? Answer. Mechanical assembly uses a mechanical fastening method for joining two (or more) parts. Also, many of the mechanical fastening methods allow for disassembly - not possible with welding and brazing. 33.2 What are some of the reasons why assemblies must be sometimes disassembled? Answer. For maintenance and repair service, to replace worn-out components, and to make adjustments. 33.3 What is the technical difference between a screw and a bolt? Answer. Both are externally threaded fasteners. A screw is generally assembled into a blind threaded hole, whereas a bolt is assembled using a nut. 33.4 What is a stud (in the context of threaded fasteners)? Answer. A stud is an externally threaded fastener that does not have the usual head possessed by a bolt. 33.5 What is torque-turn tightening? Answer. Torque-turn tightening involves the tightening of the threaded fastener to a certain low torque level, and then advancing the fastener by a specified additional amount of turn. 33.6 Define proof strength as the term applies in threaded fasteners. Answer. Proof strength can be defined as the maximum tensile stress that an externally threaded fastener can sustain without permanent deformation. 33.7 What are the three ways in which a threaded fastener can fail during tightening? Answer. (1) Stripping of the bolt or screw threads, (2) stripping of the internal fastener threads, or (3) excessive tensile load on the cross-sectional area of the bolt or screw. 33.8 What is a rivet? Answer. A rivet is an unthreaded headed pin used to join two parts by inserting the pin through holes in the parts and deforming the unheaded portion over the opposite side. 33.9 What is the difference between a shrink fit and expansion fit in assembly? Answer. In a shrink fit, the outside part is expanded by heating to fit over the mating component. Then cooling causes an interference fit with the component. In an expansion fit, the internal part is cooled so that it can be readily inserted into the mating component. Then, upon warming to room temperature, it expands to cause an interference fit with its mating part. 33.10 What are the advantages of snap fitting? Answer. (1) The method is fast, (2) no tooling is required, and (3) the parts can be designed with self-aligning features. 226 33.11 What is the difference between industrial stitching and stapling? Answer. In stitching the U-shaped fasteners are formed during the assembly process. In stapling, the fasteners are preformed. 33.12 What are integral fasteners? Answer. Integral fasteners make use of a forming operation on one of the parts to be joined to interlock the components and create a mechanically fastened joint. 33.13 Identify some of the general principles and guidelines for design for assembly. Answer. Some of the general principles and guidelines in design for assembly include: (1) Use the fewest number of parts possible to reduce the amount of assembly required. (2) Reduce the number of threaded fasteners required; instead use snap fits, retaining rings, integral fasteners, and similar fastening mechanisms that can be accomplished more rapidly. Use threaded fasteners only where justified, e.g., where disassembly or adjustment is required. (3) Standardize fasteners in order to reduce the number of sizes and styles of fasteners required in the product. (4) Reduce parts orientation difficulties by designing parts to be symmetrical and minimizing the number of asymmetric features. This allows easier handling and insertion during assembly. (5) Avoid parts that tangle. Parts with hooks, holes, slots, and curls are more likely to become entangled in parts bins, frustrating assembly workers or jamming automatic feeders. 33.14 Identify some of the general principles and guidelines that apply specifically to automated assembly. Answer. Some of the principles and guidelines that apply specifically to automated assembly include: (1) Use modularity in product design. Riley [11] suggests that each module or subassembly to be produced on a single assembly system has a maximum of 12 or 13 parts and should be designed around a base part to which other components are added. (2) Reduce the need for multiple components to be handled at once. The preferred practice for automated assembly is to separate the operations at different stations rather than to simultaneously handle and fasten multiple components at the same workstation. (3) Limit the required directions of access, that is, the number of directions in which new components are added to the existing subassembly. If all of the components can be added vertically from above, this is the ideal situation. (4) Use only high quality components. Poor quality components cause jams in feeding and assembly mechanisms that result in downtime. (5) Use of snap fit assembly, eliminating the need for threaded fasteners. Assembly is by simple insertion, usually from above. Multiple Choice Quiz There are a total of 18 correct answers in the following multiple choice questions (some questions have multiple answers that are correct). To attain a perfect score on the quiz, all correct answers must be given, since each correct answer is worth 1 point. For each question, each omitted answer or wrong answer reduces the score by 1 point, and each additional answer beyond the number of answers required reduces the score by 1 point. Percentage score on the quiz is based on the total number of correct answers. 33.1 Most externally threaded fasteners are produced by: (a) cold forming, or (b) machining. Answer. (a) 33.2 Which of the following methods is used for applying the required torque to achieve a desired preload of a threaded fastener (more than one)? (a) sense of feel by a human operator, (b) snap fit, (c) stall-motor wrenches, or (d) torque wrench. 227 Answer. (a), (c), and (d) 33.3 Which of the following are reasons for using mechanical assembly (more than one)? (a) ease of assembly, (b) ease of disassembly, (c) in some cases involves a melting of the base parts, and (d) no heat affected zone in the base parts. Answer. (a), (b), and (d). Answer (c) certainly includes the scope of mechanical assembly but it is not a reason for using it. 33.4 Which of the following are the common ways in which threaded fasteners fail during tightening (more than one)? (a) excessive pressure applied to the bolt or screw head by the tightening tool (e.g., screwdriver) resulting in failure of the head, (b) excessive shearing stresses on the threads due to inadequate length of engagement, (c) excessive tensile stresses, or (d) stripping of the internal or external threads. Answer. (b), (c), and (d) 33.5 The difference between a shrink fit and an expansion fit is that in a shrink fit the internal part is cooled to a sufficiently low temperature to reduce its size for assembly, whereas in an expansion fit, the external part is heated sufficiently to increase its size for assembly. When brought back to room temperature in either case, an interference fit is formed. (a) true, or (b) false. Answer. (b). In a shrink fit the external part is heated and then cooled to shrink it onto the internal part. In an expansion fit, the internal part is cooled to contract it for assembly; it then expands to form the interference fit. 33.6 The advantages of snap fit assembly include which of the following (more than one)? (a) assembly can be accomplished quickly, (b) no special tools are required, (c) the components can be designed with features that facilitate parts mating, and (d) the resulting joint is stronger than with most other assembly methods. Answer. (a), (b), and (c). Snap fit joints are not recognized as being stronger than joints produced by other assembly methods. 33.7 The difference between industrial stitching and stapling is that the U-shaped fasteners are formed during the stitching process while in stapling the fasteners are preformed: (a) true, or (b) false. Answer. (a) 33.8 From the standpoint of assembly cost, it is more desirable to use many small threaded fasteners rather than few large ones in order to distribute the stresses more uniformly: (a) true, or (b) false. Answer. (b) From the standpoint of assembly cost, it is more desirable to use few large threaded fasteners rather than many small ones because the large fasteners are easier to handle and since there are fewer of them, they require less assembly time. 33.9 Which of the following are considered good product design rules for automated assembly (more than one)? (a) design the assembly with the fewest number of components possible; (b) design the product using bolts and nuts wherever possible to allow for disassembly; (c) design with as many different fastener types as possible to achieve maximum flexibility in design; (d) design parts with asymmetric features to mate with other parts having corresponding (but reverse) features, so as to minimize the number of ways the parts will go together, and (e) limit the required directions of access when adding components to a base part. Answer. (a) and (e). All of the other answers go against design-for-assembly principles. 228 Problems Threaded Fasteners 33.1 A 5-mm diameter bolt is to be tightened to produce a preload = 25 N. If the torque coefficient C = 0.23, determine the torque that should be applied. Solution: T = CDF = 0.23(5)(250) = 287.5 N-mm = 0.2875 N-m. 33.2 A Metric 10 x 1.5 screw (10 mm diameter, pitch p = 1.5 mm) is to be turned into a threaded hole and tightened to one/half of its proof strength, which is 300 MPa. Determine the maximum torque that should be used if the torque coefficient C = 0.18. Solution: As = 0.25(10 - 0.9382 x 1.5)2 = 57.99 mm2 = 0.5 of 300 MPa = 150 MPa = 150 N/mm2 F = As = 150(57.99) = 8698.4 N. T = CDF = 0.18(10)(8698.4) = 15,657.1 N-mm = 15.66 N-m. 33.3 A M16x2 bolt (16 mm diameter, pitch p = 2 mm) is subjected to a torque of 12 N-m during tightening. If the torque coefficient C = 0.20, determine the tensile stress on the bolt. Solution: T = 12 N-m = 12,000 N-mm F = T/CD = 12,000/(0.2 x 16) = 3750 N. As = 0.25(16 - 0.9382 x 2)2 = 156.7 mm2 = 3750/156.7 = 23.94 N/mm2 = 23.94 MPa. 33.4 A 1/2-in diameter screw is to be preloaded to a tension force F = 1000 lb. Torque coefficient C = 0.22. Determine the torque that should be used to tighten the bolt. Solution: T = CDF = 0.22(0.50)(1000) = 110 in-lb. 33.5 A torque wrench is used on a 3/4-10 UNC screw (3/4 in nominal diameter, 10 threads/in) in an automobile final assembly plant. A torque of 125 in-lb is generated by the wrench. If the torque coefficient C = 0.20, determine the tension in the bolt. Solution: F = T/CD = 125/(0.2 x 0.75) = 833.3 lb. As = 0.25(0.75 - 0.9743/10)2 = 0.334 in2 = 833.3/0.334 = 2495 lb/in2 33.6 The designer has specified that a 3/8-16 UNC low-carbon bolt (3/8 in nominal diameter, 16 threads/in) in a certain application should be stressed to its proof stress of 33,000 lb/in2 (see Table 33.1). Determine the maximum torque that should be used if C = 0.25. Solution: As = 0.25(0.375 - 0.9743/16)2 = 0.0775 in2 F = As = 33,000(0.0775) = 2557.5 lb. T = CDF = 0.25(0.375)(2557.5) = 240 in-lb. 33.7 A 1-8 UNC low carbon steel bolt (diameter = 1.0 in, 8 threads/in) is currently planned for a certain application. It is to be preloaded to 75% of its proof strength, which is 33,000 lb/in2 (Table 33.1). However, this bolt is too large for the size of the components involved, and a higher strength but smaller bolt would be preferable. Determine: (a) the smallest nominal size of an alloy steel bolt (proof strength = 120,000 lb/in2) that could be used to achieve the same preload from the following standard UNC sizes used by the company: 1/4-20, 5/16-18, 3/8-16, 1/2-13, 5/8-11, or 3/4-10; and (b) compare the torque required to obtain the preload for the original 1-in bolt and the alloy steel bolt selected in part (a) if the torque coefficient in both cases C = 0.20. 229 Solution: (a) As = 0.25(1.0 - 0.9743/8)2 = 0.6057 in2 F = As = 0.75(33,000)(0.6057) = 14,992 lb. For the alloy bolt, = 120,000 lb/in2. As = F/= 14992/(0.75 x 120,000) = 0.1665 in2 As = 0.1665 in2 = 0.25(D - 0.9743/n)2 (D - 0.9743/n)2 = 0.1665 in2/0.25= 0.212 in2 (D - 0.9743/n) = 0.4605 in. Possible bolt sizes are: (1) 1/4-20, (2) 5/16-18, (3) 3/8-16, (4) 1/2-13, (5) 5/8-11, (6) 3/4-10 Try (1): (D - 0.9743/n) = (0.25 - 0.9743/20) = 0.2013 in. Obviously, none of the D values below 0.4605 will be sufficient. Try (5): (D - 0.9743/n) = (0.625 - 0.9743/11) = 0.5364 in. > 0.4605 in. Use 5/8-11 bolt.
(b) For the original 1-8 bolt, T = CDF = 0.2(1.0)(14,992) = 2,998 in-lb.
For the 5/8-11 bolt, T = CDF = 0.2(0.625)(14,992) = 1,874 in-lb.
Interference Fits
33.8 A dowel pin made of steel (E = 209,000 MPa) is to be press fitted into a steel collar. The pin has a
nominal diameter of 13.0 mm, and the collar has an outside diameter = 25.0 mm. (a) Compute the
radial pressure and the maximum effective stress if the interference between the shaft OD and
the collar ID is 0.02 mm. (b) Determine the effect of increasing the outside diameter of the collar
to 35.0 mm on the radial pressure and the maximum effective stress.
Solution: (a) pf = Ei(Dc
2 - Dp
2)/DpDc
2 = 209,000(0.02)(252 - 132)/(13 x 252) = 234.6 MPa
Max e = 2pfDc
2/( Dc
2 - Dp
2) = 2(234.6)(252)/(252 - 132) = 643.1 MPa
(b) When Dc = 35 mm, pf = 209,000(0.02)(352 - 132)/(13 x 352) = 277.2 MPa
Max e = 2(277.2)(352)/(352 - 132) = 643.1 MPa
33.9 A gear made of aluminum (modulus of elasticity E = 69,000 MPa) is press-fitted onto an aluminum
shaft. The gear has a diameter of 55 mm at the base of its teeth. The nominal internal diameter of
the gear = 30 mm and the interference = 0.10 mm. Compute: (a) the radial pressure between the
shaft and the gear, and (b) the maximum effective stress in the gear at its inside diameter.
Solution: (a) pf = Ei(Dc
2 - Dp
2)/DpDc
2 = 69,000(0.10)(552 - 302)/(30 x 552) = 161.5 MPa
(b) Max e = 2pfDc
2/( Dc
2 - Dp
2) = 2(161.5)(552)/ (552 - 302) = 460 MPa
33.10 A steel collar is press fitted onto a steel shaft. The modulus of elasticity of steel E = 30 x 106
lb/in2. The collar has an internal diameter = 0.998 in and the shaft has an outside diameter = 1.000
in. The outside diameter of the collar is 1.750 in. Determine: the radial (interference) pressure on
the assembly, and (b) the maximum effective stress in the collar at its inside diameter.
Solution: (a) pf = Ei(Dc
2 -Dp
2)/DpDc
2 = 30 x 106(0.002)(1.752 -1.02)/(1.0 x 1.752) = 40,408 lb/in2
(b) Max e = 2pfDc
2/( Dc
2 - Dp
2) = 2(40,408)(1.752)/ (1.752 - 1.02) = 120,000 lb/in2
33.11 The yield strength of a certain metal Y = 50,000 lb/in2 and its modulus of elasticity E = 22 x 106
lb/in2. It is to be used for the outer ring of a press-fit assembly with a mating shaft made of the
same metal. The nominal inside diameter of the ring is 1.000 in and its outside diameter = 2.500 in.
Using a safety factor SF = 2.0, determine the maximum interference that should be used with this
assembly.
Solution: Max e Y/SF , use Max e = Y/SF = 50,000/2.0 = 25,000 lb/in2 Eq. (33.9)
Max e = 2pfDc
2/(Dc
2 - Dp
2) = 25,000 lb/in2 Eq. (33.6)
230
Rearranging, pf = e(Dc
2 - Dp
2)/2Dc
2 = 25,000(2.52 - 1.02)/(2 x 2.52) = 10,500 lb/in2
pf = Ei(Dc
2 -Dp
2)/DpDc
2 Eq. (33.5)
Rearranging, i = pf DpDc
2/E(Dc
2 - Dp
2)
i = 10,500(1.0)(2.52)/(22 x 106 x (2.52 - 1.02)) = 0.00057 in.
33.12 A shaft made of aluminum is 40.0 mm in diameter at room temperature (21C). Its coefficient of
thermal expansion = 24.8 x 10-6 mm/mm per C. If it must be reduced in size by 0.20 mm in
order to be expansion fitted into a hole, determine the temperature to which the shaft must be
cooled.
Solution: (D2 - D1) = -0.20 = 24.8 x 10-6(40)(T2 - 21)
T2 - 21 = -0.20/(24.8 x 10-6 x 40) = -201.6
T2 = -201.6 + 21 = -180.6°C
33.13 A steel ring has an inside diameter = 30 mm and an outside diameter = 50 mm at room
temperature (21C). If the coefficient of thermal expansion of steel a = 12.1 x 10-6 mm/mm per
C, determine the inside diameter of the ring when heated to 500C.
Solution: D2 - D1 = D2 - 30 = 12.1 x 10-6(30)(500 - 21)
D2 = 30 + 0.174 = 30.174 mm.
33.14 A 1-inch diameter steel pin is to be heated from room temperature (70F) to 700F. If the
coefficient of thermal expansion of the pin is a = 6.7 x 10-6 in/in per F, determine the increase in
diameter of the pin.
Solution: (D2 - D1) = D1(T2 - T1) = 6.7 x 10-6(1.0)(700 - 70) = 4221 x 10-6 = 0.0042 in.
33.15 A steel collar whose outside diameter = 3.000 in at room temperature is to be shrink fitted onto a
steel shaft by heating it to an elevated temperature while the shaft remains at room temperature.
The shaft diameter = 1.500 in. For ease of assembly when the collar is heated to an elevated
temperature of 1000F, the clearance between the shaft and the collar is to be 0.007 in.
Determine: (a) the initial inside diameter of the collar at room temperature so that this clearance is
satisfied, (b) the radial pressure and (c) maximum effective stress on the resulting interference fit
at room temperature (70F). For steel, E = 30,000,000 lb/in2 and a = 6.7 x 10-6 in/in per F.
Solution: (a) If the clearance = 0.007 in., then the inside diameter of the collar must be
D2 = Dp + 0.007 = 1.500 + 0.007.
1.507 - D1 = 6.7 x 10-6 D1(1000 - 70)
1.507 - D1 = 0.00623 D1
1.507 = D1 + 0.00623 D1 = 1.00623 D1
D1 = 1.507/1.00623 = 1.4977 in.
(b) Interference i = 1.500 - 1.4977 = 0.00233 in.
pf = 30 x 106 (0.00233)(3.02 - 1.52)/(1.5 x 3.02) = 34,950 lb/in2
(c) Max e = 2(34,950)(3.02)/ (3.02 - 1.52) = 93,200 lb/in2
33.16 A pin is to be inserted into a collar using an expansion fit. Properties of the pin and collar metal
are: coefficient of thermal expansion = 12.0 x 10-6 m/m/C, yield strength = 450 MPa, and
modulus of elasticity = 209 GPa. At room temperature (20C), the outer and inner diameters of
the collar = 75.00 mm and 40.00 mm, respectively, and the pin has a diameter = 40.02 mm. The
pin is to be reduced in size for assembly into the collar by cooling to a sufficiently low temperature
that there is a clearance of 0.04 mm. (a) What is the temperature to which the pin must be cooled
231
for assembly? (b) What is the radial pressure at room temperature after assembly? (c) What is
the safety factor in the resulting assembly?
Solution: (a) D2 – D1 = D1(T2 – T1) = (40.00 – 0.04) – 40.02 = 12(10-6)(40.02)(T2 – 20)
-0.06 = 480.24(10-6) T2 – 9604.8(10-6)
-60,000(10-6) + 9604.8(10-6) = 480.24(10-6) T2
-50,395.2(10-6) = 480.24(10-6) T2
T2 = -104.9°C
(b) pf = Ei(Dc
2 - Dp
2)/DpDc
2
pf = 209(109)(0.02)(752 – 402)/(40(752) = 0.07477(109) N/m2 = 74.8 MPa
(c) Max e = 2pfDc
2/(Dc
2 – Dp
2) = 2(74.8)(752)/(752 – 402) = 209 MPa
If Y = 450 MPa and Max e = Y/SF, then SF = Y/( Max e ) = 450/209 = 2.15
232
34 RAPID PROTOTYPING
Review Questions
34.1 What is rapid prototyping? Provide a definition of the term.
Answer. Rapid prototyping consists of a family of fabrication processes developed to make
engineering prototypes in minimum possible lead times based on a computer-aided design (CAD)
model of the item.
34.2 What are the three types of starting materials in rapid prototyping?
Answer. (1) Liquid, (2) solid, and (3) powders.
34.3 Besides the starting material, what other feature distinguishes the rapid prototyping technologies?
Answer. The part build process also distinguishes the different RP technologies.
34.4 What is the common approach used in all of the material addition technologies to prepare the
control instructions for the RP system?
Answer. The text describes the common approach as a three step process: (1) Geometric
modeling, which consists of modeling the component on a CAD system to define its enclosed
volume; (2) tessellation of the geometric model, in which the CAD model is converted into a
format that approximates its surfaces by facets (triangles or polygons); and (3) slicing of the
model into layers that approximate the solid geometry.
34.5 Of all of the current rapid prototyping technologies, which one is the most widely used?
Answer. Stereolithography.
34.6 Describe the RP technology called solid ground curing.
Answer. Solid ground curing works by curing a photosensitive polymer layer by layer to create a
solid model based on CAD geometric data. Instead of using a scanning laser beam to accomplish
the curing of a given layer, the entire layer is exposed to an ultraviolet light source through a mask
that is positioned above the surface of the liquid polymer.
34.7 Describe the RP technology called laminated object manufacturing.
Answer. Laminated object manufacturing produces a solid physical model by stacking layers of
sheet stock that are each cut to an outline corresponding to the cross-sectional shape of a CAD
model that has been sliced into layers. The layers are bonded one on top of the previous prior to
cutting.
34.8 What is the starting material in fused deposition modeling?
Answer. A long filament of wax or polymer.
Multiple Choice Quiz
There are a total of 14 correct answers in the following multiple choice questions (some questions have
multiple answers that are correct). To attain a perfect score on the quiz, all correct answers must be
given, since each correct answer is worth 1 point. For each question, each omitted answer or wrong
answer reduces the score by 1 point, and each additional answer beyond the number of answers required
233
reduces the score by 1 point. Percentage score on the quiz is based on the total number of correct
answers.
34.1 Machining is never used for rapid prototyping because it takes too long: (a) true or (b) false.
Answer. (b) Desktop milling is the principal material removal technology used for rapid
prototyping.
34.2 Which of the following rapid prototyping processes starts with a photosensitive liquid polymer to
fabricate a component (more than one)? (a) ballistic particle manufacturing, (b) fused deposition
modeling, (c) selective laser sintering, (d) solid ground curing, and (e) stereolithography.
Answer. (d) and (e)
34.3 Of all of the current material addition rapid prototyping technologies, which one is the most widely
used? (a) ballistic particle manufacturing, (b) fused deposition modeling, (c) selective laser
sintering, (d) solid ground curing, and (e) stereolithography.
Answer. (e)
34.4 Which of the following RP technologies use a liquid as the starting material (more than one)? (a)
ballistic particle manufacturing, (b) fused deposition modeling, (c) laminated object manufacturing,
(d) selective laser sintering, (e) solid ground curing, and (f) stereolithography.
Answer. (a), (e), and (f)
34.5 Which one of the following RP technologies uses solid sheet stock as the starting material? (a)
ballistic particle manufacturing, (b) fused deposition modeling, (c) laminated object manufacturing,
(d) solid ground curing, and (e) stereolithography.
Answer. (c)
34.6 Which of the following RP technologies uses powders as the starting material (more than one)?
(a) ballistic particle manufacturing, (b) fused deposition modeling, (c) selective laser sintering, (d)
solid ground curing, and (e) three dimensional printing.
Answer. (c) and (e)
34.7 Rapid prototyping technologies are never used to make production parts: (a) true or (b) false.
Answer. (b) Examples include small batch sizes of plastic parts that could not be economically
injection molded, parts with intricate internal geometries, and one-of-a-kind parts such as bone
replacements.
34.8 Which of the following are problems with the current material addition rapid prototyping
technologies (more than one)? (a) inability to convert a solid part into layers, (b) limited material
variety, (c) part accuracy, and (d) part shrinkage.
Answer. (b), (c), and (d)
Problems
34.1 A prototype of a tube with a square cross-section is to be fabricated using stereolithography. The
outside dimension of the square = 100 mm and the inside dimension = 90 mm (wall thickness = 5
mm except at corners). The height of the tube (z-direction) = 80 mm. Layer thickness = 0.10 mm.
The diameter of the laser beam (“spot size”) = 0.25 mm, and the beam is moved across the
surface of the photopolymer at a velocity of 500 mm/s. Compute an estimate for the time required
to build the part, if 10 s are lost each layer to lower the height of the platform that holds the part.
Neglect the time for postcuring.
234
Solution: Layer area Ai same for all layers.
Ai = 1002 – 902 = 1900 mm2
Time to complete one layer Ti same for all layers.
Ti = (1900 mm2)/(0.25 mm)(500 mm/s)+ 10 s = 15.2 + 10 = 25.2 s
Number of layers nl = (80 mm)/(0.10 mm/layer) = 800 layers
Tc = 800(25.2) = 20,160 s = 336.0 min = 5.6 hr
34.2 Solve Problem 34.1 except that the layer thickness = 0.40 mm.
Solution: Layer area Ai same for all layers. Ai = 1002 – 902 = 1900 mm2
Time to complete one layer Ti same for all layers.
Ti = (1900 mm2)/(0.25 mm)(500 mm/s)+ 10 s = 15.2 + 10 = 25.2 s
Number of layers nl = (80 mm)/(0.40 mm/layer) = 200 layers
Tc = 200(25.2) = 5,040 s = 84.0 min = 1.4 hr
34.3 The part in Problem 34.1 is to be fabricated using fused deposition modeling instead of
stereolithography. Layer thickness is to be 0.20 mm and the width of the extrudate deposited on
the surface of the part = 1.25 mm. The extruder workhead moves in the x-y plane at a speed of
150 mm/s. A delay of 10 s is experienced between each layer to reposition the workhead.
Compute an estimate for the time required to build the part.
Solution: Use same basic approach as in stereolithography.
Layer area Ai same for all layers. Ai = 1002 – 902 = 1900 mm2
Time to complete one layer Ti same for all layers.
Ti = (1900 mm2)/(1.25 mm)(150 mm/s)+ 10 s = 10.133 + 10 = 20.133 s
Number of layers nl = (80 mm)/(0.20 mm/layer) = 400 layers
Tc = 400(20.133) = 8053.33 s = 134.22 min = 2.24 hr
34.4 Solve Problem 34.3, except using the following additional information. It is known that the
diameter of the filament fed into the extruder workhead is 1.25 mm, and the filament is fed into
the workhead from its spool at a rate of 30.6 mm of length per second while the workhead is
depositing material. Between layers, the feed rate from the spool is zero.
Solution: Cross-sectional area of filament = D2/4 = 0.25(1.25)2 = 1.227 mm2
Volumetric rate of filament deposition = (1.227 mm)(30.6 mm/s) = 37.55 mm3/s
Part volume = part cross sectional area x height = Ah
A = 1002 – 902 = 1900 mm2 and h = 80 mm. Part volume V = 1900(80) = 152,000 mm3
Tc = (152,000 mm3)/( 37.55 mm3/s) + (400 layers)(10 s delay/layer) = 4047.94 + 4000
= 8047.9 s = 134.13 min = 2.24 hr
This is very close to previous calculated value - within round-off error.
34.5 A cone-shaped part is to be fabricated using stereolithography. The radius of the cone at its base
= 35 mm and its height = 40 mm. The layer thickness = 0.20 mm. The diameter of the laser beam
= 0.22 mm, and the beam is moved across the surface of the photopolymer at a velocity of 500
mm/s. Compute an estimate for the time required to build the part, if 10 s are lost each layer to
lower the height of the platform that holds the part. Neglect postcuring time.
Solution: Volume of cube V = R2h/3 = (35)2(40)/3 = 51,313 mm3
Layer thickness t = 0.20 mm
Number of layers nl = 40 mm/(0.20 mm/layer) = 200 layers
Average volume per layer Vi= (51,313 mm3)/200 = 256.56 mm3
Since thickness t = 0.20 mm, average area/layer = (256.56 mm3)/(0.20 mm) = 1282.8 mm2
Average time per layer Ti = 1282.8/(0.22 x 500) = 11.66 + 10 = 21.66 s
235
Cycle time Tc = 200(21.66 s) = 4332.4 s = 72.2 min = 1.20 hr.
34.6 The cone-shaped part in Problem 34.5 is to be built using laminated object manufacturing. Layer
thickness = 0.20 mm. The laser beam can cut the sheet stock at a velocity of 500 mm/s. Compute
an estimate for the time required to build the part, if 10 s are lost each layer to lower the height of
the platform that holds the part and advance the sheet stock in preparation for the next layer.
Ignore cutting of the cross-hatched areas outside of the part since the cone should readily drop out
of the stack owing to its geometry.
Solution: For LOM, we need the circumference of each layer, which is the outline to be cut by
the laser beam. For a cone, the total surface area (not including the base) = R(R2 + h2)0.5
A = (35)(352 + 40)0.5 = 5844.2 mm2
Average surface area per layer = (5844.2 mm2)/(200 layers) = 29.22 mm2/layer
Since layer thickness t = 0.20 mm, circumference C = (29.22 mm2)/(0.20 mm) = 146.1 mm
Average time to cut a layer Ti = (146.1 mm)/(500 mm/s) + 10 s = 0.292 + 10 = 10.292 s
Number of layers nl = 40/0.20 = 200 layers
Tc = 200(10.292) = 2058.4 s = 34.3 min = 0.57 hr.
34.7 Stereolithography is to be used to build the part in Figure 34.1 (in text). Dimensions of the part are:
height = 125 mm, outside diameter = 75 mm, inside diameter = 65 mm, handle diameter = 12 mm,
handle distance from cup = 70 mm measured from center (axis) of cup to center of handle. The
handle bars connecting the cup and handle at the top and bottom of the part have a rectangular
cross-section and are 10 mm thick and 12 mm wide. The thickness at the base of the cup is 10
mm. The laser beam diameter = 0.25 mm, and the beam can be moved across the surface of the
photopolymer at = 500 mm/s. Layer thickness = 0.20 mm. Compute an estimate of the time
required to build the part, if 10 s are lost each layer to lower the height of the platform that holds
the part. Neglect postcuring time.
Solution: The part can be sliced into cross sections that have one of three basic shapes: (1) base,
which is 10 mm thick and includes the handle and handle bar; (2) cup ring and handle; and (3) top
of cup, which is 10 mm thick and consists of the cup ring, handle, and handle bar. Let us compute
the areas of the three shapes.
Area (1): A1 = (75)2/4 + (12)2/4 + (approximately)(12 x 32.5 – 0.5(12)2/4)
A1 = 4417.9 + 113.1 + (390.0 – 56.5) = 4864.5 mm2
Area (2): A2 = (752 – 652)/4 + (12)2/4 = 1099.6 + 113.1 = 1212.7 mm2
Area (3): A3 = (752 – 652)/4 + (12)2/4 + (approximately)(12 x 32.5 – 0.5(12)2/4)
A3 = 1099.6 +113.1 + (390.0 – 56.5) = 1546.2 mm2
Number of layers for each area:
(1) nl1 = (10 mm)/(0.2 mm/layer) = 50 layers
(2) nl2 = (125 – 10 – 10)/(0.2) = 525 layers
(3) nl3 = (10 mm)/(0.2 mm/layer) = 50 layers
Time to complete one layers for each of the three shapes:
(1) Ti1 = (4864.5 mm2)/(0.25 x 500) + 10 = 38.92 + 10 = 48.92 s
(2) Ti2 = (1212.7 mm2)/(0.25 x 500) + 10 = 9.70 + 10 = 19.70 s
(3) Ti3 = (1546.2 mm2)/(0.25 x 500) + 10 = 12.37 + 10 = 22.37 s
Total time for all layers Tc = 50(48.92) + 525(19.70) + 50(22.37)
Tc = 13,907 s + 231.78 min = 3.86 hr.
236
35 PROCESSING OF INTEGRATED CIRCUITS
Review Questions
35.1 What is an integrated circuit?
Answer. An integrated circuit is a collection of electronic devices (e.g., transistors, diodes,
resistors) that have been fabricated and electrically intraconnected onto the surface of a small flat
chip of semiconductor material.
35.2 Name some of the important semiconductor materials?
Answer. Important semiconductor materials include silicon (most important), germanium, and
gallium arsenide.
35.3 Describe the planar process.
Answer. The planar process refers to the fabrication of an IC chip by a sequence of layering
processes - adding, altering, and removing layers to create the devices and their intraconnection on
the IC chip.
35.4 What are the three major stages in the production of silicon-based integrated circuits?
Answer. The three stages are: (1) silicon processing, to produce very pure silicon and shape it
into wafers; (2) IC fabrication, in which layers are added, altered, and removed in selected regions
to form electronic devices on the face of the wafer; and (3) IC packaging, in which the wafers
are tested, cut into chips, and the chips are encapsulated in a package.
35.5 What is a clean room and explain the classification system by which clean rooms are rated?
Answer. A clean room is a room or rooms where the air is purified to reduce airborne particles.
The classification system indicates the quantity of particles of size 0.5 microns or greater per
cubic foot of air. For example, a class 100 clean room contains 100 or fewer particles of size 0.5
microns per cubic foot.
35.6 What are some of the significant sources of contaminants in IC processing?
Answer. Sources of contaminants include humans (bacteria, cigarette smoke, viruses, and hair),
and processing equipment (wear particles, oil, and dirt).
35.7 What is the name of the process most commonly used to grow single crystal ingots of silicon for
semiconductor processing?
Answer. It is the Czocralski process.
35.8 What are the alternatives to photolithography in IC processing?
Answer. Alternatives to photolithography are: electron lithography, X-ray lithography, and ion
lithography.
35.9 What is a photoresist?
Answer. A photoresist is a polymer that is sensitive to light radiation in a certain wavelength
range; the sensitivity causes either an increase or a decrease in solubility of the polymer to certain
chemicals.
35.10 Why is ultraviolet light favored over visible light in photolithography?
237
Answer. Because it has a shorter wavelength, the transferred images are sharper.
35.11 Name the three exposure techniques in photolithography.
Answer. The three exposure techniques are: (1) contact printing, (2) proximity printing, and (3)
projection printing.
35.12 What layer material is produced by thermal oxidation in IC fabrication?
Answer. SiO2 is grown on the surface of the Si wafer.
35.13 Define epitaxial deposition.
Answer. Epitaxial deposition involves growth of a crystalline structure on the surface of a
substrate which is an extension of the substrate's structure.
35.14 What are some of the important design functions of IC packaging?
Answer. Design functions of IC packaging include: provide electrical connections to external
circuits, encase chip for protection, and heat dissipation.
35.15 What is Rent's Rule?
Answer. Rent's Rule indicates the number of input/output terminals nio required for an integrated
circuit of a given number of internal circuits nc; the Rent's Rule equation is: nio = C nc
m, where C
and m are constants for a certain circuit type.
35.16 Name the two categories of component mounting to a printed circuit board.
Answer. The two types are: (1) through-hole mounting and (2) surface mount technology.
35.17 What is a DIP?
Answer. DIP stands for dual in-line package, an IC package with two rows of terminals on each
side of a rectangular body containing the IC chip.
35.18 What is the difference between postmolding and premolding in plastic IC chip packaging?
Answer. Postmolding refers to the use of transfer molding of epoxy around the chip and
leadframe to form the package; a premolded package is one in which an enclosure is molded
beforehand, and the chip and leadframe are then attached to it, adding a solid lid to complete the
package.
Multiple Choice Quiz
There are a total of 17 correct answers in the following multiple choice questions (some questions have
multiple answers that are correct). To attain a perfect score on the quiz, all correct answers must be
given, since each correct answer is worth 1 point. For each question, each omitted answer or wrong
answer reduces the score by 1 point, and each additional answer beyond the number of answers required
reduces the score by 1 point. Percentage score on the quiz is based on the total number of correct
answers.
35.1 How many electronic devices would be contained in an IC chip in order for it to be classified in
the VLSI category? (a) 1000, (b) 10,000, (c) 1 million, or (d) 100 million.
Answer. (c)
35.2 An alternative name for chip in semiconductor processing is which one of the following (one
answer)? (a) component, (b) device, (c) die, (d) package, or (e) wafer.
238
Answer. (c)
35.3 Which one of the following is the source of silicon for semiconductor processing? (a) pure Si in
nature, (b) SiC, (c) Si3N4, or (d) SiO2.
Answer. (d)
35.4 Which one of the following is the most common form of radiation used in photolithography? (a)
electronic beam radiation, (b) incandescent light, (c) infrared light, (d) ultraviolet light, or (e) X-ray.
Answer. (d)
35.5 After exposure to light, a positive resist becomes which of the following? (a) less soluble or (b)
more soluble to the chemical developing fluid.
Answer. (b)
35.6 Which of the following processes are used to add layers of various materials in IC fabrication
(more than one)? (a) chemical vapor deposition, (b) diffusion, (c) ion implantation, (d) physical
vapor deposition, (e) plasma etching, (f) thermal oxidation, or (g) wet etching.
Answer. (a), (d), and (f).
35.7 Which of the following are doping processes in IC fabrication (more than one)? (a) chemical
vapor deposition, (b) diffusion, (c) ion implantation, (d) physical vapor deposition, (e) plasma
etching, (f) thermal oxidation, or (g) wet etching.
Answer. (b) and (c).
35.8 Which one of the following impurity elements form electron acceptor (p-type) regions in silicon
wafers? (a) antimony, (b) arsenic, (c) boron, (d) nitrogen, (e) phosphorous, or (f) potassium.
Answer. (c)
35.9 Which one of the following is the most common metal for intraconnection of devices in a silicon
integrated circuit? (a) aluminum, (b) copper, (c) gold, (d) nickel, (e) silicon, or (f) silver.
Answer. (a)
35.10 Which etching process produces the more anisotropic etch in IC fabrication? (a) plasma etching,
or (b) wet chemical etching.
Answer. (a)
35.11 Which of the following are the two principal packaging materials used in IC packaging? (a)
aluminum, (b) aluminum oxide, (c) copper, (d) epoxies, or (e) silicon dioxide.
Answer. (b) and (d).
35.12 Which of the following metals are commonly used for wire bonding of chip pads to the lead frame
(two best answers)? (a) aluminum, (b) copper, (c) gold, (d) nickel, (e) silicon, or (f) silver.
Answer. (a) and (c).
Problems
Silicon Processing and IC Fabrication
35.1 A single crystal boule of silicon is grown by the Czochralski process to an average diameter of
110 mm with length = 1200 mm. The seed and tang ends are removed, which reduces the length
to 950 mm. The diameter is ground to 100 mm. A 30 mm wide flat is ground on the surface which
239
extends from one end to the other. The ingot is then sliced into wafers of thickness = 0.50 mm,
using an abrasive saw blade whose thickness = 0.33 mm. Assuming that the seed and tang
portions cut off the ends of the starting boule were conical in shape, determine: (a) the original
volume of the boule, mm3; (b) how many wafers are cut from it, assuming the entire 950 mm
length can be sliced; and (c) what is the volumetric proportion of silicon in the starting boule that is
wasted during processing?
Solution: (a) Total volume V = V1 (tang) + V2 (cylinder) + V3 (seed)
V1 = V3 = (cone in which h = 0.5(1200-950) = 125, D = 110, R = 55) = R2h/3
= 0.333(55)2(125) = 395,972 mm3.
V2 = R2L = (55)2(950) = 9,028,152 mm3
Total V = 2(395,972) + 9,028,152 = 9,820,095 mm3
(b) Number of wafers = 950/(0.50 + 0.33) = 1144.6 1144 wafers
(c) Area of one wafer Aw = Ac - As, where Ac = area of the circle of radius R = 50 mm, and As =
the area of the segment As created by the flat ground on the cylindrical surface.
Ac = R2 = (50)2 = 7854.0 mm2
The area of a segment of the circle created by the 30 mm chord As = R2/360 - 0.5R2 sin ,
where is the angle formed by two radii of the circle and the chord. . 0.5= sin-1(15/50) =
17.46.
= 34.92.
As = (50)2(34.92)/360 - 0.5(50)2 sin 34.92 = 761.8 - 715.5 = 46.3 mm2
Aw = Ac - As = 7854.0 - 46.3 = 7807.7 mm2
Volume of one wafer Vw = Awt = 7807.7(0.5) = 3903.8 mm3
Volume of 1144 wafers = 1144(3903.8) = 4,465,994 mm3
Volume wasted = 9,820,095 - 4,465,994 = 5,354,101 mm3
Proportion wasted = 5,354,101/9,820,095 = 54.52%.
35.2 A silicon boule is grown by the Czochralski process to a diameter of 5.25 inches and a length of 5
ft. The seed and tang ends are cut off, reducing the effective length to 48.00 in. Assume that the
seed and tang portions are conical in shape. The diameter is ground to 4.921 inch (125 mm). A
primary flat of width 1.625 inch is ground on the surface the entire length of the ingot. The ingot is
then sliced into wafers 0.025 inch thick, using an abrasive saw blade whose thickness = 0.0128
inch. Determine: (a) the original volume of the boule, in3; (b) how many wafers are cut from it,
assuming the entire 4 ft length can be sliced, and (c) what is the volumetric proportion of silicon in
the starting boule that is wasted during processing?
Solution: (a) Volume V = V1 (tang) + V2 (cylinder) + V3 (seed)
V1 = V3 = (cone in which h = 0.5(60-48) = 6.0, D = 5.25) = R2h/3 = 0.333(5.25/2)2(6.0)
= 43.295 in3.
V2 = D2L/4 = (5.25)2(48)/4 = 1039.082 in3
V = 2(43.295) + 1039.082 = 1125.672 in3
(b) Number of wafers = 48.0/(0.025 + 0.0128) = 48/(0.0378) = 1269.8 1269 wafers
(c) Area of one wafer Aw = Ac - As, where Ac = area of the circle of radius R = 4.921/2 = 2.4605
in., and As = the area of the segment As created by the flat ground on the cylindrical surface.
Ac = R2 = (2.4605)2 = 19.0194 in2
The area of a segment of the circle created by the 1.625 in. chord As = R2/360 - 0.5R2 sin ,
where is the angle formed by two radii of the circle and the chord. 0.5= sin-1(1.625/4.921) =
19.28.
240
= 38.56.
As = (2.4605)2(38.56)/360 - 0.5(2.4605)2 sin 38.56 = 2.0372 - 1.8869 = 0.1503 in2
Aw = Ac - As = 19.0194 - 0.1503 = 18.8691 in2
Volume of one wafer Vw = Awt = 18.8691(0.025) = 0.4717 in3
Volume of 1269 wafers = 1269(0.4717) = 598.621 in3
Volume wasted = 1125.672 - 598.621 = 527.051 in3
Proportion wasted = 527.051/1125.672 = 46.82%.
35.3 The processable area on a 125 mm diameter wafer is a 110 mm diameter circle. How many
square IC chips can be processed within this area, if each chip is 5 mm on a side? All chips must
lie completely within the processable area. Assume the cut lines (streets) between chips are of
negligible width.
Solution: Processable area A = D2/4 = (110)2/4 = 9503.3 mm2
Circumference C = D = 110= 345.6 mm
Chip area Ac = (5)2 = 25 mm2
The chips on the periphery of the processable area are lost. The number of lost chips can be
estimated by dividing the average chip dimension (assumed to be the length of the side) into the
circumference of the circle. Thus, our estimate of the chips produced is:
n = 9503.3/25 - 345.6/5 = 380.1 - 69.1 = 311 chips
35.4 Solve the previous problem, only use a wafer size of 200 mm whose processable area is 175 mm
in diameter. What is the percent increase in (a) number of chips, (b) wafer diameter, and (c)
processable wafer area, compared to the values in the previous problem?
Solution: Processable area A = D2/4 = (175)2/4 = 24,053 mm2
Circumference C = D = 175= 549.8 mm
Chip area Ac = (5)2 = 25 mm2
The chips on the periphery of the processable area are lost. The number of lost chips can be
estimated by dividing the average chip dimension (assumed to be the length of the side) into the
circumference of the circle. Thus, our estimate of the chips produced is:
n = 24,053/25 - 549.8/5 = 962 - 110 = 852 chips
(a) Increase in number of chips = (852 – 311)/311 = 174% increase
(b) Increase in wafer diameter = (200 – 125)/125 = 60% increase
(c) Increase in processable area = (24,053 - 9503.3)/ 9503.3 = 153% increase
Note: These results indicate the advantages of increasing wafer size.
35.5 A 4.0 inch wafer has a processable area that is only 3.65 inches in diameter. How many square
IC chips can be fabricated within this area, if each chip is 0.25 inch on a side? All chips must lie
completely within the processable area. Assume the cut lines (streets) between chips are of
negligible width.
Solution: Processable area A = D2/4 = (3.65)2/4 = 10.463 in2
Circumference C = D = 3.65= 11.467 in.
Chip area Ac = (0.25)2 = 0.0625 in2
The chips on the periphery of the processable area are lost. The number of lost chips can be
estimated by dividing the average chip dimension (assumed to be the length of the side) into the
circumference of the circle. Thus, our estimated number of chips produced is:
n = 10.463/0.0625 - 11.467/0.25 = 167.4 - 45.9 = 121.5 121 chips
241
35.6 Solve the previous problem, only use a wafer size of 6.0 inches whose processable area is 5.50
inches in diameter. What is the percent increase in number of chips compared to the 50%
increase in wafer diameter?
Solution: Processable area A = D2/4 = (4.65)2/4 = 16.982 in2
Circumference C = D = 4.65= 14.608 in.
Chip area Ac = (0.25)2 = 0.0625 in2
The chips on the periphery of the processable area are lost. The number of lost chips can be
estimated by dividing the average chip dimension (assumed to be the length of the side) into the
circumference of the circle. Thus, our estimated of the chips produced is:
n = 16.982/0.0625 - 14.608/0.25 = 271.7 - 58.4 = 213.3 213 chips
Note: the wafer diameter increases by about 27%, the wafer area increases by 62%, and the
number of chips increases by 76%. This is a principal motivation for using larger wafer diameters.
35.7 The surface of a silicon wafer is thermally oxidized, resulting in a SiO2 film that is 3 m thick. If
the starting thickness of the wafer was exactly 0.400 mm thick, what is the final wafer thickness?
Solution: A 3 m film requires a layer of silicon = 0.44d
Final thickness tf = 0.400 - 0.44(3 x 10-3) + 3 x 10-3 = 0.400 + 0.56(0.003) = 0.40168 mm
35.8 It is desired to etch out a region of a silicon dioxide film on the surface of a silicon wafer. The
SiO2 film is 3 m thick. The width of the etched-out area is specified to be 10 m. If the degree of
anisotropy for the etchant in the process is known to be 1.2, what should be the size of the opening
in the mask through which the etchant will operate?
Solution: A = d/u = 1.2, u = d/1.2 = 3/1.2 = 2.5 m.
Mask opening size = 10.0 - 2(2.5) = 5.0 mm
35.9 In the previous problem, if plasma etching is used instead of wet etching, and the degree of
anisotropy for plasma etching is infinity, what should be the size of the mask opening?
Solution: A = d/u = . u = d/= 0 m.
Mask opening size = 10.0 - 2(0) = 10.0 mm
IC Packaging
35.10 An integrated circuit used in a microprocessor will contain 1000 logic gates. Use Rent's Rule (C =
4.5 and m = 0.5) to determine the approximate number of input/output pins required in the
package.
Solution: Rents rule: nio = Cnc
m = 4.5(1000)0.5 = 142.3 142 input/output pins
35.11 A dual-in-line package has a total of 48 leads. Use Rent's Rule (C = 4.5 and m = 0.5) to determine
the approximate number of logic gates that could be fabricated in the IC chip for this package.
Solution: 48 = 4.5(nc)0.5
nc
0.5 = 48/4.5 = 10.667
nc = (10.667)2 = 113.8 113 logic gates
35.12 It is desired to determine the effect of package style on the number of circuits (logic gates) that
can be fabricated onto an IC chip to which the package is assembled. Using Rent's Rule (C = 4.5
and m = 0.5), compute the estimated number of devices (logic gates) that could be placed on the
chip in the following cases: (a) a DIP with 16 I/O pins on a side - a total of 32 pins; (b) a square
chip carrier with 16 pins on a side - a total of 64 I/O pins; and (c) a pin grid array with 16 by 16
pins - a total of 256 pins.
242
Solution: (a) Using Rent’s rule: nio = 4.5(nc)0.5 , find nc if nio = 32.
nc
0.5 = 32/4.5 = 7.11
nc = 50.6 50 logic gates
(b) Using Rent’s rule: nio = 4.5(nc)0.5 , find nc if nio = 64.
nc
0.5 = 64/4.5 = 14.22
nc = 202.3 202 logic gates
(c) Using Rent’s rule: nio = 4.5(nc)0.5 , find nc if nio = 256.
nc
0.5 = 256/4.5 = 56.89
nc = 3236.3 3236 logic gates
35.13 In the Equation for Rent's Rule with C = 4.5 and m = 0.5, determine the value of nio and nc at
which the number of logic gates equals the number of I/O terminals in the package.
Solution: We have two equations and two unknowns: (1) nio = 4.5nc
0.5 and (2) nio = nc.
Using nio in place of nc in Eq. (1), nio = 4.5nio
0.5
ln nio = ln 4.5 + 0.5 ln nio
ln nio - 0.5 ln nio = 0.5 ln nio = ln 4.5 = 1.50408
ln nio = 3.00816
nio = nc = 20.25. The closest possible values are nio = nc = 20 or 21.
35.14 A static memory device will have a two dimensional array with 64 by 64 cells. Compare the
number of input/output pins required using (a) Rent's Rule (C = 6.0 and m = 0.12), and (b) the
alternative computation given in Eq. (35.11).
Solution: (a) Rent’s rule: nio = 6.0 nc
0.12 = 6.0 (64 x 64)0.12 = 6.0(4096)0.12 = 16.3 16 pins
(b) Eq. (35.11): nio = 1.4427 ln (64 x 64) = 1.4427 ln 4096 = 12 pins.
35.15 To produce a 1 megabit memory chip, how many I/O pins are predicted by: (a) Rent's Rule (C =
6.0 and m = 0.12), and (b) the alternative computation method given in Eq. (35.11)?
Solution: (a) Rent’s rule: nio = 6.0 nc
0.12 = 6.0 (1,000,000)0.12 = 31.5 31 pins
(b) Eq. (35.11): nio = 1.4427 ln nc = 1.4427 ln (1,000,000) = 19.9 19 pins.
35.16 Suppose it is desired to produce a memory device that will be contained in a dual-in-line package
with 32 I/O leads. How many memory cells can be contained in the device, as estimated by: (a)
Rent's Rule (C = 6.0 and m = 0.12), and (b) the alternative computation method given in Eq.
(35.11)?
Solution: (a) Rent’s rule: nio = 6.0 nc
0.12
nc
0.12 = nio/6.0 = 32/6 = 5.333
nc = (5.333)1/.12 = (5.333)8.333 = 1,143,728 memory cells
(b) Eq. (35.11): nio = 1.4427 ln nc
32 = 1.4427 ln nc
ln nc = 32/1.4427 = 22.18
nc = exp(22.18) = 4,294,967,305 memory cells.
Yields in IC Processing
35.17 Given the following: crystal yield Yc = 50%, crystal-to- slice yield Ys = 50%, wafer yield Yw = 70%,
multiprobe yield Ym = 60%, and final test yield Yt = 90%. If a starting boule weighs 75 kg, what is
the final weight of silicon that results after final test?
243
Solution: Overall yield Y = Yc Ys Yw Ym Yt = (0.5)(0.50)(0.70)(0.60)(0.90) = 0.0945
Wf = YWi = 0.0945(75) = 7.09 kg
35.18 A silicon wafer with a nominal diameter of 100 mm is processed to fabricate square chips of 5
mm on a side. The area of the processed chips occupies 65.25% of the total wafer area on one
side. The density of point defects in the surface area is 0.027 defects/cm2. Determine the number
of good chips using: (a) the Boltzmann yield estimate, Eq. (35.14), and (b) the Bose-Einstein yield
estimate, Eq. (35.15).
Solution: Wafer area A = (100)2/4 = 7854 mm2
Chip area A = 0.6525(7854) = 5125 mm2 = 51.25 cm2
Area of one chip Ac = 5 x 5 = 25 mm.
Number of chips = 5125/25 = 205 chips total.
(a) Eq. (35.14): Ym = e-AD = e-51.25(0.027) = e-1.384 = 0.251
Number of good chips = 0.251(205) = 51.4 51 good chips
(b) Eq. (35.15): Ym = 1/(1 + AD) = 1/(1 + 51.25 x 0.027) = 1/2.384 = 0.419
Number of good chips = 0.419(205) = 85.9 85 good chips
35.19 A 5-inch diameter wafer is processed over a circular area that is 4.75 inches in diameter. The
density of point defects in the surface area is 0.32 defects/in2. Determine the multiprobe yield
using: (a) the Boltzmann yield estimate, Eq. (35.14), and (b) the Bose-Einstein yield estimate, Eq.
(35.15).
Solution: Processable area A = (4.75)2/4 = 17.72 in2
(a) Eq. (35.14): Ym = e-AD = e-17.72(0.32) = e-5.67 = 0.00344
(b) Eq. (35.15): Ym = 1/(1 + AD) = 1/(1 + 17.72 x 0.32) = 1/6.67 = 0.15
35.20 The yield of good chips in multiprobe for a certain batch of wafers is 83%. The wafers have a
nominal diameter of 150 mm with a processable area that is 135 mm in diameter. If the defects
are all assumed to be point defects that are uniformly distributed over the surface (Poisson
distribution), what is their density D?
Solution: Eq. (35.14): Ym = e-AD
Processable area A = (135)2/4 = 14,314 mm2 = 143.14 cm2
0.83 = e-143.14D
ln 0.83 = -143.14D
-0.18633 = -143.14D
D = 0.18633/143.14 = 0.00130 defects/cm2.
35.21 In the previous problem, determine the density of point defects using Bose-Einstein statistics, Eq.
(35.15), as the method of estimating yield.
Solution: Eq. (35.15): Ym = 1/(1 + AD)
Processable area A = (135)2/4 = 14,314 mm2 = 143.14 cm2
0.83 = 1/(1 + 143.14D)
0.83(1 + 143.14D) = 0.83 + 118.806D = 1
118.806D = 1 - 0.83 = 0.17
D = 0.17/118.806 = 0.00143 defects/cm2.
35.22 A silicon wafer has a processable area of 20.0 in2. The yield of good chips on this wafer is Ym =
75%. If the defects are all assumed to be point defects that are uniformly distributed over the
surface (Poisson distribution), what is the density of point defects D?
244
Solution: Eq. (35.14): Ym = e-AD
0.75 = e-20D
ln 0.75 = -20D
-0.2877 = -20D
D = 0.2877/20 = 0.01438 defects/in2.
245
36 ELECTRONICS ASSEMBLY AND PACKAGING
Review Questions
36.1 What are the functions of a well-designed electronics package?
Answer. The principal functions are: (1) power distribution and signal interconnection, (2)
structural support, (3) environmental protection, (4) heat dissipation, (5) minimize delays in signal
transmission.
36.2 Identify the levels of packaging hierarchy in electronics.
Answer. The levels are: (0) chip intraconnections, (1) chip to package, (2) component to PCB, (3)
PCB to rack or chassis, and (4) wiring and cabling inside a cabinet.
36.3 What is the difference between a track and a land on a printed circuit board?
Answer. A track is a copper conducting path on a PCB, while a land is a small copper area for
electrically attaching components.
36.4 Define what a printed circuit board (PCB) is.
Answer. A PCB is a laminated flat panel of insulating material to which electronic components
are attached and electrically interconnected.
36.5 Name the three principal types of printed circuit board?
Answer. The three types are: (1) single -sided board, (2) double -sided board, and (3) multilayer
board.
36.6 What is a via hole in a printed circuit board?
Answer. A via hole is a hole in the printed circuit board whose sides are plated with copper to
serve as a conducting path from one side of a PCB to the other or between intermediate layers in
a multilayer board.
36.7 What are the two basic resist coating methods for printed circuit boards?
Answer. The two methods are: (1) screening, such as silk screening, and (2) photolithography.
36.8 What is etching used for in PCB fabrication?
Answer. Etching is used to remove copper cladding on the PCB surface to define the tracks and
lands of the circuit.
36.9 What is continuity testing, and when is it performed in the PCB fabrication sequence?
Answer. Continuity testing is an electrical test in which contact probes are brought into contact
with track and land areas to insure the existence of electrical conduction paths. Continuity tests
are generally used: (1) after the bare board has been fabricated, and (2) after the board has been
populated with components.
36.10 What are the two main categories of printed circuit board assemblies, as distinguished by the
method of attaching components to the board?
Answer. The two categories are: (1) pin-in-hole technology, also known as through-hole
technology; and (2) surface mount technology.
246
36.11 What are some of the reasons and defects that make rework an integral step in the PCB
fabrication sequence?
Answer. Rework is required to correct the following types of defects: (1) replace defective
components, (2) insert missing components, (3) repair faulty solder joints, and (4) repair of copper
film that has lifted from the substrate surface.
36.12 Identify some of the advantages of surface mount technology over conventional through-hole
technology.
Answer. Advantages of SMT include: (1) smaller components, (2) higher packing densities, (3)
components can be mounted on both sides of the board, (4) smaller PCBs are possible for the
same function, (5) reduced number of holes drilled in the board, and (6) certain undesirable
electrical effects are reduced, such as spurious surface capacitances and inductances.
36.13 Identify some of the limitations and disadvantages of surface mount technology?
Answer. Limitations and disadvantages of SMT include: (1) components are more difficult for
humans to handle, (2) SMT components are generally more expensive than THT components, (3)
inspection, testing, and rework are more difficult, and (4) certain types of components are not
available in SMT.
36.14 What are the two methods of component placement and soldering in surface mount technology?
Answer. The two methods are: (1) adhesive bonding and wave soldering, and (2) solder paste and
reflow soldering.
36.15 What is a solder paste?
Answer. A solder paste is a suspension of solder powders in a flux binder. The flux binder
includes an adhesive that attaches the SMT components to the board surface. The solder
constitutes about 85% of the total volume of the paste.
36.16 Identify the two basic methods of making electrical connections.
Answer. The two methods are: (1) soldering, and (2) pressure connections.
36.17 Define crimping in the context of electrical connections?
Answer. Crimping involves the mechanical forming of a terminal barrel to form a permanent
connection with the stripped end of a conductor wire.
36.18 What is press-fit technology in electrical connections?
Answer. A press-fit technology in the context of electrical connections is an interference fit
between a terminal pin and the plated hole into which it is inserted.
36.19 Define what a terminal block is?
Answer. A terminal block consists of a series of evenly spaced receptacles that allow connection
of individual wires or terminals.
36.20 What is a pin connector?
Answer. A pin connector is a connector with multiple pins or blades that are inserted into the
holes of a mating receptacle to establish electrical contact.
247
Multiple Choice Quiz
There are a total of 17 correct answers in the following multiple choice questions (some questions have
multiple answers that are correct). To attain a perfect score on the quiz, all correct answers must be
given, since each correct answer is worth 1 point. For each question, each omitted answer or wrong
answer reduces the score by 1 point, and each additional answer beyond the number of answers required
reduces the score by 1 point. Percentage score on the quiz is based on the total number of correct
answers.
36.1 The second level of packaging refers to which one of the following? (a) component to printed
circuit board, (b) IC chip to package, (c) intraconnections on the chip, or (d) wiring and cabling
connections.
Answer. (a)
36.2 Surface mount technology is included within which one of the following levels of packaging? (a)
zeroth, (b) first, (c) second, (d) third, or (e) fourth.
Answer. (c)
36.3 Card-on-board (COB) packaging refers to which one of the following levels in the electronics
packaging hierarchy? (a) zeroth, (b) first, (c) second, (d) third, or (e) fourth.
Answer. (d)
36.4 Which of the following polymeric materials is commonly used as an ingredient in the insulation
layer of a printed circuit board (more than one)? (a) copper, (b) E-glass, (c) epoxy, (d) phenolic,
(e) polyethylene, and (f) polypropylene.
Answer. (c) and (d).
36.5 Typical thickness of the copper layer in a printed circuit board is which one of the following? (a)
0.100 inch, (b) 0.010 inch, (c) 0.001 inch, or (d) 0.0001 inch.
Answer. (c)
36.6 Photolithography is widely used in PCB fabrication. Which of the following is the most common
resist type used in the processing of PCBs? (a) negative resists, or (b) positive resists.
Answer. (a)
36.7 Which of the following plating processes has the higher deposition rate in PCB fabrication? (a)
electroless plating, or (b) electroplating.
Answer. (b)
36.8 In addition to copper, which one of the following is another common metal plated onto a PCB? (a)
aluminum, (b) gold, (c) nickel, or (d) tin.
Answer. (b)
36.9 Which of the following are the soldering processes used to attach components to printed circuit
boards in through-hole technology (more than one)? (a) hand soldering, (b) infrared soldering, (c)
reflow soldering, (d) torch soldering, and (e) wave soldering.
Answer. (a) and (e).
36.10 In general, which of the following technologies results in greater problems during rework? (a)
surface mount technology, or (b) through-hole technology.
248
Answer. (a)
36.11 Which of the following are methods of forming electrical connections (more than one)? (a)
soldering, (b) insulation displacement, (c) retaining rings, or (d) pressure connections.
Answer. (a), (b), and (d)
36.12 Which of the following electrical connection methods produce a separable connection (more than
one): (a) crimping of terminals, (b) terminal blocks, (c) press fitting, and (d) sockets.
Answer. (b) and (d)
249
37 MICROFABRICATION TECHNOLOGIES
Review Questions
37.1 Define microelectromechanical system.
Answer: A microelectromechanical system (MEMS) is a miniaturized system consisting of both
electronic and mechanical components.
37.2 What is the approximate size scale in microsystem technology?
Answer: The size range is 10-6 m to 10-3 m, typically.
37.3 Why is it reasonable to believe that microsystem products would be available at lower costs than
products of larger, more conventional size?
Answer: Because less material is used in microsystem products.
37.4 What is a hybrid microsensor?
Answer: A hybrid microsensor is a sensing element (transducer) combined with electronics
components in the same device.
37.5 What are some of the basic types of microsystem devices?
Answer: The text indicates four classifications: (1) microsensors, (2) microactuators, (3)
microstructures and microcomponents, and (4) microsystems and micro-instruments.
37.6 Why is silicon a desirable work material in microsystem technology?
Answer: The reasons given in the text are: (1) the microdevices in MST often include electronic
circuits, so both the circuit and the microdevice can be fabricated in combination on the same
substrate; (2) in addition to its desirable electronic properties, silicon also possesses useful
mechanical properties, such as high strength and elasticity, good hardness, and relatively low
density; (3) the technologies for processing silicon are well-established, owing to their widespread
use in microelectronics; and (4) use of single-crystal silicon permits the production of physical
features to very close tolerances.
37.7 What is meant by the term aspect ratio in microsystem technology?
Answer: The aspect ratio is the height-to-width ratio of the features produced in the MST device.
37.8 What is the difference between bulk micromachining and surface micromachining?
Answer: Bulk micromachining refers to a relatively deep wet etching process into a single-crystal
silicon substrate (Si wafer). Surface micromachining refers to the planar structuring of the
substrate surface using much more shallow layering processes.
37.9 What is meant by the term nanotechnology?
Answer: Nanotechnology refers to devices and structures that are even smaller than in
microsystems, as well as the associated fabrication processes which involve the control of feature
sizes measured on the nanometer (one nm = 10-9 m) scale.
250
Multiple Choice Quiz
There are a total of 15 correct answers in the following multiple choice questions (some questions have
multiple answers that are correct). To attain a perfect score on the quiz, all correct answers must be
given, since each correct answer is worth 1 point. For each question, each omitted answer or wrong
answer reduces the score by 1 point, and each additional answer beyond the number of answers required
reduces the score by 1 point. Percentage score on the quiz is based on the total number of correct
answers.
37.1 Microsystem technology includes which of the following (more than one)?(a) LIGA technology,
(b) microelectromechanical systems, (c) micromachines, (d) nanotechnology, (e) precision
engineering.
Answer: (a), (b), and (c).
37.2 The typical range of feature sizes in microsystem technology is which one of the following? (a) 10-
3 m to 10-2 m, (b)10-6 m to 10-3 m, (c) 10-9 m to 10-6 m.
Answer: (b).
37.3 Which of the following are current applications of microsystem technology in modern automobiles
(more than one)? (a) air-bag release sensors, (b) alcohol blood level sensors, (c) driver
identification sensors for theft prevention, (d) oil pressure sensors, (e) temperature sensors for
cabin climate control.
Answer: (a), (d), and (e).
37.4 The most common work material used in microsystem technology is which one of the following?
(a) boron, (b) gold, (c) nickel, (d) potassium hydroxide, (e) silicon.
Answer: (e).
37.5 The aspect ratio in microsystem technology is best defined by which one of the following? (a)
degree of anisotropy in etched features, (b) height-to-width ratio of the fabricated features, (c)
height-to-width ratio of the MST device, (d) length-to-width ratio of the fabricated features, (e)
thickness-to-length ratio of the MST device.
Answer: (b).
37.6 Which of the following forms of radiation have wavelengths shorter than the wavelength of
ultraviolet light used in photolithography (more than one)? (a) electron beam radiation, (b) natural
light, (c) X-ray radiation.
Answer: (a) and (c).
37.7 Bulk micromachining refers to a relatively deep wet etching process into a single-crystal silicon
substrate: (a) true or (b) false.
Answer: (a).
37.8 In the LIGA process, the letters LIGA stand for which one of the following? (a) let it go already,
(b) little itty-bitty grinding apparatus, (c) lithographic applications, (d) lithography, electrodeposition,
and plastic molding, (e) lithography, grinding, and alteration.
Answer: (d).
37.9 Photofabrication means the same process as photolithography. (a) True or (b) false.
251
Answer: (b). Photofabrication is an industrial process in which ultraviolet exposure through a
pattern mask causes a significant increase in the chemical solubility of an optically clear material,
thus permitting a suitable etchant to remove the exposed regions much more rapidly. This is not
the same as photolithography, in which resists are used to determine the regions to be etched.
37.10 The typical range of feature sizes in nanotechnology is which one of the following? (a) 10-3 m to
10-2 m, (b)10-6 m to 10-3 m, (c) 10-9 m to 10-6 m.
Answer: (c).
252
38 NUMERICAL CONTROL AND INDUSTRIAL
ROBOTICS
Review Questions
38.1 Identify and briefly describe the three basic components of a numerical control system.
Answer. The three basic components are: (1) part program, (2) machine control unit, and (3)
processing equipment. The part program is the detailed set of commands to be followed by the
processing equipment. Each command specifies a position or motion that is to be accomplished by
the workhead relative to the processed object. The machine control unit in modern NC
technology is a microcomputer which stores the program and executes it by converting each
command into actions by the processing equipment, one command at a time. The processing
equipment accomplishes the sequence of processing steps to transform the starting workpart into
a completed part. It operates under the control of the machine control unit according to the set of
instructions contained in the part program.
38.2 What is the difference between point-to-point and continuous path in a motion control system?
Answer. In point-to-point, the motion is from one location in space to the next with no regard for
the path taken between starting and final locations. In continuous path, the trajectory of the
movement is controlled.
38.3 What is the difference between absolute positioning and incremental positioning?
Answer. In absolute positioning, the locations are defined relative to the origin of the axis system.
In incremental positioning, each succeeding location is defined relative to the previous location.
38.4 What is the difference between an open loop positioning system and a closed loop positioning
system?
Answer. In a closed loop system, measurements of the output (position) are fed back to verify
that it corresponds to the desired input value. In an open loop system, there is no feedback of the
output value.
38.5 Under what circumstances is a closed loop positioning system preferable to an open loop system?
Answer. When there is a significant reaction force resisting the motion of the positioning system,
a closed loop system is preferred.
38.6 Explain the operation of an optical encoder.
Answer. An optical encoder is a sensor for measuring angular position and rotational velocity. It
consists of a light source, a photodetector, and a disk containing a series of slots through which the
light source can shine to energize the photodetector. The disk is connected, either directly or
through a gear train, to a rotating shaft whose angular position and velocity are to be measured.
As the shaft rotates, the slots cause the light source to be seen by the photocell as a series of
flashes, which are converted into an equivalent series of electrical pulses. By counting the pulses
and computing the frequency of the pulse train, angular position and rotational speed can be
determined.
38.7 Why should the electromechanical system be the limiting factor in control resolution rather than
the controller storage register?
253
Answer. Because the control resolution in the controller storage register can be increased simply
by increasing the number of bits used to define the axis location.
38.8 What is manual data input in NC part programming?
Answer. Manual data input refers to a method of programming in which the machine tool
operator accomplishes the programming of the NC machine using a menu-driven procedure.
Programming is simplified to minimize the amount of training required by the operator.
38.9 Identify some of the non-machine tool applications of numerical control.
Answer. The applications include: (1) arc welding and resistance welding, (2) electronic
component insertion, (3) electrical wire wrap machines, (4) drafting, (5) tape laying for fiber
reinforced polymer composites, and (6) coordinate measuring machines.
38.10 What are some of the benefits usually cited for NC compared to using manual alternative
methods?
Answer. Advantages of NC include: (1) reduced non- productive time, (2) lower manufacturing
lead times, (3) simpler fixtures, (4) greater flexibility, (5) improved accuracy, and (6) reduced
human error.
38.11 What is an industrial robot?
Answer. An industrial robot is a programmable machine possessing certain anthropomorphic
features. The most common feature is a manipulator (arm) that can be programmed to perform
industrial tasks.
38.12 How is an industrial robot similar to numerical control?
Answer. They are both positioning systems that can be programmed and reprogrammed.
38.13 What is an end effector?
Answer. An end effector is the special tooling that is attached to the robot's wrist to perform a
particular application. A gripper is one form of end effector.
38.14 In robot programming, what is the difference between powered leadthrough and manual
leadthrough?
Answer. In powered leadthrough, a teach pendant which controls the drive motors of the
individual joints is used to move the manipulator into the desired joint positions, which are then
recorded into memory. In manual leadthrough, the manipulator is physically moved through the
desired sequence of positions, which are recorded into memory for later execution.
38.15 What is a programmable logic controller?
Answer. A programmable logic controller (PLC) is a microcomputer-based device that uses
programmed instructions to implement logic, sequencing, timing, counting, and arithmetic functions
to control industrial machinery and processes.
Multiple Choice Quiz
There are a total of 13 correct answers in the following multiple choice questions (some questions have
multiple answers that are correct). To attain a perfect score on the quiz, all correct answers must be
given, since each correct answer is worth 1 point. For each question, each omitted answer or wrong
answer reduces the score by 1 point, and each additional answer beyond the number of answers required
254
reduces the score by 1 point. Percentage score on the quiz is based on the total number of correct
answers.
38.1 The standard coordinate system for numerical control machine tools is based on which one of the
following? (a) Cartesian coordinates, (b) cylindrical coordinates, or (c) polar coordinates.
Answer. (a)
38.2 Identify which of the following applications are point-to- point and not continuous path operations
(more than one): (a) arc welding, (b) drilling, (c) hole punching in sheet metal, (d) milling, (e) spot
welding, and (f) turning.
Answer. (b), (c), and (e).
38.3 The ability of a positioning system to return to a previously defined location is measured by which
one of the following terms? (a) accuracy, (b) control resolution, or (c) repeatability.
Answer. (c)
38.4 The APT command GORGT is which of the following (more than one)? (a) continuous path
command, (b) geometry statement involving a volume of revolution about a central axis, (c) name
of the monster in a 1960s Japanese science fiction movie, (d) point-to-point command, or (e) tool
path command in which the tool must Go Right in the next move.
Answer. (a) and (e).
38.5 The arm-and-body of a robot manipulator generally performs which one of the following functions
in an application? (a) orientation, or (b) positioning.
Answer. (b)
38.6 A SCARA robot is normally associated with which of the following applications (one answer)? (a)
arc welding, (b) assembly, (c) inspection, (d) machine loading and unloading, or (e) resistance
welding.
Answer. (b)
38.7 In robotics, spray painting applications are which of the following? (a) continuous path, or (b)
point-to-point.
Answer. (a)
38.8 Which of the following are characteristics of work situations that tend to promote the substitution
of a robot in place of a human worker (more than one)? (a) frequent job changeovers, (b)
hazardous work environment, (c) repetitive work cycle, (d) multiple work shifts, and (e) task
requires mobility.
Answer. (b), (c), and (d).
Problems
Open Loop Positioning Systems
38.1 A leadscrew with a 7.5 mm pitch drives a worktable in a NC positioning system. The leadscrew is
powered by a stepping motor which has 250 step angles. The worktable is programmed to move a
distance of 120 mm from its present position at a travel speed of 300 mm/min. Determine: (a) how
many pulses are required to move the table the specified distance, and (b) what is the required
motor speed and pulse rate to achieve the desired table speed?
255
Solution: (a) = 360/ns = 360/250 = 1.44
np = 360x/p= 360(100)/(7.5 x 1.44) = 4000 pulses
(b) N = vt /p = (300 mm/min)/(7.5 mm/rev) = 40 rev/min.
fp = vt ns/ 60p = 300(250)/(60 x 7.5) = 166.67 Hz
38.2 Referring to the previous problem, the mechanical inaccuracies in the open loop positioning system
can be described by a normal distribution whose standard deviation = 0.005 mm. The range of the
worktable axis is 500 mm, and there are 12 bits in the binary register used by the digital controller
to store the programmed position. For the positioning system, determine: (a) control resolution, (b)
accuracy, and (c) repeatability. (d) What is the minimum number of bits that the binary register
should have so that the mechanical drive system becomes the limiting component on control
resolution?
Solution: (a) CR1 = p/ns = 7.5 mm/250 = 0.03 mm.
CR2 = L/2B = 500/212 – 1 = 500/4095 = 0.122 mm.
CR = Max{CR1, CR2} = Max{0.03, 0.122} = 0.122 mm.
(b) Accuracy = 0.5 CR + 3 = 0.5(0.122) + 3(0.005) = 0.076 mm.
(c) Repeatability = 3 = 3(0.005) = ± 0.015 mm.
(d) In order for the mechanical errors to be the limiting factor in control resolution in this problem,
set CR1 = CR2.
Thus, 0.03 = 500/(2B –1)
2B – 1 = 500/0.016 = 16,667
2B = = 16,668
B ln 2 = ln 16,668
0.69315 B = 9.7212
B = 14.025 Use B = 15 bits
38.3 A stepping motor has 200 step angles. Its output shaft is directly coupled to leadscrew with pitch =
0.250 in. A worktable is driven by the leadscrew. The table must move a distance of 5.00 in from
its present position at a travel speed of 20.0 in/min. Determine: (a) how many pulses are required
to move the table the specified distance, and (b) what is the required motor speed and pulse rate
to achieve the desired table speed?
Solution: (a) = 360/ns = 360/200 = 1.8
np = 360x/p= 360(5.0)/(0.25 x 1.8) = 4000 pulses
(b) N = vt /p = (20 in/min)/(0.25 in/rev) = 80 rev/min.
fp = vt ns/ 60p = 20(200)/(60 x 0.25) = 266.67 Hz
38.4 A stepping motor with 240 step angles is coupled to a leadscrew through a gear reduction of 5:1 (5
rotations of the motor for each rotation of the leadscrew). The leadscrew has 6 threads/in. The
worktable driven by the leadscrew must move a distance = 10.00 in at a feed rate of 30.0 in/min.
Determine: (a) number of pulses required to move the table, and (b) the required motor speed and
pulse rate to achieve the desired table speed.
Solution: (a) = 360/ns = 360/240 = 1.5
np = 360 rg x/p= 360(5)(10)/(0.1667 x 1.5) = 72,000 pulses
(b) N = rg fr /p = 5(30 in/min)/(0.1667 in/rev) = 900 rev/min.
fp = rg fr ns/ 60p = 5(30)(240)/(60 x 0.1667) = 3600 Hz
256
38.5 The drive unit for a positioning table is driven by a leadscrew directly coupled to the output shaft
of a stepping motor. The pitch of the leadscrew = 0.18 in. The table must have a linear speed = 35
in/min, and a positioning accuracy = 0.001 in. Mechanical errors in the motor, leadscrew, and table
connection are characterized by a normal distribution with standard deviation = 0.0002 in.
Determine: (a) the minimum number of step angles in the stepping motor to achieve the accuracy,
(b) the associated step angle, and (c) the frequency of the pulse train required to drive the table at
the desired speed.
Solution: (a) Accuracy = 0.5 CR + 3 
0.001 = 0.5 CR + 3(0.0002) = 0.5 CR + 0.0006
0.001 - 0.0006 = 0.0004 = 0.5 CR
CR = 0.0008 in.
Assume CR = CR1
CR1 = 0.0008 = p/ns = 0.18/ns
ns = 0.18/0.0008 = 225 step angles
(b) = 360/225 = 1.6°
(c) fp = vt ns /60p = 35(225)/(60 x 0.18) = 729.167 Hz
38.6 The positioning table for a component insertion machine uses a stepping motor and leadscrew
mechanism. The design specifications require a table speed of 40 in/min and an accuracy = 0.0008
in. The pitch of the leadscrew = 0.2 in, and the gear ratio = 2:1 (2 turns of the motor for each turn
of the leadscrew). The mechanical errors in the motor, gear box, leadscrew, and table connection
are characterized by a normal distribution with standard deviation = 0.0001 in. Determine: (a) the
minimum number of step angles in the stepping motor, and (b) the frequency of the pulse train
required to drive the table at the desired maximum speed.
Solution: (a) Accuracy = 0.5 CR + 3 
0.0008 = 0.5 CR + 3(0.0001) = 0.5 CR + 0.0003
0.0008 - 0.0003 = 0.0005 = 0.5 CR
CR = 0.001 in.
Assume CR = CR1
CR1 = 0.001 = p/(rg ns ) = 0.2/2ns
ns = 0.2/(2 x 0.001) = 100 step angles
(b) fp = rg vt ns /60p = 2(40)(100)/(60 x 0.2) = 667.67 Hz
38.7 The drive unit of a positioning table for a component insertion machine is based on a stepping
motor and leadscrew mechanism. The specifications are for the table speed to be 25 mm/s over a
600 mm range and for the accuracy to be 0.025 mm. The pitch of the leadscrew = 4.5 mm, and
the gear ratio = 5:1 (5 turns of the motor for each turn of the leadscrew). The mechanical errors
in the motor, gear box, leadscrew, and table connection are characterized by a normal distribution
with standard deviation = 0.005 mm. Determine: (a) the minimum number of step angles in the
stepping motor, and (b) the frequency of the pulse train required to drive the table at the desired
maximum speed.
Solution: (a) Accuracy = 0.5 CR + 3 
0.025 = 0.5 CR + 3(0.005) = 0.5 CR + 0.015
0.025 - 0.015 = 0.010 = 0.5 CR
CR = 0.02 mm.
Assume CR = CR1
CR1 = p/(rg ns )
257
ns = 4.5/(5 x 0.02) = 45 step angles
(b) fp = rg vt ns /p = 5(25)(45)/4.5 = 1250 Hz
38.8 The two axes of an x-y positioning table are each driven by a stepping motor connected to a
leadscrew with a 10:1 gear reduction. The number of step angles on each stepping motor is 20.
Each leadscrew has a pitch = 5.0 mm and provides an axis range = 300.0 mm. There are 16 bits
in each binary register used by the controller to store position data for the two axes. (a) What is
the control resolution of each axis? (b) What are the required the rotational speeds and
corresponding pulse train frequencies of each stepping motor in order to drive the table at 600
mm/min in a straight line from point (25,25) to point (100,150)? Ignore acceleration.
Solution: (a) CR1 = p/rgns = 5.0/(10 x 20) = 0.025 mm
CR2 = L/(2B – 1)= 300/(216 – 1) = 300/65,535 = 0.00458 mm
CR = Max{0.025, 0.00458} = 0.025 mm
(b) vt = 600 mm/min from (25, 25) to (100, 150)
x = 100 - 25 = 75 mm, y = 150 - 25 = 125 mm
Angle A = tan-1(125/75) = 59
vtx = 600 cos 59 = 308.7 mm/min
Nx = rgvtx/p = 10(308.7)/5.0 = 617.4 rev/min.
fpx = Nx ns/60 = 617.4(20)/60 = 205.8 Hz
vty = 600 sin 59 = 514.5 mm/min.
Ny = rgvty/p = 10(514.5)/5.0 = 1029 rev/min.
fpx = Ny ns /60 = 1029(20)/60 = 343.0 Hz
Closed Loop Positioning Systems
38.9 A NC machine tool table is powered by a servomotor, leadscrew, and optical encoder. The
leadscrew has a pitch = 5.0 mm and is connected to the motor shaft with a gear ratio of 16:1 (16
turns of the motor for each turn of the leadscrew). The optical encoder is connected directly to
the leadscrew and generates 200 pulses/rev of the leadscrew. The table must move a distance =
100 mm at a feed rate = 500 mm/min. Determine: (a) the pulse count received by the control
system to verify that the table has moved exactly 100 mm; and (b) the pulse rate and (c) motor
speed that correspond to the feed rate of 500 mm/min.
Solution: (a) x = p np/ns ; Rearranging, np = xns/p = 100(200)/5 = 4000 pulses.
(b) fp = fr ns /60p = 500(200)/60(5) = 333.3 Hz
(c) N = rg fr /p = 16 x 500/5 = 1600 rev/min.
38.10 Same as the previous problem, except that the optical encoder is directly coupled to the motor
shaft rather than to the leadscrew.
Solution: (a) x = p np/rg ns ; Rearranging, np = rg x ns/p = 16(100)(200)/5 = 64,000 pulses.
(b) fp = rg fr ns /60p = 16(500)(200)/60(5) = 5333.3 Hz
(c) N = rg fr /p = 16 x 500/5 = 1600 rev/min.
38.11 The worktable of a numerical control machine tool is driven by a closed loop positioning system
which consists of a servomotor, leadscrew, and optical encoder. The leadscrew has 6 threads/in
and is coupled directly to the motor shaft (gear ratio = 1:1). The optical encoder generates 225
pulses per motor revolution. The table has been programmed to move a distance of 7.5 in at a
feed rate = 20.0 in/min. (a) How many pulses are received by the control system to verify that the
258
table has moved the programmed distance? What are (b) the pulse rate and (c) motor speed that
correspond to the specified feed rate?
Solution: (a) x = p np/ns ;Rearranging, np = xns/p = 7.5(225)/0.1667 = 10,125 pulses.
(b) fp = fr ns /60p = 20(225)/60(0.1667) = 450 Hz
(c) N = fr /p = 20/0.1667 = 120 rev/min.
38.12 A leadscrew coupled directly to a dc servomotor is used to drive one of the table axes of an NC
milling machine. The leadscrew has 5 threads/in. The optical encoder attached to the leadscrew
emits 100 pulses/rev of the leadscrew. The motor rotates at a maximum speed of 800 rev/min.
Determine: (a) The control resolution of the system, expressed in linear travel distance of the table
axis; (b) the frequency of the pulse train emitted by the optical encoder when the servomotor
operates at maximum speed; and (c) the travel speed of the table at the maximum rpm of the
motor.
Solution: (a) CR = p/ns = 0.2/100 = 0.002 in.
(b) fp = N ns /60 = 800(100)/60 = 1333.3 Hz
(c) vt = Np = 800(0.2) = 160 in/min.
38.13 Solve the previous problem only the servomotor is connected to the leadscrew through a gear box
whose reduction ratio = 12:1 (12 revolutions of the motor for each revolution of the leadscrew).
Solution: (a) CR = p/ns = 0.2/100 = 0.002 in.
(b) fp = N ns /60 = 800(100)/60 = 1333.3 Hz
(c) vt = N p/rg = 800(0.2)/12 = 13.33 in/min.
38.14 A leadscrew connected to a dc servomotor is the drive system for a positioning table. The
leadscrew pitch = 4 mm. The optical encoder attached to the leadscrew emits 250 pulses/rev of
the leadscrew. The motor operates at a speed = 15 rev/s. Determine: (a) The control resolution of
the system, expressed in linear travel distance of the table axis; (b) the frequency of the pulse
train emitted by the optical encoder when the servomotor operates at 14 rev/s; and (c) the travel
speed of the table at the operating speed of the motor.
Solution: (a) CR = p/ns = 4/250 = 0.016 mm.
(b) fp = N ns = 14(250) = 3500 Hz
(c) vt = Np = 14(4) = 56 mm/s
38.15 A milling operation is performed on a NC machining center. Total travel distance = 300 mm in a
direction parallel to one of the axes of the worktable. Cutting speed = 1.25 m/s and chip load =
0.05 mm. The end milling cutter has four teeth and its diameter = 20.0 mm. The axis uses a dc
servomotor whose output shaft is coupled to a leadscrew with pitch = 6.0 mm. The feedback
sensing device is an optical encoder which emits 250 pulses per revolution. Determine: (a) feed
rate and time to complete the cut, and (b) rotational speed of the motor and the pulse rate of the
encoder at the feed rate indicated.
Solution: (a) N = (1.25 x 103 mm/s)/(20mm/rev) = 19.89 rev/s.
fr = N f nt = 19.89(0.05)(4) = 3.978 mm/s.
Tm = 300/3.978 = 75.4 s = 1.26 min.
(b) N = fr /p = (3.978 mm/s)/(6 mm/rev) = 0.663 rev/s.
259
fp = ns N = 250(0.663) = 165.75 Hz
38.16 An end milling operation is carried out along a straight line path which is 325 mm in length. The
cut is in a direction parallel to the x-axis on a NC machining center. Cutting speed = 30 m/min and
chip load = 0.06 mm. The end milling cutter has two teeth and its diameter = 16.0 mm. The x-axis
uses a dc servomotor connected directly to a leadscrew whose pitch = 6.0 mm. The feedback
sensing device is an optical encoder which emits 400 pulses per revolution. Determine: (a) feed
rate and time to complete the cut, and (b) rotational speed of the motor and the pulse rate of the
encoder at the feed rate indicated.
Solution: (a) N = (30 x 103 mm/min)/(16mm/rev) = 596.8 rev/min.
fr = N f nt = 596.8(0.06)(2) = 71.62 mm/min.
Tm = 325/71.62 = 4.54 min.
(b) N = fr /p = (71.62 mm/min)/(6.0 mm/rev) = 11.94 rev/min.
fp = ns N/60 = 400(11.94)/60 = 79.58 Hz
38.17 A dc servomotor is used to drive the x-axis of a NC milling machine table. The motor is coupled
directly to the table leadscrew, which has 4 threads/in. An optical encoder is used to provide the
feedback measurement. It is connected to the leadscrew using a 1:5 gear ratio (one turn of the
leadscrew converts to 5 turns of the encoder disk). The optical encoder emits 125 pulses per
revolution. To execute a certain programmed instruction, the table must be moved from point (3.5,
1.5) to point (1.0, 7.2) in a straight-line trajectory at a feed rate of 7.5 in/min. Determine: (a) the
control resolution of the system for the x-axis; also (b) the rotational speed of the motor and (c)
the frequency of the pulse train emitted by the optical encoder when the desired feed rate is
achieved.
Solution: (a) CR = p/rg ns = 0.250/(5 x 125) = 0.0004 in.
(b) Move from (3.5, 1.5) to (1.0, 7.2) at fr = 7.5 in/min.
x = 1.0 - 3.5 = -2.5 in., y = 7.2 - 1.5 = 5.7 in.
Angle A = tan-1(5.7/-2.5) = 113.7
frx = 7.5 cos 113.7 = 7.5(-0.4017) = -3.012 in/min
N = frx /p = (-3.012/(.25) = 12.05 rev/min.
(c) fp = rg ns N/60 = 5(12.05)(125)/60 = 125.52 Hz
Industrial Robotics
38.18 The largest axis of a Cartesian coordinate robot has a total range of 750 mm. It is driven by pulley
system capable of a mechanical accuracy = 0.25 mm and repeatability = 0.15 mm. Determine
the minimum number of bits required in the binary register for the axis in the robot's control
memory.
Solution: Repeatability = 3= 0.15 mm
= 0.15/3 = 0.05 mm
Accuracy = 0.25 mm = 0.5 CR + 3 = 0.5 CR + 0.15
0.5 CR = 0.25 - 0.15 = 0.10
CR = 0.20
CR = CR2 = L/(2B – 1)= 750/(2B – 1)
750/(2B – 1)= 0.20
2B – 1 = 750/0.20 = 3750
2B = 3751
260
B ln 2 = ln 3751
0.69315 B = 8.22978 B = 11.87 ® 12 bits.
38.19 A stepper motor serves as the drive unit for the linear joint of an industrial robot. The joint must
have an accuracy of 0.25 mm. The motor is attached to a leadscrew through a 2:1 gear reduction
(2 turns of the motor for 1 turn of the leadscrew). The pitch of the leadscrew is 5.0 mm. The
mechanical errors in the system (due to backlash of the leadscrew and the gear reducer) can be
represented by a normal distribution with standard deviation = 0.05 mm. Specify the number of
step angles that the motor must have in order to meet the accuracy requirement.
Solution: Repeatability = 3 = 3(0.05) = 0.15 mm
Accuracy = 0.25 mm = 0.5 CR + 3= 0.5 CR + 0.15
0.5 CR = 0.25 - 0.15 = 0.10
CR = 0.20 mm
Assume CR = CR1 = p/rg ns
ns = p/(rg CR) = 5.0/(2 x 0.20) = 12.5 ns = 13 step angles
38.20 The designer of a polar configuration robot is considering a portion of the manipulator consisting of
a rotational joint connected to its output link. The output link is 25 in long and the rotational joint
has a range of 75. The accuracy of the joint-link combination, expressed as a linear measure at
the end of the link which results from rotating the joint, is specified as 0.030 in. The mechanical
inaccuracies of the joint result in a repeatability error = 0.030of rotation. It is assumed that the
link is perfectly rigid, so there are no additional errors due to deflection. (a) Show that the
specified accuracy can be achieved, given the repeatability error. (b) Determine the minimum
number of bits required in the binary register of the robot's control memory to achieve the
specified accuracy.
Solution: (a) Repeatability = 3 = 0.030.
0.030= 2(0.030)/360 = 0.0005236 rad.
End-of-link movement = LA where A = angle of movement in radians
LA = 25(0.0005236) = 0.0131 in.
Accuracy = 0.5 CR + 3 = 0.5 CR + 0.0131
Specified accuracy = 0.030
0.030 = 0.5 CR + 0.0131
0.5 CR = 0.030 - 0.0131 = 0.0169
CR = 0.0169/0.5 = 0.0338 in
Since CR is positive, the specified accuracy should be possible to achieve.
(b) Given CR = 0.0338 from part (a), total range = 75
Converting this to an arc distance, range = (2(75)/360) x 25 = 32.725 in.
CR = L/(2B – 1) = 0.0338
32.725/(2B – 1) = 0.0338
2B – 1 = 32.725/0.0338 = 968.2
2B = 969.2
B ln 2 = ln 969.2
0.6931 B = 6.876 B = 9.92 ® 10 bits
261
39 GROUP TECHNOLOGY AND FLEXIBLE
MANUFACTURING SYSTEMS
Review Questions
39.1 Define group technology?
Answer. GT is a general approach in which similarities among parts are identified and exploited in
design and manufacturing.
39.2 What is a part family?
Answer. A part family is a collection of parts that are similar in design or manufacturing
attributes.
39.3 Define cellular manufacturing?
Answer. Cellular manufacturing involves the production of part families using groups of machines
(generally manually operated) to produce a certain part family or a limited set of part families.
39.4 What is the composite part concept in group technology?
Answer. In GT, a composite part is a hypothetical part that includes all of the design and/or
manufacturing attributes of a given part family. The concept is useful in designing cells to produce
the part family.
39.5 Name some of the possible machine cell designs in group technology.
Answer. GT machine cell designs include: single machines, multiple machines with manual
handling, multiple machines with mechanized handling, flexible manufacturing cells, and flexible
manufacturing systems.
39.6 What is a flexible manufacturing system?
Answer. A flexible manufacturing system (FMS) is an automated group technology cell consisting
of processing stations interconnected by an automated handling system and controlled by a
computer.
39.7 What makes an automated manufacturing system flexible?
Answer. The tests of flexibility are: (1) process different part styles in non-batch mode; (2)
accept changes in schedule, (3) deal gracefully with equipment breakdowns, and (4)
accommodate new part style introductions.
39.8 Name some of the FMS software and control functions.
Answer. Functions include: (1) NC part programming, (2) NC part program download, (3)
production control, (4)machine control, (5) workpart control, (6) tool management, (7) work
transport control, and (8) general system management.
39.9 Identify some of the applications of FMS technology.
Answer. Applications include: machining, assembly, inspection, and sheet metal processing.
39.10 What are the advantages of FMS technology, compared to conventional batch operations?
262
Answer. Advantages include: (1) higher machine utilization, (2) reduced work-in-process, (3)
lower manufacturing lead times, and (4) greater flexibility in production scheduling.
Multiple Choice Quiz
There are a total of 12 correct answers in the following multiple choice questions (some questions have
multiple answers that are correct). To attain a perfect score on the quiz, all correct answers must be
given, since each correct answer is worth 1 point. For each question, each omitted answer or wrong
answer reduces the score by 1 point, and each additional answer beyond the number of answers required
reduces the score by 1 point. Percentage score on the quiz is based on the total number of correct
answers.
39.1 Production flow analysis is a method of identifying part families that uses data from which one of
the following sources? (a) bill of materials, (b) engineering drawings, (c) master schedule, (d)
production schedule, or (e) route sheets.
Answer. (e)
39.2 Most parts classification and coding systems are based on which of the following types of part
attributes (more than one)? (a) annual production rate, (b) design, (c) manufacturing, and (d)
weight.
Answer. (b) and (c).
39.3 Which of the following are part design attributes that are likely to be included in a parts
classification and coding system (more than one)? (a) annual production, (b) batch size, (c)
length-to-diameter ratio, (d) major process, (e) part dimensions, and (f) tolerances.
Answer. (c), (e), and (f).
39.4 What is the dividing line between a manufacturing cell and a flexible manufacturing system? (a)
two machines, (b) four machines, or (c) six machines.
Answer. (b)
39.5 A machine capable of producing different part styles in a batch mode of operation qualifies as a
flexible manufacturing system: (a) true or (b) false.
Answer. (b) A flexible manufacturing system does not normally operate in a batch mode.
39.6 The physical layout of a flexible manufacturing system is determined principally by which one of
the following? (a) computer system, (b) material handling system, (c) part family, (d) processing
equipment, or (e) weight of parts processed.
Answer. (b)
39.7 Industrial robots can, in general, most easily handle which of the following part types in a flexible
machining system (one best answer)? (a) heavy parts, (b) metal parts, (c) nonrotational parts, (d)
plastic parts, or (e) rotational parts.
Answer. (e).
39.8 Flexible manufacturing systems and cells are generally applied in which one of the following
areas? (a) high variety, low volume production, (b) low variety, (c) low volume, (d) mass
production, (e) medium volume, medium variety production.
Answer. (e)
263
39.9 Which of the following technologies is most closely associated with flexible machining systems
(one best answer)? (a) lasers, (b) machine vision, (c) manual assembly lines, (d) numerical control,
or (f) transfer lines.
Answer. (d)
264
40 PRODUCTION LINES
Review Questions
40.1 What is a production line?
Answer. A production line is a sequence of workstations at which individual tasks are
accomplished on each work unit as it moves from one station to the next to progressively make
the product.
40.2 Distinguish between a batch model production line and a mixed model production line.
Answer. Both lines are used to make multiple product types. A batch model line produces the
different products in batches, with a setup changeover between the products; while a mixed model
line produces different products simultaneously.
40.3 What are the advantages of the mixed model line for producing different product styles?
Answer. Advantages of the mixed model line include: (1) no downtime between different models;
(2) minimize or avoid high inventories of some models while there are stock-outs of other models;
and (3) production rates can be matched to demand rates for different models.
40.4 What are some of the limitations of a mixed model line compared to a batch model line?
Answer. Limitations include: (1) line balancing problem is more complex; (2) scheduling the
models is more difficult; and (3) getting the right parts to each workstation is more difficult
because more parts are involved.
40.5 Identify two fundamental principles on which manual assembly lines are based.
Answer. Three principles are described in the text: (1) specialization of labor, also known as
division of labor, (2) interchangeable parts, and (3) work flow principle.
40.6 Describe how manual methods are used to move parts between workstations on a production line.
Answer. The methods include: (1) work units are simply passed by hand along a flat worktable
from one station to the next, (2) sometimes collecting them in boxes and then passing the boxes
between stations; and (3) pushing the work units along a conveyor between stations.
40.7 Briefly define the three types of mechanized workpart transfer systems used in production lines.
Answer. The three work transfer systems are: (1) continuous transfer, in which parts move on a
conveyor at a steady speed; (2) synchronous transfer, in which parts all move simultaneously from
station-to-station with a stop-and-go action; and (3) asynchronous transfer, in which parts move
independently between stations with a stop-and-go action.
40.8 Why are parts sometimes fixed to the conveyor in a continuous transfer system in manual
assembly?
Answer. Because the parts are big and/or heavy and cannot be conveniently removed by a
human worker.
40.9 Why must a production line be paced at a rate higher than that required to satisfy the demand for
the product?
Answer. Because all production lines will suffer a certain amount of nonproductive time due to
reliability problems.
265
40.10 What are the reasons why the number of workstations cannot be determined simply from the ratio
Twc/Tc?
Answer. Reasons are: (1) line balancing losses, (2) repositioning time during Tc, and (3) multiple
manning.
40.11 Why is the line balancing problem different on an automated transfer line than on a manual
assembly line?
Answer. Line balancing on an automated transfer line (e.g., machining) involves allocation of
(usually) single machining operations to workstations. Line balancing on a manual assembly line
(usually) involves assigning multiple work elements to workers or stations. There are many more
possible solutions to the line balancing problem in the manual assembly case.
40.12 Repositioning time on a synchronous transfer line is known by a different name; what is that
name?
Answer. The repositioning time is called the transfer time; it is the time to move parts from one
station to the next.
40.13 Why are single station assembly cells generally not suited to high production jobs?
Answer. The entire work cycle is performed at one station, so single station cells usually operate
at relatively slow production rates.
40.14 What are some of the reasons for downtime on a machining transfer line?
Answer. Reasons include: tool changes, sudden mechanical and electrical failures, and gradual
normal wear and tear on the equipment.
Multiple Choice Quiz
There are a total of 10 correct answers in the following multiple choice questions (some questions have
multiple answers that are correct). To attain a perfect score on the quiz, all correct answers must be
given, since each correct answer is worth 1 point. For each question, each omitted answer or wrong
answer reduces the score by 1 point, and each additional answer beyond the number of answers required
reduces the score by 1 point. Percentage score on the quiz is based on the total number of correct
answers.
40.1 Batch model lines are most suited to which one of the following production situations? (a) job
shop, (b) mass production, or (c) medium production.
Answer. (c)
40.2 Manual methods of workpart transfer are probably closest to which one of the following
mechanized methods of transfer? (a) asynchronous, (b) continuous, or (c) synchronous.
Answer. (a)
40.3 Precedence constraints are best described by which of the following (one best answer)? (a)
launching sequence in a mixed model line, (b) limiting value on the sum of element times that can
be assigned to a worker or station, (c) order of work stations along the line, or (d) sequence in
which the work elements must be done.
Answer. (d)
266
40.4 Which of the following phrases are most appropriate to describe the characteristics of tasks that
are performed at automated workstations (more than one)? (a) complex, (b) consists of multiple
work elements, (c) involves a single work element, (d) involves straight-line motions, (e) requires
sensory capability, and (f) simple.
Answer. (c), (d), and (f).
40.5 The transfer line is most closely associated with which one of the following types of production
operations? (a) assembly, (b) automotive chassis fabrication, (c) machining, (d) pressworking, or
(e) spotwelding.
Answer. (c)
40.6 A dial indexing machine uses which one of the following types of workpart transfer? (a)
asynchronous, (b) continuous, (c) parts passed by hand, or (d) synchronous.
Answer. (d)
40.7 The line efficiency (proportion uptime) on an automated line can be increased by which of the
following approaches (more than one)? (a) improving the reliability of each workstation on the line,
(b) increasing the number of stations n on the line, and (c) reducing the average downtime Td.
Answer. (a) and (c).
Problems
Manual Assembly Lines
40.1 A manual assembly line is being designed for a product with annual demand = 100,000 units. The
line will operate 50 wks/year, 5 shifts/wk, and 7.5 hr/shift. Work units will be attached to a
continuously moving conveyor. Work content time = 42.0 min. Assume line efficiency E = 0.97,
balancing efficiency Eb = 0.92, and repositioning time Tr = 6 sec. Determine: (a) hourly production
rate to meet demand, and (b) number of workers required.
Solution: (a) Rp = 100,000/(50 x 5 x 7.5) = 53.33 units/hr
(b) Tc = E/Rp = 60(.97)/53.33 = 1.09125 min.
Ts = Tc - Tr = 1.09125 - 0.1 = 0.99125 min.
w = Min Int 42.0/(.92 x 0.99125) = 46.06 47 workers
40.2 In the previous problem, compute (a) the ideal minimum number of workstations nmin; and (b) the
number of workstations required if multiple manning can be used and the estimated manning level
is M = 1.4.
Solution: (a) nmin = Min Int Twc/Tc = 42.0/1.09125 = 38.5 39 stations
(b) n = w/M = 47/1.4 = 33.6 34 stations
40.3 A manual assembly line produces a small appliance whose work content time = 25.9 min. Desired
production rate = 50 units/hr. Repositioning time = 6 sec, line efficiency = 95%, and balancing
efficiency is 93%. How many workers are on the line?
Solution: Tc = E/Rp = 60(0.95)/50 = 1.14 min.
Ts = Tc - Tr = 1.14 - 0.1 = 1.04 min.
w = Min Int 25.9/(0.93 x 1.04) = 26.78 27 workers
267
40.4 A single model manual assembly line produces a product whose work content time = 48.9 min.
The line has 24 workstations with a manning level M = 1.25. The product has a Available shift
time per day = 8 hr, but downtime during the shift reduces actual production time to 7.6 hr on
average. This results in an average daily production of 256 units/day. Repositioning time per
worker Tr is 8% of cycle time Tc. Determine: (a) line efficiency, (b) balancing efficiency, and (c)
repositioning time Tr.
Solution: (a) E = 7.6/8.0 = 0.95
(b) Rp = 256/8 = 32 units/hr
Tc = 60(0.95)/32 = 1.78125 min
Ts = Tc - Tr = Tc - 0.08Tc = 0.92 Tc = 0.92(1.78125) = 1.63875 min.
w = 24(1.25) = 32 workers
Eb = Twc/wTs = 47.8/(32 x 1.63875) = 0.9325
(c) Tr = 0.08(1.78125) = 0.1425 min. = 8.55 sec.
40.5 A final assembly plant for a certain automobile model is to have a capacity of 240,000 units
annually. The plant will operate 50 weeks/yr, two shifts/day, 5 days/week, and 8.0 hours/shift. It
will be divided into three departments: (1) body shop, (2) paint shop, (3) trim-chassis-final
department. The body shop welds the car bodies using robots, and the paint shop coats the bodies.
Both of these departments are highly automated. Trim-chassis-final has no automation. There are
15.5 hours of direct labor content on each car in this department, where cars are moved by a
continuous conveyor. Determine: (a) hourly production rate of the plant, (b) number of workers
and workstations required in trim-chassis-final if no automated stations are used, the average
manning level is 2.5, balancing efficiency = 93%, proportion uptime = 95%, and a repositioning
time of 0.15 min is allowed for each worker.
Solution: (a) Rp = 240,000/(50 x 10 x 8) = 60.0 units/hr
(b) Tc = E/Rp = 60(0.95)/60 = 0.95 min.
Ts = Tc - Tr = 0.95 - 0.15 = 0.8 min.
w = Min Int Twc/EbTs = 15.5 x 60/(0.93 x .8) = 1250 workers
n = w/M = 1250/2.5 = 500 stations
40.6 A product whose total work content time = 50 minutes is to be assembled on a manual production
line. The required production rate is 30 units per hour. From previous experience with similar
products, it is estimated that the manning level will be close to 1.5. Assume E = Eb = 1.0. If 9
seconds will be lost from the cycle time for repositioning, determine: (a) the cycle time, and (b)
how many workers and (c) stations will be needed on the line?
Solution: (a) Tc = E/Rp = 1.0(60)/30 = 2.0 min/unit
(b) Ts = Tc - Tr = 2.0 - 0.15 = 1.85 min.
w = Min Int Twc/EbTs = 50/(1.0 x 1.85) = 27.03 28 workers
(c) n = 28/1.5 = 18.67 19 stations
40.7 A manual assembly line has 17 workstations with one operator per station. Total work content
time to assemble the product = 22.2 minutes. The production rate of the line = 36 units per hour. A
synchronous transfer system is used to advance the products from one station to the next, and the
transfer time = 6 seconds. The workers remain seated along the line. Proportion uptime E = 0.90.
Determine the balance delay.
Solution: Tc = E/Rp = 60(0.90)/36 = 1.50 min.
268
Ts = Tc - Tr = 1.50 - 0.1 = 1.40 min.
Eb = Twc/wTs = 22.2/(17 x 1.40) = 0.933 = 93.3%
40.8 A production line with four automatic workstations (the other stations are manual) produces a
certain product whose total assembly work content time = 55.0 min. of direct manual labor. The
production rate on the line is 45 units/hr. Because of the automated stations, uptime efficiency =
89%. The manual stations each have one worker. It is known that 10% of the cycle time is lost
due to repositioning. If the balancing efficiency Eb = 0.92 on the manual stations, find: (a) cycle
time, (b) number of workers and (c) workstations on the line. (d) What is the average manning
level on the line, where the average includes the automatic stations?
Solution: (a) Tc = E/Rp = 60(0.89)/45 = 1.1867 min.
(b) Ts = Tc - Tr = 0.9Tc = 0.9(1.1867) = 1.068 min.
w= Twc/EbTs = 55.0/(0.92 x 1.068) = 55.97 56 workers
(c) n = 56 + 4 = 60 stations
(d) M = 56/60 = 0.933
40.9 Production rate for a certain assembled product is 47.5 units per hour. The total assembly work
content time = 32 minutes of direct manual labor. The line operates at 95% uptime. Ten
workstations have two workers on opposite sides of the line so that both sides of the product can
be worked on simultaneously. The remaining stations have one worker. Repositioning time lost by
each worker is 0.2 min/cycle. It is known that the number of workers on the line is two more than
the number required for perfect balance. Determine: (a) number of workers, (b) number of
workstations, (c) the balancing efficiency, and (d) average manning level.
Solution: (a) Tc = E/Rp = 0.95(60)/47.5 = 1.2 min.
Ts = Tc - Tr = 1.2 - 0.2 = 1.0 min
If perfect balance, then Eb = 1.0 and w = Min Int Twc/EbTs = 32/(1.0 x 1.0) = 32 workers
But with 2 additional workers, w = 32 + 2 = 34 workers
(b) n = 10 + (34 - 2 x 10) = 10 + 14 = 24 stations
(c) Eb = Twc/wTs = 32/(34 x 1.0) = 0.941
(d) M = w/n = 34/24 = 1.417
40.10 The total work content for a product assembled on a manual production line is 48 min. The work is
transported using a continuous overhead conveyor which operates at a speed of 3 ft/min. There
are 24 workstations on the line, one-third of which have two workers; the remaining stations each
have one worker. Repositioning time per worker is 9 sec, and uptime efficiency of the line is 95%.
(a) What is the maximum possible hourly production rate if line is assumed to be perfectly
balanced? (b) If the actual production rate is only 92% of the maximum possible rate determined
in part (a), what is the balance delay on the line?
Solution: (a) Eb = 1.0, w = 0.333(24) x 2 + 0.667(24) x 1 = 32 workers
w = Twc/EbTs, Ts = Twc/wEb = 48/32 = 1.5 min.
Tc = Ts + Tr = 1.5 + .15 = 1.65 min.
Tp = Tc/E = 1.65/.95 = 1.737 min.
Rp = 60/Tp = 60/1.737 = 34.55 units/hr
(b) Actual Rp = 0.92(34.55) = 31.78 units/hr
Tc = 60E/Rp = 60(.95)/31.78 = 1.7935 min.
Ts = 1.7935 - .15 = 1.6435 min.
269
Eb = Twc/wTs = 48/(32 x 1.6435) = 0.9127
Automated Production Lines
40.11 An automated transfer line has 20 stations and operates with an ideal cycle time of 1.50 min.
Probability of a station failure is p = 0.008 and average downtime when a breakdown occurs is
10.0 minutes. Determine: (a) the average production rate Rp and (b) the line efficiency E.
Solution: (a) F = np = 20(0.008) = 0.16
Tp = 1.50 + 0.16(10.0) = 1.50 + 1.60 = 3.10 min.
Rp = 60/Tp = 60/3.1 = 19.35 units/hr
(b) E = Tc/Tp = 1.5/3.1 = 0.484
40.12 A dial-indexing table has 6 stations. One station is used for loading and unloading, which is
accomplished by a human worker. The other five perform processing operations. The longest
process takes 25 sec and the indexing time = 5 sec. Each station has a frequency of failure p =
0.015. When a failure occurs it takes an average of 3.0 min to make repairs and restart.
Determine: (a) hourly production rate, and (b) line efficiency.
Solution: (a) Assume p = 0 at the manual station
F = np = 1(0) + 5(.015) = 0.075
Tp = 0.5 + 0.075(3.0) = 0.5 + .225 = 0.725 min.
Rp = 60/0.725 = 82.76 units/hr
(b) E = Tc/Tp = 0.5/0.725 = 0.690
40.13 A 7-station transfer line has been observed over a 40-hour period. The processing times at each
station are:
Station 1 2 3 4 5 6 7
Process time (min) 0.80 1.10 1.15 0.95 1.06 0.92 0.80
The transfer time between stations = 6 sec. The number of downtime occurrences = 110, and hours
of downtime = 14.5 hours. Determine: (a) the number of parts produced during the week, (b) the
average actual production rate in parts/hour, and (c) the line efficiency. (d) If the balancing efficiency
were computed for this line, what would be its value?
Solution: (a) Tc = 1.15 + 0.10 = 1.25 min.
EH = 40E = 40 - 14.5 = 25.5 hrs
Q = 25.5(60)/1.25 = 1224 pc during the 40 hour period.
(b) Rp = 1224/40 = 30.6 pc/hr
(c) 40E = 25.5 E = 25.5/40 = 0.6375
(d) Twc = Ts = 0.80 + 1.10 + 1.15 + 0.95 + 1.06 + 0.92 + 0.80 = 6.78 min.
n(maximum Ts) = 7(1.15) = 8.05 min
Eb = 6.78/8.05 = 0.842
40.14 A 12-station transfer line was designed to operate with an ideal production rate = 50 parts/hour.
However, the line does not achieve this rate, since the line efficiency E = 0.60. It costs $75/hour
to operate the line, exclusive of materials. The line operates 4000 hours per year. A computer
monitoring system has been proposed that will cost $25,000 (installed) and will reduce downtime
on the line by 25%. If the value added per unit produced = $4.00, will the computer system pay for
270
itself within one year of operation. Use expected increase in revenues resulting from the computer
system as the criterion. Ignore material costs in your calculations.
Solution: Tc = 60/Rc = 60/50 = 1.2 min.
Tp = Tc/E = 1.2/.6 = 2.0 min.
Rp = 60/Tp = 60/2.0 = 30 pc/hr
In the current system:
Annual production Q = 4000Rp = 4000(30) = 120,000 units/yr
Revenues = $4.00Q = $4.00(120,000) = $480,000/yr.
Cost to operate line = $75H = $75(4000) = $300,000/yr
With computer monitoring system:
Tc = 1.2 min. and Tp = 2.0 min. FTd = Tp - Tc. This is reduced by 25% with new system.
FTd = (1 - 25%)(2.0 - 1.2) = 0.75(0.8) = 0.6 min.
Tp = 1.2 + 0.6 = 1.8 min.
Rp = 60/1.8 = 33.33 pc/hr
Annual production Q = 4000(33.33) = 133,333 units/yr
Revenues = $4.00(133,333) = $533,333/yr.
Cost to operate line = same as in current system (neglecting increased cost of new system)
Difference in revenues = $533,333 - $480,000 = $53,333. This is more than enough to justify the
$25,000 investment.
40.15 An automated transfer line is to be designed. Based on previous experience, the average
downtime per occurrence = 5.0 min, and the probability of a station failure that leads to a
downtime occurrence p = 0.01. The total work content time = 9.8 min and is to be divided evenly
amongst the workstations, so that the ideal cycle time for each station = 9.8/n. Determine: (a) the
optimum number of stations n on the line that will maximize production rate, and (b) the production
rate Rp and proportion uptime E for your answer to part (a).
Solution: (a) Maximizing Rp is equivalent to minimizing Tp.
Tp = Tc + Ftd = 9.8/n + n(0.01)(5.0) = 9.8/n + 0.05n
dTp/dn = -9.8/n2 + 0.05 = zero at minimum point
n2 = 9.8/0.05 = 196
n = (196).5 = 14 stations
(b) Tp = 9.8/14 + 0.05(14) = 0.7 + 0.7 = 1.4 min.
Rp = 60/1.4 = 42.86 pc/hr E = 0.7/1.4 = 0.50
271
41 MANUFACTURING ENGINEERING
Review Questions
41.1 Define manufacturing engineering.
Answer. Manufacturing engineering is a technical staff department responsible for planning the
manufacturing processes for a product and its components.
41.2 What are the principal activities in manufacturing engineering?
Answer. Principal activities of manufacturing engineering are: (1) process planning, (2)
day-to-day technical problem solving and continuous improvement, (3) advising the product
designers in design for manufacturability, and (4) advanced manufacturing planning, which
includes evaluation of new processing technologies, manufacturing research, and so forth.
41.3 Identify some of the details and decisions that are included within the scope of process planning.
Answer. Table 41.1 lists the following details and decisions in process planning: (1) what
processes are required and the sequence in which they are performed; (2) equipment selection;
(3) what tools, dies, molds, fixtures, and gages will be needed; (4) identification of cutting tools and
cutting conditions for machining operations; (5) methods for manual operations (e.g., assembly)
and manual portions of machine cycles (e.g., loading and unloading a production machine); (6)
work standards (time standards for each operation); (7) estimates of production costs; (8)
consideration must be given to the problem of moving materials and work-in-progress in the
factory; and (9) plant layout and facilities design.
41.4 What is a route sheet?
Answer. A route sheet is a listing of the production operations and their sequence required to
make a given part. It also lists the equipment and special tooling that is required.
41.5 What is the difference between a basic process and a secondary process?
Answer. A basic process establishes the starting geometry of the work material; e.g., casting,
rolled sheet metal, drawn metal bar. Secondary processes are used to refine the starting
geometry and transform the material into final shape and size; machining and sheet metal
pressworking are common secondary processes.
41.6 What is a precedence constraint in process planning?
Answer. A precedence constraint is a limitation on the order in which processing or assembly
operations can be performed on a given workpart; for example, a hole must be drilled before it can
be tapped.
41.7 In the make or buy decision, why is it that purchasing a component from a vendor may cost more
than producing the component internally, even though the quoted price from the vendor is lower
than the internal price?
Answer. Because purchasing the component may cause idle equipment and staff in the factory
that the company must still pay for. See Example 41.1 in the text.
41.8 Identify some of the important factors that should enter into the make or buy decision.
272
Answer. The factors include: (1) cost, (2) whether the process is available internally, (3)
production quantity, (4) Product life, (5) whether or not the component is a standard commercially
available hardware item, (6) supplier reliability, and (7) possible need for alternative sources.
41.9 Name three of the general principles and guidelines in design for manufacturability?
Answer. Table 41.5 lists the following DFM principles and guidelines: (1) Minimize number of
components. (2) Use standard commercially available components. (3) Use common parts across
product lines. (4) Design for ease of part fabrication. (5) Design parts with tolerances that are
within process capability. (6) Design the product to be foolproof during assembly. (7) Minimize
flexible components. (8) Design for ease of assembly. (9) Use modular design. (10) Shape parts
and products for ease of packaging. (11) Eliminate or reduce adjustment required.
41.10 What is concurrent engineering and what are its important components?
Answer. Concurrent engineering is an approach used by companies to reduce the time to bring a
new product to market. It includes: (1) design for manufacturability, (2) design for quality, (3)
design for life cycle, and (4) design for cost. It also includes certain organizational changes which
attempt to bring different functions in the company together during product design to consider all
possible aspects of the product.
41.11 Identify some of the enabling technologies for concurrent engineering?
Answer. Enabling technologies mentioned in the text are rapid prototyping and computer
integrated manufacturing (which includes CAD/CAM). In addition, electronic data interchange
should probably be included in the list.
41.12 What is meant by the term design for life cycle?
Answer. Design for life cycle means that factors relating to the product after it has been
manufactured should be taken into consideration in design. These factors include installability,
reliability, maintainability, serviceability, upgradeability, and disposability.
Multiple Choice Quiz
There are a total of 18 correct answers in the following multiple choice questions (some questions have
multiple answers that are correct). To attain a perfect score on the quiz, all correct answers must be
given, since each correct answer is worth 1 point. For each question, each omitted answer or wrong
answer reduces the score by 1 point, and each additional answer beyond the number of answers required
reduces the score by 1 point. Percentage score on the quiz is based on the total number of correct
answers.
41.1 Which of the following are the usual responsibilities of the manufacturing engineering department
(more than one)? (a) advising on design for manufacturability, (b) facilities planning, (c) process
improvement, (d) process planning, (e) product design, and (f) solving technical problems in the
production departments.
Answer. (a), (c), (d), and (f).
41.2 Which of the following would be considered basic processes, as opposed to secondary processes
(more than one)? (a) annealing, (b) anodizing, (c) drilling with a twist drill, (d) electroplating, (e)
forward hot extrusion to produce aluminum bars, (f) impression die forging, (g) rolling, (h) sand
casting, (i) sheet metal stamping, (j) sintering of pressed ceramic powders, (k) spot welding, (l)
surface grinding of hardened steel, (m) tempering of martensitic steel, (n) trepanning, (o) turning,
and (p) ultrasonic machining.
273
Answer. (e), (f), (g), and (h).
41.3 Which of the following would be considered secondary processes, as opposed to basic processes
(more than one)? (a) annealing, (b) anodizing, (c) drilling with a twist drill, (d) electroplating, (e)
forward hot extrusion to produce aluminum bars, (f) impression die forging, (g) rolling, (h) sand
casting, (i) sheet metal stamping, (j) sintering of pressed ceramic powders, (k) spot welding, (l)
surface grinding of hardened steel, (m) tempering of martensitic steel, (n) trepanning, (o) turning,
and (p) ultrasonic machining.
Answer. (c), (i), (l), (n), (o), and (p).
41.4 Which of the following are operations to enhance physical properties (more than one)? (a)
annealing, (b) anodizing, (c) drilling with a twist drill, (d) electroplating, (e) forward hot extrusion to
produce aluminum bars, (f) impression die forging, (g) rolling, (h) sand casting, (i) sheet metal
stamping, (j) sintering of pressed ceramic powders, (k) spot welding, (l) surface grinding of
hardened steel, (m) tempering of martensitic steel, (n) trepanning, (o) turning, and (p) ultrasonic
machining.
Answer. (a), (j), and (m).
41.5 Which one of the following types of computer-aided process planning relies on parts classification
and coding in group technology? (a) generative CAPP, (b) retrieval CAPP, (c) traditional process
planning, or (d) none of the preceding.
Answer. (b)
274
42 PRODUCTION PLANNING AND CONTROL
Review Questions
42.1 What is meant by the term make-to-stock production?
Answer. Make-to-stock is the case in which the company produces to replenish inventories of
products. Production rate is greater than demand rate, and it is appropriate to carry inventory.
42.2 How does aggregate planning differ from the master production scheduling?
Answer. Aggregate planning is scheduling by general product line; the master production
schedule indicates how many and when of each product model within the product line are to be
produced.
42.3 What are the product categories usually listed in the master production schedule.
Answer. The categories are: (1) firm customer orders, (2) sales forecasts, and (3) spare parts.
42.4 What is the difference between dependent and independent demand for products?
Answer. Independent demand means that the demand or consumption of the item is unrelated
to demand for other items. End products and spare parts experience independent demand.
Dependent demand refers to the fact that demand for the item is directly related to demand for
something else, usually because the item is a component of an end product subject to independent
demand.
42.5 Define reorder point inventory system.
Answer. In a reorder point system, an order to restock is issued when the inventory level for the
given stock item declines to some point defined as the reorder point.
42.6 In MRP, what are common use items?
Answer. Common use items are materials, components, or subassemblies that are used for more
than one item in the next level above in the product structure; for example, a starting material that
is used on more than one component, or a component that is used on more than one product.
42.7 Identify the inputs to the MRP processor in material requirements planning.
Answer. The inputs to MRP are: (1) master production schedule, (2) bill-of-materials file for
product structure, and (3) inventory record file.
42.8 What are some of the resource changes that can be made to increase plant capacity in the short
run?
Answer. Short term adjustments to increase capacity include: increase employment levels,
increase shift hours, authorize overtime, increase number of shifts, and subcontracting work to
outside vendors.
42.9 Identify the principal objective in just-in-time production, as the Japanese view it.
Answer. Reduction of in-process inventory.
42.10 How is a pull system distinguished from a push system in production and inventory control?
275
Answer. In a pull system, authorization to produce parts originates from downstream stations. In
a push system, authorization to produce derives from parts being produced by upstream stations
and forming queues in front of the production station.
42.11 What are the three phases in shop floor control?
Answer. The three phases are (1) order release, (2) order scheduling, and (3) order progress.
Multiple Choice Quiz
There are a total of 17 correct answers in the following multiple choice questions (some questions have
multiple answers that are correct). To attain a perfect score on the quiz, all correct answers must be
given, since each correct answer is worth 1 point. For each question, each omitted answer or wrong
answer reduces the score by 1 point, and each additional answer beyond the number of answers required
reduces the score by 1 point. Percentage score on the quiz is based on the total number of correct
answers.
42.1 Which one of the following terms best describes the overall function of production planning and
control? (a) inventory control, (b) manufacturing logistics, (c) manufacturing engineering, (d) mass
production, or (e) product design.
Answer. (b)
42.2 Which of the following are the categories usually listed in the master production schedule (more
than one)? (a) components used to build the final products, (b) firm customer orders, (c) general
product lines, (d) orders for maintenance and spare parts, (e) sales forecasts, and (f) spare tires.
Answer. (b), (d), and (e).
42.3 Inventory carrying costs include which of the following (more than one)? (a) equipment downtime,
(b) investment, (c) obsolescence, (d) setup, (e) spoilage, (f) stock-out, and (g) storage.
Answer. (b), (c), (e), and (g).
42.4 Which of the following are the terms in the economic order quantity formula (name three)? (a)
annual demand rate, (b) batch size, (c) cost per piece, (d) holding cost, (e) interest rate, and (f)
setup cost.
Answer. (a), (d), and (f).
42.5 Order point inventory systems are intended for which of the following (more than one)? (a)
dependent demand items, (b) independent demand items, (c) low production quantities, (d) mass
production quantities, and (e) mid- range production quantities.
Answer. (b) and (e).
42.6 With which of the following manufacturing resources is capacity requirements planning primarily
concerned (more than one)? (a) component parts, (b) direct labor, (c) inventory storage space, (d)
production equipment, and (e) raw materials.
Answer. (b) and (d).
42.7 The word kanban is most closely associated with which one of the following? (a) capacity
planning, (b) economic order quantity, (c) just-in-time production, (d) master production schedule,
or (e) material requirements planning.
Answer. (c)
276
42.8 The term machine loading refers most closely to which one of the following? (a) assigning jobs
to a work center, (b) floor foundation in the factory, (c) managing work-in- process in the factory,
(d) releasing orders to the shop, or (e) sequencing jobs through a machine.
Answer. (a)
Problems
Inventory Control
42.1 A product is made-to-stock. Annual demand is 60,000 units. Each unit costs $4.00 and the annual
holding cost rate = 25%. Setup cost to produce this product is $300. Determine: (a) economic
order quantity and (b) total inventory costs for this situation.
Solution: (a) EOQ = (2Da Csu/Ch)0.5 = (2 x 60,000 x 300/(.25 x 4.00))0.5 = 6000 units
(b) TIC = ChQ/2 + CsuDa/Q = 0.25(4.00)(6000/2) + 300(60,000/6000) = 3000 + 3000 = $6000
42.2 Given: annual demand for product X is 20,000 units; cost per unit = $6.00; holding cost rate =
2.5%/month; changeover (setup) time between products averages 2.0 hr; downtime cost during
changeover = $200/hr. Determine: (a) economic order quantity and (b) total inventory costs for
this situation.
Solution: (a) EOQ = (2Da Csu/Ch)0.5 = (2 x 20,000 x 2 x 200/(12 x .025 x 6.00))0.5 = 2981 units
(b) TIC = ChQ/2 + CsuDa/Q = 12 x 0.025(6.00)(2981/2) + 2 x 200(20,000/2981)
= 2683 + 2684 = $5387
42.3 A product is produced in batches. Batch size = 2000 units. Annual demand = 50,000 units, and unit
cost of the product = $4.00. Setup time to run a batch = 2.5 hr, cost of downtime on the affected
equipment is figured at $250/hr, and annual holding cost rate = 30%. What would the annual
savings be if the product were produced in the economic order quantity?
Solution: Current TIC = ChQ/2 + CsuDa/Q = 0.30(4.00)(2000/2) + 2.5 x 250(50,000/2000)
= 1200 + 15,625 = $16,825
EOQ = (2Da Csu/Ch)0.5 = (2 x 50,000 x 2.5 x 250/(0.30 x 4.00))0.5 = 7217 units
TIC at EOQ = ChQ/2 + CsuDa/Q = 0.30(4.00)(7217/2) + 2.5 x 250(50,000/7217)
= 4330 + 4330 = $8660
Savings = 16,825 - 8660 = $8165
42.4 A certain piece of production equipment is used to produce various components for an assembled
product of the XYZ Company. To keep in-process inventories low, it is desired to produce the
components in batch sizes of 150 units (daily requirements for assembly). Demand for each
product is 2500 units per year. Production downtime costs an estimated $200/hr. All of the
components made on the equipment are of approximately equal value: Cp = $9.00/unit. Holding
cost rate = 30%/yr. In how many minutes must the changeover (setup) between batches be
completed in order for 100 units to be the economic order quantity?
Solution: EOQ = (2Da CdtTsu /hCp)0.5
(EOQ)2 = 2Da CdtTsu/hCp
Tsu = hCp(EOQ)2/2Da Cdt = 0.3(9.00)(150)2/(2 x 2500 x 200) = 0.06075 hr = 3.65 min.
42.5 Current changeover (setup) time on a certain machine = 3.0 hr. Cost of downtime on this machine
is estimated at $200/hr. Annual holding cost per part made on the equipment, Ch = $1.00. Annual
demand for the part is 15,000 units. Determine: (a) EOQ and (b) total inventory costs for this data.
277
Also, determine: (c) EOQ and (b) total inventory costs, if the changeover time could be reduced to
six minutes.
Solution: (a) EOQ = (2Da Csu/Ch)0.5 = (2 x 15,000 x 3.00 x 200/1.00)0.5 = 4243 pc
(b) TIC = ChQ/2 + CsuDa/Q = 1.00(4243/2) + 3.00 x 200(15,000/4243)
= 2121.50 + 2121.14 = $4242.64
(c) If Tsu = 6 min = 0.1 hr, Csu = CdtTsu = 200(0.1) = $20.
EOQ = (2 x 15,000 x 20/1.00)0.5 = 775 pc
(d) TIC = 1.00(775/2) + 20(15,000/775) = 387.50 + 387.10 = $774.60
42.6 The two-bin approach is used to control inventory for a particular low-cost component. Each bin
holds 1000 units. The annual usage of the component is 40,000 units. Cost to order the component
is around $50. (a) What is the imputed holding cost per unit for this data? (b) If the actual annual
holding cost per unit is only 5 cents, what lot size should be ordered? (c) How much more is the
current two-bin approach costing the company annually, compared to the economic order
quantity?
Solution: (a) EOQ = (2Da Csu/Ch)0.5
1000 = (2 x 40,000 x 50/Ch)0.5
Ch = 2 x 40,000 x 50/1000 = $4.00 annually
(b) Given Ch = $0.05, EOQ = (2 x 40,000 x 50/0.05)0.5 = 8944.3 8944 pc
(c) For the two-bin approach in which Q = 1000, TIC = 0.05(1000/2) + 50(40,000/1000) = 250 +
2000 = $2250.00
For the EOQ = 8944, TIC = 0.05(8944/2) + 50(40,000/8944) = 223.60 + 223.61 = $447.21
Additional cost = 2250.00 - 447.21 = $1802.79
Material Requirements Planning
42.7 Quantity requirements are to be planned for component C2 in product P1. Required deliveries for
P1 are given in Figure 42.2. Ordering, manufacturing, and assembly lead times are as follows:
Item identification: P1 S1 C2 M2
Lead time (weeks): 1 2 1 2
Given the product structure in Figure 42.6, determine the time-phased requirements for M2, C2,
and S1 to meet the master schedule for P1. Assume no common use items and all on-hand
inventories and scheduled receipts are zero. Use a format similar to Figure 42.7. Ignore demand
for P1 beyond period 10.
Solution:
Period 1 2 3 4 5 6 7 8 9 10
P1 Requirements 50 75 100
Order Release 50 75 100
S1 Requirements 50 75 100
Order Release 50 75 100
C2 Requirements 200 300 400
Order Release 200 300 400
278
M2 Requirements 200 300 400
Order Release 200 300 400
42.8 Requirements are to be planned for component C5 in product P1. Required deliveries for P1 are
given in Figure 42.2. Ordering, manufacturing, and assembly lead times are as follows:
Item identification: P1 S2 C5 M5
Lead time (weeks): 1 1 3 2
Given the product structure in Figure 42.6, determine the time-phased requirements for M5, C5,
and S2 to meet the master schedule for P1. Assume no common use items. On-hand inventories
are: 200 units for M5 and 100 units for C5, zero for S2. Use a format similar to Figure 42.7.
Ignore demand for P1 beyond period 10.
Solution:
Period 1 2 3 4 5 6 7 8 9 10
P1 Requirements 50 75 100
On-hand: 0
Net Requirements 50 75 100
Order Release 50 75 100
S2 Requirements 100 150 200
On hand: 0
Net Requirements 100 150 200
Order Release 100 150 200
C5 Requirements 200 300 400
On hand: 100 100
Net Requirements 100 300 400
Order Release 100 300 400
M5 Requirements 100 300 400
On hand: 200 100 100
Net Requirements 0 200 400
Order Release 0 200 400
42.8 Solve the previous problem except that the following is known in addition to the information given:
scheduled receipts of M5 are 250 units in period (week) 3 and 50 units in period (week) 4.
Solution:
Period 1 2 3 4 5 6 7 8 9 10
P1 Requirements 50 75 100
On-hand: 0
Net Requirements 50 75 100
Order Release 50 75 100
S2 Requirements 100 150 200
On hand: 0
Net Requirements 100 150 200
Order Release 100 150 200
C5 Requirements 200 300 400
On hand: 100 100
Net Requirements 100 300 400
279
Order Release 100 300 400
M5 Requirements 100 300 400
Scheduled Receipts 250 50
On hand: 200 450 350 100
Net Requirements -350 -100 300
Order Release 0 0 300
Order Scheduling
42.10 Four products are to be manufactured in Department A, and it is desired to determine how to
allocate resources in that department to meet the required demand for these products for a certain
week. The demand and other data for the products are given as follows:
Product Weekly demand Setup time Operation time
1 750 6 hr 4.0 min.
2 900 5 hr 3.0 min.
3 400 7 hr 2.0 min.
4 400 6 hr 3.0 min.
The plant normally operates one shift (7.0 hours per shift), five days per week and there are
currently 3 work centers in the department. Propose a way of scheduling the machines to meet
the weekly demand.
Solution: Determine time to produce each product, assuming a single setup for each product:
Product 1: Time per batch = 6.0 + 750(4/60) = 6 + 50 = 56 hr.
Product 2: Time per batch = 5.0 + 900(3/60) = 5 + 45 = 50 hr
Product 3: Time per batch = 7.0 + 400(2/60) = 7 + 13.333 = 20.333 hr
Product 4: Time per batch = 6.0 + 400(3/60) = 6 + 20 = 26 hr
Total hours for all four products = 56 + 50 + 20.333 + 26 = 152.333 hr.
Available hours per week on 3 work centers if normal hours are assumed = 3 x (5 x 7) = 105
hr. This is fewer than the number of hours required. To meet the weekly production, overtime
must be used. The following schedule is proposed:
Work center Product Quantity Setup hours Run hours Hrs/product Hrs/wk center
I 1 750 6.0 50.0 56.0 56.0
II 2 900 5.0 45.0 50.0 50.0
III 3 400 7.0 13.333 20.333
4 400 6.0 20.0 26.0 46.333
_____ ______ ______ ______
Totals 24.0 128.333 152.333 152.333
42.11 In the previous problem, propose a way of scheduling to meet the weekly demand if there were
four machines instead of three.
Solution: Time to produce each product is the same as given in the preceding solution, under the
assumption that a single setup is required for each product. Available hours per week on 4 work
centers if normal hours are assumed = 4 x (5 x 7) = 140 hr. This is fewer than the number of
hours required. To meet the weekly production, overtime must be used. In order to equalize the
workload among machines as much as possible, let us propose to produce products 1 and 3 on
work centers 1 and 2 and Products 2 and 4 on work centers 3 and 4. In both cases, this will
require an additional setup
280
We want to equalize the workload on work centers I and II with Products 1 and 3.
Work center I: TI = 6.0 + QI(4/60) = 6.0 + 0.06667QI
Work center II: TII =6.0 + 0.06667(750 - QI) + 7.0 + 400(2/60) = 76.333 - 0.06667QI
Setting TI = TII: 6.0 + 0.06667QI = 76.333 - 0.06667QI
2(0.06667 QI) = 0.13334 QI = 76.333 - 6.0 = 70.333
QI = 70.333/0.13334 = 528
TI = 6.0 + 0.06667(528) = 41.20 hr
TII = 76.333 - 0.06667(528) = 41.133 hr.
We next want to equalize the workload on work centers III and IV with Products 2 and 4.
Work center III: TIII = 5.0 + QIII(3/60) = 5.0 + 0.05 QIII
Work center IV: TIV = 5.0 + 0.05(900 - QIII) + 6.0 + 400(3/60) = 76.0 - 0.05 QIII
Setting TIII = TIV: 5.0 + 0.05 QIII = 76.0 - 0.05QIII
2(0.05 QIII) = 0.10 QI = 76.0 - 5.0 = 71.0
QI = 71.0/0.10 = 710
TI = 5.0 + 0.05(710) = 40.50 hr
TII = 76.0 - 0.05(710) = 40.50 hr.
The following table summarizes the production at each work center:
Work center Product Quantity Setup hours Run hours Hrs/product Hrs/wk center
I 1 528 6.0 35.20 41.20 41.20
II 1 222 6.0 14.80 20.80
3 400 7.0 13.33 20.33 41.13
III 2 710 5.0 35.50 40.50 40.50
IV 2 190 5.0 9.50 14.50
4 400 6.0 20.00 26.00 40.50
_____ ______ ______ ______
Totals 35.0 128.33 163.33 163.33
42.12 The current date in the production calendar of the XYZ Company is day 15. There are three
orders (A, B, and C) to be processed at a particular work center. The orders arrived in the
sequence A-B-C at the work center. The following table indicates the remaining process time and
production calendar due date for each order:
Order Remaining process time Due date
A 5 days Day 25
B 16 days Day 34
C 7 days Day 24
Determine the sequence of the orders that would be scheduled using: (a) first-come-first-serve,
(b) earliest due date, (c) shortest processing time, (d) least slack time, and (e) critical ratio.
Solution: (a) FCFS: sequence = A - B - C
(b) Earliest due date: sequence = C - A - B
(c) Shortest processing time: sequence = A - C - B
(d) Least slack time:
Order A slack time = (25 - 15) - 5 = 5
Order B slack time = (34 - 15) - 16 = 3
Order C slack time = (24 - 15) - 7 = 2
Sequence = C - B - A
281
(e) Critical ratio:
Order A critical ratio = (25 - 15)/5 = 2.0
Order B critical ratio = (34 - 15)/16 = 1.1875
Order C critical ratio = (24 - 15)/7 = 1.286
Sequence = B - C - A
282
43 QUALITY CONTROL
Review Questions
43.1 What are the two principal aspects of product quality?
Answer. The two quality aspects are: (1) product features, and (2) freedom from deficiencies.
43.2 How is a process operating in statistical control distinguished from one that is not?
Answer. The process in statistical control is characterized by only random variations. A process
that is out of control exhibits additional variation that is not normal, called assignable variation.
This indicates that something is wrong with the process.
43.3 Define process capability.
Answer. Process capability is the limits of the random variations of the process when it is in
statistical control. The limits are defined as the process mean 3 standard deviations.
43.4 What is the difference between control charts for variables and control charts for attributes?
Answer. In control charts for variables, measurements of the characteristic of interest are made.
In control charts for attributes, the characteristic of interest is identified as being acceptable of not
acceptable.
43.5 Identify the two types of control charts for variables.
Answer. The two charts are: (1) x chart for sample means and (2) R chart for ranges.
43.6 What are the two basic types of control charts for attributes?
Answer. The two charts are: (1) p chart for proportion of defects in a sample, and (2) c chart for
count of defects in a sample.
43.7 When interpreting a control chart, what does one look for to identify problems?
Answer. Problems are indicated by the following: (1) x or R lie outside the LCL or UCL limits;
(2) trends or cyclical patterns in the data; (3) sudden changes in average; and (4) points
consistently near the upper or lower limits.
Multiple Choice Quiz
There are a total of 12 correct answers in the following multiple choice questions (some questions have
multiple answers that are correct). To attain a perfect score on the quiz, all correct answers must be
given, since each correct answer is worth 1 point. For each question, each omitted answer or wrong
answer reduces the score by 1 point, and each additional answer beyond the number of answers required
reduces the score by 1 point. Percentage score on the quiz is based on the total number of correct
answers.
43.1 Which of the following would be classified as examples of a product feature, rather than a
freedom from deficiency (more than one)? (a) components within tolerance, (b) location of
ON/OFF switch, (c) no missing parts, (d) product weight, and (e) reliability.
Answer. (b), (d), and (e).
283
43.2 If the product tolerance is set so that the process capability index = 1.0, then the percentage of
parts that are within tolerance will be closest to which one of the following when the process is
operating in statistical control? (a) 35%, (b) 65%, (c) 95%, (d) 99%, or (e) 100%.
Answer. (e)
43.3 Which of the following principles and/or approaches are generally credited to G. Taguchi (more
than one)? (a) acceptance sampling, (b) control charts, (c) loss function, (d) Pareto priority index,
and (e) robust design.
Answer. (c) and (e).
43.4 In a control chart, the upper control limit is set equal to which one of the following? (a) process
mean, (b) process mean plus three standard deviations, (c) upper design tolerance limit, or (d)
upper value of the maximum range R.
Answer. (b)
43.5 The R chart is used for which one of the following product or part characteristics? (a) number of
rejects in the sample, (b) number of reworked parts in a sample, (c) radius of a cylindrical part, or
(d) range of sample values.
Answer. (d)
43.6 Which one of the following best describes the situations for which the c chart is most suited? (a)
control of defective parts, (b) mean value of part characteristic of interest, (c) number of defects
in a sample, or (d) proportion of defects in a sample.
Answer. (c)
43.7 Which of the following identify an out-of-control condition in a control chart (more than one)? (a)
consistently increasing value of x , (b) points near the central line, (c) R outside the control limits
of the R chart, and (d) x outside the control limits of the x chart.
Answer. (a), (c), and (d).
Problems
Process Capability and Statistical Tolerancing
43.1 An automatic turning process is set up to produce parts with a mean diameter = 6.255 cm. The
process is in statistical control and the output is normally distributed with a standard deviation =
0.004 cm. Determine the process capability.
Solution: Process capability PC = 3= 6.255 3(0.004) = 6.255 ± 0.012 cm
The upper and lower limits of the process capability range are: 6.243 to 6.267 cm
43.2 In the previous problem, the design specification on the part is: diameter = 6.250 0.013 cm. (a)
What proportion of parts fall outside the tolerance limits? (b) If the process were adjusted so that
its mean diameter = 6.250 cm and the standard deviation remained the same, what proportion of
parts would fall outside the tolerance limits?
Solution: (a) Given process mean = 6.255 cm and = 0.004 cm and tolerance limits 2.237 to
2.263. On the lower side of the tolerance limit, using the standard normal distribution,
z = (6.237 - 6.255)/0.004 = -4.50.
Conclusion: there are virtually no defects on the lower side of the tolerance. On the upper side of
the tolerance limit, z = (6.263 - 2.255)/0.004 = +2.00
284
Using tables of the standard normal distribution, Pr(z > 2.00) = 0.0227
The proportion of defects with the current process mean = 0.0227 = 2.27%.
(b) Given process mean = 6.250 cm and = 0.004 cm and tolerance limits 6.237 to 6.263.
On the lower side of the tolerance limit, z = (6.237 - 6.250)/0.004 = -3.25. Using tables of the
standard normal distribution, Pr(z < -3.25) = 0.0006. On the upper side of the tolerance limit, z = (6.263 - 6.250)/0.004 = +3.25. Using tables of the standard normal distribution, Pr(z > 3.25) = 0.0006
The proportion of defects with the current process mean = 0.0006 + 0.0006 = 0.0012 = 0.12%.
43.3 A sheet metal bending operation produces bent parts with an included angle = 92.1. The process
is in statistical control and the values of included angle are normally distributed with a standard
deviation = 0.23. The design specification on the angle = 90 2. (a) Determine the process
capability. (b) If the process could be adjusted so that its mean = 90.0, determine the value of the
process capability index.
Solution: (a) PC = 92.1 3(0.23) = 92.10.69.
The upper and lower limits of the process capability range are: 91.41to 92.79.
(b) If = 90
T = 92- 88= 4
PCI = 4/(6 x 0.23) = 2.9 virtually no defects.
43.4 A plastic extrusion process produces extrudate with a critical cross-section dimension = 28.6 mm.
The process is in statistical control and the output is normally distributed with standard deviation =
0.53 mm. Determine the process capability.
Solution: Process capability PC = 3= 28.6 3(0.53) = 28.6 ± 1.59 mm
The upper and lower limits of the process capability range are: 27.01 to 30.19 mm.
43.5 In the previous problem, the design specification on the part is: diameter = 28.0 2.0 mm. (a)
What proportion of parts fall outside the tolerance limits? (b) If the process were adjusted so that
its mean diameter = 28.0 mm and the standard deviation remained the same, what proportion of
parts would fall outside the tolerance limits? (c) With the adjusted mean at 28.0 mm, determine the
value of the process capability index.
Solution: (a) Given process mean = 28.6 mm and = 0.53 mm and tolerance limits 26.0 to 30.0
mm. On the lower side of the tolerance limit, using the standard normal distribution,
z = (26.0 - 28.6)/0.53 = -4.01.
Conclusion: there are virtually no defects on the lower side of the tolerance.
On the upper side of the tolerance limit, z = (30.0 - 28.6)/0.53 = +2.64
Using tables of the standard normal distribution, Pr(z > 2.64) = 0.0041
The proportion of defects with the current process mean = 0.0041 = 0.41%.
(b) Given process mean = 28.0 mm and = 0.53 mm and tolerance limits 26.0 to 30.0 mm. On
the lower side of the tolerance limit, z = (26.0 - 28.0)/0.53 = -3.77. Using tables of the standard
normal distribution, Pr(z < -3.77) = approx. 0.0001 On the upper side of the tolerance limit, z = (30.0 - 28.0)/0.53 = +3.77 Using tables of the standard normal distribution, Pr(z > 3.77) = approx. 0.0001
The proportion of defects with the current process mean = 0.0001 + 0.0001 = 0.0002 = 0.02%.
(c) Process capability index PCI = 4.0/(6 x 0.53) = 1.258
285
43.6 An assembly consists of four components stacked to create an overall dimension of 2.500 inch,
with a bilateral tolerance Ta = 0.020 inch (0.010 inch). The dimensions of the individual parts are
each 0.625 inch. All parts will have identical bilateral tolerances. Determine the tolerance (a)
under a worst case design approach, and (b) using a statistical tolerancing approach.
Solution: (a) Worst case: Ti = 0.020/4 = 0.005 in. Interpret this to be ± 0.005 in.
(b) Statistical tolerancing: Ta = (4 Ti
2)0.5
Ta
2 = 4 Ti
2
(0.020)2 = 0.0004 = 4 Ti
2
Ti
2 = 0.0004/4 = 0.0001 = 10-4
Ti = 10-2 = 0.010. Interpret this to be ± 0.010 in.
43.7 An assembly is made by stacking 20 flat pieces of sheet metal to produce a thick laminated
structure. The sheet metal blanks are all cut with the same punch and die to the desired profile, so
that the thick assembly has the same profile. All of the parts are cut from the same sheet metal
coil, whose thickness specification is 1/16 inch 0.002 inch. The thickness of the final assembly is
specified as 1.250 0.010 inch. Does a statistical tolerancing approach apply in this situation?
Why?
Solution: If statistical tolerancing were used, Ta = (20 x 0.0042)0.5 = (0.00032)0.5 = 0.0179 in.
Expressing this as a bilateral tolerance, Ta = 0.00895 in., which is well within the specified
tolerance of 0.010 in.
However, this would not be a good application of statistical tolerancing because the process
making the components is the same process; specifically the rolling process making the starting
piece of sheet metal from which the laminating pieces are fabricated. The absence of
independent processes making the individual components is a violation of one of the fundamental
assumptions of statistical tolerancing. It would be more appropriate to use worst case tolerancing
here, which would yield Ta = 20(0.002) = 0.040 in., significantly beyond the specified
tolerance.
43.8 The assembly in Figure P43.8 has a critical assembly dimension C = 5.000 cm. If each part is
made from an independent process with process means for part thickness all set to 2.500 cm and
standard deviation = 0.005 cm, what is the process capability of the critical dimension C. Assume
the opposite sides of each part on the 2.50 cm dimension are parallel.
Solution: Dimension C is determined by two middle parts. a
2 = 2 i
2 = 2(0.005)2 = 0.00005
a = (0.00005)0.5 = 0.00707 cm
PC = 3= 2(2.500) 3(0.00707) = 5.000 ± 0.0212 cm
43.9 An assembly consists of three parts stacked to form a final dimension of 30.0 mm with tolerance
= 0.20 mm. The relevant part dimensions making up the 30 mm total are 5 mm, 10 mm, and 15
mm. Parts are produced by independent manufacturing operations, whose process capabilities are
proportional to their respective dimensions. Given that the part tolerances are to be a constant
proportion of the respective dimensions, determine the tolerance for each part using: (a) worst
case design and (b) statistical tolerancing.
Solution: (a) Worst case tolerancing: Ta = 0.40 mm. Let us apportion the tolerances among parts
in proportion to their size. Thus, T1 = T, T2 = 2T, T3 = 3T.
T1 + T2 + T3 = T + 2T + 3T = 6T = 0.40 mm.
T = 0.40/6 = 0.0667 mm.
T1 = 0.0667 mm = ± 0.0333 mm , T2 = 0.1333 mm = ± 0.0667 mm,
T3 = 0.200 mm = ± 0.100 mm.
286
(b) Statistical tolerancing: Ta = 0.40 mm.
Again, let us apportion the tolerances among parts in proportion to their size. Thus, T1 = T, T2 =
2T, T3 = 3T.
Ta = 0.40 = (T1 2 + T2
2 + T3
2)0.5 = (T2 + (2T)2 + (3T)3)0.5
= (T2 + 4T2 + 9T2)0.5 = (14T2)0.5 = 3.7417 T
T = 0.40/3.7417 = 0.1069
T1 = 0.1069 mm = ± 0.0535 mm , T2 = 0.2138 mm = ± 0.1069 mm,
T3 = 0.3207 mm = ± 0.1604 mm.
43.10 Figure P43.10 shows an assembly in which the critical dimension is C. Each part used in the
assembly, including the base part, has a thickness = 10.0 mm, with process capability = 0.1 mm
for the thickness. Given that the process capability index for the parts PCI = 1.0, and the PCI for
the assembly will also be 1.0, determine the recommended tolerance for C using: (a) worst case
design and (b) statistical tolerancing.
Solution: Given that PCI = 1.0, then Ti = 6= 0.2 mm and = 0.2/6 = 0.0333 mm.
Critical dimension C = 5(10) - 3(10) = 20 mm.
(a) Worst case tolerancing: there are 8 parts that determine critical dimension C, each with
independent variability. Therefore, Ta = 8Ti = 8(0.2) = 1.6 mm = ± 0.80 mm.
(b) Statistical tolerancing: Ta = (8Ti
2)0.5 = (8(0.2)2)0.5 = (0.32)0.5 = 0.566 mm = ± 0.283 mm.
43.11 Solve part (b) of the previous problem, except that the process capability index for the assembly is
a more conservative 1.5. The PCI for the individual parts is still 1.0.
Solution: Given that parts PCI = 1.0, then Ti = 6= 0.2 mm and = 0.2/6 = 0.0333 mm.
Critical dimension C = 5(10) - 3(10) = 20 mm.
Given that assembly PCI = 1.5, Ta = 1.5 PC = 1.5 a = 9 a
Statistical tolerancing: a = (8i
2)0.5 = (8(0.03333)2)0.5 = (0.008889)0.5 = 0.0943 mm
Ta = 9(0.0943) = 0.8485 = ± 0.424 mm
Control Charts
43.12 Ten samples of size n = 8 have been collected from a process in statistical control, and the
dimension of interest has been measured for each part. (a) Determine the values of the center,
LCL, and UCL for the x and R charts. The calculated values of x and R for each sample are
given below (measured values are in mm). (b) Construct the control charts and plot the sample
data on the charts.
s 1 2 3 4 5 6 7 8 9 10
x 9.22 9.15 9.20 9.28 9.19 9.12 9.20 9.24 9.17 9.23
R 0.24 0.17 0.30 0.26 0.27 0.19 0.21 0.32 0.21 0.23
Solution: x = x /m
= (9.22 + 9.15 + 9.20 + 9.28 + 9.19 + 9.12 + 9.20 + 9.24 + 9.17 + 9.23)/10 = 9.20
R = R/7 = (0.24 + 0.17 + 0.30 + 0.26 + 0.27 + 0.19 + 0.21 + 0.32 + 0.21 + 0.23)/10 = 0.24
(a) x chart: x = 9.20 mm = CL
LCL = x - A2 R = 9.20 - 0.373(0.24) = 9.1105 mm.
UCL = x + A2 R = 9.20 + 0.373(0.24) = 9.2895 mm.
287
R chart: R = 0.0133 = CL
LCL = D3 R = 0.136(0.24) = 0.0326 mm.
UCL = D4 R = 2.114(0.0133) = 0.4474 mm.
(b) Student exercise.
43.13 Seven samples of 5 parts each have been collected from an extrusion process that is in statistical
control, and the diameter of the extrudate has been measured for each part. (a) Determine the
values of the center, LCL, and UCL for x and R charts. The calculated values of x and R for
each sample are given below (measured values are in inches). (b) Construct the control charts
and plot the sample data on the charts.
s 1 2 3 4 5 6 7
x 1.002 0.999 0.995 1.004 0.996 0.998 1.006
R 0.010 0.011 0.014 0.020 0.008 0.013 0.017
Solution: x = x /m = (1.002 + 0.999 + 0.995 + 1.004 + 0.996 + 0.998 + 1.006)/7 = 1.000
R = R/7 = (0.010 + 0.011 + 0.014 + 0.020 + 0.008 + 0.013 + 0.017)/7 = 0.0133
(a) x chart: x = 1.000 in. = CL
LCL = x - A2 R = 1.000 - 0.577(0.0133) = 0.9923 in.
UCL = x + A2 R = 1.000 + 0.577(0.0133) = 1.0077 in.
R chart: R = 0.0133 = CL
LCL = D3 R = 0
UCL = D4 R = 2.114(0.0133) = 0.0281 in.
(b) Student exercise.
43.14 In 12 samples of size n = 7, the average value of the sample means is x = 6.860 cm for the
dimension of interest, and the mean of the ranges of the samples is R = 0.027 cm. Determine: (a)
lower and upper control limits for the x chart and (b) lower and upper control limits for the R
chart. (c) What is your best estimate of the standard deviation of the process?
Solution: (a) x chart: x = 6.860 cm = CL
LCL = x - A2 R = 6.860 - 0.419(0.027) = 6.8487 cm
UCL = x + A2 R = 6.860 + 0.419(0.027) = 6.8713 cm
(b) R chart: R = 0.027 = CL
LCL = D3 R = 0.076(0.027) = 0.0205 cm
UCL = D4 R = 1.924(0.027) = 0.0519 cm
(c) The x chart is based on 3 x/ n
Therefore, A2 R = 3 x/ n
x = A2 R n /3 = 0.419(0.027) 7 /3 = 0.00998 cm
43.15 In nine samples of size n = 10, the grand mean of the samples is x = 100 for the characteristic of
interest, and the mean of the ranges of the samples is R = 8.5. Determine: (a) lower and upper
288
control limits for the x chart and (b) lower and upper control limits for the R chart. (c) Based on
the data given, estimate the standard deviation of the process?
Solution: (a) x chart: x = 100 = CL
LCL = x - A2 R = 100 - 0.308(8.5) = 102.618.
UCL = x + A2 R = 100 + 0.308(8.5) = 97.382.
(b) R chart: R = 8.5 = CL
LCL = D3 R = 0.223(8.5) = 1.8955.
UCL = D4 R = 1.777(8.5) = 15.1045.
(c) The x chart is based on 3 x/ n
Therefore, A2 R = 3 x/ n
x = A2 R n /3 = 0.308(8.5) 10 /3 = 2.7596.
43.16 A p chart is to be constructed. Six samples of 25 parts each have been collected, and the average
number of defects per sample was 2.75. Determine the center, LCL and UCL for the p chart.
Solution: p = 2.75/25 = 0.11 = CL
LCL = p - 3 p(1 p) / n = 0.11 - 3 0.11(0.89) / 25 = 0.11 - 3(0.0626) = -0.078 ® 0
UCL = p + 3 p(1 p) / n = 0.11 + 3 0.11(0.89) / 25 = 0.11 + 3(0.0626) = 0.298
43.17 Ten samples of equal size are taken to prepare a p chart. The total number of parts in these ten
samples was 900 and the total number of defects counted was 117. Determine the center, LCL
and UCL for the p chart.
Solution: d = 117/10 = 11.7.
p = 11.7/90 = 0.13 = CL
LCL = p - 3 p(1 p) / n = 0.13 - 3 0.13(0.87) / 90 = 0.13 - 3(0.03545) = 0.024
UCL = p + 3 p(1 p) / n = 0.11 + 3 0.13(0.87) / 90 = 0.11 + 3(0.03545) = 0.236
43.18 The yield of good chips during a certain step in silicon processing of integrated circuits averages
91%. The number of chips per wafer is 200. Determine the center, LCL, and UCL for the p chart
that might be used for this process.
Solution: Use p = 1 - 0.91 = 0.09 = CL
LCL = p - 3 p(1 p) / n = 0.09 - 3 0.09(0.91) / 90 = 0.09 - 3(0.0202) = 0.0293
UCL = p + 3 p(1 p) / n = 0.11 + 3 0.09(0.91) / 90 = 0.09 + 3(0.0202) = 0.1507
43.19 The upper and lower control limits for a p chart are: LCL = 0.19 and UCL = 0.24. Determine the
sample size n that is used with this control chart.
Solution: p = 0.5(UCL + LCL) = 0.5(.24 + .10) = 0.17
UCL - LCL = 0.24 - 0.10 = 0.14 = 6 p(1 p) / n = 6 0.17(0.83) / n
(0.14)2 = 62 (0.17 x 0.83/n)
0.0196 = 36(0.17)(0.83)/n = 5.0796/n
n = 5.0796/0.0196 = 259.2 259
289
43.20 The upper and lower control limits for a p chart are: LCL = 0 and UCL = 0.10. Determine the
minimum possible sample size n that is compatible with this control chart.
Solution: p = 0.5(UCL + LCL) = 0.5(.10 + 0) = 0.05
LCL = p - 3 p(1 p) / n = 0
Therefore, p = 3 p(1 p) / n
0.05 = 3 0.05(0.95) / n
(0.05)2 = 0.0025 = 32 (0.05)(0.95)/n = 0.4275/n
n = 0.4275/0.0025 = 171
43.21 Twelve cars were inspected after final assembly. The number of defects found ranged between
87 and 139 defect per car with an average of 116. Determine the center and upper and lower
control limits for the c chart that might be used in this situation.
Solution: CL = 116
LCL = c - 3 c = 116 - 3 116 = 83.7 ® 83
UCL = c + 3 c = 116 + 3 116 = 148.3 ® 148
290
44 MEASUREMENT AND INSPECTION
Review Questions
44.1 How is measurement distinguished from inspection?
Answer. In measurement, an unknown quantity is compared with a known standard and a value
of the quantity is obtained using an accepted and consistent system of units. In inspection, a
product characteristic is examined to see if it conforms to design specifications.
44.2 How does gaging differ from measuring?
Answer. Gaging determines whether the product characteristic (e.g., dimension) satisfies the
specification or not. Measuring determines the actual value of the characteristic.
44.3 What are the six fundamental quantities in metrology?
Answer. The six quantities are: length, mass, time, electric current, temperature, and light
radiation.
44.4 What is accuracy in measurement?
Answer. Accuracy is the degree to which the measured value agrees with the true value of the
quantity of interest. It is a measurement procedure that is absent of systematic errors.
44.5 What is precision in measurement?
Answer. Precision in measurement is the degree to which random errors are minimized.
44.6 What is meant by the term calibration?
Answer. Calibration means checking the measuring instrument against a known standard.
44.7 Besides good accuracy and precision, what are the desirable attributes and features of a
measuring instrument?
Answer. The desirable features include: high resolution, ease of calibration, speed of response,
wide operating range, high reliability, and low cost.
44.8 What is the rule of 10?
Answer. The rule of 10 means that the measuring instrument or gage must be ten times more
precise than the object (part) being inspected.
44.9 Automated inspection can be integrated with the manufacturing process to accomplish some
action. What are these possible actions?
Answer. Possible actions discussed in text are: (1) parts sortation, and (2) feedback of data to
adjust the process.
44.10 Give an example of a non-contact inspection technique.
Answer. Non-contact inspection techniques include machine vision, laser measuring methods, and
electrical field techniques.
44.11 What is meant by the term graduated measuring device?
Answer. A graduated measuring device has markings (called graduations) on a linear or angular
scale to measure an object's feature of interest (e.g., length).
291
44.12 What are the common methods for assessing surface roughness?
Answer. Common methods are: (1) comparison of the specimen surface with standard test blocks
having known surface roughness values, and (2) stylus-type electronic instruments which measure
average roughness.
44.13 What is a coordinate measuring machine?
Answer. A CMM is an automated measuring machine consisting of a contact probe and a means
to position the probe in three dimensions relative to workpart features and surfaces; when the
probe contacts the part, the x-y-z coordinates are recorded.
44.14 Describe a scanning laser system.
Answer. The scanning laser system uses a laser beam deflected by a rotating mirror to produce a
beam of light that sweeps past an object. A photodetector on the far side of the object senses the
light beam during its sweep except for the short time when it is interrupted by the object. This time
period can be measured quickly with great accuracy. A microprocessor system measures the time
interruption that is related to the size of the object in the path of the laser beam, and converts from
time to a linear dimension.
44.15 What is a binary vision system?
Answer. In a binary vision system, the light intensity of each pixel is reduced to either of two
values (black or white, 0 or 1).
44.16 Name some of the non-optical non-contact sensor technologies available for inspection.
Answer. The technologies include: electrical fields (capacitance, inductance), radiation (X-ray),
and ultrasonic techniques (high frequency sound).
Multiple Choice Quiz
There are a total of 28 correct answers in the following multiple choice questions (some questions have
multiple answers that are correct). To attain a perfect score on the quiz, all correct answers must be
given, since each correct answer is worth 1 point. For each question, each omitted answer or wrong
answer reduces the score by 1 point, and each additional answer beyond the number of answers required
reduces the score by 1 point. Percentage score on the quiz is based on the total number of correct
answers.
44.1 In measurement and inspection for manufacturing, which one of the following fundamental
physical quantities are we most concerned with? (a) electric current, (b) length, (c) light radiation,
(d) mass, (e) temperature, or (f) time.
Answer. (b)
44.2 Which of the following are attributes of the “metric system” of linear measurement (more than
one)? (a) based on astronomical distances, (b) defined in terms of the human body, (c) originated
in Great Britain, (d) rational prefixes for units, and (e) units are subdivided decimally.
Answer. (d) and (e).
44.3 Which one of the following countries does not embrace the International System of units? (a)
China, (b) France, (c) Germany, (d) Japan, (e) Panama, (f) Russia, or (g) United States.
Answer. (g)
292
44.4 The two basic types of inspection are inspection by variables and inspection by attributes. The
second of these inspections uses which one of the following: (a) destructive testing, (b) gaging, (c)
measuring, or (d) non- destructive testing.
Answer. (b)
44.5 Automated 100% inspection can be integrated with the manufacturing process to accomplish
which of the following (more than one): (a) better design of products, (b) feedback of data to
adjust the process, (c) 100% perfect quality, and (d) sortation of good parts from defects.
Answer. (b) and (d) are mentioned in the text.
44.6 Which of the following are examples of contact inspection (more than one)? (a) calipers, (b)
coordinate measuring systems, (c) dial indicators, (d) machine vision, (e) micrometers, (f) scanning
laser systems, (g) snap gages, and (h) ultrasonic techniques.
Answer. (a), (b), (c), (e), and (g).
44.7 A surface plate is most typically made of which one of the following materials? (a) aluminum
oxide ceramic, (b) cast iron, (c) granite, (d) hard polymers, or (e) stainless steel.
Answer. (c)
44.8 Which of the following are graduated measuring instruments (more than one)? (a) bevel
protractor, (b) dial indicator, (c) divider, (d) micrometer, (e) outside calipers, (f) sine bar, (g) steel
rule, (h) surface plate, and (i) vernier caliper.
Answer. (a), (b), (d), (g), and (i).
44.9 An outside micrometer would be appropriate in the measurement of which of the following (more
than one)? (a) hole depth, (b) hole diameter, (c) part length, (d) shaft diameter, and (e) surface
roughness.
Answer. (c) and (d).
44.10 In a GO/NO-GO gage, which one of the following best describes the function of the GO gage?
(a) checks limit of maximum tolerance, (b) checks maximum material condition, (c) checks
maximum size, (d) checks minimum material condition, or (e) checks minimum size.
Answer. (b)
44.11 Which of the following are likely to be GO/NO-GO gages (more than one)? (a) gage blocks, (b)
limit gage, (c) master gage, (d) plug gage, and (e) snap gage.
Answer. (b), (d), and (e).
44.12 Which of the following are contact sensing methods used in inspection (more than one)? (a)
calipers, (b) coordinate measuring machine, (c) laser techniques, (d) machine vision, (e)
micrometer, and (f) X-ray radiation.
Answer. (a), (b), and (e).
44.13 Which one of the following is the most important application of vision systems? (a) inspection, (b)
object identification, (c) safety monitoring, or (d) visual guidance and control of a robotic
manipulator.
Answer. (a)